Download as pdf or txt
Download as pdf or txt
You are on page 1of 139

5/17/24, 8:14 PM Exam Automation

College Knowledge
.

Test / Exam Name: Electric Charges And Field Standard: 12th Science Subject: Physics
Student Name: Section: Roll No.:
Questions: 653 Time: 01:00 hh:mm Marks: 1329

Q1. Which group among the following is insulator? 1 Mark


A Silver, copper, gold B Paper, glass, cotton
C The human body, wood, iron D Glass, copper, paper
Ans: B Paper, glass, cotton
Explanation:
Glass, paper, and cotton are good quality insulators. The rest options contain one or more conducting materials. Silver is the best conductor material available in
nature. But it is costly, so it can’t be used in the electricity distribution system.
Q2. For two statements are given-one labelled Assertion (A) and the other labelled Reason (R). Select the correct answer to these questions from the codes (a), (b), (c) 1 Mark
and (d) as given below.
Assertion: If there exists coulomb attraction between two bodies both of them must be charged
Reason: In coulomb attraction of two bodies are positively charged
A Both A and R are true and R is the correct explanation of A. B Both A and R are true but R is not the correct explanation of A.
C A is true but R is false. D A is false and R is also false.
Ans: D A is false and R is also false.
Q3. For two statements are given-one labelled Assertion (A) and the other labelled Reason (R). Select the correct answer to these questions from the codes (a), (b), (c) 1 Mark
and (d) as given below.
Assertion (A): If a conducting medium is placed between two charges, then electric force between them becomes zero.
Reason (R): Reduction in a force due to introduced material is inversely proportional to its dielectric constant.
A Both A and R are true, and R is the correct explanation of A. B Both A and R are true, but R is not the correct explanation of A.
C A is true but R is false. D A is false and R is also false.
Ans: A Both A and R are true, and R is the correct explanation of A.
Explanation:
F0
E0 q F0
The dielectric constant of any material is K = E
= F = F
q
F0
⇒F= K
, where F0 is force when conductor is not present between the charge. Fis a force after introduction of conductor between charges. Since dielectric
constant of a conducting medium is infinity therefore F = 0.
Q4. Is the force acting between two point charges q1 and q2 kept at some distance apart in air attractive or repulsive when, 1 Mark
1. q1q2 > 0?
2. q1q2 < 0?

Ans:1. Repulsive.
2. Attractive.
Q5. For two statements are given-one labelled Assertion (A) and the other labelled Reason (R). Select the correct answer to these questions from the codes (a), (b), (c) 1 Mark
and (d) as given below.
Assertion: A point charge is brought in an electric field at a nearby point will increase or decrease depending on the nature of charge.
Reason: The electric field is independent of the nature of charge.
A Both A and R are true and R is the correct explanation of A. B Both A and R are true but R is not the correct explanation of A.
C A is true but R is false. D A is false and R is also false.
Ans: C A is true but R is false.
Q6. K q1q2 1 Mark
The electrostatic force between two point charges q1​and q2​at separation 'r' is given by F = . ​The constant K:
r2

A Depends on the system of units only. B Depends on the medium between the charges only.
C Depends on both the system of units and the medium between the charges. D Is independent of both the system of units and the medium between the
charges.
Ans: C Depends on both the system of units and the medium between the charges.
Explanation:
1
K= and its units is N−m2/ C2
4πϵ ∘ k
It depend on system of unit and dielectric constant (k), that's medium between charges
Q7. The band gap between the valence band and conduction band is the measure of ______? 1 Mark
A The conductivity of the material. B The resistivity of the material.
C Charge density. D Ease of ionization.
Ans: A The conductivity of the material.
Explanation:
The more the band gap between the valence band and conduction band, the worse is the conductivity of the material. For conductors, there are overlapping
bands. So, conductors can carry electricity.
Q8. A hemisphere is uniformely charged positively. The electric field at a point on a diameter away from the centre is directed: 1 Mark
A Perpendicular to the diameter. B Parallel to the diameter.
C At an angle tilted towards the diameter. D At an angle tilted away from the diameter.
Ans: A Perpendicular to the diameter.
In case of a uniformly positive charged hemisphere, if a point situated at a point on a diameter away from the centre, the electric field should be perpendicular to
the diameter. In this case the component of electric field intensity parallel to the diameter cancel out.
Q9. A positive charge Q is uniformly distributed along a circular ring of radius R.A. small test charge q is placed at the centre of the ring Fig. Then: 1 Mark

https://bls.smartstudies.co.in/#/exam/pdf-preview/b5b87181-b79b-4b4a-8ebf-8f94fb605fc8/1 1/139
5/17/24, 8:14 PM Exam Automation

A If q > 0 and is displaced away from the centre in the plane of the ring, it will B If q < 0 and is displaced away from the centre in the plane of the ring, it will
be pushed back towards the centre. never return to the centre and will continue moving till it hits the ring.
C If q < 0, it will perform SHM for small displacement along the axis. D q at the centre of the ring is in an unstable equilibrium within the plane of
the ring for q > 0.
Ans: A If q > 0 and is displaced away from the centre in the plane of the ring, it will be pushed back towards the centre.
B If q < 0 and is displaced away from the centre in the plane of the ring, it will never return to the centre and will continue moving till it hits the ring.
C If q < 0, it will perform SHM for small displacement along the axis.
Here, the positive charge Q is uniformly distributed on the outer surface of the ring. Thus, the electric field inside the ring is zero. So, the effect of electric force on
charge q due to the positive charge Q is at the centre zero.
When q < 0, force between Q and q is attractive. If q is displaced away from the centre in the plane of the ring, the net force on one side decrease and other side
increase by the same amount. So, it will be pushed back towards the centre.
When this charge is displaced towards the axis of the ring, it will gain potential energy and will be pulled by the ring (towards its centre). Due to increase in kinetic
energy, it will cross the centre and again gains potential energy. Afterwards, it will be again pulled towards the centre and this process (SHM) will continue due to
inter conversion of energies.
When q > 0, force between Q and q is repulsive. If q is displaced away from the centre in the plane of the ring, again, the net force on one side decrease and other
side increase by the same amount. So, it will be pushed back towards the centre.
Within the plane of the ring, the equilibrium is stable as the charge q tends to move back to its position.
Q10. When an object possess electric charge then the object is said to be: 1 Mark
A Charged. B Electrified. C Both a and b. D None.
Ans: C Both a and b.
Q11. Two field lines can ____. 1 Mark
A Never cross each other. B May cross each other. C Both a and b. D None.
Ans: A Never cross each other.
Q12. Insulation breakdown may occur at _______? 1 Mark
A High temperature B Low temperature C At any temperature D Depends on pressure
Ans: A High temperature
Explanation:
At high temperatures, electrons of insulators get excited and then the electrons can overcome the large energy band gaps between valence and conduction bands.
So a large number of electrons travel to the conduction band and they act as conductor i.e. insulation breakdown occurs.
Q13. Which one is not the property of charge? 1 Mark
A Charge is additive. B Charge is conserved.
C Quantization of charge. D A charge is self-destructive.
Ans: D A charge is self-destructive.
Explanation:
Electric charge possesses the properties of quantization, conservation of charge. It cannot be destroyed i.e. it is not self-destructive.
Q14. 1 micro coulomb = 1 Mark
A 106C. B 10-6C. C 10C. D None.
Ans: B 10-6C.
Q15. When we wear nylon dresses during winter then there is ______ current which gets produced due to contact with out body. Fill in the Blank. 1 Mark
A Magnetic B Electrostatic C Potential D kinetic
Ans: B Electrostatic
Explanation:
When we wear nylon dresses during winter then there is electrostatic current which gets produced due to contact with out body.
Q16. In case of Coulomb’s law, the value of k is given by: 1 Mark
-9 2 2 9 2 2 9 2 2 -9 2 2
A 9 × 10 Nm /C B 9 × 10 Nm /C C 10 Nm /C D 10 Nm /C
9 2 2
Ans: B 9 × 10 Nm /C
Q17. What happens when a glass rod is rubbed with silk? 1 Mark
A Gains protons from silk. B Gains electrons from silk.
C Gives electrons to silk. D Gives protons to silk.
Ans: A Gains protons from silk.
Explanation:
Excess electrons are transferred from glass to silk when a glass rod is rubbed with silk. Hence, the glass rod becomes positive, and silk becomes negative.
Q18. The Gaussian surface: 1 Mark
A Can pass through a continuous charge distribution. B Cannot pass through a continuous charge distribution.
C Can pass through any system of discrete charges. D Can pass through a continuous charge distribution as well as any system of
discrete charges.
Ans: B Cannot pass through a continuous charge distribution.
Q19. Two uncharged bodies on rubbing, get charged due to: 1 Mark
A Conduction B Friction C Induction D None of these
Ans: B Friction
Explanation:
When two bodies are rubbed together, there is transfer of electrons because of friction.
Hence, two uncharged bodies on rubbing, gets equal and opposite charges.
Q20. The unit of electric field is not equivalent to: 1 Mark

https://bls.smartstudies.co.in/#/exam/pdf-preview/b5b87181-b79b-4b4a-8ebf-8f94fb605fc8/1 2/139
5/17/24, 8:14 PM Exam Automation

A N / C. B J / C. C V / m. D J / Cm.
Ans: B J / C.
Q21. Earth is the source of __________? 1 Mark
A An infinite positive and negative charge. B Positive charge.
C Negative charge. D Zero charge.
Ans: A An infinite positive and negative charge.
Explanation:
Earth can be considered as an infinite source of positive and negative charges.
This can be justified by the fact that if we connect any positive or negatively charged body to the ground, all of its charges will go to earth.
Q22. 1 emu = __________ C 1 Mark
9 -10
A 10 B 3 × 10 C 4.8 × 10 D 0.1
Ans: A 10
Explanation:
1 emu charge is equal to 10-coulomb charge while 1 coulomb is equal to 3 × 109 esu charge. Emu and C both are the units of charge and hence their conversion
formula is necessary for many numerical problems.
Q23. Coulomb's law is a confirmation of______ 1 Mark
A inverse cube law B product law C inverse square law D None of the above
Ans: C inverse square law
Explanation:
According to Coulomb's Law-
1
F∝
r2
Hence, Coulomb's Law is a confirmation of inverse square law.
Q24. Choose the correct answer. 1 Mark
K. ( q 1 × q 2 )
F=
r2
This is given by which law?
A Faraday’s law B Newton’s law C Coulomb’s law D Fleming’s law
Ans: C Coulomb’s law
Explanation:
K. ( q 1 × q 2 )
F=
r2
is given by Coulomb’s Law.
Q25.
Let there be a spherically symmetric charge distribution with charge density varying as p(r) = p 0 ( 5
4

r
R ) upto r = R and p(r) = 0 for r > R where r is the distance from 1 Mark

the origin the electric field at a distance r(r < R) from the origin is given by.
5 r 4πp 0 r 5
A p r
0 (3 − R ) B
3ϵ 0 (3 − R )
r

C 4p 0 r 5 r D p0r 5 r
4ϵ ( 4
− R) (
3ϵ 0 4
− R )
0

Ans: A p r 5 − r
0 (3 R)

Q26. Which of the following is false for electric lines of force ? 1 Mark
A They always start from positive charges and terminate on negative charges. B They are always perpendicular to the surface of a charged conductors.
C They always form closed loops. D They are parallel and equally spaced in a region of uniform electric field.
Ans: C They always form closed loops.
Explanation:
Electric lines of force cannot form closed loop as they originate from only positive charges but do not originate from negative charges but only terminate into them.
Q27. A capacitor has been charged by a dc source. What are the magnitudes of conduction and displacement currents, when it is fully charged? 1 Mark

Ans:Magnitude of conduction & displacement current's are zero.


Q28. ......... Charge is produced by friction 1 Mark
A Stationary. B No. C Attractive. D All.
Ans: A Stationary.
Explanation:
The stationary charge (i.e static electricity) is produced by friction.
Q29. A point charge + q is placed at a distance d from an isolated conducting plane. The field at a point P on the other side of the plane is: 1 Mark
A Directed perpendicular to the plane and away from the plane. B Directed perpendicular to the plane but towards the plane.
C Directed radially away from the point charge. D Directed radially towards the point charge.
Ans: A Directed perpendicular to the plane and away from the plane.
Q30. For two statements are given-one labelled Assertion (A) and the other labelled Reason (R). Select the correct answer to these questions from the codes (a), (b), (c) 1 Mark
and (d) as given below.
Assertion (A): Charge is quantized.
Reason (R): Charge which is less than I C is not possible.
A Both A and R are true, and R is the correct explanation of A. B Both A and R are true, but R is not the correct explanation of A.
C A is true but R is false. D A is false and R is also false.
Ans: C A is true but R is false.
Explanation:
The charge q on a body is given as q = ne where n is any integer positive or negative. The charge on the electron is q = 1.6 × 10-19C which is less than 1C.
Q31. The surface charge destiny of tin charged disc of radius R is The value of the electric filed at the centre of the disc is σ With reapect to the field the centre the 1 Mark
σ 2ϵ 0
electric field along the axis at distance R from the centre of the disc.
A Reduces by 70.7%. B Reduces by 29.3%. C Reduces by 9.7%. D Reduces by 14.6%.
Ans: A Reduces by 70.7%.
Q32. The electric field lines are far apart where electric field is: 1 Mark
A Strong. B Weak. C Moderate. D None.
Ans: B Weak.
Q33. For two statements are given-one labelled Assertion (A) and the other labelled Reason (R). Select the correct answer to these questions from the codes (a), (b), (c) 1 Mark
and (d) as given below.
Assertion (A): If a point charge q is placed in front of an infinite grounded conducting plane surface, the point charge will experience a force.

https://bls.smartstudies.co.in/#/exam/pdf-preview/b5b87181-b79b-4b4a-8ebf-8f94fb605fc8/1 3/139
5/17/24, 8:14 PM Exam Automation
Reason (R): This force is due to the induced charge on the conducting surface which is at zero potential.
A Both A and R are true, and R is the correct explanation of A. B Both A and R are true, but R is not the correct explanation of A.
C A is true but R is false. D A is false and R is also false.
Ans: A Both A and R are true, and R is the correct explanation of A.
Q34. A force ‘F’ is acting between two charges in air. If the space between them be completely filled with a medium K = 4, the force will be: 1 Mark
A F B 4F C F D 2F
4
Ans: C F
4

Explanation:
By Coulomb's law, the force between two charges (q1​, q2​) in air is given by
q 1q 2
F=
4πϵ 0r 2
where r= distance between two charges.
Now the force between two charges (q1​, q2​) in medium with dielectric
q 1q 2
constant K = 4 becomes, F =
4πKϵ 0r 2
F F
So, F = K
= 4
Q35. Gold-leaf electroscope can be used _______? 1 Mark
A Only to detect the presence of charge. B To detect the presence of charge as well as its nature (positive or negative).
C To measure the surface charge density. D To measure current.
Ans: B To detect the presence of charge as well as its nature (positive or negative).
Explanation:
If the apparatus is initially in contact with some positively charged body and then comes in contact with another charged body and if the divergence increases we
can deduce that the second body is also positively charged. If the divergence decreases then the second body must be negatively charged.
Q36. If a body is charged by rubbing it, its weight 1 Mark
A Remains precisely constant. B Increases slightly.
C Decreases slightly. D May increase slightly or may decrease slightly.
Ans: D May increase slightly or may decrease slightly.
Explanation:
If a body is charged by rubbing it, its weight may increase slightly or may decrease slightly.
Q37. The quantisation of charge indicates that: 1 Mark
A Charge, which is a fraction of charge on an electron, is not possible. B A charge cannot be destroyed.
C Charge exists on particles. D There exists a minimum permissible charge on a particle.
Ans: A Charge, which is a fraction of charge on an electron, is not possible.
Explanation:
The quantisation of charge means that when we say something has some charge, we mean by that that how many times the charge of electrons it has. Because
the whole charge is associated with an electron.
Q38. The charge is negative, then the electric lines of forces are: 1 Mark
A Straight lines converging towards the charge. B Concentric circle with charge at the centre.
C Straight lines radiating away from the charge. D Non of these.
Ans: A Straight lines converging towards the charge.
Explanation:
Lines of force originate from a positive charge and terminate to a negative charge. Hence, when the charge is negative, then the electric lines of force are straight
lines converging towards the charge.
Q39. Which among the following is the proper way of earthing? 1 Mark
A B C D

Ans: D

Explanation:
Earthing is done from the far end of the body so that a positive charge on the body (from the figure) goes to the ground. If it is done at both far and near end, all the
positive and negative charge from the ground and the body becomes electrically neutral.
Q40. An electric filament bulb can be worked from: 1 Mark
A DC supply only. B AC supply only. C Battery supply only. D All above.
Ans: D All above.
Explanation:
An electric filament bulb can be worked from
Q41. Which of the following is the best insulator? 1 Mark
A Carbon. B Paper. C Graphite. D Ebonite.
Ans: D Ebonite.
Q42. The charges 1, 2, 3 are moving in uniform transverse magnetic field then: 1 Mark

https://bls.smartstudies.co.in/#/exam/pdf-preview/b5b87181-b79b-4b4a-8ebf-8f94fb605fc8/1 4/139
5/17/24, 8:14 PM Exam Automation

A Particle 1 positive and particle 3 negative. B Particle 1 negative and particle 3 positive.
C Particle 1 negative and particle 2 neutral. D Particle 1 and 3 are positive and particle 2 neutral.
Ans: A Particle 1 positive and particle 3 negative.
Q43. When we rub our comb with hair and then take near to the paper pieces, all pieces get stuck to the comb. This happens due tyo which phenomena? 1 Mark
A Electrostatic Induction. B Magnetic effect.
C Potential effect. D Kinetic effect.
Ans: A Electrostatic Induction.
Explanation:
When negative charges is created in the comb by rubbing it with hair and pieces of paper are exposed to the electric field created by the charge, the part of paper
closer to the comb will be positively charged due to electromagnetic induction, and the farther would be negatively charged, causing the paper to be attracted to
the comb.
Q44. The electric potential decreases uniformly from 120V to 80V as one moves on the x-axis from x = -1cm to x = + 1cm. The electric field at the origin. 1 Mark
A Must be equal to 20Vcm-1 B May be equal to 20Vcm-1
C May be greater than 20Vcm-1 D May be lees than 20Vcm-1
Ans: B May be equal to 20Vcm-1 C May be greater than 20Vcm-1
Explanation:

△V = − E. dr
(V f − V i) = − E. dr = E x(B − A)
(80 − 120) = − E x. (2)
40
Ex = = 20v/m
2
If electric field lines lies in 'x ' direction than it may be equal to 20v/cm.
If Electric field lines liesin 'x - y' direction than it may be greater than 20v/cm.
Q45. Which of the following quantities do not depend on the choice of zero potential or zero potential energy? 1 Mark
A Potential at a point. B Potential difference between two points.
C Potential energy of a two-charge system. D Change in potential energy of a two-charge system.
Ans: C Potential energy of a two-charge system. D Change in potential energy of a two-charge system.
Explanation:

KQ KQ
Vp = r
− r
=0
Q46. Two small balls having the same mass and charge and located on the same vertical at heights h1 and h2 are thrown in the same direction along the horizontal at the 1 Mark
same velocity v. The first ball touches the ground at a distance l from the initial vertical. At what height H2 will the second ball be at this instant ? The air drag and
the effect of the charges induced on the ground should be neglected.
A h +h −g ℓ B h −h −g ℓ
1 2 ( V )2 1 2 ( V )2
C h +h −g ℓ 1
D h1 + h2
1 2 (V ) 2
−g

− g ( V )2

Ans: A h + h − g ℓ 2
1 2 (V )
Q47. For two statements are given-one labelled Assertion (A) and the other labelled Reason (R). Select the correct answer to these questions from the codes (a), (b), (c) 1 Mark
and (d) as given below.
Assertion (A): Range of Coulomb force is infinite.
Reason (R): Coulomb force acts between two charged particles.
A Both A and R are true, and R is the correct explanation of A. B Both A and R are true, but R is not the correct explanation of A.
C A is true but R is false. D A is false and R is also false.
Ans: B Both A and R are true, but R is not the correct explanation of A.
Explanation:

https://bls.smartstudies.co.in/#/exam/pdf-preview/b5b87181-b79b-4b4a-8ebf-8f94fb605fc8/1 5/139
5/17/24, 8:14 PM Exam Automation

Coulomb's force or electric force given by Coulomb's law is valid over nuclear distances ( ≅ 10 − 15m) as well as over very large distances.
k ∣ q1 ∥ q2 ∣
From Coulomb's law, F = . This generally holds only for charged objects whose size are much smaller than the distance between them.
r2
Q48. The charge on proton is: 1 Mark
A +e B -e C 1/e D -1/e
Ans: A +e
Q49. Electric charge is measured in: 1 Mark
A Coulombs B Amperes C Volts D Watts
Ans: A Coulombs
Explanation:
The standard unit of charge is coulomb (C). Here ampere is the unit of current, volts is the unit of electric potential difference and watts is the unit of power.
Q50. If one penetrates a uniformly charged spherical cloud, electric field strength: 1 Mark
A Decreases directly as the distance from the centre. B Increases directly as the distance from the centre.
C Remains constant. D None of these.
Ans: C Remains constant.
Q51. If the electric flux entering and leaving a closed surface in air are ϕ 1 and ϕ 2 respectively, the net electric charge enclosed within the surface is ________. 1 Mark

Ans:If the electric flux entering and leaving a closed surface in air are ϕ 1 and ϕ 2 respectively, the net electric charge enclosed within the surface is (ϕ 2 − ϕ 1)ε 0.
Explanation:
According to Gauss theorem,
Q
(ϕ 2 − ϕ 1) = ε0
⇒ Q(ϕ 2 − ϕ 1)ε 0.

The flux enters the enclosure if one has a nagative charge (-q2) and flux goes out has a +ve charge (+q1). As one does not know whether
ϕ 1 > ϕ 2, ϕ 2 > ϕ 1, Q = q 1 ∼ q 2.
Q52. In the diagram, three point charges (labeled 1, 2 and 3) are shown, along with the electric field around them. Which charge(s) is/ are positive? 1 Mark
A 2 only B 1 and 2 C 1 and 3 D 1, 2 and 3
Ans: C 1 and 3
Explanation:
The electric field always originate at the positive charge and terminate at the negative charge.
From figure, the electric field lines originate at the points 1 and 3 but ends at point 2.
Thus points 1 and 3 represent positive charge and 2 represents the negative charge.
Q53. When a charged body is brought near an electroscope: 1 Mark
A The strips of foils opens up. B The strips of foils close.
C The strips open and close simultaneously. D The strips neither open nor close.
Ans: A The strips of foils opens up.
Explanation:
When a charged body is brought near the top conducting disc of an electroscope, the disc gets induced with charges. These charges run down to the metal strips
hanging from the paper clip. Now as both the strips get similar charge, those repel each other and thus open up.
Q54. As the net electric flux through a closed surface is zero, the total charge contained in the closed surface is also: 1 Mark
A Unity. B Zero. C Positive. D Negative.
Ans: B Zero.
Q55. A Solid sphare of radius R has a charge Q distributed in its Volume with a charge density P = Kra Where K and a are constants and r is the distance from its centre 1 Mark
R 1
from its centre. if the electric filed at r = 2
is 8
times that at r = R, the value of a is.

A 3 B 5 C 2 D 7
Ans: C 2
Q56. In the figure shown here, A is a conducting sphere and B is a closed spherical surface. If a-q change is placed at C near A, then the electric flux through the closed 1 Mark
surface is -

A Zero B Positive
C Negative D None of the above can be predicted
Ans: C Negative
Explanation:
As the electric field inside the spherical conductor has to be zero the equal and opposite charges will be distributed on the conductor surface such that the
induced electric field counters the electric field due to negative charge. For negative charges will be distributed on right and positive on left side
As the spherical region is on right side q inside < 0.
Q57. A hollow sphere of charge does not have electric field at: 1 Mark
A Outer point. B Interior point. C Beyond 2m. D Beyond 100m.
Ans: B Interior point.
Q58. When we remove polyester or woollen clothes in dark, we can see a spark and hear a crackling sound. Which of the following is responsible for it? 1 Mark
A Static electricity. B Current electricity. C Reflection of light. D Refraction of light.
Ans: A Static electricity.
Q59. Which orientation of an electric dipole in a uniform electric field would correspond to stable equilibrium? 1 Mark

Ans:Parallel / ϕ = 0 / diagram only.


Q60. The electric field inside a conductor. 1 Mark
A Must be zero. B May be non-zero.
C Must be non-zero. D (a) and (c) are correct.
Ans: A Must be zero.
Q61. For two statements are given-one labelled Assertion (A) and the other labelled Reason (R). Select the correct answer to these questions from the codes (a), (b), (c) 1 Mark
and (d) as given below.
Assertion: The properties that the force with which two charges attract or repel each other are not affected by the presence of a third charge
Reason: On rubbing, electrons from Silk cloth move to the glass rod.

https://bls.smartstudies.co.in/#/exam/pdf-preview/b5b87181-b79b-4b4a-8ebf-8f94fb605fc8/1 6/139
5/17/24, 8:14 PM Exam Automation

A Both A and R are true and R is the correct explanation of A. B Both A and R are true but R is not the correct explanation of A.
C A is true but R is false. D A is false and R is also false.
Ans: D A is false and R is also false.
Q62. If an electron has an initial velocity in a direction different from that of an electric field, the path of the electron is: 1 Mark
A A straight line. B A circle. C An ellipse. D A parabola.
Ans: D A parabola.
Q63. A point positive charge is brought near an isolated conducting sphere. The electric field is best given by: 1 Mark

A Fig (i). B Fig (ii). C Fig (iii). D Fig (iv).


Ans: A Fig (i).
Key concept:
The explanation to this problem-can be done by keeping two things in mind.
Let us discuss the phenomenon of induction involved in this case. When a positive point charge is brought near an isolated conducting sphere without touching the
sphere, then the free electrons in the sphere are attracted towards the positive charge. Thus, the left surface of sphere has an excess of negative charge and the
right surface of sphere has an excess of positive charge. It should be noted that both kinds of charges are bound in the metal sphere and cannot escape. They,
therefore, reside on the surface of the sphere.

An electric field lines start from a positive point charge and ends at negative charge induced on the left surface of sphere. Also, electric field line emerges from a
positive charge, in case of single charge and ends at infinity.
Here, all these conditions are fulfilled in Fig. (i).
Q64. Superconductors have __________? 1 Mark
A Almost zero resistivity B Very high resistivity
C Temperature-dependent resistivity D Moderate value of resistivity
Ans: A Almost zero resistivity
Explanation:
Semiconductors are those which carry current with almost zero resistivity at a very low temperature (example: Lead at -272 - degree centigrade). So, they have a
very huge amount of current flow through them.
Q65. The electric current flowing in a wire in the direction from B to A is decreasing. Find out the direction of the induced current in the metallic loop kept above the wire 1 Mark
as shown.

Ans:Clockwise.
Alternate Answer

https://bls.smartstudies.co.in/#/exam/pdf-preview/b5b87181-b79b-4b4a-8ebf-8f94fb605fc8/1 7/139
5/17/24, 8:14 PM Exam Automation
Q66. There are uniform electric and magnetic fields in a region pointing along X-axis. An α − particle is projected along Y-axis with a velocity v. The shape of the 1 Mark
trajectory will be:
A Circular in XZ plane. B Circular in YZ plane.
C Helical with its axis parallel to X-axis. D Helical with its axis parallel to Y-axis.
Ans: C Helical with its axis parallel to X-axis.
Explanation:
If θ = 0 ∘ then due to magnetic force path is circular but due to force qE 0( ↑ )q will have accelerated motion along y-axis. So combined path of q will be a helical path
with variable pitch so (a) and (b) are wrong.
If θ = 10 ∘ , then due to vcosθ, path is circular and due to qE0 and vcosθ, q has accelerated motion along y-axis so combined path is a helical path with variable pitch
(C) is correct.
If θ = 90 ∘ then FB = 0 and due to qE0 motion is accelerated along y-axis (D).
Q67. The filament of an electric bulb is made of: 1 Mark
A Carbon B Aluminium C Tungsten D Nickel
Ans: C Tungsten
Explanation:
The electric bulbs convert electrical energy to heat and light energy and thus the filaments of the bulbs are expected to withstand high temperatures.
While at the same time ensure that their structure does not get affected via heating effects from the current passing through.
As a metal, Tungsten is a good electrical conductor. It is also a good thermal conductor. In pure form, tungsten has the highest known melting point among all
metals and lowest vapour pressure and greatest tensile strength out of all the metals.
This means that tungsten is able to maintain its form irrespective of the large heating effects from electrical current. Because of this, It can reach a high
temperature before melting.
Q68. A charge "Q" and "2Q" are 0.05 meters apart and isolated. The ratio of the electrostatic force on the charge Q to the force on charge 2Q is: 1 Mark
A 2:1 B 1:1 C 1:2 D 1:4
Ans: B 1 : 1
Explanation:
According to Newton's 3rd law, every action has equal and opposite reaction.
Thus the force exerted by charge on Q due to 2Q is equal but opposite in direction to the force exerted by charge on 2Q due to Q i.e
FQ​= F2Q​
FQ
⇒ F 2Q
=1

Q69. Dr. Gilbert tried to hold a brass rod by hand and induce static electricity in it by friction. Why did he fail? 1 Mark
A Static electricity is induced only on insulators while brass is conducting. B The induced static charge flew through his hand and body to the ground.
C The static charge was not produced at all. D It is not possible to induce static electricity by friction.
Ans: B The induced static charge flew through his hand and body to the ground.
Explanation:
The human body is a good conductor and hence the charge produced by friction flew through his body to the ground. We can tackle the problem simply by holding
the brass rod with the help of some insulated holder.
Q70. If two charges of 1 coulomb each are placed 1km apart in vacuum, the force between them will be: 1 Mark
A 9 × 103N B 9 × 10-3N C 1.1 × 10-4N D 10-6N
Ans: A 9 × 103N
Explanation:
Q = 1C r = 1000m
KQQ 9 × 10 3
F= 2 = = 9 × 10 3N
r ( ( 10 ) 3 ) 2
Q71. Define the term electric dipole moment. Is it a scalar or a vector quantity? 1 Mark

Ans:It is the product of charge (q) and the seperation (2a) between the pair of charges.
Alternate Answer
In terms of p = Torque/normal electric field It is a vector quantity.
Q72. Answer the following questions. 1 Mark
1. Write SI unit of electric field intensity.
2. Write SI unit of electric dipole moment.

Ans:1. SI unit of electric field intensity is newton/ coulomb (NC-1).


2. SI unit of electric dipole moment is coulomb × metre (Cm).
Q73. What happens when some charge is placed on a soap bubble? 1 Mark
A Its radius decreases. B Its radius increases. C The bubble collapses. D None of these.
Ans: B Its radius increases.
Q74. If a body is negatively charged, then it has. 1 Mark
A Excess of electrons. B Excess of protons.
C Deficiency of electrons. D Deficiency of neutrons
Ans: A Excess of electrons.
Q75. Like charges ____ each other. 1 Mark
A Attacks. B Repels. C Both a and b. D None.
Ans: B Repels.
Q76. Which among the following cannot be the charge of a charged body? 1 Mark
A 4.8 × 10-14 Coulomb B 6.4 × 10-15 Coulomb
C 5 × 10-14 Coulomb D 3.2 × 10-10 Coulomb
Ans: C 5 × 10-14 Coulomb
Explanation:
Charge of a body must be an integral multiple of charge of an electron. 5 × 10-14 is not an integral multiple of (1.602 × 10-19 Coulomb) which is equal to the charge
of an electron. But we can see that 4.8, 6.4, 3.2 are integral multiples of 1.6.
Q77. For two statements are given-one labelled Assertion (A) and the other labelled Reason (R). Select the correct answer to these questions from the codes (a), (b), (c) 1 Mark
and (d) as given below.
Assertion (A): A small metal ball is suspended in a uniform electric field with an insulated thread. If high-energy X-ray beam falls on the ball, the ball will be
deflected in the electric field.
Reason (R): X-rays emits photoelectron and metal becomes negatively charged.
A Both A and R are true, and R is the correct explanation of A. B Both A and R are true, but R is not the correct explanation of A.
C A is true but R is false. D A is false and R is also false.
Ans: C A is true but R is false.

https://bls.smartstudies.co.in/#/exam/pdf-preview/b5b87181-b79b-4b4a-8ebf-8f94fb605fc8/1 8/139
5/17/24, 8:14 PM Exam Automation
Explanation:
When high energy X-ray beam falls on the ball, the metal will emit photoelectrons, thus leaving the positive charge on the ball. As a result of this, ball is deflected in
the direction of the electric field.
Q78. Why do the electric field lines never cross each other? 1 Mark

Ans:At the point of intersection of the two field lines, there will be two directionsfor the electric field. This is not acceptable.
Q79. The process of sharing the charges with the earth is called as: 1 Mark
A Grounding. B Earthing. C Both a and b. D None.
Ans: C Both a and b.
Q80. The SI unit of linear charge density is: 1 Mark
A C/m B C C Cm D None
Ans: A C/m
Q81. An electric dipole will experience a net force when it is placed in. 1 Mark
A A uniform electric field. B A non-uniform electric field.
C Both (a) and (b). D None of these.
Ans: B A non-uniform electric field.
Q82. A lightning arrester must have the following property. 1 Mark
A Discontinuity. B Poor conductivity. C Needle end. D Low melting point.
Ans: C Needle end.
Explanation:
Needle end can arrest the lightning easily. Besides, the device must provide a continuous path to electricity so that current passes to ground. It must have a high
melting point else it will meltdown due to the heat generated during carrying lightning current.
Q83. The electric field inside a spherical shell of uniform surface charge density is: 1 Mark
A Zero. B Constant different from zero.
C Proportional to the distance from the curve. D None of the above.
Ans: A Zero.
Q84. The three basic properties possessed by the electric charge are: 1 Mark
A Quantisation. B Additivity. C Conservation. D All.
Ans: D All.
Q85. The total charge of the electric dipole is: 1 Mark
A Negative. B Positive. C Infinity. D Zero.
Ans: D Zero.
Q86. Which of the following are insulators. 1 Mark
A Glass. B Plastic. C Nylon. D All.
Ans: D All.
Q87. The property between two charged ebonite rods is: 1 Mark

A Attraction B Repulsion C Both D None


Ans: B Repulsion
Explanation:
If two ebonite rods A and B are separately rubbed with fur and then they are brought near each other, they repel each other.
Q88. The black shapes in the figure are closed surfaces. The electric field lines are in red. For which case the net flux through the surfaces is non-zero? 1 Mark

A In all cases net flux is non-zero B Only (c) and (d)


C Only (a) and (b) D Only (b), (c) and (d)
Ans:
https://bls.smartstudies.co.in/#/exam/pdf-preview/b5b87181-b79b-4b4a-8ebf-8f94fb605fc8/1 9/139
5/17/24, 8:14 PM Exam Automation
C Only (a) and (b)
Explanation:
As charge is enclosed in a, b, so by Gauss's Law there will be a flux enclosed by the given surface.
Q89. The dimensions of electric field are: 1 Mark
A [M L T-3 A-1]. B [M L T-2 A-1]. C [M L T-3 A-2]. D None.
Ans: A [M L T-3 A-1].
Q90. An electron revolves around the nucleus of hydrogen atom in a circle of radius 5 × 10-11 m. The intensity of electric field at a point in the orbit of the electron is: 1 Mark
11 -8
A 5.76 × 10 N/C B 9.216 × 10 N/C C 0 D 4 N/C
11
Ans: A 5.76 × 10 N/C
Explanation:
We know hydrogen atom consist of one proton and one electron
R = 5 × 10-11m
kq 1 ( q 2 ) 9 × 10 9 × ( 1.6 × 10 − 19 ) 2
We know attractive force = 2
=
r ( 5 × 10 − 11 ) 2
We know F = Eq
F
∴E= q
\therefore\text{E}=\frac{9\times10^9\times(1.6\times10^{-19})}{(5\times10^{-11})^2}=5.76\times10^{11}\text{N/C.}​​
Q91. The SI unit of dipole moment is: 1 Mark
A C. B C/m. C Cm. D None.
Ans: C Cm.
Q92. Why do the electrostatic field lines not form closed loops? 1 Mark

Ans:They start from positive charges and end on negative charges.


Alternate Answer
Electric field is conservative in nature.
Q93. An inflated balloon was pressed against a wall after it has been rubbed with a piece of synthetic cloth. It was found that the balloon sticks to the wall. What force 1 Mark
might be responsible for the attraction between the balloon and the wall?
A Gravitational B Magnetic C Electrostatic D Adhesive
Ans: C Electrostatic

Explanation:
Electrostatic force is the phenomenon that results from slow-moving or stationary electrical charges. Specifically, electrostatic force is the physical reaction that
holds together the electromagnetic field created by subatomic particles, such as electrons and protons. In order for electrostatic forces to remain cohesive, these
particles need to independently maintain both positive and negative charges and react to each other accordingly.
When a balloon is rubbed with a cloth, the electrons from the atoms of the cloth enter the atoms of the balloon and the number of electrons in the atoms of balloon
becomes more than the number of protons. Because of that, there is a negative charge in the balloon, and in the cloth the number of electrons in its atoms become
less than the number of protons, so it's positively charged.
Therefore, due to this electrostatic force, the balloon sticks to the wall.
Q94. The net electric flux through a closed surface is: 1 Mark
A Unity. B Negative. C Positive. D Zero.
Ans: D Zero.
Q95. The property which differentiate the two kinds of charges is called as: 1 Mark
A Magnitude of charge. B Direction of charge. C Polarity of charge. D None.
Ans: C Polarity of charge.
Q96. +q, +2q, +3q, +4q, ……(up to +20q) charges are situated at coordinates (0, 0), (1, 0), (2, 0), ….. (Up to 20). What is the total charge stored in the system? 1 Mark
A +20q B +210q C +420q D +190q
Ans: B +210q
Explanation:
As charge is additive, total charge will be (1+2+3+4+…. +20)\times\text{q}=\frac{20\times21}{2}\times\text{q}= 210\text{q}. But if the polarities of the charges are
different i.e. some of them are positive and some are negative, then the result will be different. We have to add separately the positive charges and the negative
charges.
Q97. Unlike charges _____ each other. 1 Mark
A Attacks. B Repels. C Both a and b. D None.
Ans: A Attacks.
Q98. If two bodies A and B (A bigger in size than B) are rubbed together, then: 1 Mark
A A and B get equal and opposite charges. B A and B get equal and similar charges.
C A gets more charge than B, but of opposite kind. D A gets less charge than B, but of same kind.
Ans: A A and B get equal and opposite charges.
Explanation:
Static electricity is created when two objects are rubbed together, causing an object to give up or gain electrons. The electric charges that are created do not
depend on the size of the objects. Therefore, when two bodies are rubbed together, both the bodies transfer electrons to and fro between the bodies and
they obtain equal and opposite charges. The body that gives away electrons, gains positive charge and the body that obtains electrons gains negative charge.
Q99. Define dielectric constant of a medium. What is its S.I. unit? 1 Mark

Ans:Dielectric Constant of a medium is the ratio of intensity of electric field in free space to that in the dielectric medium.
Alternate Answer
It is the ratio of capacitance of a capacitor with dielectric medium to that without dielectric medium.
Q100. A positive point charge Q is brought near an isolated metal cube. 1 Mark
A The cube becomes negatively charged. B The cube becomes positively charged.
C The interior becomes positively charged and the surface becomes D The interior remains charge free and the surface gets nonuniform charge
negatively charged. distribution.
Ans: D The interior remains charge free and the surface gets nonuniform charge distribution.
Explanation:
A positive point charge Q is brought near an 'Isolated metal cube'. So charge distributed on the surface of isolated metal is non uniformly and the interior remains
charge free.
Q101. For two statements are given-one labelled Assertion (A) and the other labelled Reason (R). Select the correct answer to these questions from the codes (a), (b), (c) 1 Mark
and (d) as given below.
Assertion (A): No two electric lines of force can intersect each other.
Reason (R): Tangent at any point of electric line of force gives the direction of electric field.

https://bls.smartstudies.co.in/#/exam/pdf-preview/b5b87181-b79b-4b4a-8ebf-8f94fb605fc8/1 10/139
5/17/24, 8:14 PM Exam Automation

A Both A and R are true, and R is the correct explanation of A. B Both A and R are true, but R is not the correct explanation of A.
C A is true but R is false. D A is false and R is also false.
Ans: A Both A and R are true, and R is the correct explanation of A.
Explanation:
If the two electric tines of force can intersect each other then at the point of intersection, we can draw two tangents to the two lines of force. This would mean two
directions of electric field intensity at the point of intersection, which is not possible.
Q102. The plates of a parallel-plate capacitor are given equal positive charges. What will be the potential difference between the plates? What will be the charges on the 1 Mark
facing surfaces and on the outer surfaces?

Ans:It is given that the plates of the capacitor have the same charges. In other words, they are at the same potential, so the potential difference between them is zero.
(Figure)
Let us consider that the charge on face II is q so that the induced charge on face III is -q and the distribution is according to the figure.
Now, if we consider Gaussian surface ABCD, whose faces lie inside the two plates, and calculate the field at point P due to all four surfaces, it will be
\text{E}_1=\frac{\text{Q}-\text{q}}{2\epsilon_0\text{A}}
\text{E}_2=\frac{\text{q}}{2\epsilon_0\text{A}}
\text{E}_3=-\frac{\text{q}}{2\epsilon_0\text{A}}
\text{E}_4=-\frac{\text{Q}+\text{q}}{2\epsilon_0\text{A}} (It is -ve because point P is on the left side of face IV.)
Now, as point P lies inside the conductor, the total field must be zero.
\therefore\text{E}_1+\text{E}_2+\text{E}_3+\text{E}_4=0
\text{Q}-\text{q}+\text{q}-\text{q}+\text{Q}+\text{q}=0
\therefore\text{q}=0
Hence, on faces II and III, the charge is equal to zero; and on faces I and IV, the charge is Q.
Thus, it seems that the whole charge given is moved to the outer surfaces, with zero charge on the facing surfaces.
Q103. What do you understand by the static or frictional electricity? 1 Mark
A Discarging of two bodies on rubbing them together is called static or B Charging of two bodies on rubbing them together is called static or frictional
frictional electricity. electricity.
C Electricity which is static is called static or frictional electricity. D Electricity on high friction bodies is called static or frictional electricity.
Ans: B Charging of two bodies on rubbing them together is called static or frictional electricity.
Explanation:
Static electricity is produced when a suitable combination of substances (e.g., glass rod and silk, ebonite rod and cat's skin, sealing wax and wool) are rubbed
together and get electrified due to friction.
Q104. Electric switches and appliances should be tested only with: 1 Mark
A Right hand B Left hand C Both hands D Electric tester
Ans: D Electric tester
Explanation:
We should not touch the electric appliance with bare hands. To check the supply in the circuit, the electric tester should be used.
Q105. Figure shows three point charges, + 2q, – q and + 3q. Two charges + 2q and – q are enclosed within a surface ‘S’. What is the electric flux due to this configuration 1 Mark
through the surface ‘S’?

Ans:Electric flux \Phi = \frac{\text{q}}{\varepsilon_{0}}.


Q106. For two statements are given-one labelled Assertion (A) and the other labelled Reason (R). Select the correct answer to these questions from the codes (a), (b), (c) 1 Mark
and (d) as given below.
Assertion (A): The electric flux emanating out and entering a closed surface are 8 × 103 and 2 × 103Vm respectively. The charge enclosed by the surface
is 0.053\mu\text{C}.
Reason (R): Gauss's theorem in electrostatics may be applied to verify.
A Both A and R are true, and R is the correct explanation of A. B Both A and R are true, but R is not the correct explanation of A.
C A is true but R is false. D A is false and R is also false.
Ans: A Both A and R are true, and R is the correct explanation of A.
Explanation:
According to Gauss's theorem in electrostatics,\phi=\frac{\text{q}}{\epsilon_0}
\phi=\frac{\text{q}}{\epsilon_0}=8.85\times10^{-12}[8\times10^3-2\times10^3]
=53.10\times10^{-9}\text{C}=0.053\mu\text{C}.
Q107. Identify the wrong statement in the following. Coulomb's law correctly describes the electric force that: 1 Mark
A Binds the electrons of an atom to its nucleus. B Binds the protons and neutrons in the nucleus of an atom.
C Binds atoms together to form molecules. D Binds atoms and molecules together to form solids.
Ans: B Binds the protons and neutrons in the nucleus of an atom.
Explanation:
Nuclear force binds the protons and neutrons in the nucleus of an atom.
Q108. Five charges q1, q2, q3, q4, and q5 are fixed at their positions as shown in. S is a Gaussian surface. The Gauss’s law is given by 1 Mark
\oint\limits_\text{S}\text{E.ds}=\frac{\text{q}}{\epsilon_0}
Which of the following statements is correct?

.
A E on the LHS of the above equation will have a contribution from q1, q5 and B E on the LHS of the above equation will have a contribution from all charges
q3 while q on the RHS will have a contribution from q2 and q4 only. while q on the RHS will have a contribution from q2 and q4 only.
C E on the LHS of the above equation will have a contribution from all charges D Both E on the LHS and q on the RHS will have contributions from q2 and q4
while q on the RHS will have a contribution from q1, q3 and q5 only. only

https://bls.smartstudies.co.in/#/exam/pdf-preview/b5b87181-b79b-4b4a-8ebf-8f94fb605fc8/1 11/139
5/17/24, 8:14 PM Exam Automation
Ans: B E on the LHS of the above equation will have a contribution from all charges while q on the RHS will have a contribution from q2 and q4 only.
Key concept: According to Gauss' law, the term qenclosed on the right side of the equation \oint\limits\text{E.ds} = qenclosed/ϵ0 includes the sum of all charges
enclosed by the surface called (Gaussian surface).
In left side equation, the electric field is due to all the charges present both inside as well as outside the Gaussian surface.
Hence in given question, E on LHS of the above equation will have a contribution from all charges while q on the RHS will have a contribution from q2 and q4 only.
Hence option (b) is correct.
Q109. The tangent at any point of field line gives the direction of: 1 Mark
A Electric field at that point. B Electric force on positive charge at that point.
C Both (1) & (2). D Rotation of charge.
Ans: A Electric field at that point.
Explanation:
A target is present at any point of field line.
The target at any point of the field lines generaly gives the elective field at that point.
Q110. A metallic particle having no net charge is placed near a finite metal plate carrying a positive charge. The electric force on the particle will be: 1 Mark
A Towards the plate. B Away from the plate. C Parallel to the plate. D Zero.
Ans: A Towards the plate.
Explanation:
Plase: The electric force on the particle will be owards the plate. Because the distance bewteen attraction force is Less than the distance between repulsive force.
So Attractive Force > Repulsive force.

Q111. Electric field lines cannot forms the: 1 Mark


A Open loops. B Closed loops. C Both a and b. D None.
Ans: B Closed loops.
Q112. The potential difference between two points in vacuum is V0. If vacuum is replaced by a medium of dielectric constant K, the new value of potential difference will 1 Mark
be __________.

Ans:The potential difference between two points in vacuum is V0. If vacuum is replaced by a medium of dielectric constant K, the new value of potential difference will
be \frac{\text{V}_0}{\text{K}}.
Explanation:
As we know the relation between electric field and voltage difference between two points as.
\Delta\text{V}=\text{E.d}
So as the medium is changed by other dielectric medium then we have
\text{E}_\text{med}=\frac{\text{E}}{\text{k}}
So we have,
\Delta\text{V}'=\text{E}_\text{med}\text{d}
Now divide above two equations,
\frac{\Delta\text{V}}{\Delta\text{V}'}=\frac{\text{E}}{\text{E}_\text{med}}
Now we have
\frac{\Delta\text{V}}{\Delta\text{V}'}=\text{k}
So we have,
\Delta\text{V}'=\frac{\Delta\text{V}}{\text{k}}
\Delta\text{V}'=\frac{\text{V}_0}{\text{k}}
Q113. The amount of force exerted on a unit positive charge in an electric field is known as _____? 1 Mark
A Electric field intensity. B Electric flux.
C Electric potential. D Electric lines of force.
Ans: A Electric field intensity.

https://bls.smartstudies.co.in/#/exam/pdf-preview/b5b87181-b79b-4b4a-8ebf-8f94fb605fc8/1 12/139
5/17/24, 8:14 PM Exam Automation
Explanation:
The zone near a charge where its attraction or repulsion force works, is known as the electric field of that charge. Theoretically, it is up to infinite but practically it
has limitations. If a unit positive charge is kept in that field, it will undergo some force which is known as electric field intensity at that point.
Q114. The SI unit of an electric charge is: 1 Mark
A Coulomb. B C. C Both a and b. D A.
Ans: C Both a and b.
Q115. For two statements are given-one labelled Assertion (A) and the other labelled Reason (R). Select the correct answer to these questions from the codes (a), (b), (c) 1 Mark
and (d) as given below.
Assertion: The properties that the force with which two charges attract or repel each other are not affected by the presence of a third charge
Reason: Forces on any charge due to the number of other charges is the vector sum of all the forces on that charge due to other charges, taken one at a time.
A Both A and R are true and R is the correct explanation of A. B Both A and R are true but R is not the correct explanation of A.
C A is true but R is false. D A is false and R is also false.
Ans: B Both A and R are true but R is not the correct explanation of A.
Q116. Which is the best conductor of electricity? 1 Mark
A Iron B Silver C Copper D Carbon
Ans: B Silver
Explanation:
Silver is the best conductor of electricity because it contains maximum number of free electrons.
Also the resistance of silver is very low.
Q117. Coulomb's Law agrees with________? 1 Mark
A Newtons 3rd Law of Motion. B Newtons 1st Law of Motion.
C Newtons 2nd Law of Motion. D All of the above.
Ans: A Newtons 3rd Law of Motion.
Explanation:
Coulomb's law agrees with Newton's 1st, 2nd and 3rd law of motion. Coulomb's law states that \text{F}=\text{K}\frac{\text{q}_1\text{q}_2}{\text{r}^2} here the two
charge particle is similar to the objects of the Newton's 1st law. Newton's 3rd law is same as the two similar charge particles repel to each other. Which coincides
with Newton's 3rd law every action there is an equal and opposite reaction.
Q118. Which one of the following is the correct diagram of charge distribution in a hollow sphere? 1 Mark
A B C D

Ans: A

Explanation:
The same charges repel each other. So, they try to maintain maximum distance from each other and hence they try to remain at the outer surface of an object. If
the charge remains on the inner surface or throughout the body, they will repel each other with greater force.
Q119. Torque acting on an electric dipole placed in an electric field is maximum when the angle between the electric field and the dipole moment is _______. 1 Mark

Ans:Torque acting on an electric dipole placed in an electric field is maximum when the angle between the electric field and the dipole moment is 90°.
Explanation:
The torque acting on an electric dipole.
\tau=\vec{\text{p}}\times\vec{\text{E}}
=\text{pE}\sin\theta
Torque will maximum when value of \sin\theta will be maximum or \theta=90^\circ.
Q120. The law, governing the force between electric charges is known as. 1 Mark
A Ampere's law. B Ohm's law. C Faraday's law. D Coulomb's law.
Ans: D Coulomb's law.
Q121. A point charge q is placed in a cavity in a metal block. If a charge Q is brought outside the metal, will the charge q feel an electric force? 1 Mark

Ans:Yes, the charge Q will feel an electric force, as the charge q given to the metal block appears on the surface. Hence, it exerts an electric force on the charge Q.
Q122. One brass plate is inserted between two charges. The force between two charges will: 1 Mark
A Remain the same. B Increase. C Decrease. D Fluctuate.
Ans: B Increase.
Q123. The dimensions of dipole moment are: 1 Mark
A [L T A]. B [L T-1 A-1]. C [L T A-1]. D None.
Ans: A [L T A].
Q124. A girl brings a positively charged rod near a thin neutral stream of water flowing from a tap. She observes, that the water stream, bends towards her. Instead, if she 1 Mark
is to bring a negativity charged rod near to the stream, it will:
A Bend in the same direction. B Bend in the opposite direction.
C Not bend at all. D Bend in the opposite direction above and below the rod.
Ans: A Bend in the same direction.
Explanation:
Water is dipolar in nature. Hence when water enters an electric field, the molecules rearrange themselves.
The case 1 in the figure, where a positively charged rod is brought near the water, the negative charge moves towards the rod and positive charge moves away
from the rod.
Hence, the attractive force is greater than the repulsive force and as a result, the water stream bends towards the rod.

https://bls.smartstudies.co.in/#/exam/pdf-preview/b5b87181-b79b-4b4a-8ebf-8f94fb605fc8/1 13/139
5/17/24, 8:14 PM Exam Automation
To the contrast, in case 2, when a negatively charged rod is brought near the water, the molecules rearrange such that positive charge moves towards the rod and
the negative charge moves away from it as shown in the figure.
In this case also, the attractive forces dominate the repulsive forces, and hence, the stream still bends towards the rod.

Q125. A rubber balloon is given a charge Q distributed uniformly over its surface. Is the field inside the balloon zero everywhere if the balloon does not have a spherical 1 Mark
surface?

Ans:No, the field is not zero everywhere, as the electric field vector due to the charge distribution does not cancel out at any place inside the balloon because of its
non-spherical shape.
Q126. According to Coulomb’s law, the coulomb force measured between two point charges is inversely proportional to: 1 Mark
A Product of magnitude of charges. B Square of the distance between the two charges.
C Both a and b. D None.
Ans: B Square of the distance between the two charges.
Q127. For two statements are given-one labelled Assertion (A) and the other labelled Reason (R). Select the correct answer to these questions from the codes (a), (b), (c) 1 Mark
and (d) as given below.
Assertion (A): Three equal charges are situated on a circle of radius r such that they form on equilateral triangle, then the electric field intensity at the centre is
zero.
Reason (R): The force on unit positive charge at the centre, due to the three equal charges are represented by the three sides of a triangle taken in the same order.
Therefore, electric field intensity at centre is zero.
A Both A and R are true, and R is the correct explanation of A. B Both A and R are true, but R is not the correct explanation of A.
C A is true but R is false. D A is false and R is also false.
Ans: A Both A and R are true, and R is the correct explanation of A.
Explanation:
Resultant of electric intensity at O due to B and C is equal and opposite to that due to A.


Q128. Which is not an electrostatic phenomenon: 1 Mark
A Comb rubbed with hair attracts paper bits. B A blanket in winters in dark when scrambled gives electrical sparks.
C Two people shaking hands feel a small spark in winters. D Heat is generated in a resistor carrying current.
Ans: D Heat is generated in a resistor carrying current.
Explanation:

(d) is based on Joule's Law.


Rest are electrostatic phenomenon.
Q129. The device, which can be used to detect whether an object is charged, is: 1 Mark
A Telescope B Microscope C Electroscope D None of the above
Ans: C Electroscope
Explanation:
Electroscope is a device that is used for detection and identification of charge on objects.
Q130. Dipole moment depends on _______? 1 Mark
A Charge only. B Charge and length of a dipole.
C Charge, length of a dipole and dielectric constant of the medium. D Charge and dielectric constant of the medium.
Ans: B Charge and length of a dipole.

https://bls.smartstudies.co.in/#/exam/pdf-preview/b5b87181-b79b-4b4a-8ebf-8f94fb605fc8/1 14/139
5/17/24, 8:14 PM Exam Automation
Explanation:
Dipole is defined as two equal but opposite charges, kept at a small distance and having a dipole moment. The dipole moment is simply the product of the electric
charge and length of the dipole. The dielectric constant of the medium doesn’t affect the dipole moment i.e. dipole moment of an electric dipole will be the same in
water as well as in the air.
Q131. The dimensions of volume charge density is: 1 Mark
A [L T A]. B [L-1 T A]. C [L-2 T A]. D [L-3 T A].
Ans: D [L-3 T A].
Q132. A charge Q is uniformly distributed over the surface of a spherical shell of radius R. The work done in bringing a test charge Q0 from its centre to its surface is: 1 Mark
A \frac{1}{4\pi\varepsilon_0}\frac{\text{QQ}_0}{\text{R}} B \frac{1}{4\pi\varepsilon_0}\frac{\text{QQ}_0}{2\text{R}}
C \frac{\text{QQ}_0}{\varepsilon_0\text{R}} D \text{Zero}
Ans: D \text{Zero}
Explanation:
Work done will be 0.
Q133. Two like charges are placed a certain distance apart in air. If a brass plate is introduced between them, the force between the two charges will: 1 Mark
A Increase. B Decrease. C Remain the same. D Become zero.
Ans: C Remain the same.
Explanation:
They are like charges so they will repel each other.
If a metal plate is in between them, then its electrons will get pushed away if the two charges are negative and pulled if positive.
In both the cases, the brass plate's electrons undergo forces in opposite directions.
Q134. When some charge is transferred to ...A... it readily gets distributed over the entire surface of ... A... If some charge is put on ... B..., it stays at the same place. 1 Mark
Here, A and B refer to.
A Insulator, conductor. B Conductor, insulator. C Insulator, insulator. D Conductor, conductor.
Ans: B Conductor, insulator.
Q135. The nature of charge on the object can be detected by: 1 Mark
A Microscope B Electroscope C Electrometer D Telescope
Ans: B Electroscope
Explanation:
A pre-charged electroscope detects the nature of charge on a charged body.

Q136. What is the angle between the directions of electric field at any (i) Axial point and (ii) Equatorial point due to an electric dipole? 1 Mark

Ans:The directions of electric field \vec{\text{E}}_1 at axial point P1 and electric field \vec{\text{E}}_2 at equatorial point P2 are shown in fig. Obviously, angle
between \vec{\text{E}}_1 and \vec{\text{E}}_2 is 180^\circ.
Q137. What is the geometrical shape of equipotential surfaces due to a single isolated charge? 1 Mark

Ans:Spherical.
Q138. What happens when some charge is placed on a soap bubble? 1 Mark
A Its radius decreases B Its radius increases C The bubble collapses D None of these
Ans: B Its radius increases
Explanation:
The radius of soap bublle increases because of outward force acting on the bubble due to charging.
Q139. Two point charges 2 C and -6 C attract each other with a force of 12 N. A negative charge of 2 C is added to each of the above charges, now the force between 1 Mark
them is:
A 32 N B 8N C zero D 4N
Ans: C zero
Explanation:
When -2C is added to 2C and -6C, they become 0C and -8C respectively. Since one of the charges is now zero, the force between them is zero.
Q140. For a single charge the field lines start or ends at: 1 Mark
A Positive charge. B Negative charge. C Infinity. D None.
Ans: C Infinity.
Q141. Quantisation of charge was experimentally demonstrated by: 1 Mark
A Einstein’s photoelectric effect. B Frank-Hertz experiment.
C Davisson and Germer experiment. D Millikan’s oil drop experiment.
Ans: D Millikan’s oil drop experiment.
Q142. An electric dipole is kept in a non-uniform electric field. It experiences. 1 Mark
A A force and a torque. B A force but not a torque.
C A torque but no force. D Neither a force nor a torque.
Ans: A A force and a torque.
Q143. Two charged particles having charge 2.0 × 10-8C each are joined by an insulating string of length 1m and the system is kept on a smooth horizontal table. Find the 1 Mark
tension in the string.
https://bls.smartstudies.co.in/#/exam/pdf-preview/b5b87181-b79b-4b4a-8ebf-8f94fb605fc8/1 15/139
5/17/24, 8:14 PM Exam Automation

Ans:

Force exerted =\frac{\text{kq}_1^2}{\text{r}^2}


=\frac{9\times10^9\times2\times2\times10^{-16}}{1^2}
=3.6\times10^{-6} Is the force exerted on the string.
Q144. The safety fuse should have: 1 Mark
A high resistance and high melting point. B high resistance and low melting point.
C low resistance and high melting point. D low resistance and low melting point.
Ans: B high resistance and low melting point.
Explanation:
The safety fuse should have high resistance and low melting point.
Q145. The electric field at a point on equatorial line of a dipole and direction of the dipole moment: 1 Mark
A Will be parallel. B Will be in opposite direction.
C Will be perpendicular. D Are not relate.
Ans: B Will be in opposite direction.
Q146. A problem of practical interest is to make a beam of electrons turn at 90° corner. This can be done with the parallel plates shown in figure. An electron with kinetic 1 Mark
energy 8.0 × 10-17J enters through a small hole in the bottom plate. The strength of electric field that is needed if the electron is to emerge from an exit
hole 1.0cm away from the entrance hole, traveling at right angles to its original direction is _______ ×105N/ C.

A 3 B 1 C 6 D 2.5
Ans: A 3
Q147. The field lines of a single positive charge are: 1 Mark
A Radially outward. B Radially inward. C Both a and b. D None.
Ans: A Radially outward.
Q148. An electric dipole consisting of charges +q and -q separated by a distance r, is kept symmetrically at the centre of an imaginary sphere of radius R (> r). Another 1 Mark
point charge Q is also kept at the centre of the sphere. The net electric flux coming out of the sphere will be:
A \frac{-(2\text{q}+\text{Q})}{4\pi\varepsilon_0} B \frac{\text{Q}}{\varepsilon_0}
C \frac{2\text{q}+\text{Q}}{\varepsilon_0} D \frac{-\text{Q}}{\varepsilon_0}
Ans: B \frac{\text{Q}}{\varepsilon_0}
Explanation:
As we know that net flux passing through the closed surface is the ratio of total enclosed charge in the surface and permittivity of the medium.
So it is given as,
\phi=\frac{\text{q}_\text{net}}{\varepsilon_0}
Here we know that net charge of the surface is given as,
\text{q}_\text{net}=\text{q}+(-\text{q})+\text{Q}
\text{q}_\text{net}=\text{Q}
So the flux through the surface is given as,
\phi=\frac{\text{Q}}{\varepsilon_0}
Q149. If a glass rod is rubbed on silk cloth and then brought near the bits of paper it...... then. 1 Mark
A Attracts B Repel C Both D None
Ans: A Attracts
Explanation:
If a ordinary glass rod is rubbed on the silk cloth is brought near bits of paper, it is found that they have acquired the attractive property. The glass rod get charged
on rubbing.
Q150. The direction of electric field created by a negative charge is ___________? 1 Mark
A Directed outwards. B Directed towards the charge.
C Maybe outwards or towards the charge. D Circular in shape.
Ans: B Directed towards the charge.
Explanation:
If a unit positive charge is kept near a negative charge, the unit positive charge will be attracted towards the negative charge. That means the electric field is
towards the negative charge. But in case of positive charge, the field is directed away from the charge.
Q151. The concept of solid angle is a natural extension of a angle to dimensions. 1 Mark
A plane, two B line, three C plane, three D line, two
Ans: C plane, three
Explanation:
The concept of solid angle is a natural extension of two-dimensional plane angle to three-dimensional. It is measured in terms of the unit called steradian,
abbreviated as sr.
Q152. The term on the RHS of the Gauss’s law shows the 1 Mark
A Charges enclosed by the surface. B Sum of charges enclosed by the surface.
C Both a and b. D None.
Ans: B Sum of charges enclosed by the surface.
Q153. Gauss's law is valid for: 1 Mark
A Any closed surface. B Only regular close surfaces.
C Any open surface. D Only irregular open surfaces.

https://bls.smartstudies.co.in/#/exam/pdf-preview/b5b87181-b79b-4b4a-8ebf-8f94fb605fc8/1 16/139
5/17/24, 8:14 PM Exam Automation
Ans: A Any closed surface.
Q154. If the sizes of charged bodies are very small compared to the distances between them, we treat them as ____________? 1 Mark
A Zero charges. B Point charges. C Single charge. D No charges.
Ans: B Point charges.
Explanation:
If the sizes of charged bodies are very small compared to the distances between them, we treat them as point charges.
Q155. The branch of physics which deals with the study of forces, Fields and potential arising from static charges is called as: 1 Mark
A Electromagnetics. B Thermodynamics. C Electrostatics. D None.
Ans: C Electrostatics.
Q156. Two lithium nuclei in lithium vapour at room temperature do not combine to form a carbon nucleus because: 1 Mark
A Carbon nucleus is an unstable particle. B It is not energetically favourable particle.
C Nuclei do not come very close due to Coulombic repulsion. D Lithium nucleus is more tightly bound than a carbon nucleus.
Ans: C Nuclei do not come very close due to Coulombic repulsion.
Explanation:
Due to Coulombic repulsion (i.e. nuclei are having same charge) these do not combine at room temperature.
Q157. The force of attraction or repulsion between two charges is given by __________. 1 Mark
A Gauss's law B Kepler's law C Coulomb's law D Gravitational law
Ans: C Coulomb's law
Explanation:
Coulomb's law: The force exerted by one point charge is proportional to the product of the charge and varies inversely as the sure of the distance separating the
charges.
Q158. Sometimes you get a mild spark when you touch the metal surface of a car. What might be the reason? 1 Mark
A Due to magnetic attraction, a spark is created. B Due to electrostatic charge residing on the metal as well as on skin, mild
spark is observed.
C Car tries to become neutral by donating electrons. D Its the only path to ground.
Ans: B Due to electrostatic charge residing on the metal as well as on skin, mild spark is observed.
Explanation:
Negative electric charge often resides on the surface of metal of car. When you touch the car, electrostatic discharge occurs via your body and you get a mild
spark.
Q159. If a linear isotropic dielectric is placed in an electric field of strength E, then the polarization P is: 1 Mark
A Independent of E. B Inversely proportional to E.
C Directly proportional to E. D Directly proportional to E.
Ans: D Directly proportional to E.
Q160. 6 A particle of charge q and mass m moves rectilinearly under the action of electric field E = A – Bx, where A and B are positive constants and x is distance from 1 Mark
the point where particle was initially at rest then the distance travelled by the particle before coming to rest and acceleration of particle at that moment are
respectively:
A \frac{2\text{A}}{\text{B}},0 B 0,-\frac{\text{qA}}{\text{m}}
C \frac{2\text{A}}{\text{B}},-\frac{\text{qA}}{\text{m}} D \frac{-2\text{A}}{\text{B}},-\frac{\text{qA}}{\text{m}}
Ans: C \frac{2\text{A}}{\text{B}},-\frac{\text{qA}}{\text{m}}
Q161. Which of the following materials has a negative temperature coefficient of resistance? 1 Mark
A Copper B Aluminium C Carbon D Brass
Ans: C Carbon
Explanation:
Carbon has a negative temperature coefficient of resistance.
Q162. A charge 'q' is placed at the centre of a cube of side l.What is the electric flux passing through each face of the cube? 1 Mark

Ans:\phi_{e} = \text{q}60_{0}.
Q163. What is the electric flux through a cube of side 1 cm which encloses an electric dipole? 1 Mark

Ans:Zero.
Q164. Positive electric flux indicates that electric lines of force are directed: 1 Mark
A Outwards. B Inwards. C Either (a) or (b). D None of these.
Ans: A Outwards.
Q165. The force that a unit positive charge would experiences if placed at that point is called as _____ due to charge at a point in space. 1 Mark
A Electric charge. B Electric field. C Both a and b. D None.
Ans: B Electric field.
Q166. Debye is the unit of: 1 Mark
A Electric flux. B Electric dipole moment.
C Electric potential. D Electric field intensity.
Ans: B Electric dipole moment.
Q167. An electric dipole is placed at an angle of 30° to a non-uniform electric field. The dipole will experience: 1 Mark
A A translational force only in the direction of the field. B A translational force only in the direction normal to the direction of the field.
C A torque as well as a translational force. D A torque only.
Ans: C A torque as well as a translational force.
Q168. What is the dielectric constant of a metal? 1 Mark
A -1 B 0 C 1 D Infinite
Ans: D Infinite
Explanation:
The dielectric constant of metals is infinite. The dielectric constant of metal is infinite, as the net electric field inside the metal is zero.
Q169. What happens when the end of the paper clip of electroscope is touched by hands? 1 Mark
A Electric current is produced. B Foil strips remain as it is.
C Foil strips collapse. D None of the above.
Ans: C Foil strips collapse.
Explanation:
When the paper clip of electroscope is touched by hands, the foil strips collapse because the charges on foil strips gets discharged through our body.

https://bls.smartstudies.co.in/#/exam/pdf-preview/b5b87181-b79b-4b4a-8ebf-8f94fb605fc8/1 17/139
5/17/24, 8:14 PM Exam Automation

Q170. The term on the RHS of the Gauss’s law shows the 1 Mark
A Charges enclosed by the surface. B Sum of charges enclosed by the surface.
C Both a and b. D None.
Ans: B Sum of charges enclosed by the surface.
Q171. Two spherical bobs, one metallic and the other of glass, of the same size are allowed to fall freely from the same height above the ground. Which of the two would 1 Mark
reach earlier and why?

Ans:Glass. In glass there is no effect of electromagnetic induction, due to presence of Earth’s magnetic field, unlike in the case of metallic ball.
Q172. For two statements are given-one labelled Assertion (A) and the other labelled Reason (R). Select the correct answer to these questions from the codes (a), (b), (c) 1 Mark
and (d) as given below.
Assertion (A): A point charge is lying at the centre of a cube of each side. The electric flux emanating from each surface of the cube is \frac{1^\text{th}}{6} of total
flux.
Reason (R): According to Gauss theorem, total electric flux through a closed surface enclosing a charge is equal to \frac{1}{\epsilon_0} times the magnitude of the
charge enclosed.
A Both A and R are true, and R is the correct explanation of A. B Both A and R are true, but R is not the correct explanation of A.
C A is true but R is false. D A is false and R is also false.
Ans: B Both A and R are true, but R is not the correct explanation of A.
Explanation:
The electric flux through the cube, \phi=\frac{\text{q}}{\epsilon_0}. A cube has six faces of equal area. Therefore, electric flux through each face =\frac{1}
{6}\phi=\frac{1}{6}(\frac{\text{q}}{\epsilon_0}).
Q173. A charged particle is free to move in an electric field. It will travel: 1 Mark
A Always along a line of force. B Along a line of force, if its initial velocity is zero.
C Along a line of force, if it has some initial velocity in the Direction of an acute D None of the above.
angle with the line of force.
Ans: B Along a line of force, if its initial velocity is zero.
Q174. ————— gives the information on field strength, direction, and nature of the charge. 1 Mark
A Electric current. B Electric flux. C Electric field. D Electric potential.
Ans: C Electric field.
Explanation:
Electric field gives the information on field strength, direction, and nature of the charge.
Q175. The molecules having centres of positive charges and negative charges at same place, has the dipole moment as: 1 Mark
A Zero. B Infinity. C Positive. D Negative.
Ans: A Zero.
Q176. Two large conducting spheres carrying charges Q1 and Q2 are kept with their centres r distance apart. The magnitude of electrostatic force between them is not 1 Mark
exactly \frac{1}{4\pi\varepsilon_0}\frac{\text{Q}_1\text{Q}_2}{\text{r}^2} because:
A These are not point charges. B Charge distribution on the spheres is not uniform.
C Charges on spheres will shift towards the centres of their respective D Charges will shift towards the portions of the spheres which are closer and
spheres. facing towards each other.
Ans: B Charge distribution on the spheres is not uniform.
Explanation:
The given relation is Coulomb’s law which is true for point charges. In the present case, as the spheres are brought closer, the distribution of charge on them
becomes non­-uniform.
Q177. When a glass rod is rubbed with a piece of silk cloth the rod ? 1 Mark
A And the cloth both acquire positive charge. B Becomes positively charged while the cloth has a negative charge.
C And the cloth both acquire negative charge. D Becomes negatively charged while the cloth has a positive charge.
Ans: B Becomes positively charged while the cloth has a negative charge.
Explanation:
It is a convention to call the charge acquired by a glass rod when it is rubbed with silk as positive. The other kind of charge is said to be negative.
Q178. The S.I. unit of electric flux is: 1 Mark
A Weber. B Newton per coulomb. C Volt × metre. D Joule per coulomb.
Ans: C Volt × metre.
Q179. As shown in figure a dust particle with mass m = 5.0 × 10-9 kg and charge q0 = 2.0nC starts from rest at point a and moves in a straight line to point b. What is its 1 Mark
speed v at point b?

A 26 ms-1 B 34 ms-1 C 46 ms-1 D 14 ms-1


Ans: C 46 ms-1
Q180. In annihilation process, in which an electron and a positron transform into two gamma rays, which property of electric charge is displayed? 1 Mark
A Additivity of charge. B Quantisation of charge.
C Conservation of charge. D Attraction and repulsion.
Ans: C Conservation of charge.
Q181. Complete the following statements with an appropriate word/ term be filled in the blank space(s). 1 Mark
The force of attraction or repulsion between two charges is given by ......................
A Gauss's law B Kepler's law C Coulomb's law D Gravitational law
Ans: C Coulomb's law

https://bls.smartstudies.co.in/#/exam/pdf-preview/b5b87181-b79b-4b4a-8ebf-8f94fb605fc8/1 18/139
5/17/24, 8:14 PM Exam Automation
Explanation:
Coulomb's law: The force exerted by one point charge is proportional to the product of the charge and varies inversely as the sure of the distance separating the
charges.
Q182. Find the dimensional formula of \in_0. 1 Mark

Ans:By Coulomb's Law,


\text{F}=\frac{1}{4\pi\in_0}\frac{\text{q}_1\text{q}_2}{\text{r}^2}
\Rightarrow\in_0=\frac{1}{4\pi\text{F}}\frac{\text{q}_1\text{q}_2}{\text{r}^2}
Using [\text{F}]=\big[\text{MLT}^{-2}\big]
[\text{r}]=\big[\text{M}^0\text{L}^1\text{T}^0\big]
[\text{q}]=\big[\text{M}^2\text{L}^0\text{T}^1\text{A}^1\big], we get
[\in_0]=\big[\text{M}^{-1}\text{L}^{-3}\text{T}^4\text{A}^2\big]
Q183. Water molecule is the example of: 1 Mark
A Polar molecule. B Non polar molecule. C Both a and b. D None.
Ans: A Polar molecule.
Q184. Three equal positive charges are kept at the corners of an equilateral triangle. What will be the vector sum of the forces acting on the particles? 1 Mark
A Directed towards the center. B Directed radially outside.
C Acts along one of the sides of the triangle. D Zero.
Ans: D Zero.
Explanation:
The magnitude of the force acting on each of the charges will be the same and they will differ from each other by 120 degrees. Therefore their resultant force will
always be zero.
Q185. If a comb is rubbed on dry hairs, and then brought near paper pieces, it: 1 Mark
A Attracts paper. B Repels paper.
C Remains indifferent to paper. D Either attracts or repels paper.
Ans: A Attracts paper.
Explanation:
If we bring small bits of paper near a rubber (or plastic) comb, it is found that the comb does not attract the bits of paper. But if the same comb is first rubbed on
dry hairs and then it is brought near the paper pieces, it attracts them.
Q186. In metals ____ are the mobile charges. 1 Mark
A Electrons. B Holes. C Both a and b. D None.
Ans: A Electrons.
Q187. The property which differentiates two kinds of charges is called ____________? 1 Mark
A Equality of charge B Polarity of charge C Fraction of charge D None of the option
Ans: B Polarity of charge
Explanation:
The property which differentiates two kinds of charges is called polarity of charge.
Q188. An electron and a proton are both released from rest 1 meter from a large stationary negative charge, considering only the force from the large stationary negative 1 Mark
charge on the proton and electron. Which of the following is true?
A The magnitude of force is greater on the electron. B The magnitude of force is greater on the proton.
C The magnitude of force is the same on both the electron and proton. D The direction of the force, but not the magnitude, is the same on both the
electron.
Ans: C The magnitude of force is the same on both the electron and proton.
Explanation:
The electron has negative charge ,−e and proton has positive charge ,+e.
Thus, the force on electron due to the large stationary charge will be repulsive and force on proton due to the large stationary charge will be attractive.
Since both electron and proton released from same distance and they have equal magnitude of charge so the magnitude of force on both the electron and proton
will be same.
Q189. The ampere-sec stands for the unit of: 1 Mark
A Power B Current C Energy D Charge
Ans: D Charge
Explanation:
Charge is given by the relation, q = I × t
so its unit is ampere - sec
Q190. Coulomb’s law is true for: 1 Mark
–11 –15
A Atomic distances (= 10 m). B Nuclear distances (= 10 m).
C Charged as well as uncharged particles. D All the distances.
Ans: D All the distances.
Q191. Which among the following is false? 1 Mark
A Coulomb force is a central force. B The force between two charges depend on the medium between them.
C Coulomb force is a weak force. D The net force on a charge is the vector sum of the forces acting on it due to
several other charges.
Ans: C Coulomb force is a weak force.
Explanation:
Coulomb force is a strong force. It can be shown that the Coulomb force between two electrons is 1043 times than gravitational force. All the other options are basic
properties of Coulomb’s Law.
Q192. For two statements are given-one labelled Assertion (A) and the other labelled Reason (R). Select the correct answer to these questions from the codes (a), (b), (c) 1 Mark
and (d) as given below.
Assertion (A): If a point charge be rotated in a circle around a charge, the work done will be zero.
Reason (R): Work done is equal to dot product of force and distance.
A Both A and R are true, and R is the correct explanation of A. B Both A and R are true, but R is not the correct explanation of A.
C A is true but R is false. D A is false and R is also false.
Ans: A Both A and R are true, and R is the correct explanation of A.
Explanation:
Work done will be zero because in rotating the charge in a circle, force is along the radius, and direction of motion is perpendicular to it.
\therefore Work done, \text{W}=\vec{\text{F}}\cdot\vec{\text{S}}=\text{FS}\cos\theta=\text{FS}\cos90^\circ=0
Q193. Two charges are placed a certain distance apart in air. If a brass plate is introduced between them, the force between them will: 1 Mark
A Increase B Decrease C Remain the same D Become zero
Ans: C Remain the same

https://bls.smartstudies.co.in/#/exam/pdf-preview/b5b87181-b79b-4b4a-8ebf-8f94fb605fc8/1 19/139
5/17/24, 8:14 PM Exam Automation
Explanation:
If two like charges are placed in air, then they will experience a force of repulsion from each other.
If a metal plate is introduced between them, then nothing will happen.
Because, if they are positively charged, then both will try to attract electrons of the plate along opposite sides and thus no net force will act on the electrons of the
plate and the plate will remain as it is.
If the charges are negative then both will try to repel electrons from both sides and nothing will happen.
Q194. A charge q1​exerts a force of 45N on a charge of q2 ​= 10-5C located at a point 0.2m from q1​. The magnitude of q1​is: 1 Mark
A 4 × 10-8C B 2 × 10-5C C 3 × 10-6C D 5 × 10-8C
-5
Ans: B 2 × 10 C
Q195. A closed surface S is constructed around a conducting wire connected to a battery and a switch (figure). As the switch is closed, the free electrons in the wire start 1 Mark
moving along the wire. In any time interval, the number of electrons entering the closed surface S is equal to the number of electrons leaving it. On closing the
switch, the flux of the electric field through the closed surface:

A Is increased. B Is decreased. C Remains unchanged. D Remains zero.


Ans: C Remains unchanged.
Explanation:

The flux of the electric field through the closed surface is remain unchanged.
Because the no. of electric field lines enter in the closed path = no. of electric field lines existsin the closed path.
Q196. A proton and an electron are placed in a uniform electric field. 1 Mark
A The electric forces acting on them will be equal. B The magnitudes of the forces will be equal.
C Their accelerations will be equal. D The magnitudes of their accelerations will be equal.
Ans: B The magnitudes of the forces will be equal.
Explanation:
Proton contain positive charge = 1.6 × 10-19 = e
Electron contain negative charge = -1.6 × 10-19 = -e
Force inoniforma electric field is = qE
\therefore\ The Magniludes of Electric force is equal But direction of electric force is opposite to each other.
mass of proton = 1.67 × 10-27Kg.
mass of electron = 9.1 × 10-31Kg.
So that Magnitudes of their acceleration will be unequal.
Q197. 1 Coulomb = ________ Electro Static Unit. 1 Mark
9 8 -19 9
A 3 × 10 B 3 × 10 C 1.602 × 10 D 2 × 10
9
Ans: A 3 × 10
Explanation:
1 Coulomb means 3 × 109 Electro Static Unit of charge. Coulomb and ESU are two different units of charge and their conversion formula is necessary. Coulomb is
the unit of charge in the SI system and esu is the unit of charge in the CGS system.
Q198. The SI unit of volume charge density is: 1 Mark
2. 3.
A C/m. B C/m C C/m D Cm.
3.
Ans: C C/m
Q199. The force per unit charge is known as ____________. 1 Mark
A Electric current B Electric potential C Electric field D Electric space
Ans: C Electric field
Explanation:
The force per unit charge is known as the electric field.
Q200. A charge q is placed at the centre of the open end of a cylindrical vessel (figure). The flux of the electric field through the surface of the vessel is: 1 Mark

https://bls.smartstudies.co.in/#/exam/pdf-preview/b5b87181-b79b-4b4a-8ebf-8f94fb605fc8/1 20/139
5/17/24, 8:14 PM Exam Automation

A \text{Zero} B \frac{\text{q}}{\epsilon_0}
C \frac{\text{q}}{2\epsilon_0} D \frac{2\text{q}}{\epsilon_0}
Ans: C \frac{\text{q}}{2\epsilon_0}
Explanation:

In the charge 'q' in side closed cylinder, due to charge the flux of the electric field through the surface of the vessel is \frac{\text{q}}{\epsilon_0}. Here A charge q
is placed at the centre of the open end of a cylindrical, By symmetricily Half line of flux goes outside & half flux line goes inside. So we can say that the flux of the
electric field through the surface of the vessel is \frac{\text{q}}{2\epsilon_0}.
Q201. Figure shows electric field lines in which an electric dipole p is placed as shown. Which of the following statements is correct? 1 Mark

A The dipole will not experience any force. B The dipole will experience a force towards right.
C The dipole will experience a force towards left. D The dipole will experience a force upwards.
Ans: C The dipole will experience a force towards left.
We know electric field emerges radially outward from positive point charge.
In the figure given above, space between field lines is increasing (or density of electric field line is decreasing). In other words, the electric force is decreasing
while moving from left to right.
Thus, the force on charge ‒q is greater than the force on charge +q in turn dipole will experience a force towards left direction.
Q202. Two charged particles are placed at a distance 1.0cm apart. What is the minimum possible magnitude of the electric force acting on each charge? 1 Mark

Ans:Minimum charge of a body is the charge of an electron,


\Rightarrow\text{q}=1.6\times10^{-19}\text{C}
Given,
Separation between the charges, r = 1cm = 10-2cm
By Coulomb's Law, force,
\text{F}=\frac{1}{4\pi\epsilon_0}\frac{\text{q}_1\text{q}_2}{\text{r}^2}
=\frac{9\times10^9\times1.6\times1.6\times10^{-19}\times10^{-19}}{10^{-2}\times10^{-2}}
=23.04\times10^{-38+9+2+2}
=23.04\times10^{-25}
=2.3\times10^{-24}
Q203. Which of the following are conductors. 1 Mark
A Metals. B Human body. C Earth. D All.
Ans: D All.
Q204. Force between the protons and neutrons in a nucleus is: 1 Mark
A Only coulombian B Only nuclear C Both (a) and (b) D None of these
Ans: B Only nuclear
Explanation:

CONCEPT:
Strong nuclear force: The strong attractive force which binds together the protons and neutrons in a nucleus is called a strong nuclear force.
This force cannot be an electrostatic force because positively charged protons strongly repel each other at such a small separation of the order 10-15m.
This strong nuclear force is the strongest of all the fundamental forces, about 100 times stronger than the electromagnetic force.
Important properties of the strong nuclear force:

https://bls.smartstudies.co.in/#/exam/pdf-preview/b5b87181-b79b-4b4a-8ebf-8f94fb605fc8/1 21/139
5/17/24, 8:14 PM Exam Automation
It is the strongest interaction known in nature, which is about 1038 times stronger than the gravitational force.
It is a short-range force that operates only over the size of the nucleus (= 10-15m).
It is basically an attractive force but becomes repulsive when the distance between the nucleons becomes less than 0.5 fermi (1 fermi = 10-15m).
It varies inversely with some higher power (> 2) of distance.
It is a non-central and non-conservative force.
From the above, it is clear that the force between a neutron and a proton inside the nucleus is only nuclear attractive.
Q205. Two charges q1, q2 exert some amount of force on each other. What will happen to the force on q1 if another charge q3 is brought close to them? 1 Mark
A The force will increase. B The force will decrease.
C The force remains the same. D The force may increase or decrease depending on whether q3 is positive or
negative.
Ans: D The force may increase or decrease depending on whether q3 is positive or negative.
Explanation:
If both the charges q1 and q2 are positive then they will repel each other. Now if the third charge q3 is also positive, the repulsive force will increase. But if q3 is
negative, the repulsive force will be reduced by some amount due to the attractive force between q1 and q3.
Q206. The ratio of electric force between two electrons to two protons separated by the same distance in air is: 1 Mark
0 6 4
A 10 B 10 C 10 D None of these
0
Ans: A 10
Q207. In figure two positive charges, q2​and q3​ fixed along the y-axis, exert a net electric force in the +x direction on a charge q1​fixed along the x-axis. If a positive 1 Mark
charge Q is added at (x, 0), the force on q1​:
A Shall increase along the positive x-axis. B Shall decrease along the positive x-axis.
C Shall point along the negative x-axis. D Shall increase but the direction changes because of the intersection
of Q with q1 and q3​
Ans: A Shall increase along the positive x-axis.
Explanation:
Since positive charge q2​and q3​exert a net force in the +X direction on the charge Q1​fixed along the X-axis, the charge q1​ is negative as shown in figure. Obviously,
due to addition of positive charge Q ad (x, 0) the force on −q shall increase along the positive X-axis.
Q208. The electric flux through a closed Gaussian surface depends upon: 1 Mark
A Net charge enclosed and permittivity of the medium. B Net charge enclosed, permittivity of the medium and the size of the
Gaussian surface.
C Net charge enclosed only. D Permittivity of the medium only.
Ans: A Net charge enclosed and permittivity of the medium.
Explanation:
The electric flux through a closed Gaussian surface is given by,
\oint\vec{\text{E}}.\vec{\text{ds}}=\frac{\text{q}}{\in}
Where, q is the net charge enclosed by the Gaussian and \in is the permittivity of the medium.
Hence, the correct answer is Net charge enclosed and permittivity of the medium.
Q209. Figure shows a closed surface which intersects a conducting sphere. If a positive charged is placed at the point P, the flux of the electic field through the closed 1 Mark
surface:

A Will remain zero. B Will become positive. C Will become neagative. D Will become undefined.
Ans: B Will become positive.
Explanation:
Charge distribution:

https://bls.smartstudies.co.in/#/exam/pdf-preview/b5b87181-b79b-4b4a-8ebf-8f94fb605fc8/1 22/139
5/17/24, 8:14 PM Exam Automation

Due to charge distribution, intersection by the closed surface of conducting sphere containing the positive charge. So we can say that the flux of the electric field
through the closed surface will become positive.
\therefore\phi=\frac{\text{q}_{\text{in}}}{\epsilon_0}\Rightarrow+\text{ve}
Q210. When a comb is rubbed with hair, it attracts paper bits. Choose the right explanation: 1 Mark
A Bits of paper gets attracted due to gravitational force. B Due to electromagnetic effect, bits of paper are attracted.
C Comb gets charged by friction and attracts bits of paper. D None of these.
Ans: C Comb gets charged by friction and attracts bits of paper.
Explanation:
When comb is rubbed to hair, it induces negative charge due to friction. When it is brought close to paper bits, they get polarized and the positive part sticks to the
comb due to electrostatic attraction.
Q211. For two statements are given-one labelled Assertion (A) and the other labelled Reason (R). Select the correct answer to these questions from the codes (a), (b), (c) 1 Mark
and (d) as given below.
Assertion: Electrostatic field line start at positive charges and end at negative charges
Reason: Field lines are continuous curve without any breaks and they form a closed loop
A Both A and R are true and R is the correct explanation of A. B Both A and R are true but R is not the correct explanation of A.
C A is true but R is false. D A is false and R is also false.
Ans: C A is true but R is false.
Q212. If \int_\text{S}\text{E.dS}=0 over a surface, then: 1 Mark
A The electric field inside the surface and on it is zero. B The electric field inside the surface is necessarily uniform.
C The number of flux lines entering the surface must be equal to the number D All charges must necessarily be outside the surface.
of flux lines leaving it.
Ans: C The number of flux lines entering the surface must be equal to the number of flux lines leaving it. D All charges must necessarily be outside the surface.
Given,
\int_\text{s}\text{E.dS}=0
It means the algebraic sum of number of flux lines entering the surface and number of flux lines leaving the surface.
Now, from Gauss' law,
\int_\text{S}\text{E.dS}=\frac{\text{q}}{\epsilon_0}
Where q is charge enclosed by the surface.
Now,
\int_\text{s}\text{E.dS}=0
q = 0 i.e., net charge enclosed by the surface must be zero.
Hence, all other charges must necessarily be outside the surface.
Q213. The SI unit of surface charge density is: 1 Mark
A C/m B C/m2 C Cm D Cm2
2
Ans: B C/m
Q214. Consider the situation of the previous problem. The force on the central charge due to the shell is: 1 Mark
A Towards left. B Towards right. C Upward. D Zero.
Ans: D Zero.
Explanation:

Force on the central charge due to shell is right \frac{\text{Kq}^2}{\text{r}^2} towards.


Q215. Two spheres A and B of exactly same mass are given equal positive and negative charges respectively. Their masses after charging. 1 Mark

https://bls.smartstudies.co.in/#/exam/pdf-preview/b5b87181-b79b-4b4a-8ebf-8f94fb605fc8/1 23/139
5/17/24, 8:14 PM Exam Automation

A Remains unaffected. B Mass of A > mass of B.


C Mass of A < mass of B. D Nothing can be said.
Ans: C Mass of A < mass of B.
Q216. An electron is released from rest in a region of space with a non-zero electric field. Which of the following statements is true? 1 Mark
A The electron will begin moving towards a region of higher potential. B The The electron will begin moving towards a region of lower potential.
C The The electron will begin moving among a line of constant potential. D Nothing can be concluded unless the direction of the electric field is known.
Ans: A The electron will begin moving towards a region of higher potential.
Explanation:
Negative charge itself goes from low to high potential.
Q217. In a charge free region, electric field lines can be taken to be continuous curves: 1 Mark
A With breaks. B Without breaks. C Both a and b. D None.
Ans: B Without breaks.
Q218. The value of electric permittivity of free space is: 1 Mark
A 9 × 109 NC2/m2 B 8.85 × 10–12Nm2/C2 sec
C 8.85 × 10–12 C2/Nm2 D 9 × 109C2/Nm2
Ans: C 8.85 × 10–12 C2/Nm2
Q219. For two statements are given-one labelled Assertion (A) and the other labelled Reason (R). Select the correct answer to these questions from the codes (a), (b), (c) 1 Mark
and (d) as given below.
Assertion (A): As force is a vector quantity, hence electric field intensity is also a vector quantity.
Reason (R): 'Tile unit of electric field intensity is newton per coulomb.
A Both A and R are true, and R is the correct explanation of A. B Both A and R are true, but R is not the correct explanation of A.
C A is true but R is false. D A is false and R is also false.
Ans: B Both A and R are true, but R is not the correct explanation of A.
Explanation:
The electric field intensity is equal to force experienced by unit positive test charge q0 placed at that point i.e.
\vec{\text{E}}=\frac{\vec{\text{F}}}{\text{q}_0} thus \vec{\text{E}} is also a vector quality,
As, \text{E}=\frac{\text{F}}{\text{q}}=\frac{\text{newton}}{\text{coulomb}}.
Q220. A charged body has an electric flux F associated with it. Now if the body is place inside a conducting shell then the electric flux outside the shell is: 1 Mark
A zero B Greater than F C Less than F D Equal to F
Ans: A zero
Explanation:
A conducting shell does not allow to escape any flux outside, thus confining all fluxes within it.so,flux outside the shell will be zero.
[this happens due to electrostatic sheilding by the conducting shell]
Q221. Define Electric Flux. Write its SI unit. 1 Mark

Ans:Electric flux \triangle\phi, through an area element \overrightarrow{\triangle S}, is defined by


\triangle\phi=\overrightarrow{E}.\overrightarrow{\triangle S}=E\triangle S\cos\theta
where \theta is the angle between\overrightarrow{E}\text{ }\text{and}\text{ }\overrightarrow{\triangle S}.
S.I unit of electric flux is NC^{-1}m^2.
Q222. A thin, metallic spherical shell contains a charge Q on it. A point charge q is placed at the centre of the shell and another charge q1 is placed outside it as shown in 1 Mark
figure. All the three charges are positive. The force on the charge at the centre is:

A Towards left. B Towards right. C Upward. D Zero.


Ans: D Zero.
Explanation:

Electric field due to Q & q, inside the spherical shell is zero.


Q223. Gauss’s law is true only if force due to a charge varies as: 1 Mark
A r–1 B r–2 C r–3 D r–4
–2
Ans: B r
Q224. An electric dipole is placed at the centre of a sphere. 1 Mark
Mark the correct options:
A The flux of the electric field through the sphere is zero. B The electric field is zero at every point of the sphere.
https://bls.smartstudies.co.in/#/exam/pdf-preview/b5b87181-b79b-4b4a-8ebf-8f94fb605fc8/1 24/139
5/17/24, 8:14 PM Exam Automation
C The electric field is not zero anywhere on the sphere. D The electric field is zero on a circle on the sphere.
Ans: A The flux of the electric field through the sphere is zero. C The electric field is not zero anywhere on the sphere.
Explanation:
\therefore Net charge of electric dipole is always zero.

\phi=\frac{\text{q}_{\text{inside}}}{\epsilon_0} Here inside = 0


The flux of the electric field through the sphere is zero. But the electric field is not zero any where on the sphere. Take 'A' point any where on the sphere, and find
the electric field on the surface. You will get non zero electric field on the sphere.
Q225. Two unlike charges attract each other with a force of 10N. If the distance between them is doubled, the force between them is: 1 Mark
A 40N B 20N C 5N D 2.5N
Ans: D 2.5N
Explanation:
We know \text{F}=\frac{1}{4\pi\epsilon_0}\frac{\text{q}_1\text{q}_2}{\text{r}^2}=10\text{N}
if distance is doubled.
\text{F}_{\text{new}}=\frac{1}{4\pi\epsilon_0}.\frac{\text{q}_1\text{q}_2}{(2\text{r)}^2}=\frac{10}{4}
= 2.5N
Q226. 1C is the charge which is placed at a distance of 1m from the another charge of same magnitude in vacuum experiences an electrical force of repulsion of 1 Mark
magnitude:
A 9 × 10-9N B 9 × 109N C 109N D 10-9N
Ans: B 9 × 109N
Q227. What is the unit of surface charge density in the SI unit? 1 Mark
2 3
A C B C/ m C C/ m D C/ m
2
Ans: C C/ m
Explanation:
Surface charge density means how much charge is stored in the unit surface area of a conductor.
So, it’s unit will be=\frac{\text{the unit of the charge}}{\text{the unit of area}}. In SI the unit of charge is Coulomb and the unit of area is m2.
Q228. For two statements are given-one labelled Assertion (A) and the other labelled Reason (R). Select the correct answer to these questions from the codes (a), (b), (c) 1 Mark
and (d) as given below.
Assertion (A): The surface densities of two spherical conductors of different radii are equal. Then the electric field intensities near their surface are also equal.
Reason (R): Surface density is equal to charge per unit area.
A Both A and R are true, and R is the correct explanation of A. B Both A and R are true, but R is not the correct explanation of A.
C A is true but R is false. D A is false and R is also false.
Ans: B Both A and R are true, but R is not the correct explanation of A.
Explanation:
As, \sigma_1=\sigma_2 (Given)
\therefore\frac{\text{q}_1}{4\pi\text{r}_1^2}=\frac{\text{q}_2}{4\pi\text{r}_2^2},
Or \frac{\text{q}_1}{\text{q}_2}=\frac{\text{r}_1^2}{\text{r}^2_2}
[Let r1 and r2 be two different radii) Then the ratio of electric field intensities near the surface of spherical conductors,
\frac{\text{E}_1}{\text{E}_2}=\frac{\text{q}_1}{4\pi\epsilon_0\text{r}^2_1}\times\frac{4\pi\epsilon_0\text{r}^2_2}{\text{q}_2}
=\frac{\text{q}_1}{\text{q}_2}\times\frac{\text{r}^2_2}{\text{r}^2_1}=\frac{\text{q}_1}{\text{q}_2}\times\frac{\text{q}_2}{\text{q}_1}=1
i.e. E1 = E2.
Q229. Electrostatic field lines ____. 1 Mark
A Forms any closed loops. B Do not form any closed loops.
C Both a and b. D None.
Ans: B Do not form any closed loops.
Q230. .......... Property develops in comb due to friction. 1 Mark
A Attractive B Repulsive C Both D None
Ans: A Attractive
Explanation:
An attractive property is develop in the comb due to friction. The comb on rubbing over dry hairs is aid to be charged.
Q231. Gravitational force has only on sign but coulomb force has: 1 Mark
A Positive sign. B Negative sign. C Both a and b. D None.
Ans: C Both a and b.
Q232. Two identical thin rings, each of radius a meter, are coaxially placed at a distance R meter apart. If Q1 coulomb and Q2 coulomb are respectively the charges 1 Mark
uniformly spread on the two rings, the work done in moving a charge q coulomb from the centre of one ring to that of the other is:

https://bls.smartstudies.co.in/#/exam/pdf-preview/b5b87181-b79b-4b4a-8ebf-8f94fb605fc8/1 25/139
5/17/24, 8:14 PM Exam Automation

A \text{Zero} B \frac{\text{q}(\text{Q_1 - Q_2)}(\sqrt{2}-1)}{4\sqrt{2}\pi\epsilon_0\text{a}}


C \frac{\text{q}\sqrt{2}(\text{Q_1 + Q_2)}}{4\pi\epsilon_0\text{a}} D \frac{\text{q}(\text{Q_1 + Q_2)}(\sqrt{2} + 1)}{4\sqrt{2}\pi\epsilon_0\text{a}}
Ans: B \frac{\text{q}(\text{Q_1 - Q_2)}(\sqrt{2}-1)}{4\sqrt{2}\pi\epsilon_0\text{a}}
Q233. A charge is being induced to a sphere with the help of another charged sphere. The extent of induction increase if ____________? 1 Mark
A The distance between the sphere increases. B The distance between the sphere decreases.
C The two spheres are kept in contact with each other. D Remains same irrespective of the distance between the spheres.
Ans: D Remains same irrespective of the distance between the spheres.
Explanation:
The extent of induction increases if both the spheres are kept close to each other because the more electrons in the uncharged sphere are attracted or repelled by
the charged sphere, so the extent of induction increases.
Q234. Two charges of 10C and -15C are separated in air by 1m.The ratio of force exerted by one on the other is: 1 Mark
A 1:2 B 2:1 C 1:1 D None of these
Ans: C 1 : 1
Explanation:
From Coulomb's law, the forces exerted by one on the other are equal and opposite direction i.e ∣F12​∣ = ∣-F21​∣
Thus the required ration will be 1 : 1
Q235. A soap bubble is given negative charge, its radius will: 1 Mark
A Increase B Decrease C Remain unchanged D Fluctuate
Ans: A Increase
Explanation:

The charge will be distributed uniformly over the surface of the bubble by symmetry.
As we know, they want to step much farther apart, as charges repel, the only path outward, taking with them the soap surface.
At the same point, the rise in the restored force of the soap film (surface tension) would be equal and contrary to the electrostatic force, which will result in a new
(larger) radius of equilibrium.
Because of the ionic similarities, this can happen to both positive and negatively charged bubbles.Therefore, when a negative charge is given to the soap bubble,
then its radius will increase.
Q236. In the thunderstorm, the charges accumulate near the upper edges of clouds are: 1 Mark
A Negatively charged. B Positively charged. C Neutral. D None of these.
Ans: B Positively charged.
Explanation:
We have to find the nature of charges on upper edges of clouds in a thunderstorm, the air currents move upwards while the water droplets move downward. This
vigorous movement causes separation of charges. This leads to the positive charges collected near the upper edges of clouds and the negative charges to
accumulate near the lower edges of the clouds.
Q237. The force of repulsion between two point charges is F, when these are 1m apart. Now the point charges are replaced by conducting spheres of radii 5cm having 1 Mark
the charge same as that of point charges. The distance between their centres is 1m, then the force of repulsion will:
A increase B decrease
C remain same D become \frac{10\text{F}}{9}
Ans: C remain same
Explanation:
From Coulombs law
\text{F}\propto\frac{\text{q}_1\text{q}_2}{\text{r}^2}
As the charges and distance are same in either cases, the force between the charges remains constant.
Q238. The amount of charge flowing through the wire in 1s if the current flowing 1 A then the charge flowing will be: 1 Mark
A One coulomb. B 2 coulomb. C 3 coulomb. D None.
Ans: A One coulomb.
Q239. The dimensions of linear charge density is: 1 Mark
A [L T A]. B [L-1 T A]. C [L-1 T-1 A-1]. D None.
Ans: B [L-1 T A].
Q240. Which one of the following is a safe place during lightning? 1 Mark
A Under a tree B Under a light post
C House with lightning arrester D High wall
Ans: C House with lightning arrester
Explanation:
Lightning arrester arrests lightning and allows a safe path of electricity to ground. Thus it is a safe place inside a house that has a lightning arrester on the top of it.
Else, electricity finds a high tower or tree or wall and travels through them to ground. Therefore these are not safe places to take shelter during lightning.
Q241. The dimensions of electric flux are: 1 Mark
A [M L-3 T-3 A-1]. B [M L-3 T-3 A-1]. C [M L3 T-3 A-1]. D None.
Ans: C [M L3 T-3 A-1].
Q242. For two statements are given-one labelled Assertion (A) and the other labelled Reason (R). Select the correct answer to these questions from the codes (a), (b), (c) 1 Mark
and (d) as given below.
Assertion: Coulomb’s force is the dominating force in the universe
Reason: Coulomb’s force is weaker then the gravitational
A Both A and R are true and R is the correct explanation of A. B Both A and R are true but R is not the correct explanation of A.
C A is true but R is false. D A is false and R is also false.
Ans: D A is false and R is also false.
Q243. Opening up of strips of an electroscope is an indication of: 1 Mark
A Proximity of a conductor. B Proximity of a charged body.
C Proximity of an uncharged body. D Earth connection.
Ans: B Proximity of a charged body.

https://bls.smartstudies.co.in/#/exam/pdf-preview/b5b87181-b79b-4b4a-8ebf-8f94fb605fc8/1 26/139
5/17/24, 8:14 PM Exam Automation
Explanation:
When a charged body comes close to an electroscope, charges are induced to the electroscope. These charges flow down to the metal strips which open up due
to repulsion for having similar charges.
Q244. Coulomb’s Law is valid for ______? 1 Mark
A Only point charge. B For both point charge and distributed charge.
C Only distributed charges. D Neither distributed nor point charge.
Ans: A Only point charge.
Explanation:
Coulomb’s Law explains the force between two point charges only. A distributed charge can be considered as the sum of infinite point charges and thus the force
between two distributed charge systems can be explained.
Q245. Mark the correct options: 1 Mark
A Gauss's law is valid only for symmetrical charge distributions. B Gauss's law is valid only for charges placed in vacuum.
C The electric field calculated by Gauss's law is the field due to the charges D The flux of the electric field through a closed surface due to all the charges
inside the Gaussian surface. is equal to the flux due to the charges enclosed by the surface.
Ans: D The flux of the electric field through a closed surface due to all the charges is equal to the flux due to the charges enclosed by the surface.
Explanation:
The flux of the electric field through a closed surface due to all the charges is equal to the flux due to the charges enclosed by the surface.
Q246. Electric lines of force: 1 Mark
A Exist everywhere. B Exist only in the immediate vicinity of electric charges.
C Exist only when both positive and negative charges are near one another. D Are imaginary.
Ans: D Are imaginary.
Q247. Two equal balls having equal positive charge ‘q’ coulombs are suspended by two insulating strings of equal length. What would be the effect on the force when a 1 Mark
plastic sheet is inserted between the two?

Ans:Force decreases.
Q248. On rubbing, when one body gets positively charged and other negatively charged, the electrons transferred from positively charged body to negatively charged 1 Mark
body are:
A Valence electrons only. B Electrons of inner shells.
C Both valence electrons and electrons of inner shell. D Yet to be established.
Ans: A Valence electrons only.
Q249. The rubber used in the wheels of aero-plane is _________? 1 Mark
A Perfect insulator. B Slightly conducting.
C Can be an insulator or conducting. D Semiconductor.
Ans: B Slightly conducting.
Explanation:
Due to high friction during takeoff, a huge amount of charge is produced on the rubber of the wheel of a plane. This charge must be sent to the ground. So, the
rubber used is slightly conducting, else the huge charge produced may produce a spark and can cause an accident.
Q250. Electric field is a _______? 1 Mark
A Scalar quantity B Vector quantity
C Tensor quantity D Quantity that has properties of both scalar and vector
Ans: B Vector quantity
Explanation:
A scalar quantity is a quantity with magnitude only but no direction. But a vector quantity possesses both magnitude and direction. An electric field has a very
specific direction (away from a positive charge or towards a negative charge). Hence electric field is a vector quantity. Moreover, we have to use a vector addition
for adding two electric fields.
Q251. Define electric dipole moment. Write its S.I. unit. 1 Mark

Ans:Electric dipole moment is defined as the numerical product of charge and distance between the charges, and is directed from negative to positive charge.
Alternate Answer

\overrightarrow{\text{p}} = \text{q}( 2 \overrightarrow{\text{a}})


Unit: coulomb metre or Cm.
Q252. What should be the shape of a conductor that can hold a charge for long? 1 Mark
A Cubical B Conical C Sharp-edged D Spherical
Ans: D Spherical
Explanation:
Sphere has uniform charge distribution over its entire surface. It doesn’t have any sharp edges hence very little chance of charge accumulation at those edges and
hence very little chance of discharge of stored charge.
Q253. For two statements are given-one labelled Assertion (A) and the other labelled Reason (R). Select the correct answer to these questions from the codes (a), (b), (c) 1 Mark
and (d) as given below.
Assertion: Consider two identical charges placed distance 2d apart, along the x-axis. The equilibrium of a positive test charge placed at the point O midway
between them is stable for displacements along the x-axis
Reason: Force on test charge is zero.
A Both A and R are true and R is the correct explanation of A. B Both A and R are true but R is not the correct explanation of A.
C A is true but R is false. D A is false and R is also false.
Ans: B Both A and R are true but R is not the correct explanation of A.
Q254. Lightning is a ________ phenomenon: 1 Mark
A Electromagnetic B Current electricity C Electrochemical D Electrostatic
Ans: D Electrostatic
Explanation:
Lightning is a sudden electrostatic discharge during an electrical storm between electrically charged regions of a cloud, between that cloud and another cloud, or
between a cloud and the ground.
Q255. For two statements are given-one labelled Assertion (A) and the other labelled Reason (R). Select the correct answer to these questions from the codes (a), (b), (c) 1 Mark
and (d) as given below.

https://bls.smartstudies.co.in/#/exam/pdf-preview/b5b87181-b79b-4b4a-8ebf-8f94fb605fc8/1 27/139
5/17/24, 8:14 PM Exam Automation
Assertion: A point charge is brought in an electric field at a nearby point will increase or decrease depending on the nature of charge.
Reason: The electric field is independent of the nature of charge.
A Both A and R are true and R is the correct explanation of A. B Both A and R are true but R is not the correct explanation of A.
C A is true but R is false. D A is false and R is also false.
Ans: D A is false and R is also false.
Q256. A charge Q is uniformly distributed over a large plastic plate. The electric field at a point P close to the centre of the plate is 10Vm-1. If the plastic plate is replaced 1 Mark
by a copper plate of the same geometrical dimensions and carrying the same charge Q, the electric field at the point P will become:
A Zero B 5Vm-1 C 10Vm-1 D 20Vm-1
Ans: A Zero
Explanation:
Electric field at centre of the plastic Platcis same as the centre of the copper plate.
Q257. An attractive force of 9N acts between +5C and −5C at some distance. These charges are allowed to touch each other and are then again placed at their initial 1 Mark
position. The force acting between them will be:
A infinite B 9×109N C 1N D zero
Ans: D zero
Explanation:
According to the question
\text{F}=\frac{1}{4\pi\epsilon_0}\frac{(5\times10^{-6})\times(-5\times10^{-6})}{\text{r}^2}
On touching them the charge on each will be zero.
\therefore\text{F}=\frac{1}{4\pi\epsilon_0}\frac{(0)}{\text{r}^2}=0
Q258. The adjoining figure shows a negatively charged electroscope. If a negatively charged rod is brought close to, but not touching, the knob, the two leaves will: 1 Mark

A Move closer together B Move farther apart C Not move at all D None of these
Ans: B Move farther apart
Explanation:
In this case, the given electroscope is negatively charged. If a negatively charged rod is brought close to, but not touching, the knob, the two leaves will move far
apart as the like charges repel each other.
Q259. Charge motion within the Gaussian surface gives changing physical quantity: 1 Mark
A Electric field. B Electric flux. C Charge. D Gaussian surface area.
Ans: A Electric field.
Q260. For two statements are given-one labelled Assertion (A) and the other labelled Reason (R). Select the correct answer to these questions from the codes (a), (b), (c) 1 Mark
and (d) as given below.
Assertion: When charges are shared between any two bodies no charges is really lost but some loss of energy does occur
Reason: Some of the energy is dissipated in the form of heat, sparkling etc
A Both A and R are true and R is the correct explanation of A. B Both A and R are true but R is not the correct explanation of A.
C A is true but R is false. D A is false and R is also false.
Ans: A Both A and R are true and R is the correct explanation of A.
Q261. An electric dipole is placed in an electric field generated by a point charge. 1 Mark

A The net electric force on the dipole must be zero. B The net electric force on the dipole may be zero.
C The torque on the dipole due to the field must be zero. D The torque on the dipole due to the field may be zero.
Ans: D The torque on the dipole due to the field may be zero.
Explanation:
In the uniform Electric field the net electric force on the dipole is alwas zero.

In uniform Electric field the torque on the dipole due to field may be zero.

https://bls.smartstudies.co.in/#/exam/pdf-preview/b5b87181-b79b-4b4a-8ebf-8f94fb605fc8/1 28/139
5/17/24, 8:14 PM Exam Automation

T=0
Here t Þ Torque
\text{T}=2\text{qEl}\sin\text{q }\otimes\neq0
Q262. For distance far away from centre of dipole the change in magnitude of electric field with change in distance from the centre of dipole is: 1 Mark
A Zero. B Same in equatorial plane as well as axis of dipole.
C More in case of equatorial plane of dipole as compared to axis of dipole. D More in case of axis of dipole as compared to equatorial plane.
Ans: C More in case of equatorial plane of dipole as compared to axis of dipole.
Q263. An electric line of force in the xy−plane is given by equation x2 + y2 = 1. A particle with unit positive charge, initially at rest at the point x = 1, y = 0 in the xy−plane. 1 Mark
A Not move at all. B Will move along straight line.
C Will move along the circular line of force. D Information is insufficient to draw any conclusion.
Ans: C Will move along the circular line of force.
Explanation:
Charge will move along the circular line of force because x2 + y2 = 1 is the equation of circle in xy−plane.
Q264. A solid sphere of radius R1 and volume charge density \text{p}=\frac{\text{p}_{0}}{\text{r}} is enclosed by a hollow sphere of radius R2 with negative surface 1 Mark
charge destiny \sigma_2 such that the total charge in the system is zero \text{p}_0 is positive constant and r is the distance from the centre of sphere the
ratio \frac{\text{R}_2}{\text{R}_1} is:
A \frac{\sigma}{\text{p}_0} B \sqrt{\frac{2\sigma}{\text{p}_0}}
C \sqrt{\frac{\text{p}_0}{2\sigma}} D \frac{\text{p}_0}{\sigma}
Ans: C \sqrt{\frac{\text{p}_0}{2\sigma}}
Q265. Electric field is a _______? 1 Mark
A Scalar quantity B Vector quantity
C Tensor quantity D Quantity that has properties of both scalar and vector
Ans: B Vector quantity
Explanation:
A scalar quantity is a quantity with magnitude only but no direction. But a vector quantity possesses both magnitude and direction. An electric field has a very
specific direction (away from a positive charge or towards a negative charge). Hence electric field is a vector quantity. Moreover, we have to use a vector addition
for adding two electric fields.
Q266. Current carrier in conductors is ____? 1 Mark
A Electron B Proton C Neutron D Positron
Ans: A Electron
Explanation:
Conductors have lots of free electrons that can carry electricity if the potential difference is applied across them. Protons are positively charged particle and
Neutron are electrically neutral. They don’t carry electricity in conductors.
Q267. Electric charges are distributed in a small volume. The flux of the electric field through a spherical surface of radius 10cm surrounding the total charge is 25V-m. 1 Mark
The flux over a concentric sphere of radius 20cm will be:
A 25V-m B 50V-m C 100V-m D 200V-m
Ans: A 25V-m
Explanation:
F is not depend on the shape & size of the closed volume surface.
qm Þ qm is the net charge inside the closed volume surface.
Q268. When the object has no electric charge then the object is said to be: 1 Mark
A Charged. B Electrified. C Electrically neutral. D None.
Ans: C Electrically neutral.
Q269. hoose the correct statement from the following: 1 Mark
A Electric lines of force never cross each other. B Electric lines of force end at a positive.
C The electric field charge inside a conductor are infinity. D Lines of electric field point towards regions of lower potential.
Ans: D Lines of electric field point towards regions of lower potential.
Explanation:
If the lines cross each other at a given location, then there must be two distinctly different values of electric field with their own individual direction at that given
location. This could never be the case. Therefore the lines representing the field cannot cross each other at any given location.
Electric field end at a negative charge.
Along the electric field line potential drops.
Q270. Which one of the following is similar between electrostatic force and gravitational force? 1 Mark
A Force can be attractive or repulsive. B The force depends on the medium between the bodies.
C Both the forces are strong forces. D Force is inversely proportional to the distance between the bodies.
Ans: D Force is inversely proportional to the distance between the bodies.
Explanation:
Gravitational force cannot be repulsive and it is a very weak force. Gravitational force does not depend on the medium. But both the forces are inversely
proportional to the distance between them.
Q271. Like coulomb force, ______ also satisfies superposition principle. 1 Mark
A Electric charge. B Electric field. C Both a and b. D None.

https://bls.smartstudies.co.in/#/exam/pdf-preview/b5b87181-b79b-4b4a-8ebf-8f94fb605fc8/1 29/139
5/17/24, 8:14 PM Exam Automation
Ans: B Electric field.
Q272. Quantisation of charge implies: 1 Mark
A Charge cannot be destroyed. B Charge exists on particles.
C There is a minimum permissible charge on a particle. D Charge, which is a fraction of a coulomb is not possible.
Ans: D Charge, which is a fraction of a coulomb is not possible.
Q273. For two statements are given-one labelled Assertion (A) and the other labelled Reason (R). Select the correct answer to these questions from the codes (a), (b), (c) 1 Mark
and (d) as given below.
Assertion: The charge on any body can be increased or decreased in terms of e
Reason: Quantization of charge means that the charge of a body is integral multiple of e
A Both A and R are true and R is the correct explanation of A. B Both A and R are true but R is not the correct explanation of A.
C A is true but R is false. D A is false and R is also false.
Ans: A Both A and R are true and R is the correct explanation of A.
Q274. Two identical blocks are kept on a frictionless horizontal table connected by a spring of stiffness K and of original length \ell_0 A total charge Q is distributed on the 1 Mark
block such that maximum elongation of sparing at equilibrium is equal to x value of q is:
A 2\ell_0\sqrt{4\pi\epsilon_0(\ell_0+\text{x})} B 2\text{x}\sqrt{4\pi\epsilon_0\text{k}(\ell_0+\text{x})}
C 2(\ell_0+\text{x})\sqrt{4\pi\epsilon_0\text{kx}} D (\ell_0+\text{x})\sqrt{4\pi\epsilon_0\text{kx}}
Ans: C 2(\ell_0+\text{x})\sqrt{4\pi\epsilon_0\text{kx}}
Q275. Two small spheres with masses M1​ and M2​ hang on weightless, insulating threads with lengths L1​and L2​. The two spheres carry a charge of Q1​ and Q2​respectively. 1 Mark
The spheres hang such that they are level with one another and the threads are inclined to the vertical at angles \theta_1​and \theta_2. Which of the following
conditions is required if\theta_1​=θ_2​.
A M1 = M2​ B ∣Q1​∣ = ∣Q2​∣ C L1​= L2​ D None of these
Ans: A M1 = M2​
Explanation:
\theta_1​=θ_2​,​if their masses are the same because the force of repulsion due to charges is the same.
The force Mg \sin\theta opposes the force of repulsion. Even if Q1​and Q2​ are different, the force of repulsion is the same for both as they are mutual.
Q276. An electron and a proton are both released from rest 1 meter from a large stationary negative charge, considering only the force from the large stationary negative 1 Mark
charge on the proton and electron. Which of the following is true?
A Neither the proton nor the electron will accelerate. B The initial acceleration of the proton is greater.
C Both the electron and proton have the same non-zero initial acceleration. D The initial acceleration of the electron is greater.
Ans: D The initial acceleration of the electron is greater.
Explanation:
The magnitude of electric force between electron and proton is given by \text{F}=\frac{1}{4\pi\epsilon_0}\frac{\text{e(-e)}}{\text{d}^2}
The force on each of them is equal in magnitude and is attractive.
The acceleration of a species is given by \text{a}=\frac{\text{F}}{\text{m}}
Q277. A body is positively charged, it implies that: 1 Mark
A There is only positive charge in the body. B There is positive as well as negative charge in the body, but the positive
charge is more than negative charge.
C There is equal positive and negative charge in the body, but the positive D Negative charge is displaced from its position.
charge lies in the outer regions.
Ans: B There is positive as well as negative charge in the body, but the positive charge is more than negative charge.
Q278. Refer to the arrangement of charges in Fig. and a Gaussian surface of radius R with Q at the centre. Then: 1 Mark

A Total flux through the surface of the sphere is \frac{-\text{Q}}{\epsilon_0}.. B Field on the surface of the sphere is \frac{-\text{Q}}
{4\pi\epsilon_0\text{R}^2}.
C Flux through the surface of sphere due to 5Q is zero. D Field on the surface of sphere due to -2Q is same everywhere.
Ans: A Total flux through the surface of the sphere is \frac{-\text{Q}}{\epsilon_0}.. C Flux through the surface of sphere due to 5Q is zero.
Gauss' law states that total electric flux of an enclosed surface is given by \frac{\text{q}}{\epsilon_0}.
Here, q is the net charge enclosed by the Gaussian surface.
From the figure,
Net charge inside the surface is = Q - 2Q = -Q.
Net flux through the surface of the sphere =\frac{-\text{Q}}{\epsilon_0}
Here, charge 5Q lies outside the Gaussian surface, so it will not make no contribution to electric flux through the given surface.
Q279. If the sizes of charged bodies are very small as compared to the distance between them then the charged bodies are called as: 1 Mark
A Charged objects. B Bulk of charges. C Point charges. D None.
Ans: C Point charges.
Q280. Figure shows a point charge +Q, located at a distance \frac{\text{R}}{2} from the centre of a spherical metal shell. Draw the electric field lines for the given system. 1 Mark

https://bls.smartstudies.co.in/#/exam/pdf-preview/b5b87181-b79b-4b4a-8ebf-8f94fb605fc8/1 30/139
5/17/24, 8:14 PM Exam Automation

Ans:

Q281. On charging by conduction, mass of a body may: 1 Mark


A Increase. B Decreases. C Increase or decrease. D None of these.
Ans: C Increase or decrease.
Q282. If the potential at each point on a conductor is same to each other, then: 1 Mark
A Electric lines of force may begin or end on the same conductor. B No electric lines of force may begin or end on the same conductor.
C The electric field inside the conductor may be non-zero. D None of the above.
Ans: B No electric lines of force may begin or end on the same conductor.
Explanation:
Electric field lines explain the orientation of the electric field in the region and it points tangential to the point at which it is drawn. Thus at any point, the tangent to
the electric field line matches the direction of the electric field.
The density of the field lines at a point determines the strength of the electric field at that point.
Therefore, if the potential at each point of the conductor is the same, then it means that there are no electric lines that may begin or end on the conductor.
Q283. Charge is the property associated with matter due to which it produces and experiences. 1 Mark
A Electric effects only. B Magnetic effects only.
C Both electric and magnetic effects. D None of these.
Ans: C Both electric and magnetic effects.
Q284. In winter season, a mild spark is often seen when a man touches somebody else's skin. Why? 1 Mark
A Due to lack of humidity and rubbing with clothes, charge accumulate on B Due to cold, electrostatic charge on body finds a lower resistance path to
human body which is discharged via sparking. the skin of other's body.
C The static charge on sweaters worn by the two persons is different, hence D Similar to the lightning, extremely high potential exists on both the bodies
discharge through sparking occurs. and hence they discharge through sparking.
Ans: A Due to lack of humidity and rubbing with clothes, charge accumulate on human body which is discharged via sparking.
Explanation:
Due to friction between clothes and skin, electrostatic charge is built up on skin. Hence, electrical discharge may occur when a man touches somebody else. This
phenomenon is more significant in winters because due to low humidity, charge has a tendency to stay longer on the body.
Q285. Conservation of electric charges means that the total charge of an isolated system remains ____ with time. 1 Mark
A Changed. B Unchanged. C Both a and b. D None.
Ans: B Unchanged.
Q286. Induction occurs due to ______? 1 Mark
A Movement of electron. B Leakage of charge.
C Ionization of atoms. D Uniform charge distribution.
Ans: A Movement of electron.
Explanation:
When a positive or negative charged body comes close to another body, the electrons of the second body redistribute themselves. Electrons of the second body
are attracted by the positively charged body and repelled by the negatively charged body.
Q287. If an object is positively charged, theoretically the mass of the object ______? 1 Mark
-31 -31
A Increases slightly by a factor of 9.11 × 10 kg. B Decreases slightly by a factor of 9.11 × 10 kg.
C Remains the same. D May increase or decrease.
Ans: B Decreases slightly by a factor of 9.11 × 10-31kg.
Explanation:
If an object is positively charged it loses some of its electrons. The mass of an electron is 9.11 × 10-31kg. So, if a positively charged body loses ‘n’ number of
electrons, it mass decrease by the amount n × 9.11E-31kg.
Q288. Electric flux through a spherical surface shown in the figure, is ________. 1 Mark

https://bls.smartstudies.co.in/#/exam/pdf-preview/b5b87181-b79b-4b4a-8ebf-8f94fb605fc8/1 31/139
5/17/24, 8:14 PM Exam Automation

Ans:Electric flux through a spherical surface shown in the figure, is \frac{\text{q}_2-\text{q}_1}{\varepsilon_0}.

Explanation:
Electric flux through the surface =\frac{\text{q}_\text{in}}{\varepsilon_0}
From the given figure,
\text{q}_\text{in}=+\text{q}_2-\text{q}_1
\therefore\phi=\frac{\text{q}_\text{in}}{\varepsilon_0}=\Big(\frac{\text{q}_2-\text{q}_1}{\varepsilon0}\Big)
Q289. The laws of forces that govern the force between two electric charges were discovered by: 1 Mark
A Faraday B Ampere C Ohm D Coulumb
Ans: D Coulumb
Explanation:
Coulomb stated that the force (F) of attraction or repulsion between two points charges q1​ and q2 separated by a distance r is proportional to the product of the
magnitude of the two charges, and inversely proportional to the square of the distance of separation between them. i.e., \text{F}\propto\frac{\text{q}_1\text{q}_2}
{\text{r}^2}
Q290. How many number of electrons are present in -1C of charge: 1 Mark
A 3 × 1018 electrons. B 6 × 1018 electrons.
C 1018 electrons. D None.
Ans: B 6 × 1018 electrons.
Q291. Identify the correct statement about the charges q1​and q2​: 1 Mark

A q1​ and q2​, both are positive. B q1​ and q2​, both are negative.
C q1​ is positive and q2​is negative. D q2​is positive and q1​is negative.
Ans: B q1​ and q2​, both are negative.
Explanation:
The filed of lines is outward from the positive charge and inward to the negative charge.
Here the field of lines is towards the charges q1​ and q2​. Thus, both charges are negative.
Q292. For two statements are given-one labelled Assertion (A) and the other labelled Reason (R). Select the correct answer to these questions from the codes (a), (b), (c) 1 Mark
and (d) as given below.
Assertion (A): Electric force acting on a proton and an electron, moving in a uniform electric field is the same, where as acceleration of electron is 1836 times that
of a proton.
Reason (R): Electron is lighter than proton.
A Both A and R are true, and R is the correct explanation of A. B Both A and R are true, but R is not the correct explanation of A.
C A is true but R is false. D A is false and R is also false.
Ans: A Both A and R are true, and R is the correct explanation of A.
Explanation:
As F = qE, and charge (q) on an electron and on a proton has the same magnitude, therefore force (F) on each is same. Now, acceleration \text{a}=\frac{\text{F}}
{\text{m}}.
Mass of electron =\frac{1}{1836} mass of a proton. Therefore, acceleration of electron is 1836 times that of a proton.
Q293. Two nucleons are at a separation of 1 fm. The net force between them is F1​if both are neutrons, F2​if both are protons, and F3​if one is a proton and the other is a 1 Mark
neutron then:
A F1​> F2 ​> F3​ B F3​= F1 ​> F2​
C F1​= F3 ​> F2​ D F1​= F2 ​> F3​
Ans: B F3​= F1 ​> F2​
Explanation:

https://bls.smartstudies.co.in/#/exam/pdf-preview/b5b87181-b79b-4b4a-8ebf-8f94fb605fc8/1 32/139
5/17/24, 8:14 PM Exam Automation
Two nucleons are at separation = 1fm​. The net force between them = F1​are neutrons F2​is both are protons and F3​if one is a proton and the other is a neutron then,
nuclear forces are the strong forces of attraction which hold together the nucleons (neutrons and protons) in the tiny nucleus of an atom, in spite of strong
electrostatic forces of repulsion between protons.
Nuclear forces act between a pair of neutrons, a pair of protons and also between a neutron, proton pair with the same strength.
This shows that nuclear forces are independent of charges. The attraction nuclear force is the same force any pair of nucleons.
Thus, F1​= F3​
When there are no electrostatic forces, but F2​= Nuclear force − repulsive electrostatic force.
Q294. Number of electric lines of force emanating from 1C of positive charge in vacuum is: 1 Mark
-12 9
A 8.85 × 10 B 9 × 10
C \frac{1}{4}\pi\times9\times10^{9} D 1.13 × 1011
Ans: D 1.13 × 1011
Explanation:
\phi from 1C of charge is =\frac{\text{Q}_{\text{enclosed​}}}{\epsilon_{\circ}}
=\frac{1}{8.8501\times10^{-12}}
= 1.129 × 1011
Q295. Gauss’s law is true for. 1 Mark
A Any closed surface. B For particular surfaces C Both a and b. D None.
Ans: A Any closed surface.
Q296. The value of the basic unit of the charge is: 1 Mark
A 1.6 × 10 + 19C. B 1.6 × 10 - 19C. C 1.6 × 10 - 20C. D None.
Ans: B 1.6 × 10 - 19C.
Q297. The physical quantity having SI unit NC-1 m is ________. 1 Mark
-1
Ans:The physical quantity having SI unit NC m is electric potential.
Q298. Four charges are kept at the corner points of a square. The net force on a charge kept at the center of the square is _________? 1 Mark
A Along diagonal. B Zero.
C Along one side. D Depends on the nature of the charges.
Ans: D Depends on the nature of the charges.
Explanation:
Depending on the nature and quantity of charges, the net force on the central charge may vary its magnitude and direction. If all the charges are the same the net
force will be zero.
Q299. Electric flux per unit solid angle is defined as: 1 Mark
A Electric force B Electric field intensity
C Electric potential D Electric power
Ans: B Electric field intensity
Explanation:
Electric field intensity is the strength of an electric field at any point. It is equal to the electric force per unit charge experienced by a test charge placed at that
point.
Q300. Electric lines of force about a negative point charge are: 1 Mark
A Circular anticlockwise. B Circular clockwise. C Radial and inwards. D Radial and outwards.
Ans: C Radial and inwards.
Explanation:

Q301. Force between two charges at rest is____? 1 Mark


A electrostatic force. B elector magnetic force. C real force. D dynamic force.
Ans: A electrostatic force.
Explanation:
Force between two charges at rest is electrostatic force of coulomb's law.
Q302. Does the charge given to a metallic sphere depend on whether it is hollow or solid? Give reason for your answer. 1 Mark

Ans:No, Because the charge resides only on the surface of the conductor.
Q303. In which orientation, a dipole placed in a uniform electric field is in (i) stable, (ii) unstable equilibrium? 1 Mark

Ans:When dipole is (i) parallel to field (ii) antiparallel to the field.


Q304. A point charge is brought in an electric field. The electric field at a nearby point. 1 Mark
A Will increase if the charge is positive. B Will decrease if the charge is negative.
C May increase if the charge is positive. D May decrease if the charge is negative.
Ans: C May increase if the charge is positive. D May decrease if the charge is negative.
Explanation:
The electric field at a near by point may increase if the charge is positive or may decreases if the charge is negative.
Q305. If the flux of the electric field through a closed surface is zero, then: 1 Mark
A The electric field must be zero everywhere on the surface. B The electric field may not be zero everywhere on the surface.
C The charge inside the surface must be zero. D The charge in the vicinity of the surface must be zero.
Ans: C The charge inside the surface must be zero.
Q306. Two negative charges are kept at a certain distance in the air medium. What will happen if a dielectric slab is inserted between them? 1 Mark
A The slab will get heated. B Current will flow through the slab.
C Two charges will attract each other. D The net force between the charges will be reduced.
Ans: D The net force between the charges will be reduced.
Explanation:
Force between two charges is inversely proportional to the relative permittivity of the medium between them. A dielectric slab has relative permittivity more than
that of air. So the force between the charges decreases. No current or heating will be noticed.
Q307. What is the C.G.S. unit of charge? 1 Mark

https://bls.smartstudies.co.in/#/exam/pdf-preview/b5b87181-b79b-4b4a-8ebf-8f94fb605fc8/1 33/139
5/17/24, 8:14 PM Exam Automation

A Stat Coulomb B Coulomb C EMU D Pascal


Ans: A Stat Coulomb
Explanation:
The C.G.S. unit of charge is Stat Coulomb or ESU but in the SI system the unit is Coulomb. EMU is another unit of charge in the SI system. Pascal is the unit of
pressure.
Q308. If the electric force between two unknown charges is attractive, identify which of the following statement must be true? 1 Mark
A One charge is positive and the other charge is negative. B Both charges are positive.
C Both charges are negative. D The two charges must be equal in magnitude.
Ans: A One charge is positive and the other charge is negative.
Explanation:
The direction of electric force between the charges depends upon the nature of the charges.
When the charges are of same nature, that is, both positive or both negative, the force between them is repulsive in nature.
When the charges are of opposite nature, that is, one positive and one negative, the force between them is attractive in nature.
Q309. Suppose a charge +Q1 is given to the positive plate and a charge -Q2 to the negative plate of a capacitor. What is the ''charge on the capacitor''? 1 Mark

Ans:

Given:
Charge on the positive plane = +Q1
Charge on the negative plate = -Q2
To calculate: Charge on the capacitor
Let ABCD be the Gaussian surface such that faces AD and BC lie inside plates X and Y, respectively.
Let q be the charge appearing on surface II. Then, the distribution of the charges on faces I, III and IV will be in accordance with the figure.
Let the area of the plates be A and the permittivity of the free space be∈0∈0.
Now, to determine q​ in terms of Q1 and Q2, we need to apply Gauss's law to calculate the electric field due to all four faces of the capacitor at point P. Also, we
know that the electric field inside a capacitor is zero.
Electric field due to face I at point P, \text{E}_1=\frac{\text{Q}_1-\text{q}}{2\epsilon_0\text{A}}
Electric field due to face II at point P, \text{E}_2=\frac{+\text{q}}{2\epsilon_0\text{A}}
Electric field due to face III at point P, \text{E}_3=\frac{-\text{q}}{2\epsilon_0\text{A}}
Electric field due to face IV at point P, \text{E}_4=-\Big(\frac{-\text{Q}_2+\text{q}}{2\epsilon_0\text{A}}\Big) (Negative sign is used as point P lies on the LHS of face
IV.)
Since point P lies inside the conductor,
\text{E}_1+\text{E}_2+\text{E}_3+\text{E}_4=0
\therefore\text{Q}_1-\text{q}+\text{q}-\text{q}-(-\text{Q}_2+\text{q})=0
\Rightarrow\text{q}=\frac{\text{Q}_1+\text{Q}_2}{2}
Thus, the charge on the capacitor is \frac{\text{Q}_1+\text{Q}_2}{2}, which is the charge on faces II and III.
Q310. In a region of constant electric field, the field lines are _______. 1 Mark
A Non uniformly spaced perpendicular straight lines. B Uniformly spaced perpendicular straight lines.
C Uniformly spaced parallel straight lines. D Both b and c.
Ans: C Uniformly spaced parallel straight lines.
Q311. What will be the nature of charge on the metal paper clip of electroscope when a negatively charged body is brought in contact with it? 1 Mark
A Positive B Negative C Partially positive D None of the above
Ans: B Negative
Explanation:
The nature of charge will be negative. When a negatively charged body come in context with uncharged metal paper clip then some amount of negative change
will transfer from changed body to metal clip. This phenomenon of change transfer is called Conduction.
Q312. According to Coulomb’s law, the coulomb force measured between two point charges is directly proportional to: 1 Mark
A Product of magnitude of charges. B Square of the distance between the two charges.
C Both a and b. D None.
Ans: A Product of magnitude of charges.
Q313. The Electric flux through the surface: 1 Mark

A In Fig. 1.3 (iv) is the largest. B In Fig. 1.3 (iii) is the least.
C In Fig. 1.3 (ii) is same as Fig. 1.3 (iii) but is smaller than Fig. 1.3 (iv) D Is the same for all the figures.

https://bls.smartstudies.co.in/#/exam/pdf-preview/b5b87181-b79b-4b4a-8ebf-8f94fb605fc8/1 34/139
5/17/24, 8:14 PM Exam Automation
Ans: D Is the same for all the figures.
Key concept: According to Gauss' law of electrostatics, the total electric flux out of a closed surface is equal to the charge enclosed divided by the permittivity,
\text{i.e.,}\phi=\frac{\text{Q}_\text{enclosed}}{\epsilon_0}
Thus, electric flux through a surface doesn't depend on the shape, size or area of a surface but it depends on the amount of charge enclosed by the surface.
In given figures the charge enclosed are same that means the electric flux through all the surfaces should be the same. Hence option (d) is correct.
Q314. A region surrounding a stationary electric dipoles has: 1 Mark
A Magnetic field only. B Electric field only.
C Both electric and magnetic fields. D No electric and magnetic fields.
Ans: B Electric field only.
Q315. Mark out the correct options. 1 Mark
A The total charge of the universe is constant. B The total positive charge of the universe is constant.
C The total negative charge of the universe is constant. D The total number of charged particles in the universe is constant.
Ans: A The total charge of the universe is constant.
Explanation:
The total charge (Positive + negative) oif the universe is constant.
Q316. The electric field in a region is directed outward and is proportional to the distance r from the origin. Taking the electric potential at the origin to be zero, 1 Mark
A It is uniform in the region. B It is proportional to r.
C It is proportional to r2.​​ D It increases as one goes away from the origin.
Ans: C It is proportional to r2.​​
Explanation:
\text{DV}=-\text{E}.\text{dr}
Given (\text{E}\propto\text{r})
(\text{V}-0)=-\text{E}.\text{dr}
þ\text{E}\propto\text{r}^2
Q317. The formation of a dipole is due to two equal and dissimilar point charges placed at a: 1 Mark
A Short distance. B Long distance. C Above each other. D None of these.
Ans: A Short distance.
Q318. The total charge of the isolated system is always: 1 Mark
A Not conserved. B Conserved. C Both a and b. D None.
Ans: B Conserved.
Q319. For two statements are given-one labelled Assertion (A) and the other labelled Reason (R). Select the correct answer to these questions from the codes (a), (b), (c) 1 Mark
and (d) as given below.
Assertion (A): ln electrostatics, electric lines of force can never be closed loops, as a line can never start and end on the same charge.
Reason (R): The number of electric lines of force originating or terminating on a charge is proportional to the magnitude of charge.
A Both A and R are true, and R is the correct explanation of A. B Both A and R are true, but R is not the correct explanation of A.
C A is true but R is false. D A is false and R is also false.
Ans: B Both A and R are true, but R is not the correct explanation of A.
Explanation:
Electrostatic field lines of force can never form any closed loop. Because electric field originate from positive charge and terminates on negative charge.
Q320. A charge Q is kept at the centre of a circle of radius r. A test charge q0 is carried from a point X to the point Y on this circle such that arc XY subtends an angle of 1 Mark
60° at the centre of the circle. The amount of work done in this process will be:
A \frac{1}{4\pi\varepsilon_0}\frac{\text{Qq}_0}{2\text{r}} B \frac{1}{4\pi\varepsilon_0}\frac{\sqrt3\text{Qq}_0}{2\text{r}}
C \text{Zero} D \frac{1}{4\pi\varepsilon_0}\frac{\sqrt3\text{Qq}_0}{\text{r}}
Ans: C \text{Zero}
Explanation:
Work done will be 0.
Q321. The materials having high resistance to the passage of electricity are called as: 1 Mark
A Conductors. B Insulators. C Semiconductors. D None.
Ans: B Insulators.
Q322. A sample of HCI gas is placed in an electric field of 2.5 × 104NC-1. The dipole moment of each HCI molecule is 3.4 × 10-30Cm. Find the maximum torque that can 1 Mark
act on a molecule.

Ans:\text{E}=2.5\times104
\text{P}=3.4\times10^{-30}
\text{T}=\text{PE}\sin\theta
=\text{P}\times\text{E}\times1
=3.4\times10^{-30}\times2.5\times10^4
=8.5\times10^{-26}\text{Nm}
Q323. A thin insulator rod is placed between two unlike point charges +q1​ and q2​. For this situation tick the correct alternative (s): 1 Mark

A The total force acting on charge +q1​will increase. B The total force acting on charge −q2​ will increase.
C The total force acting on charge −q2​will decrease. D The force acting on charge +q1​ due to −q2​will remain same.
Ans: D The force acting on charge +q1​ due to −q2​will remain same.
Explanation:
Without rod the force between two charges is\text{F}=\frac{\text{q}_1(-\text{q}_2)}{4\pi\epsilon_0\text{r}} where \epsilon0= permittivity of free space
When rod inserted between charges, the force\text{F}=\frac{\text{q}_1(-\text{q}_2)}{4\pi\epsilon} where \epsilon= permittivity of insulator medium.
Since \epsilon_0​>\epsilon\text{so F}′>\text{F}
Q324. The solid angle subtended by the total surface area of a sphere at the centre is: 1 Mark
A 4\pi B 2\pi C \pi D 3\pi
Ans: A 4\pi
Explanation:
A steradian is the solid angle subtended at the center of a sphere of radius r by a section of its surface area of magnitude equal to r2. Since the surface area
is 4\pi\text{r}^2, there are 4\pi steradians surrounding a point in space.

https://bls.smartstudies.co.in/#/exam/pdf-preview/b5b87181-b79b-4b4a-8ebf-8f94fb605fc8/1 35/139
5/17/24, 8:14 PM Exam Automation
Q325. How does the electric flux due to a point charge enclosed by a spherical Gaussian surface get affected when its radius is increased? 1 Mark

Ans:Electric flux remains unaffected.


Q326. The SI unit of electric flux is: 1 Mark
A V/m. B V. C Vm. D None.
Ans: C Vm.
Q327. Number of electrons in 1 Coulomb charge is _________? 1 Mark
21 20 25 23
A 6.25 × 10 B 6.25 × 10 C 6.25 × 10 D 6.25 × 10
21
Ans: A 6.25 × 10
Explanation:
Electron has the charge of 1.602 × 10-19 Coulomb. So, the number of electrons in 1 Coulomb of charge is \frac{1}{1.602\times10^{-19}}=6.25\times10^{21}
This huge number of electrons can be accumulated to form 1C of charge.
Thus, we can conclude that we need Avogadro’s number of electrons (approximately) to get 100C charges.
Q328. The molecules having permanent electric dipole moment even in the absence of electric field are called as: 1 Mark
A Non polar molecules. B Polar molecules. C Both a and b. D None.
Ans: B Polar molecules.
Q329. A large nonconducting sheet M is given a uniform charge density. Two uncharged small metal rods A and B are placed near the sheet as shown in figure: 1 Mark

A M attracts A. B M attracts B. C A attracts B. D B attracts A.


Ans: D B attracts A.
Explanation:

Charge distribution or M, A & B


So, we can say that M attracts A, M attracts B, A attracts B and 'B' attracts A.
Q330. If two charged particles of same mass and charge are projected in a uniform electric field with the same speed, then? 1 Mark
A Both have same momentum at any instant. B Both have same kinetic energy at any instant.
C Both have same magnitude of momentum at any instant. D They may move on a straight line.
Ans: D They may move on a straight line.
Q331. The fact that electric charge is always an integral multiple of e is termed as: 1 Mark
A Conservation of charge. B Quantisation of charge. C Both a and b. D None.
Ans: B Quantisation of charge.
Q332. The SI unit of electric field is: 1 Mark
A Vm. B V/m. C V. D None.
Ans: B V/m.
Q333. The surface that we choose for application of Gauss’s law is called as: 1 Mark
A Gaussian surface. B Electrical surface. C Electrostatic surface. D None.
Ans: A Gaussian surface.
Q334. If a body is charged by rubbing it, its weight: 1 Mark
A Always decreases slightly. B Always increases slightly.
C May increase slightly or may decrease slightly. D Remains precisely the same.
Ans: C May increase slightly or may decrease slightly.
Explanation:
If a body is charged by rubbing it, then it may lose or gain electrons. Since electrons have a mass of (9.1 × 10-31kg). So, a slight weight may increase or decrease
slightly.
Q335. An electric field can deflect: 1 Mark
A Neutrons. B X-rays.
C \gamma-\text{rays}. D \alpha-\text{particles}.
Ans: D \alpha-\text{particles}.
Q336. Coulomb force and gravitational force follows the: 1 Mark
A Conservation law. B Inverse square law. C Both a and b. D None.
Ans: B Inverse square law.

https://bls.smartstudies.co.in/#/exam/pdf-preview/b5b87181-b79b-4b4a-8ebf-8f94fb605fc8/1 36/139
5/17/24, 8:14 PM Exam Automation
Q337. An electroscope is used: 1 Mark
A To detect presence of charge on a body. B To determine the nature of charge on a body.
C To measure the exact amount of charge on a body. D For both A and B.
Ans: D For both A and B.
Explanation:
Electroscope is a device which is use to detect whether a body is carrying charge and also to detect the nature of charge on the body.
Q338. Figure shows a charge q placed at the centre of a hemisphere. A second charge Q is placed at one of the positions A, B, C and D. In which position(s) of this 1 Mark
second charge, the flux of the electric field through the hemisphere remains unchanged?

A A B B C C D D
Ans: A A C C
Explanation:

The flux of electric field through the hemishpere remains unchanged dur to 'A & C' point because these point lies on the line joining the centre of hemisphere.
Q339. A positively charged rod is brought near an uncharged conductor. If the rod is then suddenly withdrawn, the charge left on the conductor will be: 1 Mark
A Positive. B Negative C Zero. D Cannot say.
Ans: C Zero.
Q340. A point charge is kept at the centre of metallic insulated spherical shell. Then: 1 Mark
A Electric field out side the sphere is zero. B Electric field inside the sphere is zero.
C Net induced charge on the sphere is zero. D Electric potential inside the sphere is zero.
Ans: C Net induced charge on the sphere is zero.
Q341. When a body is charged by induction, then the body. 1 Mark
A Becomes neutral. B Does not lose any charge.
C Loses whole of the charge on it. D Loses part of the charge on it.
Ans: B Does not lose any charge.
Q342. Additivity of the electric charges means that the total charge of a system is the _____ of all individual charges in the system. 1 Mark
A Vector sum. B Algebraic sum. C Vector difference. D None.
Ans: B Algebraic sum.
Q343. Electric field lines of force: 1 Mark
A Exist everywhere. B Exist only in the immediate vicinity of electric charges.
C Exist only when both positive and negative charges are near one another. D Are imaginary.
Ans: D Are imaginary.
Explanation:
Electric lines of force are imaginary.
Q344. Electroscope is used: 1 Mark
A To detect and test small electric charges. B To calculate the amount of electric charge flowing through the conductor in
the given interval of time.
C To find out the presence of antimatter. D To test the presence of magnetic field.
Ans: A To detect and test small electric charges.
Explanation:
Electroscope is an old electrical instrument, used to detect the presence and magnitude of electric charges on a body.

https://bls.smartstudies.co.in/#/exam/pdf-preview/b5b87181-b79b-4b4a-8ebf-8f94fb605fc8/1 37/139
5/17/24, 8:14 PM Exam Automation

Q345. Why is gold used in the Gold-leaf electroscope? 1 Mark


A Gold is easily available in nature. B Gold is malleable.
C Gold is conducting in nature. D Gold is cheap.
Ans: B Gold is malleable.
Explanation:
Though gold is a costly metal it is used in electroscope because of the property malleability. This means very thin and light sheets can be formed from gold simply
by hammering or rolling and hence the deflection of the light gold plates increases.
Q346. Coulomb's Law is true for: 1 Mark
A Atomic distances (=10-11m) B Nuclear distances (=10-15m)
C Charged as well as uncharged particles D All the distances
Ans: D All the distances
Explanation:
Coulomb's law is true for all distances whether it is small and large. Hence it is called a long range force.
Coulomb's force \text{F}=\frac{\text{q}_1\text{q}_2}{4\pi\epsilon_\circ\text{r}^2}
\Rightarrow\text{F}\propto\frac{1}{\text{r}^2}
Q347. Field lines start from____ charges and ends at _____ charges. 1 Mark
A Negative, positive. B Positive, negative. C Positive, positive. D Negative, negative.
Ans: B Positive, negative.
Q348. Two charges of +1 μ\text{C} +5 μ\text{C} are placed 4cm apart, the ratio of the force exerted by both charges on each other will be- 1 Mark
A 1:1 B 1:5 C 5:1 D 25 : 1
Ans: A 1 : 1
Explanation:
From Newton's Third law of motion, force exerted by charge 1μ\text{C} on.
Charge 5μ\text{C} is equal to the force by charge 5μ\text{C} on charge.
1μ\text{C} in magnitude.
Q349. Two charges are placed at certain distance apart. A metallic sheet is placed between them. What will happen to the force between the charges? 1 Mark
A Increases B Decreases
C Remains the same D May increase or decrease depending upon the shape of the metallic sheet
Ans: B Decreases
Explanation:
Force will decreases because when medium is inserted between then it will reduces the forces by its permitivity. the force becomes
\text{Fe}_\text{m}=\frac{\text{Fe}}{\epsilon_\text{r}}
Q350. The three basic properties possessed by the electric charge are: 1 Mark
A Quantisation. B Additivity. C Conservation. D All.
Ans: D All.
Q351. For two statements are given-one labelled Assertion (A) and the other labelled Reason (R). Select the correct answer to these questions from the codes (a), (b), (c) 1 Mark
and (d) as given below.
Assertion (A): A point charge is brought in an electric field. The field at a nearby point is increase, whatever be the nature of the charge.
Reason (R): The electric field is independent of the nature of charge.
A Both A and R are true, and R is the correct explanation of A. B Both A and R are true, but R is not the correct explanation of A.
C A is true but R is false. D A is false and R is also false.
Ans: D A is false and R is also false.
Explanation:
Electric field at the nearby point will be resultant of existing field and field due to the charge brought. lt may increase or decrease if the charge is positive or
negative depending on the position of the point with respect to the charge brought.
Q352. Two point charges of the same polarities are hung with the help of two threads and kept close. The angle between the threads will be _________ if the system is 1 Mark
taken to space.
A 180 degree B 90 degree C 45 degree D 60 degree
Ans: A 180 degree
Explanation:
There is gravitational field on earth, so if we hang the two same charges there will be an interaction of vertical gravitational field and horizontal electric field. The
system will achieve equilibrium by creating a certain angle between the threads and hence the vertical and horizontal components of forces will balance. But in
space, there is no gravity. So the charges will be at 180-degree separation.
Q353. Figure, shows some of the electric field lines corresponding to an electric field. The figure suggests that, 1 Mark

A E A > E8 > Ee B E A = EB = Ee
C E A = Ec > EB D E A = Ec < EB .
Ans: C EA = Ec > EB
Explanation:
Higher separation, Lower electric field. Because Electric field inversly proportional the square of separation.

https://bls.smartstudies.co.in/#/exam/pdf-preview/b5b87181-b79b-4b4a-8ebf-8f94fb605fc8/1 38/139
5/17/24, 8:14 PM Exam Automation
E A = Ec > EB
Q354. There exists an electric field of 1N/ C along Y direction. The flux passing through the square of 1m placed in XY plane inside the electric field is? 1 Mark
2 2 2
A 1.0Nm / C B 10.0Nm / C C 2.0Nm / C D Zero
Ans: D Zero
Explanation:
The flux passing through the square of 1m placed in XY plane inside the electric field is zero because by Gauss theorem we can say here closed circuit not formed.
Q355. The constant k in Coulomb's law depends upon: 1 Mark
A Nature of medium B System of units C Intensity of charge D Both (a) and (b)
Ans: A Nature of medium
Explanation:
The value of \text{K}=\frac{1}{4\pi\epsilon_0}=8.854\times10^{-12}\text{C}^2\text{N}^{-1}\text{m}^{-2}
Where ε0​is permittivity of free space.
Q356. For two statements are given-one labelled Assertion (A) and the other labelled Reason (R). Select the correct answer to these questions from the codes (a), (b), (c) 1 Mark
and (d) as given below.
Assertion (A): Charging is due to transfer of electrons.
Reason (R): Mass of a body decreases slightly when it is negatively charged.
A Both A and R are true, and R is the correct explanation of A. B Both A and R are true, but R is not the correct explanation of A.
C A is true but R is false. D A is false and R is also false.
Ans: C A is true but R is false.
Explanation:
A body becomes negatively charged only when some electrons are transferred to the body i.e. the body gains some electrons. Hence its mass increases slightly.
Mass of a body decreases only when body gives some electrons to some other body.
Q357. If an object possesses an electric charge, it is said to be electrified or ... A ... When it has no charge, it is said to be ... B ... Here, A and B refer to: 1 Mark
A Charged, neutral. B Neutral, charged. C Discharged, charged. D Active, reactive.
Ans: A Charged, neutral.
Q358. Force on any charge due to a number of other charges is the _____ of all the forces on that charge taken one at a time. 1 Mark
A Vector difference. B Vector sum. C Both a and b. D None.
Ans: B Vector sum.
Q359. For two statements are given-one labelled Assertion (A) and the other labelled Reason (R). Select the correct answer to these questions from the codes (a), (b), (c) 1 Mark
and (d) as given below.
Assertion (A): Sharper is the curvature of spot on a charged body lesser will be the surface charge density at that point.
Reason (R): Electric field is non-zero inside a charged conductor.
A Both A and R are true, and R is the correct explanation of A. B Both A and R are true, but R is not the correct explanation of A.
C A is true but R is false. D A is false and R is also false.
Ans: D A is false and R is also false.
Explanation:
Surface of a charged conductor is always an equipotential surface, whatever may be its shape. Hence, \sigma\text{R}= constant, at every point on the surface of
charged conductor i.e. at the sharpest point (R → 0) of the surface, charge density will be maximum. A uniformly charged conductor exerts no electrostatic force
on a point charge located anywhere inside the conductor or electric field is zero.
Q360. The electric field lines are crowded where electric field is: 1 Mark
A Strong. B Weak. C Moderate. D None.
Ans: A Strong.
Q361. Two equally charged spheres attract each other. Now if they touch each other then they will _____? 1 Mark
A Attract each other with the same amount of force. B Repel each other.
C Attract each other with less amount of force. D Don’t attract or repel.
Ans: D Don’t attract or repel.
Explanation:
The two spheres are oppositely charged by the same amount. So if they come in contact, both of their charges are neutralized. So they become uncharged and
don’t attract or repel each other.
Q362. What happens to the plates of the apparatus if we measure alternating charge using a Gold-leaf oscilloscope? 1 Mark
A It doesn’t diverge at all. B It diverges momentarily.
C The plates give a proper divergence. D The degree of divergence increases and decreases repeatedly.
Ans: C The plates give a proper divergence.
Explanation:
The divergence of the plates of the Gold-leaf oscilloscope depends only on the presence of a charge, not on the quality of charge i.e. positive or negative. So, if
the charge changes from positive to negative and vice versa the degree of divergence of the plates remains the same.
Q363. The charge on electron is: 1 Mark
A +e B -e C 1/e D -1/e
Ans: B -e
Q364. If a positively charged sphere is taken close to another uncharged sphere then which of the following statements is true? 1 Mark
A Induction and attraction occur simultaneously. B Induction occurs before the attraction.
C Attraction occurs before induction. D Attraction or repulsion may occur.
Ans: B Induction occurs before the attraction.
Explanation:
If two bodies are taken close to each other, a positively charged body induces a negative charge on another body and then they attract each other. So, induction
occurs before attraction. This phenomenon is also true for magnetic induction.
Q365. For two statements are given-one labelled Assertion (A) and the other labelled Reason (R). Select the correct answer to these questions from the codes (a), (b), (c) 1 Mark
and (d) as given below.
Assertion (A): For charge to be in equilibrium, sum of the forces on charge due to rest of the two charges must be zero.
Reason (R): A charge is lying at the centre of the line joining two similar charges each which are fixed. The system will be in equilibrium if that charge is one fourth
of the similar charges.
A Both A and R are true, and R is the correct explanation of A. B Both A and R are true, but R is not the correct explanation of A.
C A is true but R is false. D A is false and R is also false.
Ans: C A is true but R is false.
Explanation:
According to Coulomb's law, \text{F}=\text{k}\frac{\text{Q}_1\text{Q}_2}{\text{r}^2}.
The force on q due to A,
\text{F}_\text{A}=\frac{1}{4\pi\epsilon_0}\cdot\frac{\text{Qq}}{(\frac{\text{r}}{2})^2} to the right
https://bls.smartstudies.co.in/#/exam/pdf-preview/b5b87181-b79b-4b4a-8ebf-8f94fb605fc8/1 39/139
5/17/24, 8:14 PM Exam Automation

Due to B,
\text{F}_\text{B}=\frac{1}{4\pi\epsilon_0}\cdot\frac{\text{Qq}}{(\frac{\text{r}}{2})^2} to the right
\therefore Their sum is zero whether q is +\frac{\text{Q}}{4}or -\frac{\text{Q}}{4} or any other value.
Therefore, it is not true that the third charge has to be \frac{\text{Q}}{4} only. It can be any value.
Q366. Charge is: 1 Mark
A Transferable. B Associated with mass. C Conserved. D All of these.
Ans: D All of these.
Q367. In a region of space having a uniform electric field E, a hemispherical bowl of radius r is placed. The electric flux \phi through the bowl is: 1 Mark
A 2\pi\text{R}\text{E} B 4\pi\text{R}^2\text{E}
C 2\pi\text{R}^2\text{E} D \pi\text{R}^2\text{E}
Ans: C 2\pi\text{R}^2\text{E}
Explanation:
\phi=\text{E}(\text{ds})\cos\theta=\text{E}(2\pi\text{r}^2)\cos0^{\circ}=2\pi\text{r}^2\text{E}
Q368. Define the term ‘dielectric constant’ of a medium in terms of capacitance of a capacitor. 1 Mark

Ans:Dielectric constant of a medium may be defined as the ratio of capacitance of a capacitor with dielectric between its plates to its capacitance with its plates
separated in vacuum.
Alternate Answer
\text{k}=\frac{c}{c_{\circ}}
C = Capacitance with dielectric as medium.
Co = Capacitance with vacuum as medium.
Q369. An electric dipole is placed at the centre of a sphere then: 1 Mark
A The flux of the electric field through the sphere is not zero. B The electric field is zero at every point of the sphere.
C The electric field is not zero anywhere on the sphere. D The electric field is zero on a circle on the sphere
Ans: C The electric field is not zero anywhere on the sphere.
Q370. For two statements are given-one labelled Assertion (A) and the other labelled Reason (R). Select the correct answer to these questions from the codes (a), (b), (c) 1 Mark
and (d) as given below.
Assertion (A): If there exists coulomb attraction between two bodies, both of them may not be charged.
Reason (R): ln coulomb attraction two bodies are oppositely charged.
A Both A and R are true, and R is the correct explanation of A. B Both A and R are true, but R is not the correct explanation of A.
C A is true but R is false. D A is false and R is also false.
Ans: B Both A and R are true, but R is not the correct explanation of A.
Explanation:
Coulomb attraction exists even when one body is charged, and the other is uncharged.
Q371. When a glass rod is rubbed with silk, what kind of charge will be developed on glass? 1 Mark
A B C D Cannot predict.

Ans: A

Explanation:
According to electrostatic series, glass is situated above silk. So, if the glass is rubbed with silk, the positive charge on glass and negative charge on silk is
produced. Similarly, we can predict what is the nature of charge if we rub two di-electric substances together.
Q372. Unlike charges _____ each other. 1 Mark
A Attacks. B Repels. C Both a and b. D None.
Ans: A Attacks.
Q373. Electric lines of force about a positive point charge are: 1 Mark
A Radially outwards. B Circular clockwise.
C Radially inwards. D Parallel straight lines.
Ans: A Radially outwards.
Explanation:
Electric lines of force about a positive point charge are always radially outwards.

Q374. In electrolytes, the mobile charges are: 1 Mark

https://bls.smartstudies.co.in/#/exam/pdf-preview/b5b87181-b79b-4b4a-8ebf-8f94fb605fc8/1 40/139
5/17/24, 8:14 PM Exam Automation

A Positive ions. B Negative ions. C Both a and b. D None.


Ans: C Both a and b.
Q375. When a comb rubbed with dry hair attracts pieces of paper. This is because the. 1 Mark
A Comb polarizes the piece of paper. B Comb induces a net dipole moment opposite to the direction of field.
C Electric field due to the comb is uniform. D Comb induces a net dipole moment perpendicular to the direction of field.
Ans: A Comb polarizes the piece of paper.
Q376. An electron falls from rest through a vertical distance h in a uniform and vertically upward directed electric field E. The direction of electric field is now reversed, 1 Mark
keeping its magnitude the same. A proton is allowed to fall from rest in it through the same vertical distance h. The time of fall of the electron, in comparison to the
time of fall of the proton is:
A Smaller. B 5 times greater. C Equal. D 10 times greater.
Ans: A Smaller.
Q377. An electric dipole has a pair of equal and opposite point charges q and –q separated by a distance 2x. The axis of the dipole is: 1 Mark
A From positive charge to negative charge. B From negative charge to positive charge.
C Perpendicular to the line joining the two charges rawn at the centre and D Perpendicular to the line joining the two charges drawn at the centre and
pointing upward direction. pointing downward direction.
Ans: B From negative charge to positive charge.
Q378. Draw the pattern of electric field lines, when a point charge –Q is kept near an uncharged conducting plate. 1 Mark

Ans:

Q379. A hemisphere shell is uniformly charged positively. The electric field at a point on a diameter away from the centre (inside the boundary of hemisphere shell) is 1 Mark
directed:
A Perpendicular to the diameter. B Parallel to the diameter.
C At an angle tilted towards the diameter. D At an angle tilted away from the diameter.
Ans: A Perpendicular to the diameter.
Explanation:
It has to be perpendicular as when you join the other half to make complete sphere, only this orientation of the electric field will give a vector sum equal to 0, inside
the sphere.
Q380. NaCl molecule is bound due to the electric force between the sodium and the chlorine ions when one electron. of sodium is transferred to chlorine. Taking the 1 Mark
separation between the ions to be 2.75 × 10cm, find the force of attraction between them· State the assumptions (if any) that you have made.

Ans:\text{F}=\frac{\text{kq}_1\text{q}_2}{\text{r}^2}
=\frac{9\times10^9\times1.6\times1.6\times10^{-19}\times10^{-19}}{(2.75\times10^{-10})^2}
=\frac{23.04\times10^{-29}}{7.56\times10^{-20}}
=3.04\times10^{-9}
Q381. For two statements are given-one labelled Assertion (A) and the other labelled Reason (R). Select the correct answer to these questions from the codes (a), (b), (c) 1 Mark
and (d) as given below.
Assertion (A): The electric tines of forces diverges from a positive charge and converge at a negative charge.
Reason (A): A charged particle free to move in an electric field always move along an electric line of force.
A Both A and R are true, and R is the correct explanation of A. B Both A and R are true, but R is not the correct explanation of A.
C A is true but R is false. D A is false and R is also false.
Ans: C A is true but R is false.
Explanation:
If the charged particle is initially at rest in an electric field, it will move along the electric line of force. But when the initial velocity of charged particle makes some
angle with the line of force then the resultant path is not along the line of force. Because electric line of force may not coincide with the line of velocity of the
charge.
Q382. If the net electric flux through a closed surface is zero, then we can infer: 1 Mark
A No net charge is enclosed by the surface. B Uniform electric field exists within the surface.
C Electric potential varies from point to point inside the surface. D Charge is present inside the surface.
Ans: A No net charge is enclosed by the surface.
Explanation:
Net electric flux is highly dependent on the net electric charge enclosed by the surface.
In a closed surface, if the net electric flux is zero, then the net electric charge will be also zero.
Since electric flux is defined as the rate of flow of electric field in a closed area and if the electric flux is zero, the overall electric charge within the closed
boundary will be also zero.
Q383. Electric flux over a surface in an electric field may be: 1 Mark
A Positive. B Negative. C Zero. D All of the above.
Ans: D All of the above.
Q384. 1 Mark

https://bls.smartstudies.co.in/#/exam/pdf-preview/b5b87181-b79b-4b4a-8ebf-8f94fb605fc8/1 41/139
5/17/24, 8:14 PM Exam Automation
Two charges of magnitudes – 2Q and + Q are located at points (a, 0) and (4a, 0) respectively. What is the electric flux due to these charges through a sphere of
radius ‘3a’ with its centre at the origin?

Ans:Electric flux,\varphi = \frac{-2\text{Q}}{\varepsilon_{0}}.


Concept:
1. Mark the position of the charges on number line.
2. Draw a sphere of radius 3a about the origin and observe that which charge is inside the sphere, and then use Gauss theorem.

Q385. Two point charges q1 and q2 are situated at a distance d. There is no such point in between them where the electric field is zero. What can we deduce? 1 Mark
A There is no such point. B The charges are of the same polarity.
C The charges are of opposite polarity. D The charges must be unequal.
Ans: C The charges are of opposite polarity.
Explanation:
If both the charges are of the same polarity (maybe of unequal magnitude), there must be a point in between them where the electric field intensities of the
charges are of equal magnitude and in opposite direction. Hence they balance each other and the net field intensity must be zero. But if the charges are of
opposite polarities their field intensities aid each other and net field intensity can never be zero.
Q386. 1 milli coulomb = 1 Mark
A 103C B 10C C 10 - 2C D 10 - 3C
Ans: D 10 - 3C
Q387. The mass of the body is: 1 Mark
A Always positive. B Negative. C Both a and b. D None.
Ans: A Always positive.
Q388. In two positive charges q2 and q3 fixed along the y axis, exert a net electric force in the +x direction on a charge q1 fixed along the x axis. If a positive charge Q is 1 Mark
added at (x, 0), the force on q1.

A Shall increase along the positive x-axis. B Shall decrease along the positive x-axis.
C Shall point along the negative x-axis. D Shall increase but the direction changes because of the intersection of Q
with q2 and q3.
Ans: A Shall increase along the positive x-axis.
Net force on charge q1, by other charges q2 and q3 is along the + x-direction, so nature of force between q1 & q2 and q1 & q3 is attractive. This is possible when
charge q1 is negative.
Now, if a positive charge Q is placed at (x, 0), then, the force on q1 shall increase. The direction will be along positive x axis.
Q389. Two point charges + Q and + q are separated by a certain distance. If + Q > + q then in between the charges the electric field is zero at a point. 1 Mark
A Closer to + Q. B Closer to + q.
C Exactly at the mid-point of line segment joining + Q and + q. D No where on the line segment joining + Q and + q.
Ans: B Closer to + q.
Q390. A charged conductor has its charge only on its outer surface. This statement is true for which of the following? 1 Mark
A For all conductors. B Only for spherical conductors.
C For hollow conductors. D For those conductors which don’t have sharp edges.
Ans: A For all conductors.
Explanation:
Charge remains on the outer surface of a conductor, irrespective of the shape and size of the conductor and also for hollow and solid conductors both. But if there
is a sharp edge in the conductor, surface charge density will be more at that point. The surface charge density is uniform in the case of a sphere.
Q391. For two statements are given-one labelled Assertion (A) and the other labelled Reason (R). Select the correct answer to these questions from the codes (a), (b), (c) 1 Mark
and (d) as given below.
Assertion: The positively charged particle is placed in front of a spherical uncharged conductor. The number of lines forces terminating on the spare will be more
than those emerging from it
Reason: The surface charge density at a point on the sphere nearest to the point charge will be negative and maximum in magnitude compared to other points on
the sphere
A Both A and R are true and R is the correct explanation of A. B Both A and R are true but R is not the correct explanation of A.
C A is true but R is false. D A is false and R is also false.
Ans: D A is false and R is also false.
Q392. The direction of electric field is from: 1 Mark
A Positive to negative plate. B Negative to positive plate.
C Both a and b. D None.
Ans: A Positive to negative plate.
Q393. An arbitrary surface encloses a dipole. What is the electric flux through this surface? 1 Mark

Ans:Zero.
According to Gauss, law, the electric flux through an enclosed surface is given
by \oint_\text{s}\overrightarrow{\text{E}}.\text{d}\overrightarrow{\text{S}}=\frac{\text{q}_\text{enclosed}}{\epsilon_0}
The net charge on a dipole is given by -q + q = 0, hence qenclosed = 0
Hence the electric flux through a surface enclosing dipole =\frac{\text{-q}+\text{q}}{\epsilon_0}=\frac{\text{q}}{\epsilon_0}=0.

https://bls.smartstudies.co.in/#/exam/pdf-preview/b5b87181-b79b-4b4a-8ebf-8f94fb605fc8/1 42/139
5/17/24, 8:14 PM Exam Automation
Q394. An electron enters an electric field with its velocity in the direction of the electric lines of field then: 1 Mark
A The path of the electron will be a circle. B The path of the electron will be a parabola.
C The velocity of the electron will decrease just after entry. D The velocity of the electron will increase just after entry.
Ans: C The velocity of the electron will decrease just after entry.
Explanation:
When an electron enters an electric field with its velocity in the direction of the electric lines of field, then the velocity of the electron will decrease just after entry
due to the lines of force of the electric field.
Q395. Two identically charged spheres when suspended by strings of equal lengths make an angle of 30∘ with each other. The angle remains the same when they are 1 Mark
immersed in a liquid of density less than the density of the material of the spheres. Choose the correct option
A The electric force between them increases. B The electric force between them decreases.
C The net downward force will increase. D The net downward force will remain unchanged.
Ans: B The electric force between them decreases.
Q396. The ratio of electric force and gravitational force between a proton and electron is: 1 Mark
-39 39 39
A 2.4 × 10 B 2.4 × 10 C 2.4 D 10
39
Ans: B 2.4 × 10
Q397. The Electric field at a point is: 1 Mark
A Always continuous. B Continuous if there is no charge at that point.
C Discontinuous only if there is a negative charge at that point. D Discontinuous if there is a charge at that point.
Ans: B Continuous if there is no charge at that point. D Discontinuous if there is a charge at that point.
(b, d) We cannot define electric field at the position of a charge, so we cannot say that electric field is always continuous.
Hence option (a) is ruled out and option (d) is the correct choice.
The electric field due to any charge will be continuous, if there is no other charge in the medium. It will be discontinuous if there is a charge at the point under
consideration, hence option (b) is correct.
Q398. The materials which allow to pass electricity through them easily are called as: 1 Mark
A Insulators. B Conductors. C Semiconductors. D Superconductors.
Ans: B Conductors.
Q399. The electric field inside a hollow conducting sphere: 1 Mark
A Increases towards the centre. B Decreases towards the centre.
C Is finite and constant throughout. D Is zero.
Ans: D Is zero.
Explanation:
As there are no charges inside the hollow conducting sphere, as all charges reside on it surface. So, electric field inside the hollow conducting sphere is zero.
Q400. The field lines of a single negative charge are: 1 Mark
A Radially outward. B Radially inward. C Both a and b. D None.
Ans: B Radially inward.
Q401. The dimensions of surface charge density is: 1 Mark
A [L-2 T A]. B [L-1 T A]. C [L T A]. D None.
Ans: A [L-2 T A].
Q402. What is the net flux of the uniform electric field of Exercise 1.15 through a cube of side 20 cm oriented so that its faces are parallel to the coordinate planes? 1 Mark

Ans:All the faces of a cube are parallel to the coordinate axes. Therefore, the number of field lines entering the cube is equal to the number of field lines piercing out of
the cube. As a result, net flux through the cube is zero.
Q403. In 1959 Lyttleton and Bondi suggested that the expansion of the Universe could be explained if matter carried a net charge. Suppose that the Universe is made up 2 Marks
of hydrogen atoms with a number density N, which is maintained a constant. Let the charge on the proton be: ep = -(1 + y)e where e is the electronic charge.
Show that the velocity of expansion is proportional to the distance from the centre.

Ans:Net force experience bt the hydrogen atom is given by


\text{F}=\text{F}_\text{C}-\text{F}_\text{G}=\frac{1}{3}\frac{\text{Ny}^2\text{e}^2\text{R}}{\epsilon_0}-\frac{4\pi}{3}\text{Gm}_\text{p}^2\text{NR}
Because of this net force, the hydrogen atom experiences an acceleration such that
\text{m}_\text{p}\frac{\text{d}^2\text{R}}{\text{dt}^2}=\text{F}=\frac{1}{3}\frac{\text{Ny}^2\text{e}^2\text{R}}{\epsilon_0}-\frac{4\pi}
{3}\text{Gm}_\text{p}^2\text{NR}
=\Big(\frac{1}{3}\frac{\text{Ny}^2\text{e}^2}{\epsilon_0}-\frac{4\pi}{3}\text{Gm}_\text{p}^2\text{N}\Big)\text{R}
\therefore\ \frac{\text{d}^2\text{R}}{\text{dt}^2}=\frac{1}{\text{m}_\text{p}}\bigg[\frac{1}{3}\frac{\text{Ny}^2\text{e}^2}{\epsilon_0}-\frac{4\pi}
{3}\text{Gm}_\text{p}^2\text{N}\bigg]
\Rightarrow\ \frac{\text{d}^2\text{R}}{\text{dt}^2}=\alpha^2\text{R}\ .....(\text{iv})
where, \alpha^2=\frac{1}{\text{m}_\text{p}}\bigg[\frac{1}{3}\frac{\text{NY}^2\text{e}^2}{\epsilon_0}-\frac{4\pi}{3}\text{Gm}_\text{p}^2\text{N}\bigg]
The solution of Eq. (iv) is given by \text{R}=\text{Ae}^{\alpha\text{t}}+\text{Be}^{-\alpha\text{t}}. We are looking for expansion, here, so B = 0
and \text{R}=\text{Ae}^{\alpha\text{t}}.
⇒ velocity of expansion,
\text{v}=\frac{\text{dR}}{\text{dt}}=\text{Ae}^{\alpha\text{t}}(\alpha)=\alpha\text{Ae}^{\alpha\text{t}}=\alpha\text{R}
Hence, \text{v}\propto\text{R}, i.e., velocity of expansion is proportional to the distance from the centre.
Q404. 1. Explain the meaning of the statement ‘electric charge of a body is quantised’. 2 Marks
2. Why can one ignore quantisation of electric charge when dealing with macroscopic i.e., large scale charges?

Ans:1. Electric charge of a body is quantized. This means that only integral (1, 2, …., n) number of electrons can be transferred from one body to the other. Charges are
not transferred in fraction. Hence, a body possesses total charge only in integral multiples of electric charge.
2. In macroscopic or large scale charges, the charges used are huge as compared to the magnitude of electric charge. Hence, quantization of electric charge is of
no use on macroscopic scale. Therefore, it is ignored and it is considered that electric charge is continuous.
Q405. It is said that the separation between the two charges forming an electric dipole should be small. Small compared to what? 2 Marks

Ans:The separation between the two charges forming an electric dipole should be small compared to the distance of a point from the centre of the dipole at which the
influence of the dipole field is observed.
Q406. Show that the electric field at the surface of a charged conductor is given by \overrightarrow{\text{E}} = \frac{\sigma}{\varepsilon_{o}}\hat{\text{n}},where σ is the 2 Marks
surface charge density and \hat{\text{n}}is a unit vector normal to the surface in the outward direction.

Ans:

\text{E}\delta\text{S} = \frac{\sigma\delta\text{S}}{\varepsilon_{0}}
\Rightarrow\text{E} = \frac{\sigma}{\varepsilon_{0}}

https://bls.smartstudies.co.in/#/exam/pdf-preview/b5b87181-b79b-4b4a-8ebf-8f94fb605fc8/1 43/139
5/17/24, 8:14 PM Exam Automation
In vector form
\overrightarrow{\text{E}} = \frac{\sigma\hat{\text{n}}}{\varepsilon_{0}}
Alternate Answer
Also accept the derivation of electric field on the surface of spherical shell.
\oint_{s}\overrightarrow{\text{E}}.\overrightarrow{\text{ds}} = \frac{\text{q}}{\varepsilon_{o}}
\text{E}\times4\pi\text{r}^{2} = \frac{\text{q}}{\varepsilon_{0}}
\text{E} = \frac{\text{q}}{4\pi\text{r}^{2}\varepsilon_{0}}
\Rightarrow\overrightarrow{\text{E}} = \frac{\sigma}{\varepsilon_{0}}\hat{\text{n}}.
Q407. Consider a gold nucleus to be a sphere of radius 6.9 fermi in which protons and neutrons are distributed. Find the force of repulsion between two protons situated 2 Marks
at largest separation. Why do these protons not fly apart under this repulsion?

Ans:

Let two protons be at a distance be 13.8 femi,


\text{F}=\frac{1}{4\pi\epsilon_0}\frac{\text{q}_1\text{q}_2}{\text{r}^2}
\text{F}=\frac{9\times10^9\times1.6\times10^{-38}}{(14.8)^2\times10^{-30}}
\text{F}=1.2\text{N}
Q408. The electric field E due to a point charge at any point near it is defined as \text{E} = \DeclareMathOperator*{\median}{\text{lim}} 2 Marks
\median_{\text{q}\rightarrow0}\frac{\text{F}}{\text{q}} where q is the test charge and F is the force acting on it. What is the physical significance
of \DeclareMathOperator*{\median}{\text{lim}} \median_{\text{q}\rightarrow0} in this expression? Draw the electric field lines of a point charge Q when (a) Q>0
and (b) Q<0.

Ans:

Q409. Two point charges 4Q, Q are separated by 1 m in air. At what point on the line joining the charges is the electric field intensity zero? 2 Marks
Also calculate the electrostatic potential energy of the system of charges, taking the value of charge, Q = 2 × 10–7 C.

Ans:1. \frac{\text{K}4\text{Q}}{\text{x}^{2}} =\frac{\text{KQ}}{(\text{r} - \text{x})^{2}}


\text{x} = \bigg(2\big/3\bigg)\text{m}\text{ or } \text{x} = 2\text{m }(\text{from} 4\text{Q})
2. \text{u} = \frac{\text{kq}_{1}\text{q}_{2}}{\text{r}}
\text{u} = 1.44\times10^{-3}\text{J}.
Q410. Given a uniform electric field\overrightarrow{\text{E}}: 4 x 103 \hat{i} N/C, find the flux of this field through a square of 5cm on a side whose plane is parallel to the 2 Marks
y-z plane. What would be the flux through the same square if the plane makes a 30^{\circ} angle with the x-axis?

Ans:\phi = \text{EA}\cos\theta
= 4\times 10^{8}\times25\times{10}^{-4}\cos0
= 10 \text{NM}^{-2}\text{m}^{-1}.
\phi= 4\times 10^{3}\times25\times{10}^{-4}\cos60
= 5 \text{NM}^{-2}\text{m}^{-1}.
Q411. A charge Q is placed at a distance \frac{\text{a}}{2} above the centre of a horizontal, square surface of edge a as shown in figure. Find the flux of the electric field 2 Marks
through the square surface.

https://bls.smartstudies.co.in/#/exam/pdf-preview/b5b87181-b79b-4b4a-8ebf-8f94fb605fc8/1 44/139
5/17/24, 8:14 PM Exam Automation

Ans:Given:
Edge length of the square surface = a
Distance of the charge Q from the square surface =\frac{\text{a}}2{}
Area of the plane = a2
Assume that the given surface is one of the faces of the imaginary cube.
Then, the charge is found to be at the centre of the cube.
A charge is placed at a distance of about a2a2 from the centre of the surface.
The electric field due to this charge is passing through the six surfaces of the cube.
Hence flux through each surface,
\phi=\frac{\text{Q}}{\epsilon_0}\times\frac{1}{6}=\frac{\text{Q}}{6\epsilon_0}
Thus, the flux through the given surface is \frac{\text{Q}}{6\epsilon_0}.
Q412. In the figure shown, calculate the total flux of the electrostatic field through the spheres S1 and S2. The wire AB shown here has a linear charge density \lambda 2 Marks
given by \lambda=\text{kx} where x is distance measured along the wire, from the end A.

Ans:Total charge on wire AB


\text{Q}_\text{AB}=\int\limits^\text{l}_{0}\lambda\text{dx}=\int\limits^\text{l}_{0}\text{kx dx}=\text{k}\Big[\frac{\text{x}^2}{2}\Big]^\text{l}_0=\frac{1}{2}\text{kl}^2
By Gauss theorem,
Total flux through \text{S}_1=\frac{\text{Q}}{\epsilon_0}
Total flux through \text{S}_2=\frac{\text{Q}+\text{Q}_\text{AB}}{\epsilon_0}=\bigg(\frac{\text{Q}+\frac{1}{2}\text{kl}^2}{\epsilon_0}\bigg)
Q413. A positive charge q is placed in front of a conducting solid cube at a distance d from its centre. Find the electric field at the centre of the cube due to the charges 2 Marks
appearing on its surface.

Ans:

We know,
Electric field at a point due to a given charge

https://bls.smartstudies.co.in/#/exam/pdf-preview/b5b87181-b79b-4b4a-8ebf-8f94fb605fc8/1 45/139
5/17/24, 8:14 PM Exam Automation
'\text{E}'=\frac{\text{Kq}}{\text{r}^2} Where q = charge, r = Distance between the point and the charge
So, '\text{E}'=\frac{1}{4\pi\in_0}\times\frac{\text{q}}{\text{d}^2} [\therefore\text{r}=\text{‘d’}\text{here}]
Q414. A thin straight infinitely long conducting wire having charge density λ is enclosed by a cylindrical surface of radius r and length l, its axis coinciding with the length 2 Marks
of the wire. Find the expression for the electric flux through the surface of the cylinder.

Ans:Charge enclosed by the cylindrical surface \text{q} = \lambda l


Flux \varphi =\frac{\text{q}}{\varepsilon_{0}}
= \frac{\lambda l }{\varepsilon_{0}}
Alternate Answer
\varphi = \oint\overrightarrow{\text{E}}.\text{d}\overrightarrow{\text{S}} = \frac{\text{q}}{\varepsilon_{0}}
\varphi = \frac{\lambda l }{\varepsilon_{0}}.
Q415. A metallic sphere is placed in a uniform electric field. Which one of paths a, b, c and d shown in the figure will be followed by the field lines and why? 2 Marks

Ans:

Path (d) is followed by electric field line.


Reason: There are no electric field lines within a metallic sphere and field lines are normal at each point of the surface.
Q416. An electric dipole of dipole moment 20 × 10-6C is enclosed by closed surface. What is the net electric flux coming out of this surface? 2 Marks

Ans:Zero.
Reason: Net charge enclosed by surface = Net charge on dipole = q - q = 0
\therefore Total electric flux =\frac{1}{\epsilon_0}\times net charge enclosed = 0
Q417. Define electric field strength. Is it a vector or a scalar quantity? 2 Marks

Ans:The electric field strength at a point in an electric field is defined as the electrostatic force acting on a unit positive charge when placed at that point and its
direction is along the direction of electrostatic force.
Electric field strength is a vector quantity.
Q418. A photon and a proton have the same de-Broglie wavelength \lambda. Prove that the energy of the photon is \Big(\frac{2\text{m}\lambda\text{c}}{\text{h}}\Big) 2 Marks
times the kinetic energy of the proton.

Ans:Energy of photon \text{E}_\text{p}=\frac{\text{hc}}{\lambda}


For proton \lambda=\frac{\text{h}}{\text{mv}}
\text{mv}=\frac{\text{h}}{\lambda}
Kinetic energy of proton \text{E}_\text{k}=\frac{1}{2}\text{mv}^2
\text{E}_\text{k}=\frac{1}{2}\frac{\text{h}^2}{\text{m}\lambda^2}
\text{E}_\text{p}=\Big(\frac{2\text{m}\lambda\text{c}}{\text{h}}\Big)\text{E}_\text{k}
Q419. Define electric flux. Write its S.I. unit. 2 Marks
A charge q is enclosed by a spherical surface of radius R. If the radius is reduced to half, how would the electric flux through the surface change?

Ans:Electric lines of force passing through the surface normally.


Alternate Answer
Electric flux \Delta\Phi through an area element \Delta\text{S} is defined by
\Delta\phi = \text{E}\cdot\Delta\text{S} = \text{E}\Delta\text{S}\cos\theta
SI unit: νolt-meter or NM2/C
On decreasing the radius of spherical surface to half there will be no effect on the electric flux.
Q420. Given a uniform electric field \overrightarrow{\text{E}} = 2 x 103 \hat{i} N/C find the flux of this field through a square of side 20 cm, whose plane is parallel to the 2 Marks
y-z plane. What would be the flux through the same square, if the plane makes an angle of 30^{\circ} with the x-axis?

Ans:\phi = \text{EA}\cos\theta
= 2\times 10^{3}\times{4}\times 10^{-2} \cos 0^{0}
=80\ NC^{-1}m^2
\phi=2 \times10^{3}\times4\times10^{2}\cos60^{0}
40NC^{-1}{m}^2
Q421. Can a gravitational field be added vectorially to an electric field to get a total field? 2 Marks

Ans:No, a gravitational field cannot be added vectorially to an electric field. This is because for electric influence, one or both the bodies should have some net charge
and for gravitational influence both the bodies should have some mass. Also, gravitational field is a weak force, while electric field is a strong force.
Q422. Represent graphically the variation of electric field with distance, for a uniformly charged plane sheet. 2 Marks

Ans:Image
Electric field due to a uniformly charged plane sheet.
\text{E}=\frac{\sigma}{2\epsilon_0}
Which is independent of distance.
So, it represents a straight line parallel to distance axis.
Q423. A point charge produces an electric field of magnitude 5.0NC-1 at a distance of 40cm from it. What is the magnitude of the charge? 2 Marks

Ans:

https://bls.smartstudies.co.in/#/exam/pdf-preview/b5b87181-b79b-4b4a-8ebf-8f94fb605fc8/1 46/139
5/17/24, 8:14 PM Exam Automation

\text{E}=\frac{1}{4\pi\epsilon_0}\frac{\text{q}}{\text{r}^2}
\Rightarrow5.0=9\times10^9\times\frac{9}{(0.4)^2}
\Rightarrow\text{q}=8.9\times10^{-11}\text{C}
Q424. The charge on a proton is +1.6 × 10-19C and that on an electron is -1.6 × 10-19C. Does it mean that the electron has a charge 3.2 × 10-19C less than the charge of a 2 Marks
proton?

Ans:An electron and a proton have equal and opposite charges of magnitude 1.6 × 10-19C. But it doesn't mean that the electron has 3.2 × 10-19C​ less charge than the
proton.
Q425. The force between two point charges kept at a distance r apart in air is F. If the same charges are kept in water at the same distance, how does the force between 2 Marks
them change?

Ans:The force in air \text{F}_\text{a}=\frac{1}{4\pi\epsilon_0}\frac{\text{q}_1\text{q}_2}{\text{r}^2}


The force in water \text{F}_\omega=\frac{1}{4\pi\epsilon_0\text{K}}\frac{\text{q}_1\text{q}_2}{\text{r}^2}
\therefore\ \frac{\text{F}_\omega}{\text{F}_\text{a}}=\frac{1}{\text{K}}
Dielectric constant of water is 81, so the force in water reduces to \frac{1}{81} times.
Q426. When a glass rod is rubbed with a silk cloth, charges appear on both. A similar phenomenon is observed with many other pairs of bodies. Explain how this 2 Marks
observation is consistent with the law of conservation of charge.

Ans:Rubbing produces charges of equal magnitude but of opposite nature on the two bodies because charges are created in pairs. This phenomenon of charging is
called charging by friction. The net charge on the system of two rubbed bodies is zero. This is because equal amount of opposite charges annihilate each other.
When a glass rod is rubbed with a silk cloth, opposite natured charges appear on both the bodies. This phenomenon is in consistence with the law of conservation
of energy. A similar phenomenon is observed with many other pairs of bodies.
Q427. Considering the case of a parallel plate capacitor being charged, show how one is required to generalize Ampere’s circuital law to include the term due to 2 Marks
displacement current.

Ans:According to Ampere’s circuital Law


\oint\overrightarrow{\text{B}}\text{d}\overrightarrow{\text{l}} = \mu_{0}\text{I}

Applying ampere’s circuital law to fig (a) we see that, during charging, the right hand side in Ampere’s circuital law equals \mu_{0}\text{I}
However on applying it to the surfaces of the fig (b) or fig (c), the right hand side is zero.
Hence, there is a contradiction. We can remove the contradiction by assuming that there exists a current (associated with the changing electric field during
charging), known as the displacement current.
When this current ( = \frac{\text{d}\phi_{E}}{\text{dt}}) is added on the right-hand side, Ampere’s circuital law, the inconsistency disappears.
It was, therefore necessary, to generalize the Ampere’s circuital law, as
\oint\overrightarrow{\text{B}}\text{d}\overrightarrow{\text{l}} = \mu_{0}\text{I}_{c} + \mu_{0}\in_{o}\frac{\text{d}\phi_{E}}{\text{dt}}.
Q428. Consider two particles A and B having equal charges and placed at some distance. The particle A is slightly displaced towards B. Does the force on B increase as 2 Marks
soon as the particle A is displaced? Does the force on the particle A increase as soon as it is displaced?

Ans:Electrostatic force follows the inverse square law, \text{F}=\frac{\text{k}}{\text{r}^2}. This means the the force on two particles carrying charges increases on
decreasing the distance between them. Therefore, as particle A is slightly displaced towards B, the force on B as well as a A will increase.
Q429. A charge Q is placed at the centre of a cube. Find the flux of the electric field through the six surfaces of the cube. 2 Marks

Ans:According to Gauss's Law, flux passing through any closed surface is equal to \frac{1}{\epsilon_0} times the charge enclosed by that surface.
\Rightarrow\phi=\frac{\text{q}}{\epsilon_0},
where \phi is the flux through the closed surface and q is the charge enclosed by that surface.
The charge is placed at the centre of the cube and the electric field is passing through the six surfaces of the cube. So, we can say that the total electric flux
passes equally through these six surfaces.
Thus, flux through each surface,
\phi'=\frac{\text{Q}}{6\epsilon_0}
Q430. A uniformly charged conducting sphere of 2.4 m diameter has a surface charge density of 80.0 μC/m2. 2 Marks

https://bls.smartstudies.co.in/#/exam/pdf-preview/b5b87181-b79b-4b4a-8ebf-8f94fb605fc8/1 47/139
5/17/24, 8:14 PM Exam Automation
1. Find the charge on the sphere.
2. What is the total electric flux leaving the surface of the sphere?

Ans:Given,
Diameter of the sphere = 2.4
\therefore Radius of sphere, r = \frac{2.4}{2} = 1.2 m
Surface charge density of conducting sphere, \sigma = 80 × 10-6 C/m2
1. Charge on sphere
\sigma.\text{A}=\sigma.4\pi\text{r}^2
q = 80 × 10-6 × 4 × 3.14 × (1.2)2
q = 1.45 × 10-3 C
2. The total electric flux leaving the surface of the sphere using the gauss formula,
\phi=\frac{\text{q}}{\epsilon_0}
=\frac{1.45\times10^{-3}}{8.854\times10^{-12}}=1.6\times10^8 \text{Nm}^2/\text{C}
Q431. A capacitor, made of two parallel plates each of plate area A and separation d, is being charged by an external ac source. Show that the displacement current 2 Marks
inside the capacitor is the same as the current charging the capacitor.

Ans:

In Fig. conduction current is flowing in the wires, causes charge on the plates
So, \text{I}_{c} = \frac{\text{dq}}{\text{dt}}
According to Maxwell, displacement current between plates
\text{I}_{d} = \varepsilon_{0}\frac{\text{d}\varphi_{E}}{\text{dt}}\text{ where }\Phi_{E} =\text{ Electric flux}
Using Gauss’s Theorem, if one of the plate is inside the tiffin type Gaussian surface
\Phi_{E} = \frac{\text{q}}{\varepsilon_{0}}
So \text{I}_{d} = \varepsilon_{0}\frac{\text{d}}{\text{dt}}\big(\frac{\text{q}}{\varepsilon_{0}}\big)
\text{I}_{d} = \frac{\text{dq}}{\text{dt}}
From equation (2) and (3), Both conduction current and displacement currents are equal.
Q432. Plot a graph showing the variation of coulomb force (F) versus\bigg(\frac{1}{\text{r}^{2}}\bigg); where r is the distance between the two charges of each pair of 2 Marks
charges: (1 μC, 2 μC) and (2 μC, – 3 μC). Interpret the graphs obtained.

Ans:1.

Alternate Answer

(Attractive force is greater than repulsive force since magnitude of the slope is more for attraction.)
Q433. Two point electric charges of unknown magnitude and sign are placed at some distance ‘d’ apart. The electric field intensity is zero at a point, not between the 2 Marks
charges but on the line joining them.
Write two essential conditions for this to happen.

Ans:1. The two charges must be of opposite kind.


2. The magnitude of two charges must be unequal. The charge closer to the point of observation should be of smaller magnitude.
Q434. An electric dipole of length 4 cm, when placed with its axis making an angle of 60o with a uniform electric field, experiences a torque of 4\sqrt{3}Nm. Calculate the 2 Marks
potential energy of the dipole, if it has charge \pm8 nC.

Ans:\tau = \text{pE}\sin\theta
4\sqrt{3} = \text{pE}\sin60^{0} = \text{pE}\frac{\sqrt{3}}{2}
\Rightarrow\text{pE} = 8
Potential energy
\text{U} = - \text{pE}\cos\theta
= - 8 \text{ x }\cos60^{0} = -4\text{J}.
Q435. Find the electric force between two protons separated by a distance of 1 fermi (1 fermi = 10-15m). The protons in a nucleus remain at a separation of this order. 2 Marks

Ans:We know:
Charge on a proton, q = 1.6 × 10-19 C
Given, separation between the charges, r = 10-15 m
By Coulomb's Law, electrostatic force,
\text{F}=\frac{1}{4\pi\in_0}\frac{\text{q}_1\text{q}_2}{\text{r}^2}
\Rightarrow\text{F}=96\times10^9\times\frac{(1.6\times10^{-19})^2}{(10^{-15})^2}
\Rightarrow\text{F}=230\text{N}
Q436. The dimensions of an atom are of the order of an Angstrom. Thus there must be large electric fields between the protons and electrons. Why, then is the 2 Marks
electrostatic field inside a conductor zero?

Ans:

https://bls.smartstudies.co.in/#/exam/pdf-preview/b5b87181-b79b-4b4a-8ebf-8f94fb605fc8/1 48/139
5/17/24, 8:14 PM Exam Automation
In an atom, number of electrons and protons are equal and the electric fields bind the atoms to neutral entity. Fields are caused by excess charges. There can be
no excess charge on the inter surface of an isolated conductor.
Q437. Distinguish between nuclear fusion and nuclear fission. Give one example for each. 2 Marks

Ans:
Nuclear Fission Nuclear fusion
Two light nuclei combine to form a heavy nucleus with relea
A heavy nucleus splits into two or more lighter nuclei with the release of energy.
se of energy.
Example: ^{1}_{0}\text{n}+\ ^{235}_{92}\text{U}\xrightarrow{\ \ \ }\ ^{236}_{92}\text{U}\xrightarrow{\ Example: 4^{1}_{1}\text{H}+\ ^{4}_{2}\text{He}+2\text{v}+6
\ \ }\ ^{144}_{56}\text{Ba}+\ ^{89}_{36}\text{Kr}+3\ ^{1}_{0}\text{n} \gamma+26.7\text{MeV}+2\text{e}^{-}

Q438. A point charge is placed at the centre of a closed Gaussian spherical surface of radius r. Electric flux passing through the surface is \Phi How is the electric flux 2 Marks
\Phi through the surface affected when the following changes are made in turn:
1. The spherical surface is replaced by a cylindrical surface of the same radius?
2. The point charge is replaced by an electric dipole?
Justify your answer in each case.

Ans:1. Since the charge inside the Gaussian surface remains the same, the electric flux through it remains unchanged.
2. Since the net charge inside the surface is zero, the electric flux passing through the surface also becomes zero.
Q439. Given a uniform electric field\overrightarrow{\text{E}}: 5 x 103 \hat{i} N/C, find the flux of this field through a square of l0 cm on a side whose plane is parallel to 2 Marks
the y-z plane. What would be the flux through the same square if the plane makes a 300 angle with the x-axis?

Ans:\phi = \text{EA}\cos\theta
= 5\times 10^{3}\times 10^{-2} \cos 0^{0}\text{NC}^{-1}\text{m}^{2}
= 50 \text{NC}^{-1}\text{m}^{2}
\phi = 5\times10^{3}\times10^{-2}\cos60^{o}\text{NC}^{-1}\text{m}^{2}
= 25 \text{NC}^{-1}\text{m}^{2}.
Q440. A wire is bent in the form of a regular hexagon and a total charge q is distributed uniformly on it. What is the electric field at the centre? You may answer this part 2 Marks
without making any numerical calculations.

Ans:

Since it is a regular hexagon. So, it forms an equipotential surface. Hence the charge at each point is equal. Hence the net entire field at the centre is Zero.
Q441. An infinite line charge produces a field of 9 × 104 N/C at a distance of 2 cm. Calculate the linear charge density. 2 Marks

Ans:Given, Electric field , E = 9 x 104 N/C


Distance, r = 2 x 10-2 m
Using the formula of electric field for uniformly charged wire,
\text{E}=\frac{\lambda}{2\pi\text{r}\epsilon_0}
\therefore\lambda=\text{E}.\ 2\pi\text{r}.\epsilon_0
\lambda= 9 × 104 × 2\pi × 2 × 10-2 × 8.854 × 10-12
= 10 × 10-6
linear charge density - \lambda = 10 µC/m.
Q442. A charge Q is placed at the centre of an imaginary hemispherical surface. Using symmetry arguments and the Gauss's law, find the flux of the electric field due to 2 Marks
this charge through the surface of the hemisphere (figure).

Ans:From Guass's law, flux through a closed surface,


\phi=\frac{\text{Q}_{\text{en}}}{\epsilon_0},
where
Qen = charge enclosed by the closed surface
Let us assume that a spherical closed surface in which the charge is enclosed is Q.
The flux through the sphere,
\phi=\frac{\text{Q}}{\epsilon_0}

https://bls.smartstudies.co.in/#/exam/pdf-preview/b5b87181-b79b-4b4a-8ebf-8f94fb605fc8/1 49/139
5/17/24, 8:14 PM Exam Automation
Hence for a hemisphere(open bowl), total flux through its curved surface,
\phi'=\frac{\text{Q}}{\epsilon}\times\frac{1}{2}=\frac{\text{Q}}{2\epsilon_0}
Q443. Find the flux of the electric field through a spherical surface of radius R due to a charge of 10-7C at the centre and another equal charge at a point 2R away from 2 Marks
the centre.

Ans:Given:
Let charge Q be placed at the centre of the sphere and Q' be placed at a distance 2R from the centre.
Magnitude of the two charges = 10-7C
According to Gauss's Law, the net flux through the given sphere is only due to charge Q that is enclosed by it and not by the charge Q' that is lying outside.
So, only the charge located inside the sphere will contribute to the flux passing through the sphere.
Thus,
\phi=\int{\text{E}}.\text{d}{\text{s}}=\frac{\text{Q}}{\epsilon_0}=\frac{10^{-7}}{8.85\times10^{-12}}
\Rightarrow\phi=1.1\times10^{4}\text{Nm}^2\text{C}^{-1}
Q444. The distance of the field point on the axis of a small dipole is doubled. By what factor will the electric field, due to the dipole change? 2 Marks

Ans:For a small dipole, \text{E}_\text{axis}=\frac{1}{4\pi\epsilon_0}\frac{2\text{p}}{\text{r}^3}\propto\frac{1}{\text{r}^3}


When the distance r is doubled, the electric field strength becomes 1/8 times the original field.
Q445. The distance of the field point, on the equatorial plane of a small electric dipole is halved. By what factor does the electric field due to the dipole change? 2 Marks

Ans:For small dipole, \text{E}_\text{equator}=\frac{1}{4\pi\epsilon_0}\frac{\text{p}}{\text{r}^3}\propto\frac{1}{\text{r}^3}


When r is halved, the electric field strength become 8-times of the original field.
Q446. Show that there can be no net charge in a region m which the electric field is uniform at all points. 2 Marks

Ans:It is given that the electric field is uniform. If we consider a surface perpendicular to the electric field, we find that it is an equipotential surface. Hence, if a test
charge is introduced on the surface, then work done will be zero in moving the test charge on it.
But if there is some net charge in this region, the test charge introduced on the surface will experience a force due to this charge. This force has a component
parallel to the surface; thus, work has to be done in moving this test charge. Thus, the surface cannot be said to be equipotential. This implies that the net charge in
the region with uniform electric field is zero.
Q447. Is there any lower limit to the electric force between two particles placed at a separation of 1cm ? 2 Marks

Ans:Yes, there's a lower limit to the electric force between two particles placed at a separation of 1cm, which is equal to the magnitude of force of repulsion between
two electrons placed at a separation of 1cm.
Q448. Explain giving reason, how the magnifying power of a compound microscope depends on the (i) wavelength of incident light, and (ii) focal length of the objective 2 Marks
lens.

Ans:Since magnifying power of compound microscope \text{m}=\frac{\text{L}}{\text{f}_\text{o}}\times\frac{\text{D}}{\text{f}_\text{e}}


1. Reciprocal to the focal length of objective lens.
2. "Magnifying power of compound microscope is not independent of wavelength, instead it varies inversely with the wavelength of light". \text{M}=\frac{\text{LD}}
{\text{F}_0\text{F}_\text{e}}
As focal length is directly proportional to wavelength.
Q449. A block of mass m having a charge q is placed on a smooth horizontal table and is connected to a wall through an unstressed spring of spring constant k as shown 2 Marks
in figure. A horizontal electric field E parallel to the spring is switched on. Find the amplitude of the resulting SHM of the block.

Ans:

https://bls.smartstudies.co.in/#/exam/pdf-preview/b5b87181-b79b-4b4a-8ebf-8f94fb605fc8/1 50/139
5/17/24, 8:14 PM Exam Automation

\text{F}=\text{qE},\ \text{F}=-\text{Kx}
Where x = amplitude
\text{x}=\frac{-\text{qE}}{\text{K}}
Q450. The electric field E due to any point charge near it is defined as \text{E}=\lim\limits_{\text{q}\rightarrow0}\frac{\text{F}}{\text{q}} where q is the test charge and F is 2 Marks
the force acting on it. What is the physical significance of \lim\limits_{\text{q}\rightarrow0} in this expression?
Draw the electric lines of point charge Q when,
1. Q > 0
2. Q < 0.

Ans:The physical significance of \lim\limits_{\text{q}\rightarrow0} in the definition of electric field \text{E}=\lim\limits_{\text{q}\rightarrow0}\frac{\text{F}}{\text{q}}


The point test charge q produces its own electric field, hence it will modify the electric field strength to be measured. Therefore, the test charge used to measure
the electric field must be too small.
The electric lines of force are shown in figure above.
Q451. A block of mass m containing a net positive charge q is placed on a smooth horizontal table which terminates in a vertical wall as shown in figure. The distance of 2 Marks
the block from the wall is d. A horizontal electric field E towards right is switched on. Assuming elastic collisions (if any) find the time period of the resulting
oscillatory motion. Is it a simple harmonic motion?

Ans:The block does not undergo. SHM since here the acceleration is not proportional to displacement and not always opposite to displacement. When the block is
going towards the wall the acceleration is along displacement and when going away from it the displacement is opposite to acceleration.
Time taken to go towards the wall is the time taken to goes away from it till velocity is:

https://bls.smartstudies.co.in/#/exam/pdf-preview/b5b87181-b79b-4b4a-8ebf-8f94fb605fc8/1 51/139
5/17/24, 8:14 PM Exam Automation

\text{d}=\text{ut}+\Big(\frac{1}{2}\Big)\text{at}^2
\Rightarrow\text{d}=\frac{1}{2}\times\frac{\text{qE}}{\text{m}}\times\text{t}^2
\Rightarrow\text{t}^2=\frac{2\text{dm}}{\text{qE}}
\Rightarrow\text{t}=\sqrt{\frac{2\text{md}}{\text{qE}}}
\therefore\ Total time taken for to reach the wall and com back (Time period)
=2\text{t}=2\sqrt{\frac{2\text{md}}{\text{qE}}}
=\sqrt{\frac{2\text{md}}{\text{qE}}}
Q452. The electric force experienced by a charge of 1.0 × 10-6C is 1.5 × 10-3N. Find the magnitude of the electric field at the position of the charge. 2 Marks

Ans:\text{F}_\text{e}=1.5\times10^{-3}\text{N},\ \text{q}=1\times10^{-6}\text{C},
\text{F}_\text{e}=\text{q}\times\text{E}
\Rightarrow\text{E}=\frac{\text{F}_\text{e}}{\text{q}}
=\frac{1.5\times10^{-3}}{1\times10^{-6}}
=1.5\times10^3\text{N/C}
Q453. Define electric flux. Write its S.I. units. A spherical rubber balloon carries a charge that is uniformly distributed over its surface. As the balloon is blown up and 2 Marks
increases in size, how does the total electric flux coming out of the surface change? Give reason.

Ans:Electric flux is the total number of electric field lines passing normally through a given surface.
Alternate Answer
Electric Flux \int_{s}\vec{\text{E}}.\text{d}\vec{s}
S.I. unit Nm2/C
Alternate Answer
V.m No Change
As the total charge enclosed remains the same.
Q454. At what separation should two equal charges, 1.0C each, be placed so that the force between them equals the weight of a 50kg person? 2 Marks

Ans:Given:
Magnitude of charges, \text{q}_1=\text{q}_2=1\text{C}
Electrostatic force between them,
F = Weight of a 50kg person
\text{mg}=50\times9.8=490\text{N}
\text{mg}=490\text{N}
Let the required distance be r.
By Coulomb's Law, electrostatic force,
\text{F}=\frac{1}{4\pi\epsilon_0}\frac{\text{q}_1\text{q}_2}{\text{r}^2}
\Rightarrow490=\frac{9\times10^9\times1\times1}{\text{r}^2}
\Rightarrow\text{r}^2=\frac{9\times10^9}{490}
\Rightarrow\text{r}=\sqrt{\frac{9}{49}\times10^8}
=\frac{3}{7}\times10^4\text{m}
=4.3\times10^3\text{m}
Q455. 1. An electrostatic field line is a continuous curve. That is, a field line cannot have sudden breaks. Why not? 2 Marks
2. Explain why two field lines never cross each other at any point?

Ans:1. An electrostatic field line is a continuous curve because a charge experiences a continuous force when traced in an electrostatic field. The field line cannot have
sudden breaks because the charge moves continuously and does not jump from one point to the other.
2. If two field lines cross each other at a point, then electric field intensity will show two directions at that point. This is not possible. Hence, two field lines never
cross each other.
Q456. Why does a phonograph-record attract dust particles just after it is cleaned? 2 Marks

Ans:When a phonograph record is cleaned, it develops a charge on its surface due to rubbing. This charge attracts the neutral dust particles due to induction.
Q457. A hydrogen atom contains one proton and one electron. It may be assumed that the electron revolves in a circle of radius 0.53 angstrom (1 angstrom = 10-10m and 2 Marks
is abbreviated as A) with the proton at the centre. The hydrogen atom is said to be in the ground state in this case. Find the magnitude of the electric force between
the proton and the electron of a hydrogen atom in its ground state.

Ans:\text{R}=0.53\text{A}^\circ=0.53\times10^{-10}\text{m}
\text{F}=\frac{\text{Kq}_1\text{q}_2}{\text{r}^2}
=\frac{9\times10^9\times1.6\times1.6\times10^{-38}}{0.53\times0.53\times10^{-10}\times10^{-10}}
=82.02\times10^{-9}\text{N}
Q458. A parallel plate capacitor of capacitance C is charged to a potential V. It is then connected to another uncharged capacitor having the same capacitance. Find out 2 Marks
the ratio of the energy stored in the combined system to that stored initially in the single capacitor.

Ans:Energy stored in a capacitor = \frac{1}{2}\text{QV} = \frac{1}{2}\text{CV}^{2} = \frac{1}{2}\frac{\text{Q}^{2}}{\text{C}}


Capacitance of the (parallel) combination = C+C=2C
Here, total charge, Q, remains the same
\therefore\text{ initial energy } = \frac{1}{2}\frac{\text{Q}^{2}}{\text{C}}
And final energy = \frac{1}{2}\frac{\text{Q}^{2}}{2\text{C}}
\therefore\frac{\text{ final energy }}{\text{ initial energy }} = \frac{1}{2}
Parallel or (ii) Series combination to remain constant (=V) and obtain the answers as (i) 2:1 or (ii) 1:2.
Q459. A circular wire-loop of radius a carries a total charge Q distributed uniformly over its length. A small length dL of the wire is cut off. Find the electric field at the 2 Marks
centre due to the remaining wire.

Ans:\frac{\text{Charge}}{\text{Unit length}}=\frac{\text{Q}}{2\pi\text{a}}=\lambda; Charge of \text{d}\ell=\frac{\text{Qd}\ell}{2\pi\text{a}}\text{C}


Initially the electric field was ‘0’ at the centre. Since the element ‘dℓ’ is removed so, net electric field must \frac{\text{K}\times\text{q}}{\text{a}^2}

https://bls.smartstudies.co.in/#/exam/pdf-preview/b5b87181-b79b-4b4a-8ebf-8f94fb605fc8/1 52/139
5/17/24, 8:14 PM Exam Automation
Where q = charge of element \text{d}\ell
\text{E}=\frac{\text{Kq}}{\text{a}^2}
=\frac{1}{4\pi\in_0}\times\frac{\text{Q}\text{d}\ell}{2\pi\text{a}}\times\frac{1}{\text{a}^2}
=\frac{\text{Qd}\ell}{8\pi^2\in_0\text{a}^3}
Q460. A charge q is placed at the centre of the line joining two equal charges Q. 3 Marks
Show that the system of three charges will be in equilibrium if \text{q}=-\frac{\text{Q}}{4}.

Ans:

Charge q is in equilibrium since charges A and B exert equal and opposite forces on it.
For equilibrium of charge Q at B;
FBC + FAB = 0
\Rightarrow\ \frac{1}{4\pi\epsilon_0}\frac{\text{qQ}}{(\text{l}/2)^2}+\frac{1}{4\pi\epsilon_0}\frac{\text{Q.Q}}{\text{l}^2}=0
\Rightarrow\ \frac{1}{4\pi\epsilon_0}\frac{\text{Q}}{\text{l}^2}(4\text{q}+\text{Q})=0\Rightarrow\ \text{}q=-\frac{\text{Q}}{4}
Q461. 1. Define torque acting on a dipole of dipole moment \overrightarrow{p} placed in a uniform electric field \overrightarrow{\text{E}}. Express it in the vector form 3 Marks
and point out the direction along which it acts
2. What happens if the field is non-uniform?
3. What would happen if the external field \overrightarrow{\text{E}} is increasing (i) parallel to \overrightarrow{p} and (ii) anti-parallel to \overrightarrow{p}?

Ans:1. \tau=pE\sin\theta\text{ };\text{ }\theta= angle between dipole moment(\overrightarrow{p}) and electric field(\overrightarrow{\text{E}})
\tau=\overrightarrow{p}\times\overrightarrow{\text{E}}
Direction of torque is perpendicular to the plane containing and given by right-hand screw rule.
Alternate Answer

Direction of torque is out of the plane of the paper.


2. If the field is non uniform the net force on the dipole will not be zero. There will be translatory motion of the dipole.
3.
1. Net force will be in the direction of increasing electric field.

2. Net force will be in the direction opposite to the increasing field. [or in the direction of decreasing field]

Q462. The electric field in a region is given by \overrightarrow{\text{E}}=\frac{3}{5}\text{E}_0\overrightarrow{\text{i}}+\frac{4}{5}\text{E}_0\overrightarrow{\text{j}} with 3 Marks


\text{E}_0=2.0\times10^3\text{NC}^{-1}. Find the flux of this field through a rectangular surface of area 0.2m2 parallel to the y-z plane.

Ans:Given:
Electric field strength, \overrightarrow{\text{E}}=\frac{3}{5}\text{E}_0\hat{\text{i}}+\frac{4}{5}\text{E}_0\hat{\text{j}}
where \text{E}_0=2.0\times10^3\text{N/C}
The plane of the rectangular surface is parallel to the y-z plane. The normal to the plane of the rectangular surface is along the x axis.
Only \frac{3}{5}\text{E}_0\hat{\text{i}} passes perpendicular to the plane; so, only this component of the field will contribute to flux.
On the other hand, \frac{4}{5}\text{E}_0\hat{\text{i}} moves parallel to the surface.
Surface area of the rectangular surface, a = 0.2m2
Flux,
\phi=\overrightarrow{\text{E}}.\overrightarrow{\text{a}}=\text{E}\times\text{a}
\phi=\Big(\frac{3}5{}\times2\times10^3\Big)\times(2\times10^{-1})\text{Nm}^2/\text{C}
\phi=0.24\times10^3\text{Nm}^2/\text{C}
\phi=240\text{Nm}^2/\text{C}
Q463. A particle having a charge of 2.0 × 10-4C is placed directly below and at a separation of 10cm from the bob of a simple pendulum at rest. The mass of the bob is 3 Marks
100g. What charge should the bob be given so that the string becomes loose?

Ans:

https://bls.smartstudies.co.in/#/exam/pdf-preview/b5b87181-b79b-4b4a-8ebf-8f94fb605fc8/1 53/139
5/17/24, 8:14 PM Exam Automation

Mass of the bob = 100g = 0.1kg


So Tension in the string = 0.1 × 9.8 = 0.98N.
For the Tension to be 0, the charge below should repel the first bob.
\Rightarrow\text{F}=\frac{\text{kq}_1\text{q}_2}{\text{r}^2} \big[\text{T}-\text{mg}+\text{F}=0\ \Rightarrow\text{T}=\text{mg}-\text{f},\ \text{T}=\text{mg}\big]
\Rightarrow0.98=\frac{9\times10^9\times2\times10^{-4}\times\text{q}^2}{(0.01)^2}
\Rightarrow\text{q}_2=\frac{0.98\times1\times10^{-2}}{9\times2\times10^5}
=0.054\times10^{-9}\text{N}
Q464. Consider the situation shown in figure. What are the signs of q1 and q2? If the lines are drawn in proportion to the charge, what is the ratio \frac{\text{q}_1} 3 Marks
{\text{q}_2}?

Ans:The electric lines of force are entering charge q1; So, it is is negative. On the other hand, the lines of force are originating from charge q2; so, it positive.
If the lines are drawn in proprotion to the charges, then
\frac{\text{q}_1}{\text{q}_2}=\frac{6}{18}
\Rightarrow\frac{\text{q}_1}{\text{q}_2}=\frac{1}{3}
6 lines are entering q1 and 18 are coming our of q2.
Q465. A circular ring of radius r made of a nonconducting material is placed with its axis parallel to a uniform electric field. The ring is rotated about a diameter through 3 Marks
180°. Does the flux of electric field change? If yes, does it decrease or increase?

Ans:It is given that the circular ring, made of a non-conducting material, of radius r is placed with its axis parallel to a uniform electric field.This means that both the
electric field and the area vector are parallel to each other (area vector is always perpendicular to the surface area). Thus, the flux through the ring is given
by \vec{\text{E}}.\vec{\text{S}}={\text{ES}}\cos0=\text{E}(\pi\text{r}^2).
Now, when the ring is rotated about its diameter through 180°, the angle between the area vector and the electric field becomes 180°. Thus, the flux becomes -
\text{E}(\pi\text{r}^2).
Q466. Draw the electric field lines due to a uniformly charged thin spherical shell when charge on the shell is (a) positive and (b) negative. 3 Marks

Ans:The electric field lines are shown in the figure. For a positively charged shell, the field lines are directed in radially outward direction and for negatively charged
shell, these are directed in radially inward direction.

Q467. Using Gauss's law obtain the expression for the electric field due to a uniformly charged thin spherical shell of radius R at a point outside the shell. Draw a graph 3 Marks
showing the variation of electric field with r, for r > R and r < R.

https://bls.smartstudies.co.in/#/exam/pdf-preview/b5b87181-b79b-4b4a-8ebf-8f94fb605fc8/1 54/139
5/17/24, 8:14 PM Exam Automation
Ans:

From Gauss's theorem, {\phi = \oint\overrightarrow{\text{E}}}.\text{d}\overrightarrow{\text{S}} = \frac{\text{q}_{m}}{\varepsilon_{0}}


Flux \phithrough S'.
\phi = \oint\limits_{s'}\overrightarrow{\text{E}}.\text{d}\overrightarrow{\text{S}} = \oint\limits_{s'}\text{EdS} = \text{E}.4\pi\text{r}^{2}
\Rightarrow\text{E}.4\pi\text{r}^{2} = \frac{\text{q}_{m}}{\varepsilon_{0}}\Rightarrow\text{E} = \frac{1}{4\pi\varepsilon_{0}}\frac{\text{q}_{m}}{\text{r}^{2}}

Q468. Consider a sphere of radius R with charge density distributed as 3 Marks


\rho\text{(r)}=\text{kr for r}\leq\text{R}
=0\text{ for f}>\text{R}.
Find the electric field at all points r.

Ans:The expression of charge density distribution in the sphere suggests that the electric field is radial.
Let us consider a sphere S of radius R and two hypothetic spheres of radius r < R and r > R.
Let us first consider for point r < R, electric field intensity will be given by,

\oint\overrightarrow{\text{E}}.\text{d}\overrightarrow{\text{S}}=\frac{1}{\epsilon_0}\int\rho\text{dV}
Here \text{dV}=4\pi\text{r}^2\text{dr}
\Rightarrow\ \oint\overrightarrow{\text{E}}.\text{d}\overrightarrow{\text{S}}=\frac{1}{\epsilon_0}4\pi\text{K}\int\text{r}^3\text{dr}\ \ (\because\
\rho(\text{r})=\text{Kr})
\Rightarrow\ (\text{E})4\pi\text{r}^2=\frac{4\pi\text{K}}{\epsilon_0}\frac{\text{r}^4}{4}
We get, \text{E}=\frac{1}{4\epsilon_0}\text{Kr}^2
As charge density is positive, it means the direction of E is radially outwards.
Now consider points r > R, electric field intensity will be given by
\oint\overrightarrow{\text{E}}.\text{d}\overrightarrow{\text{S}}=\frac{1}{\epsilon_0}\int\rho\text{dV}
\Rightarrow\ \text{E}(4\pi\text{r}^2)=\frac{4\pi\text{K}}{\epsilon_0}\oint\text{r}^3\text{dr}=\frac{4\pi\text{K}}{\epsilon_0}\frac{\text{R}^4}{4}
Which given, \text{E}=\frac{\text{K}}{4\epsilon_0}\frac{\text{r}^4}{\text{r}^2}
Here also the charge density is again positive. So, the direction of E is radially outward.
Q469. Three identical metal plates with large surface areas are kept parallel to each other as shown in figure. The leftmost plate is given a charge Q, the rightmost a 3 Marks
charge -2Q and the middle one remains neutral. Find the charge appearing on the outer surface of the rightmost plate.

Ans:

https://bls.smartstudies.co.in/#/exam/pdf-preview/b5b87181-b79b-4b4a-8ebf-8f94fb605fc8/1 55/139
5/17/24, 8:14 PM Exam Automation

Consider the Gaussian surface as shown in the figure.


Let the charge on the outer surface of the left-most plate be q. Thus, the charges on the plates are distributed as shown in the diagram.
The net field at point P due to all the induced charges must be zero, as it is lying inside the metal surface.
Let the surface area of the plates be A.
Electric field at point P due to the charges on plate X:
Due to charge (+Q - q) is \frac{\text{Q}-\text{q}}{2\text{A}\epsilon_0} in the right direction
Due to charge (+q) is \frac{\text{q}}{2\text{A}\epsilon_0} in the right direction
Electric field at point P due to charges on plate Y:
Due to charge (-q) is \frac{\text{q}}{2\text{A}\epsilon_0} in the right direction
Due to charge (+q) is \frac{\text{q}}{2\text{A}\epsilon_0} in the left direction
Electric field at point P due to charges on plate Z:
Due to charge (-q) is \frac{\text{q}}{2\text{A}\epsilon_0} in the right direction
Due to charge (-2Q + q) is \frac{\text{2Q}-\text{q}}{2\text{A}\epsilon_0} in the right direction
The net electric field at point P:
\frac{\text{Q}-\text{q}}{2\text{a}\epsilon_0}+\frac{\text{q}}{2\text{A}\epsilon_0}-\frac{\text{q}}{2\text{A}\epsilon_0}-\frac{\text{q}}
{2\text{A}\epsilon_0}+\frac{\text{q}}{2\text{A}\epsilon_0}+\frac{\text{2Q}-\text{q}}{2\text{A}\epsilon_0}=0
\frac{\text{Q}-\text{q}}{2\text{A}\epsilon_0}+\frac{\text{2Q}-\text{q}}{2\text{A}\epsilon_0}=0
\text{Q}-\text{q}+\text{2Q}-\text{q}=0
\text{3Q}-2\text{q}=0
\text{q}=\frac{\text{3Q}}{2}
Thus, the charge on the outer plate of the right-most plate
-2\text{Q}+\text{q}=-2\text{Q}+\frac{3\text{Q}}{2}=-\frac{\text{Q}}{2}
Q470. Define electric field intensity. Write its SI unit. Write the magnitude and direction of electric field intensity due to an electric dipole of length 2a at the midpoint of 3 Marks
the line joining the two charges.

Ans:Electric Field Intensity: The electric field intensity at any point in an electric field is defined as the electric force per unit positive test charge placed at that point
i.e.,
\vec{\text{E}}=\lim\limits_{\text{q}_0\rightarrow0}\frac{\vec{\text{F}}}{\text{q}_0}
The test charge q0 has to be vanishingly small so that it does not affect the electric field of the main charge.
The SI unit of electric field intensity is newton/ coulomb.

Electric Field Strength at mid-point of dipole: The electric field strength at mid-point C due to charge +q is -q along the same direction.
E = E 1 + E2
=\frac{1}{4\pi\epsilon_0}\frac{\text{q}}{\text{a}^2}+\frac{1}{4\pi\epsilon_0}\frac{\text{q}}{\text{a}^2}=\frac{1}{4\pi\epsilon_0}\frac{2\text{q}}{\text{a}^2}
Its direction is from +q to -q.
Q471. Two particles A and B having charges of +2.00 × 10-6C and of -4.00 × 10-6C respectively are held fixed at a separation of 20.0cm. Locate the point(s) on the line 3 Marks
AB where
1. The electric field is zero.
2. The electric potential is zero.

Ans:\text{q}_2=2\times10^{-6}\text{C},\ \text{q}_1^2=-4\times10^{-6}\text{C},
\text{r}=20\text{cm}=0.2\text{m}
(E1 = electric field due to q1, E2 = electric field due to q2)
\Rightarrow\frac{(\text{r}-\text{x})^2}{\text{x}^2}=\frac{-\text{q}_2}{\text{q}_1}\Rightarrow\frac{(\text{r}-1)^2}{\text{x}}=\frac{-\text{q}_2}{\text{q}_1}
=\frac{4\times10^{-6}}{2\times10^{-6}}=\frac{1}{2}
\Rightarrow\Big(\frac{\text{r}}{\text{x}}-1\Big)=\frac{1}{\sqrt{2}}
=\frac{1}{1.414}
\Rightarrow\frac{\text{r}}{\text{x}}=1.414+1
=2.414
\Rightarrow\text{x}=\frac{\text{r}}{2.414}
=\frac{20}{2.414}
=8.285\text{cm}
Q472. Given a uniformly charged plane/sheet of surface charge density, \sigma=2\times10^{17}\text{C/m}^2. 3 Marks
Image
1. Find the electric field intensity at a point A, 5mm away from the sheet on the left side.
2. Given a straight line with three points X, Y and Z placed 50cm away from the charged sheet on the right side. At which of these points, the field due to the
sheet remain the same as that of point A and why?

Ans:1. At A, \text{E}=\frac{\sigma}{2\epsilon_0}=\frac{2\times10^{17}\text{Cm}^{-2}}{2\times8.854\times10^{-12}\text{C}^2\text{N}^{-1}\text{m}^{-2}}
\text{E}=1.1\times1028 \text{N/C}
Directed away from the sheet.

https://bls.smartstudies.co.in/#/exam/pdf-preview/b5b87181-b79b-4b4a-8ebf-8f94fb605fc8/1 56/139
5/17/24, 8:14 PM Exam Automation
2. Point Y, Because at 50cm, the charge sheet acts as a finite sheet and thus the magnitude remains same towards the middle region of the planar sheet.
Q473. Two particles, carrying charges -q and +q and having equal masses m each, are fixed at the ends of a light rod of length a to form a dipole. The rod is clamped at 3 Marks
an end and is placed in a uniform electric field E with the axis of the dipole along the electric field. The rod is slightly tilted and then released. Neglecting gravity
find the time period of small oscillations.

Ans:Consider the rod to be a simple pendulum.


For simple pendulum,

\text{T}=2\pi\sqrt{\frac{\ell}{\text{g}}} \big(\ell= length, q = acceleration\big)


Now, force experienced by the charges
F = Eq Now, acceleration =\frac{\text{F}}{\text{m}}
=\frac{\text{Eq}}{\text{m}}
Hence length = a
so, Time period =2\pi\sqrt{\frac{\text{a}}{\Big(\frac{\text{Eq}}{\text{m}}\Big)}}
=2\pi\sqrt{\frac{\text{ma}}{\text{Eq}}}
Q474. If a charge is placed at rest in an electric field, will its path be along a line of force? Discuss the situation when the lines of force are straight and when they are 3 Marks
curved.

Ans:If a charge is placed at rest in an electric field, its path will be tangential to the lines of force. When the electric field lines are straight lines then the tangent to them
will coincide with the electric field lines so the charge will move along them only. When the lines of force are curved, the charge moves along the tangent to them.
Q475. A polythene piece rubbed with wool is found to have a negative charge of 3 × 10-7 C. 3 Marks
1. Estimate the number of electrons transferred (from which to which?)
2. Is there a transfer of mass from wool to polythene?

Ans:1. When polythene is rubbed against wool, a number of electrons get transferred from wool to polythene. Hence, wool becomes positively charged and polythene
becomes negatively charged.
Amount of charge on the polythene piece, q = -3 × 10-7 C
Amount of charge on an electron, e = -1.6 × 10-19 C
Number of electrons transferred from wool to polythene = n
n can be calculated using the relation,
q = ne
\text{n} = \frac{\text{q}}{\text{e}}
=\frac{-3\times10^{-7}}{-1.6\times10^{-19}}
= 1.87 × 1012
Therefore, the number of electrons transferred from wool to polythene is 1.87 × 1012.
2. Yes.
There is a transfer of mass taking place. This is because an electron has mass, me = 9.1 × 10-3 kg
Total mass transferred to polythene from wool, m = me × n
= 9.1 × 10-31 × 1.85 × 1012
= 1.706 × 10-18 kg
Hence, a negligible amount of mass is transferred from wool to polythene.
Q476. In the figure given below, find the: 3 Marks
1. Equivalent capacitance of the network between points A and B.
Given: \text{C}_1=\text{C}_5=8\mu\text{F},\text{C}_2=\text{C}_3=\text{C}_4=4\mu\text{F}.
2. Maximum charge supplied by the battery.
3. Total energy stored in the network.

Ans:1. We need to calculate the equivalent capacitance According to diagram,


The equivalent capacitance between A and B is 2\mu\text{F}. Because the potential shifted A to D and B to E.

https://bls.smartstudies.co.in/#/exam/pdf-preview/b5b87181-b79b-4b4a-8ebf-8f94fb605fc8/1 57/139
5/17/24, 8:14 PM Exam Automation
So, The equivalent capacitance will be 2\mu\text{F}.
2. We need to calculate the maximum charge.
Using formula of charge:
Q = CV
Where, C = capacitance
V = potential
Put the value into the formula,
Q=2×5
Q = 10C
3. We need to calculate the total energy stored in the network.
Using formula of store energy:
\text{E}=\frac12\text{CV}^2
Put the value into the formula,
\text{E}=\frac12\times2\times5^2
\text{E}=25\text{J}.
Q477. A spherical volume contains a uniformly distributed charge of density 2.0 × 10-4Cm-3. Find the electric field at a point inside the volume at a distance 4.0cm from 3 Marks
the centre.

Ans:Given :
Volume charge density, \rho=2\times10^{-4}\text{C/m}^3
Let us assume a concentric spherical surface inside the given sphere with radius = 4cm = 4 × 10-2m
The charge enclosed in the spherical surface assumed can be found by multiplying the volume charge density with the volume of the sphere. Thus,
\text{q}=\rho\times\frac{4}3{}\pi\text{r}^3
\Rightarrow\text{q}=\big(2\times10^{-4}\big)\times\frac{4}{3}\pi\text{r}^3
The net flux through the spherical surface,
\phi=\frac{\text{q}}{\epsilon_0}
The surface area of the spherical surface of radius r cm:
\text{A}=4\pi\text{r}^2
Electric field,
\text{E}=\frac{\text{q}}{\epsilon_0\times\text{A}}
\text{E}=\frac{2\times10^{-4}\times4\pi\text{r}^3}{\epsilon_0\times3\times4\pi\text{r}^2}
\text{E}=\frac{2\times10^{-4}\times\text{r}}{3\times\epsilon_0\times}
The electric field at the point inside the volume at a distance 4.0cm from the centre,
\text{E}=\frac{(2\times10^{-4})\times(4\times10^{-2})}{3\times(8.85\times10^{-12})}\text{N/C}
\text{E}=3.0\times10^{5}\text{N/C}
Q478. Two identically charged particles are fastened to the two ends of a spring of spring constant 100Nm-1 and natural length 10cm. The system rests on a smooth 3 Marks
horizontal table. If the charge on each particle is 2.0 × 10-8C, find the extension in the length of the spring. Assume that the extension is small as compared to the
natural length. Justify this assumption after you solve the problem.

Ans:\text{K}=100\text{N/m},\ \ell=10\text{cm}=10^{-1}\text{m}
\text{q}=2.0\times10^{-8}\text{c},\ \text{Find}\ell=?

Force between them \text{F}=\frac{\text{kq}_1\text{q}_2}{\text{r}^2}


=\frac{9\times10^92\times10^{-8}\times2\times10^{-8}}{10^{-2}}
=36\times10^{-5}\text{N}
So, \text{F}=-\text{kx} or \text{x}=\frac{\text{F}}{-\text{K}}
=\frac{36\times10^{-5}}{100}
=36\times10^{-7}\text{cm}
=3.6\times10^{-6}\text{m}
Q479. Consider two hollow concentric spheres, S1 and S2, enclosing charges 2Q and 4Q respectively as shown in the figure. 3 Marks
1. Find out the ratio of the electric flux through them.
2. How will the electric flux through the sphere S1 change if a medium of dielectric constant '\epsilon_\text{r}' is introduced in the space inside S1 in place of air?
Deduce the necessary expression.

Ans:Using Gauss's Theorem \oint\vec{\text{E}}.\vec{\text{ds}}=\frac{\text{q(T)}}{\epsilon_0}


Electric flux through sphere S1, \Phi_1=\frac{2(\text{Q})}{\epsilon_0}
Electric flux through sphere S2, \Phi=\frac{(2\text{Q}+4\text{Q})}{\epsilon_0}=\frac{6\text{Q}}{\epsilon_0}
\text{Ratio}=\frac{\Phi_1}{\Phi}=\frac{\frac{2\text{Q}}{\epsilon_0}}{\frac{6\text{Q}}{\epsilon_0}}=\frac{1}{3}
If a medium of dielectric constant \text{K}(=\epsilon_\text{r}) is filled in the sphere S1, electric flux through sphere,
\Phi'_1=\frac{2\text{Q}}{\epsilon_\text{r}\epsilon_0}=\frac{2\text{Q}}{\text{K}\epsilon_0}.
Q480. A spherical shell made of plastic, contains a charge Q distributed uniformly over its surface. What is the electric field inside the shell? If the shell is hammered to 3 Marks
deshape it without altering the charge, will the field inside be changed? What happens if the shell is made of a metal?
https://bls.smartstudies.co.in/#/exam/pdf-preview/b5b87181-b79b-4b4a-8ebf-8f94fb605fc8/1 58/139
5/17/24, 8:14 PM Exam Automation

Ans:As the shell is made of plastic, it is non-conducting. But as the charge is distributed uniformly over the surface of the shell, the sum of all the electric field vectors
at the centre due to this kind of distribution is zero. But when the plastic shell is deformed, the distribution of charge on it becomes non-uniform. In other words,
the sum of all the electric field vectors is non-zero now or the electric field exists at the centre now.
In case of a deformed conductor, the field inside is always zero.
Q481. Which among the curves shown in figure cannot possibly represent electrostatic field lines? 3 Marks

Ans:1. Field lines are wrongly drawn because electric field lines must be normal to the surface of the conductor at each point.
2. Field lines are wrongly drawn because field lines cannot start from a negative charge.
3. Field lines are correctly drawn, because they are originating from a positive charge.
4. Field lines are wrongly drawn as the field lines cannot intersect.
5. Field lines are wrongly drawn because they cannot form closed loops.
Q482. When a charged comb is brought near a small piece of paper, it attracts the piece. Does the paper become charged when the comb is brought near it? 3 Marks

Ans:When a charged comb is brought near a small piece of paper, it attracts the piece due to induction. There's a distribution of charges on the paper. When a charged
comb is brought near the pieces of paper then an opposite charge is induced on the near end of the pieces of paper so the charged comb attracts the opposite
charge on the near end of paper and similar on the farther end. The net charge on the paper remains zero.
Q483. Three equal charges, 2.0 × 10-6C each, are held fixed at the three corners of an equilateral triangle of side 5cm. Find the Coulomb force experienced by one of the 3 Marks
charges due to the rest two.

Ans:

Three charges are held at three corners of a equilateral trangle.


Let the charges be A, B and C.
It is of length 5cm or 0.05m
Force exerted by B on A = F1
Force exerted by C on A = F2
So, force exerted on A = resultant F1 = F2
\Rightarrow\text{F}=\frac{\text{kq}_2}{\text{r}^2}
=\frac{9\times10^9\times2\times2\times2\times10^{-12}}{5\times5\times10^{-4}}
=\frac{36}{25}\times10
=14.4
Now, force on A = 2 × F cos 30° since it is equilateral \triangle.
⇒ Force on \text{A}=2\times1.44\times\sqrt{\frac{3}{2}}
=24.94\text{N}.
Q484. Careful measurement of the electric field at the surface of a black box indicates that the net outward flux through the surface of the box is 8.0 × 103 Nm2 /C. 3 Marks
1. What is the net charge inside the box?
2. If the net outward flux through the surface of the box were zero, could you conclude that there were no charges inside the box? Why or Why not?

Ans:1. Net outward flux through the surface of the box, (\phi) = 8.0 x 103 Nm2/C
For a body containing net charge a, flux is given by the relation,
\phi = \frac{\text{q}}{\in_0}
\in_0 = Permittivity of free space
= 8.854 × 10-12 × 8.0 × 103
q = \in_0\phi
= 8.854 × 10-12 × 8.0 × 103
https://bls.smartstudies.co.in/#/exam/pdf-preview/b5b87181-b79b-4b4a-8ebf-8f94fb605fc8/1 59/139
5/17/24, 8:14 PM Exam Automation
-8
= 7.08 × 10
= 0.07 µC
Therefore, the net charge inside the box is 0.07 µC.
2. No
Net flux piercing out through a body depends on the net charge contained in the body. If net flux is zero, then it can be inferred that net charge inside the body is
zero. The body may have equal amount of positive and negative charges.
Q485. 1. Two electric field lines cannot cross each other. Also, they cannot form closed loops. Give reasons. 3 Marks
2. A particle of charge 2\mu\text{C} and mass 1.6g is moving with a velocity \text{4}\hat{\text{i}}\text{ ms}^{-1}. At t = 0 the particle enters in a region having an
electric field \vec{\text{E}} (in NC-1) =80\hat{\text{i}}+60\hat{\text{j}}. Find the velocity of the particle at t = 5 s.

Ans:1. Electric field does not direct in two directions that is why electric field lines never cross over each other. Electric field lines always point in one direction, at any
point. When two lines intersect each other, tangents are drawn at that point indicating two directions of electric field lines, which is impossible therefore electric
field lines cannot cross over each other.
2. Given that,
\text{q}=\text{charge}=2\mu\text{C}=2\times10^{-6}\text{C}
Mass =\text{m}=1.6\text{g}=1.6\times10^{-3}\text{kg}
\text{V}=\text{velocity}=\text{4}\hat{\text{i}}\text{m}/\text{s}
\hat{\text{E}}=(80\hat{\text{i}}+60\hat{\text{j}})\frac{\text{N}}{\text{C}}
Since particle has initial velocity only in x-direction.
\text{u}_\text{x}=\text{v}=\text{4m/ s};\text{u}_\text{y}=0\text{m/ s}
Force =\text{F}=\text{q}\vec{\text{E}}
\Rightarrow\vec{\text{F}}=2\times10^{-6}(80\hat{\text{i}}+60\hat{\text{j}})=(160\hat{\text{i}}+120\hat{\text{j}})\times10^{-6}
\vec{\text{a}}=\text{acceleration}=\vec{\text{a}}=\frac{\vec{\text{f}}}{\text{m}}=\frac{(160\hat{\text{i}+120\hat{\text{j}}})\times10^{-6}}{1.6\times10^{-3}}
\vec{\text{a}}=(100\hat{\text{i}}+75\hat{\text{j}})\times10^{-3}=(\text{a}_\text{x}\hat{\text{i}}+\text{a}_\text{y}\hat{\text{j}})
Time = t = 5sec
\text{u}_\text{x}=\text{u}_\text{x}+\text{a}_\text{x}\text{t}=0+75\times10^{-3}\times5=0.375\text{m/ s}
Where ux, vz are initial and final velocity in x-direction and uy and vy are initial and final velocity in y-direction.
\therefore Final velocity \text{u}=\text{v}_\text{x}\hat{\text{i}}+\text{v}_\text{y}\hat{\text{j}}=4.5\hat{\text{i}}+0.375\hat{\text{j}}
\text{v}={(4.5\hat{\text{i}}+0.375\hat{\text{j}})}\text{m/ s}
Q486. Two large, thin metal plates are parallel and close to each other. On their inner faces, the plates have surface charge densities of opposite signs and of magnitude 3 Marks
17.0 × 10–22 C/m2. What is E:
1. In the outer region of the first plate.
2. In the outer region of the second plate, and
3. Between the plates?

Ans:Given,
Surface charge density, \sigma = 17. 0 x 10-22 C /m2
1. To the left of the plates, electric fields are equal and opposite as plates are close to each other electric field is zero as surface charge density in outer side is
zero.
2. To the right of the plates, electric fields are equal and opposite as plates are close to each other electric field is zero.
3. Electric fields between the plates are in same direction as total E.F. on both sides of plate due to \sigma surface charge density = \frac{\sigma}
{\epsilon_0} electric field of inner side of plate =\frac{\sigma}{2\epsilon_0}
and for both plate E =\frac{\sigma}{2\epsilon_0}+\frac{\sigma}{2\epsilon_0}
\text{E}=\frac{\sigma}{\epsilon_0}=\sigma\times4\pi\times9\times10^9
E = 17.0 × 10-22 × 4 × 3.14 × 9 × 109
E = 1921.7 × 10-13
= 1.92 × 10-10 N/C.
Q487. Suppose an attractive nuclear force acts between two protons which may be written as \text{F}=\text{Ce}^{-\text{kr}}/\text{r}^2. 3 Marks
1. Write down the dimensional formulae and appropriate SI units of C and k.
2. Suppose that k = 1 fermi-1 and that the repulsive electric force between the protons is just balanced by the attractive nuclear force when the separation is 5
fermi. Find the value of C.

Ans:Expression of electrical force \text{F}=\text{C}\times\text{e}^{\frac{-\text{kr}}{\text{r}^2}}


Since e-kr is a pure number. So, dimensional formulae of \text{F}=\frac{\text{dimensional formulae of C}}{\text{dimensional formulae of r}^2}
Or, \big[\text{MLT}^{-2}\big]\big[\text{L}^2\big]= dimensional formulae of \text{C}=\big[\text{ML}^3\text{T}^{-2}\big]
Unit of C = unit of force × unit of r2 = Newton × m2 = Newton-m2
Since -kr is a number hence dimensional formulae of
\text{k}=\frac{1}{\text{dim entional formulae of r}}=\big[\text{L}^{-1}\big] Unit of k = m-1
Q488. A charged particle having a charge of -2.0 × 10-6C is placed close to a nonconducting plate having a surface charge density 4.0 × 10-5Cm-2. Find the force of 3 Marks
attraction between the particle and the plate.

Ans:The electric field due to a conducting thin sheet,


\text{E}=\frac{\sigma}{2\epsilon_0}
The magnitude of attractive force between the particle and the plate,
\text{F}=\text{qE}
\text{F}=\frac{\text{q}\times\sigma}{2\epsilon_0}
\text{F}=\frac{\big(2.0\times10^{-6}\big)\times\big(4.0\times10^{-6}\big)}{2\times\big(8.85\times10^{-12}\big)}
\text{F}=0.45\text{N}

https://bls.smartstudies.co.in/#/exam/pdf-preview/b5b87181-b79b-4b4a-8ebf-8f94fb605fc8/1 60/139
5/17/24, 8:14 PM Exam Automation

Q489. Two identical particles, each having a charge of 2.0 × 10-4C and mass of 10g, are kept at a separation of 10cm and then released. What would be the speeds of the 3 Marks
particles when the separation becomes large?

Ans:

\text{m}=10\text{g}
\text{F}=\frac{\text{KQ}}{\text{r}}
=\frac{9\times10^9\times2\times10^{-4}}{10\times10^{-2}}
\text{F}=1.8\times10^{-7}
\text{F}=\text{m}\times\text{a}
\Rightarrow\text{a}=\frac{1.8\times10^{-7}}{10\times10^{-3}}
=1.8\times10^{-3}\text{m/s}^2
\text{V}^2-\text{u}^2=2\text{as}
\Rightarrow\text{V}^2=\text{u}^2+2\text{as}
\text{V}=\sqrt{0+2\times1.8\times10^{-3}\times10\times10^{-2}}
=\sqrt{3.6\times10^{-4}}
=0.6\times10^{-2}
=6\times10^{-3}\text{,m/s}.
Q490. Does the force on a charge due to another charge depend on the charges present nearby? 3 Marks

Ans:Coulomb's Law states that the force between two charged particle is given by,
\text{F}=\frac{\text{q}_1\text{q}_2}{4\pi\in_0\text{r}^2}
Where,
q1 and q2 are the charges on the charged particles .
r = separation between the charged particles.
\in_0 = parmittivity of free space.
According to the Law of Superposition, the electrostatic force between two charged particles are unaffected due to the presence of other charges.
Q491. Twelve negative charges of same magnitude are equally spaced and fixed on the circumference of a circle of radius R as shown in Fig. (i). Relative to potential 3 Marks
being zero at infinity, find the electric potential and electric field at the centre C of the circle. (b) If the charges are unequally spaced and fixed on an arc of 120° of
radius R as shown in Fig. (ii), find electric potential at the centre C.

Ans:1. Finding electric potential at the centre Finding electric field at the centre
Finding electric potential at the centre
Electric potential due to point charge
\text{V}=\frac{\text{kq}}{\text{R}}
Value of each charge = - q, Total charge = -12q
Total potential \text{V}=\frac{\text{k(-12}\text{q)}}{\text{R}}
\text{V}=\frac{-12\text{ kq}}{\text{R}}=\frac{-12\text{q}}{4\pi\in_0\text{R}}
By symmetry the resultant of all electric field vectors becomes zero. So electric field is zero.
2. Electric potential is a scalar quantity and does not depend on placement of charges
Therefore \text{V}=\frac{-12\text{ kq}}{\text{R}}=\frac{-12\text{q}}{4\pi\in_0\text{R}}
Q492. A 10cm long rod carries a charge of +50\mu\text{C} distributed uniformly along its length. Find the magnitude of the electric field at a point 10cm from both the 3 Marks
ends of the rod.

Ans:

https://bls.smartstudies.co.in/#/exam/pdf-preview/b5b87181-b79b-4b4a-8ebf-8f94fb605fc8/1 61/139
5/17/24, 8:14 PM Exam Automation

\text{G}=50\mu\text{C}=50\times10^{-6}\text{C}
We have, \text{E}=\frac{2\text{KQ}}{\text{r}} for a charged cylinder
\Rightarrow\text{E}=\frac{2\times9\times10^9\times50\times10^{-6}}{5\sqrt{3}}
=\frac{9\times10^{-5}}{5\sqrt{3}}
=1.03\times10^{-5}
Q493. A signal of low frequency fm is to be transmitted using a carrier wave of frequency fc. Derive the expression for the amplitude modulated wave and deduce 3 Marks
expressions for the lower and upper sidebands produced. Hence, obtain the expression for modulation index.

Ans:Let a carrier wave be given by


\text{c}(\text{t})=\text{A}_\text{c}\sin\omega_\text{c}\text{t} where \omega_\text{c}=2\pi\text{f}_\text{c}
And signal wave be
\text{m}(\text{t})=\text{A}_\text{m}\sin\omega_\text{m}\text{t} where \omega_\text{m}=2\pi\text{f}_\text{m}
The modulated signal is
\text{C}_\text{m}(\text{t})=(\text{A}_\text{c}+\text{A}_\text{m}\sin\omega_\text{m}\text{t})\sin\omega_\text{c}\text{t}
\text{C}_\text{m}(\text{t})=\text{A}_\text{c}\Big(1+\frac{\text{A}_\text{m}}{\text{A}_\text{c}}\sin\omega_\text{c}\text{t}\Big)\sin\omega_\text{c}\text{t}
\text{C}_\text{m}(\text{t})=\text{A}_\text{c}\sin\omega_\text{c}\text{t}+\mu\frac{\text{A}_\text{c}}{2}\cos(\omega_\text{c}-\omega_\text{m})\\\text{t}-
\mu\frac{\text{A}_\text{c}}{2}\cos(\omega_\text{c}+\omega_\text{m})\text{t}
The modulation index \mu=\frac{\text{A}_\text{m}}{\text{A}_\text{c}}
Lower frequency band \omega_\text{c}-\omega_\text{m}
Upper frequency band \omega_\text{c}+\omega_\text{m}
Q494. You are given three capacitors of 2\mu\text{F}, 3\mu\text{F} and 4\mu\text{F}, respectively. 3 Marks
1. Form a combination of all these capacitors of equivalent capacitance \frac{13}{3}\mu\text{F}.
2. What is the maximum and minimum value of the equivalent capacitance that can be obtained by connecting these capacitors ?

Ans:1.

2. \text{C}_{\text{max}}=2+3+4=9\mu\text{F}
\frac{1}{\text{c}_{\text{min}}}=\frac{1}{2}+\frac{1}{3}+\frac{1}{4}=\frac{12}{13}\mu\text{F}
Q495. Estimate the number of electrons in 100g of water. How much is the total negative charge on these electrons? 3 Marks

Ans:Molecular mass of water = 18g


Number of molecules in 18g of H2O = Avogadro's number
= 6.023 × 1023
Number of electrons in 1 molecule of H2O = (2 × 1) + 8 = 10
Number of electrons in 6.023 × 1023 molecules of H2O = 6.023 × 1024
That is, number of electrons in 18g of H2O = 6.023 × 1024
So, number of electrons in 100g of H2O =\frac{6.023\times10^{24}}{18}\times100
=3.34\times10^{25}
\therefore Total charge =3.34\times10^{25}\times1.6\times10^{-19}
=5.34\times10^6\text{C}
Q496. A point charge +10 μC is a distance 5cm directly above the centre of a square of side 10cm, as shown in Fig. 1.34. What is the magnitude of the electric flux 3 Marks
through the square?
(Hint: Think of the square as one face of a cube with edge 10cm.)

https://bls.smartstudies.co.in/#/exam/pdf-preview/b5b87181-b79b-4b4a-8ebf-8f94fb605fc8/1 62/139
5/17/24, 8:14 PM Exam Automation

Ans:The square can be considered as one face of a cube of edge 10cm, with a centre where charge q is placed. According to Gauss's theorem for a cube, total electric
flux is through all its six faces.
\phi_{\text{Total}}=\frac{\text{q}}{\in_0}
Hence, electric flux through one face of the cube i.e., through the square, \phi=\frac{\phi_{\text{Total}}}{6}
=\frac{1}{6}\frac{\text{q}}{\in_0}
Where,
\in_0 = Permittivity of free space
= 8.854 × 10-12 N-1C-1m-2
q = 10 µC = 10 × 10-6 C
\therefore\phi=\frac{1}{6}\times\frac{10\times10^{-6}}{8.854\times10^{-12}}
= 1.88 × 105 Nm2C-1
Therefore, electric flux through the square is 1.88 x 105 Nm2C-1.
Q497. What is the force between two small charged spheres having charges of 2 × 10-7C and 3 × 10-7C placed 30cm apart in air? 3 Marks

Ans:Repulsive force of magnitude 6 × 10-3 N


Charge on the first sphere, q1 = 2 × 10-7 C
Charge on the second sphere, q2 = 3 × 10-7 C
Distance between the spheres, r = 30 cm = 0.3 m
Electrostatic force between the spheres is given by the relation,
\text{F}=\frac{\text{q}_1\text{q}_2}{4\pi\in_0\text{r}^2}
Where, \in_0 = Permittivity of free space
\frac{1}{4\pi\in_0}=9\times10^9\text{Nm}^2\text{C}^{2}
\text{F}=\frac{9\times10^9\times2\times10^{-7}\times3\times10^{-7}}{(0.3)^2}=6\times10^{-3}\text{N}
Hence, force between the two small charged spheres is 6 × 10-3 N. The charges are of same nature. Hence, force between them will be repulsive.
Q498. 1. Derive the expression for the capacitance of a parallel plate area A and plate separation d. 3 Marks
2. Two charged spherical conductors of radii R1 and R2 when conducting wire acquire charges q1 and q2 respectively. surface charge densities in terms of their
radii.

Ans:1.

https://bls.smartstudies.co.in/#/exam/pdf-preview/b5b87181-b79b-4b4a-8ebf-8f94fb605fc8/1 63/139
5/17/24, 8:14 PM Exam Automation

Electric field between the plates of capacitor E = \frac{\sigma}{\varepsilon_{0}} =\frac{\text{Q}}{\text{A}\varepsilon_{0}}


\therefore potential difference
\text{V} = \text{Ed} = \frac{\text{Qd}}{\text{A}\varepsilon_{0}}
Capacitance
\text{C} = \frac{\text{Q}}{\text{V}} = \frac{\varepsilon_{0}\text{A}}{\text{d}}
2. When the two charged spherical conductors are connected by a conducting wire , they acquire the same potential
i.e \frac{\text{Kq}_{1}}{\text{R}_{1}} = \frac{\text{Kq}_{2}}{\text{R}_{2}}\Rightarrow\frac{\text{q}_{1}}{\text{q}_{2}} = \frac{\text{R}_{1}}{\text{R}_{2}}
Hence, ratio of surface charge densities
\frac{\sigma_{1}}{\sigma_{2}} = \frac{\text{q}_{1}/4\pi\text{R}_{1}^{2}}{\text{q}_{2}/4\pi\text{R}_{2}^{2}}
= \frac{\text{q}_{1}\text{R}_{2}^{2}}{\text{q}_{2}\text{R}_{1}^{2}}
= \frac{\text{R}_{1}}{\text{R}_{2}}\times\frac{\text{R}_{2}^{2}}{\text{R}_{1}^{2}} = \frac{\text{R}_{2}}{\text{R}_{1}}.
Q499. A conducting sphere of radius 10 cm has an unknown charge. If the electric field 20 cm from the centre of the sphere is 1.5 × 103 N/C and points radially inward, 3 Marks
what is the net charge on the sphere?

Ans:Given,
Radius of conducting sphere, r = 10 cm = 0. 1 m
Electric field , E = 1.5 x 103 N/C at distance, d = 20 cm = 0. 2 m
As, \text{E}=\frac{1}{4\pi\in_0}\frac{\text{q}}{\text{r}^2}
\Rightarrow \text{q = E.}\ 4\pi\epsilon_0.\text{r}^2
q = 1.5 × 103 × 4\pi × 8.854 × 10-12 × (0.2)2
q = 6.67 × 10-9 C
Here, since electric field is directed radially inward charge q is negative.
Thus,
q = - 6. 67 x 10-9 C
= -6.67 nC.
Q500. The number of electrons in an insulator is of the same order as the number of electrons in a conductor. What is then the basic difference between a conductor and 3 Marks
an insulator?

Ans:The outer electrons of an atom or molecule in a conductor are only weakly bound to it and are free to move throughout the body of the material.
On the other hand, in insulators, the electrons are tightly bound to their respective atoms and cannot leave their parent atoms and move through a long distance.
Q501. Two insulating small spheres are rubbed against each other and placed 1cm apart. If they attract each other with a force of 0.1N, how many electrons were 3 Marks
transferred from one sphere to the other dunng rubbing?

Ans:\text{F}=0.1\text{N}
\text{r}=1\text{cm}=10^{-2} (As they rubbed with each other. So the charge on each sphere are equal)
So, \text{F}=\frac{\text{kq}_1\text{q}_2}{\text{r}^2}
\Rightarrow0.1=\frac{\text{kq}^2}{(10^{-2})^2}
\Rightarrow\text{q}^2=\frac{0.1\times10^{-4}}{9\times10^9}
\Rightarrow\text{q}^2=\frac{1}{9}\times10^{-14}
\Rightarrow\text{q}=\frac{1}{3}\times10^{-7}
1.6\times10^{-19}\text{c} Carries by 1 electron
1 c carried by \frac{1}{1.6\times10^{-19}}
0.33\times10^{-7} c carries by
\frac{1}{1.6\times10^{-19}} \times0.33\times10^{-7}
=0.208\times10^{12}
=2.08\times10^{11}
Q502. 1. Draw the equipotential surfaces due to an electric dipole. 3 Marks
2. Derive an expression for the electric field due to a dipole of dipole moment \overrightarrow{\text{p}} at a point on its perpendicular bisector.

Ans:1.

https://bls.smartstudies.co.in/#/exam/pdf-preview/b5b87181-b79b-4b4a-8ebf-8f94fb605fc8/1 64/139
5/17/24, 8:14 PM Exam Automation

2.

The magnitude of the electric fields due to the two charges +q and -q are
\text{E}_\text{+q}=\frac{1}{4\pi\epsilon_0}\frac{\text{q}}{(\text{r}^2+\text{a}^2)}
\text{E}_\text{-q}=\frac{1}{4\pi\epsilon_0}\frac{\text{q}}{(\text{r}^2+\text{a}^2)}
The components normal to the dipole axis cancel away and the components along the dipole axis add up Hence total Electric field =-
(\text{E}_\text{+q}+\text{E}_\text{-q})\cos\hat{\text{p}}
\text{E}=-\frac{2\text{qa}}{4\pi\epsilon_0(\text{r}^2+\text{a}^2)^\frac{3}{2}}\hat{\text{p}}
Q503. The kinetic energy of a charged particle decreases by 10J as it moves from a point at potential 100V to a point at potential 200V. Find the charge on the particle. 3 Marks

Ans:K.C. decreases by 10J.


Potential = 100v to 200v.
So, change in K.E = amount of work done
\Rightarrow10\text{J}=(200-100)\text{v}\times\text{q}_0
\Rightarrow100\text{q}_0=10\text{v}
\Rightarrow\text{q}_0=\frac{10}{100}
=0.1\text{C}
Q504. Find the speed of the electron in the ground state of a hydrogen atom. The description of ground state is given in the previous problem. 3 Marks

Ans:Fe from previous problem No. 18=8.2\times10^{-8}\text{N},\ \text{Ve}=?


Now, \text{M}_\text{e}=9.12\times10^{31}\text{kg},\ \text{r}=0.53\times10^{-10}\text{m}
Now, \text{Fe}=\frac{\text{M}_\text{e}\text{v}^2}{\text{r}}
\Rightarrow\text{v}^2=\frac{\text{Fe}\times\text{r}}{\text{m}_\text{e}}
=\frac{8.2\times10^{-8}\times0.53\times10^{-10}}{9.1\times10^{-31}}
=0.4775\times10^{13}
=4.775\times10^{12}\text{m}^2/\text{s}^2
\text{v}=2.18\times10^6\text{m/s}

https://bls.smartstudies.co.in/#/exam/pdf-preview/b5b87181-b79b-4b4a-8ebf-8f94fb605fc8/1 65/139
5/17/24, 8:14 PM Exam Automation
Q505. The electrostatic force on a small sphere of charge 0.4 μC due to another small sphere of charge –0.8 μC in air is 0.2 N. 3 Marks
1. What is the distance between the two spheres?
2. What is the force on the second sphere due to the first?

Ans:1. Electrostatic force on the first sphere, F = 0.2 N


Charge on this sphere, q1 = 0.4 µC = 0.4 × 10-6 C
Charge on the second sphere, q2 = -0.8 µC = 0.8 × 10-6 C
Electrostatic force between the spheres is given by the relation,
\text{F}=\frac{\text{q}_1\text{q}_2}{4\pi\in_0\text{r}^2} And \frac{1}{4\pi\in_0}=9\times10^{9}\text{Nm}^2\text{C}^{-2}
Where, \in_0\ = Permittivity of free space
And, \frac{1}{4\pi\in_0} = 9\times10^9\text{Nm}^{-2}\text{C}^{-2}
\text{r}^2=\frac{\text{q}_1\text{q}_2}{4\pi\in_0\text{F}}
= 144 × 10-4
r = \sqrt{144\times10^{-4}}=0.12\text{m}
The distance between the two spheres is 0.12m.
2. Both the spheres attract each other with the same force. Therefore, the force on the second sphere due to the first is 0.2N.
Q506. Define electric flux. Write its SI unit. 3 Marks
Using Gauss’s law, deduce an expression for electric field intensity due to an infinitely long straight uniformly charged wire.

Ans:The electric flux is defined as


\phi_E=\overrightarrow{E}.\overrightarrow{A}={EA}\cos\theta
Its S.I unit is (Nm^{2}C^{-1})
The Gaussian surface is cylindrical and field is radial. At the cylindrical part of the surface, cylindrical part of the surface, \overrightarrow{E} is normal to the
surface at every point and its magnitude is constant (since it depends only on r). By Gauss’s theorem : \oint \overrightarrow{E}.{d}\overrightarrow{S}=\frac{q}
{\epsilon_0}
\therefore(2\pi{rl})=\frac{\lambda{l}}{\epsilon_0}
\text{or}\text{ }\text{ } E=\frac{\lambda}{2\pi\epsilon_0r}

Q507. What is electric flux? Write its S.I. units. 3 Marks


Using Gauss’s theorem, deduce an expression for the electric field at a point due to a uniformly charged infinite plane sheet.

Ans:Electric Flux: Total number of electric lines of force crossing a certain area normally.
Alternate Answer
The surface integral of electric field over a closed surface.
Alternate Answer
\phi = \oint\overrightarrow{\text{E}}.\text{d}\overrightarrow{\text{s}}
1. S.I. Units: Nm2 /C Or V-m
2.

Derivation: \phi= \oint_{s}\overrightarrow{\text{E}}.\overrightarrow{\text{ds}}= \frac{\text{q}}{\varepsilon_\circ}


2\text{EA} =\frac{\sigma\text{A}}{\varepsilon_\circ}
\therefore \text{E} =\frac{\sigma}{2\varepsilon_{\circ}}
Q508. State Gauss's theorem in electrostatics. Apply this theorem to derive an expression for electric field intensity at a point near an infinitely long straight charged wire. 3 Marks

Ans:1. Statement: Net electric flux through to a closed surface is equal to \frac{1}{\varepsilon_\circ} times the total net charge enclosed within the surface.
(If the student just writes \oint\text{E}.\text{ds} =\frac{\text{q}}{\in_\circ}, award )
2. Diagram:-

3. Derivation:-
\oint\text{E}.\text{ds} \int\limits_{s_1}\overline{E}.\text{d}\overline{s}_{1} + \int\limits_{s_2}\overline{E}.\text{ds}_{2} +
\int\limits_{s_3}\overline{E}.\text{ds}_{3}
= 0 + 0 + 2\pi\text{r}\ell

https://bls.smartstudies.co.in/#/exam/pdf-preview/b5b87181-b79b-4b4a-8ebf-8f94fb605fc8/1 66/139
5/17/24, 8:14 PM Exam Automation
Also, \text{q} = \lambda \ell (where \lambda is charge per unit length)
(\text{E}).(2\pi\text{r}\ell) = \frac{1}{\varepsilon_\circ}\lambda\ell OR \text{E}2\pi\text{r}\ell\frac{\text{q}}{\varepsilon_\circ}
\text{E} = \frac{\lambda}{2\pi\varepsilon_\circ\text{r}} OR \text{E} = \frac{\text{q}}{2\pi\varepsilon_\circ\text{r}\ell}.
Q509. A capacitor of unknown capacitance is connected across a battery of V volts. The charge stored in it is 360 \muC. When potential across the capacitor is reduced 3 Marks
by 120 V, the charge stored in it becomes 120 \muC.
Calculate:
1. The potential V and the unknown capacitance C.
2. What will be the charge stored in the capacitor, if the voltage applied had increased by 120 V?

Ans:1. Initial voltage, V1 = V volts and charge stored, Q1 = 360 µC.


Q1 = CV1 …(1)
Changed potential, V2 = V − 120
Q2 = 120 µC
Q2 = CV2 ...(2)
By dividing (2) from (1), we get \frac{\text{Q}_{1}}{\text{Q}_{2}}=\frac{\text{C}{V}_{1}}{\text{C}{V}_{2}} \Rightarrow \frac{360}{120}=\frac{\text{V}}{\text{V}-
{120}}
\therefore {\text{C}}=\frac{\text{Q}_{1}}{\text{V}_{1}}=\frac{{360}\times{10}{^-}{^6}}{180}={2}\times{10}{^-}{^6} \text{F}={2}\mu{\text{F}}
2. If the voltage applied had increased by 120 V, then \text{V}_{3}={180}+{120}={300}\text{V}.
Hence, charge stored in the capacitor, \text{Q}_{3}=\text{C}\text{V}_{3}={2}\times{10}{^-}{^6}\times{300}={600}\mu\text{C}.
Q510. A long charged cylinder of linear charge density +\lambda_1 is surrounded by a hollow coaxial conducting cylinder of linear charge density -\lambda_2. Use 3 Marks
Gauss’s law to obtain expressions for the electric field at a point (i) in the space between the cylinders, and (ii) outside the larger cylinder.

Ans:As Gauss’s Law states:


\oint\overrightarrow{E}.\vec{ds}=\frac{q}{\epsilon_0}

1. \oint\overrightarrow{E_1}.\vec{ds}=\frac{\lambda_1l}{\epsilon_0}
\Longrightarrow\overrightarrow{E_1}=\frac{\lambda_1}{2\pi\epsilon_0r_1}\hat{r_1}
2. \oint\overrightarrow{E_2}.\vec{ds}=\frac{(\lambda_1-\lambda_2)l}{\epsilon_0}
\Longrightarrow\overrightarrow{E_2}=\frac{(\lambda_1-\lambda_2)}{2\pi\epsilon_0r_2}\hat{r_2}
Q511. Two equal charges are placed at a separation of 1.0m. What should be the magnitude of the charges so that the force between them equals the weight of a 50kg 3 Marks
person?

Ans:Let the magnitude of each charge be


Separation between them, r = 1m
Force between them, F = 50 × 9.8 = 490N
By Coulomb's Law force,
\text{F}=\frac{1}{4\pi\in_0}\frac{\text{q}_1\text{q}_2}{\text{r}^2}
\Rightarrow490=9\times10^9\times\frac{\text{q}^2}{1^2}
\Rightarrow\text{q}^2=54.4\times10^{-9}
\Rightarrow\text{q}=\sqrt{54.4\times10^{-9}}
=23.323\times10^{-5}\text{C}
\text{q}=2.3\times10^{-4}\text{C}
Q512. A charge Q is distributed uniformly over a metallic sphere of radius R. Obtain the expressions for the electric field (E) and electric potential (V) at a point 0 < x < R. 3 Marks
Show on a plot the variation of E and V with x for 0 < x < 2R.

Ans:

By Gauss theorem
\oint \overrightarrow{E}.d\vec{s}=\frac{q}{E_0}
q = 0 in interval 0 < x < R
\Rightarrow E=0
E=-\frac{dv}{dr}
\Rightarrow V= constant=\frac{1}{4\pi E_0}\frac{Q}{R}

https://bls.smartstudies.co.in/#/exam/pdf-preview/b5b87181-b79b-4b4a-8ebf-8f94fb605fc8/1 67/139
5/17/24, 8:14 PM Exam Automation

Q513. Three charges are arranged on the vertices of an equilateral triangle as shown in figure. Find the dipole moment of the combination. 3 Marks

Ans:

Let -q & -q are placed at A & C


Where 2q on B
So length of A = d
So the dipole moment = (q × d) = P
So, Resultant dipole moment
\text{P}=\Big[(\text{qd})^2+(\text{qd})^2+2\text{qd}\times\text{qd}\cos60^\circ\Big]^{\frac{1}{2}}

https://bls.smartstudies.co.in/#/exam/pdf-preview/b5b87181-b79b-4b4a-8ebf-8f94fb605fc8/1 68/139
5/17/24, 8:14 PM Exam Automation
=\big[3\text{q}^2\text{d}^2\big]^{\frac{1}{2}}
=\sqrt{3}\text{qd}
=\sqrt{3}\text{p}
Q514. Consider a uniform electric field E = 3 × 103 î N/C. 3 Marks
1. What is the flux of this field through a square of 10 cm on a side whose plane is parallel to the yz plane?
2. What is the flux through the same square if the normal to its plane makes a 60° angle with the x-axis?

Ans:1. Electric field intensity, \vec{\text{E}} = 3 × 103 î N/C


Magnitude of electric field intensity, |\vec{\text{E}}| = 3 × 103 N/C
Side of the square, s = 10 cm= 0.1 m
Area of the square, A = s2 = 0.01 m2
The plane of the square is parallel to the y-z plane. Hence, angle between the unit vector normal to the plane and electric field, \theta = 0°
Flux (\phi) through the plane is given by the relation,
\phi = |\vec{\text{E}}|\text{A}\cos\theta
= 3 × 103 × 0.01 × \cos0^{\circ}
= 30 Nm2/C
2. Plane makes an angle of 60° with the x-axis. Hence, e = 60°
Flux, \phi = |\vec{\text{E}}|\text{A}\cos\theta
= 3 × 103 × 0.01 × \cos60^{\circ}
= 30 × \frac{1}{2} = 15 Nm2/C
Q515. Calculate the wavelength of the second line of Lyman series in a spectrum of hydrogen atom. (Take Rydberg constant, R = 1.1 x 107m-1) 3 Marks

Ans:Calculation of wave length of second line of Lyman series


\frac{1}{\lambda}=\text{R}\Big[\frac{1}{\text{n}^2_1}-\frac{1}{\text{n}^2_2}\Big]
For second line of Lyman series
n1 = 1, n2 = 3
\frac{1}{\lambda}=1.1\times10^7\Big[\frac{1}{1^2}-\frac{1}{3^2}\Big]
=1.1\times10^7\Big[\frac{1}{1^1}-\frac{1}{3^2}\Big]
=1.1\times10^7\times\frac{8}{9}
=\frac{8.8}{9}\times10^7\text{m}^{-1}
\lambda=\frac{9}{8.8}\times10^{-7}
=1.023\times10^{-7}\text{m}
=1023\mathring{\text{A}}
Q516. Figure 1.33 shows tracks of three charged particles in a uniform electrostatic field. Give the signs of the three charges. Which particle has the highest charge to 3 Marks
mass ratio?

Ans:Opposite charges attract each other and same charges repel each other. It can be observed that particles 1 and 2 both move towards the positively charged plate
and repel away from the negatively charged plate. Hence, these two particles are negatively charged. It can also be observed that particle 3 moves towards the
negatively charged plate and repels away from the positively charged plate. Hence, particle 3 is positively charged.
The charge to mass ratio (emf) is directly proportional to the displacement or amount of deflection for a given velocity. Since the deflection of particle 3 is the
maximum, it has the highest charge to mass ratio.
Q517. A charge is distributed uniformly over a ring of radius ‘a’. Obtain an expression for the electric intensity E at a point on the axis of the ring. Hence show that for 3 Marks
points at large distances from the ring, it behaves like a point charge.

Ans:

Net Electric Field at point \text{P} =\int^{2\pi\text{a}}_{\circ}\text{dE}\cos\theta


dE = Electric field due to a small element having charge dq
= \frac{1}{4\pi\varepsilon_\circ}\frac{\text{dq}}{\text{r}^{2}}
Let \lambda = Linear charge density
= \frac{\text{dq}}{\text{dl}}
\text{dq} = \lambda\text{dl}
Hence \text{E} = \int^{2\pi\text{a}}_{\circ}\frac{1}{4\pi\varepsilon_\circ}.\frac{\lambda\text{dl}}{\text{r}^{2}}\times\frac{x}{r} ,
where \cos\theta =\frac{\text{x}}{\text{r}}
= \frac{\lambda\text{x}}{4\pi\varepsilon_\circ\text{r}^{3}}(2\pi\text{a})
=\frac{1}{4\pi\varepsilon_\circ}\frac{\text{Qx}}{(\text{x}^{2} + \text{a}^{2})^{\frac{3}{2}}}

https://bls.smartstudies.co.in/#/exam/pdf-preview/b5b87181-b79b-4b4a-8ebf-8f94fb605fc8/1 69/139
5/17/24, 8:14 PM Exam Automation
where total charge \text{Q} = \lambda\times2\pi\text{a}
At large distance i.e. x >> a
\text{E}\simeq\frac{1}{4\pi\varepsilon_\circ}.\frac{\text{Q}}{\text{x}^{2}}
This is the Electric field due to a point charge at distance x.
Q518. 1. Obtain the expression for the energy stored per unit volume in a charged parallel plate capacitor. 3 Marks
2. The electric field inside a parallel plate capacitor is E. Find the amount of work done in moving a charge q over a closed rectangular loop a b c d a.

Ans:1. Work done by the source of potential, in storing an additional charge (dq) , is
dW = V.dq
But V = q / C
= > \text{dW} =\frac{\text{q}}{\text{C}}\text{dq}
Total work done in storing the charge Q,
\int\text{dw} = \int_{0}^{Q}\frac{\text{q}}{\text{C}}\text{dq}
\text{W} = \frac{1}{\text{C}}\bigg(\frac{\text{q}^{2}}{2}\bigg)_{0}^{Q} = \frac{\text{Q}^{2}}{2\text{C}}
This work is stored as electrostatic energy in the capacitor.
\because\text{Q= CV}, \therefore \text{U} = \frac{1}{2}\text{CV}^{2}
Energy stored per unit volume \frac{\frac{1}{2}\text{CV}^{2}}{\text{Ad}} = \frac{\frac{1}{2}\big(\frac{\varepsilon_{0}\text{A}}{\text{d}}\big)(\text{Ed)}^{2}}
{\text{Ad}}
= \frac{1}{2}\varepsilon_{o}\text{E}^{2}
2. Work done in moving the charge q from a to b , and from c to d is zero because Electric field is perpendicular to the displacement.
Work done from b to c = - Workdone from d to a
\therefore Total work done in moving a charge q over a closed loop = 0.
Q519. 1. Obtain the expression for the torque \overrightarrow\tau experienced by an electric dipole of dipole moment \overrightarrow{\text{P}} in a uniform electric 3 Marks
field\overrightarrow{\text{E}}
2. What will happen if the field were not uniform?

Ans:1.

Force on + q, \overrightarrow{\text{F}}=q\overrightarrow{\text{E}}
Force on - q, \overrightarrow{\text{F}}=-q\overrightarrow{\text{E}}
Magnitude of torque
\tau=q\text{E}\times2a \text{ sin}\theta
\tau=2q\text{a}\times{\text{E}} \text{ sin}\theta
\overrightarrow\tau=\overrightarrow{\text{P}}\times\overrightarrow{\text{E}}
2. If the electric field is non uniform, the dipole experiences a translatory force as well as a torque.
Q520. A thin conducting spherical shell of radius R has charge Q spread uniformly over its surface. Using Gauss’s law, derive an expression for an electric field at a 3 Marks
point outside the shell.
Draw a graph of electric field E(r) with distance r from the centre of the shell for 0\underline{<}\text{r}\underline{<}\infty.

Ans:\oint\overrightarrow{\text{E}}.\overrightarrow{\text{ds}} = \frac{\text{q}}{\varepsilon_\circ}
\text{E}\times4\pi\text{r}^{2} =\frac{\text{Q}}{\varepsilon_\circ}
\text{E} = \frac{1}{4\pi\varepsilon_\circ}\frac{\text{Q}}{\text{r}^{3}}

Q521. Two small identical electric dipoles AB and CD, each of dipole moment \vec{\text{p}} are kept at an angle of 120° to each other in an external electric 3 Marks
field \vec{\text{E}} pointing along the x-axis as shown in the figure. Find the:
1. Dipole moment of the arrangement.

https://bls.smartstudies.co.in/#/exam/pdf-preview/b5b87181-b79b-4b4a-8ebf-8f94fb605fc8/1 70/139
5/17/24, 8:14 PM Exam Automation
2. Magnitude and direction of the net torque acting on it.

Ans:Given, AB and CD are dipoles kept at an angle of 120o to each other.


Resultant magnetic dipole moment is given by,
\vec{\text{p}_\text{r}}=\sqrt{\text{p}^2+\text{p}^2+2\text{ppcos}120^\circ}
=\sqrt{2\text{p}^2+2\text{p}^2\cos120^\circ}
=\sqrt{2\text{p}^2+(2\text{p}^2)\times\Big(-\frac{1}{2}\Big)}
\sqrt{2\text{p}^2-\text{p}^2=\text{p}}
Resultant magnetic dipole makes an angle 60o with Y-axis or 30o with x-axis.
Now, torque is given by,
\tau=\vec{\text{p}}\times\vec{\text{E}}
=\text{pE}\sin\theta
=\text{pE}\sin30^\circ
=\frac12\text{pE}
Direction of torque is along negative Z-direction.
Q522. A water particle of mass 10.0mg and having a charge of 1.50 × 10-6C stays suspended in a room. What is the magnitude of electric field in the room? What is its 3 Marks
direction?

Ans:

\text{m}=10,\ \text{mg}=10\times10^{-3}\text{g}\times10^{-3}\text{kg},
\text{q}=1.5\times10^{-6}\text{C}
But \text{qE}=\text{mg}
\Rightarrow(1.5\times10^{-6})\text{E}=10\times10^{-6}\times10
\Rightarrow\text{E}=\frac{10\times10^{-4}\times10}{1.5\times10^{-6}}
=\frac{100}{1.5}=66.6\text{N/C}
=\frac{100\times10^3}{1.5}=\frac{10^{5+1}}{15}
=6.6\times10^{3}
Q523. Four point charges qA = 2 μC, qB = –5 μC, qC = 2 μC, and qD = –5 μC are located at the corners of a square ABCD of side 10 cm. What is the force on a charge of 1 3 Marks
μC placed at the centre of the square?

Ans:The given figure shows a square of side 10 cm with four charges placed at its corners. O is the centre of the square.

https://bls.smartstudies.co.in/#/exam/pdf-preview/b5b87181-b79b-4b4a-8ebf-8f94fb605fc8/1 71/139
5/17/24, 8:14 PM Exam Automation

Where,
(Sides) AB = BC = CD = AD cm
(Diagonals) AC = BD =10\sqrt{2} cm
AO = OC = DO = OB cm 5\sqrt{2} cm
A charge of amount 1µ C is placed at point O.
Force of repulsion between charges placed at corner A and centre O is equal in magnitude but opposite in direction relative to the force of repulsion between the
charges placed at corner C and centre O. Hence, they will cancel each other. Similarly, force of attraction between charges placed at corner B and centre O is
equal in magnitude but opposite in direction relative to the force of attraction between the charges placed at corner D and centre O. Hence, they will also cancel
each other. Therefore, net force caused by the four charges placed at the corner of the square on 1µ C charge at centre O is zero.
Q524. A charge of 1.0C is placed at the top of your college building and another equal charge at the top of your house. Take the separation between the two charges to 3 Marks
be 2.0km. Find the force exerted by the charges on each other. How many times of your weight is this force?

Ans:Given:
\text{q}_1=\text{q}_2=\text{q}=1.0\text{C}
Distance between the charges, \text{r}=2\text{km}=2\times10^3\text{m}
By Coulomb's Law, electrostatic force,
\text{F}=\frac{1}{4\pi\in_0}\frac{\text{q}_1\text{q}_2}{\text{r}^2}
\text{F}=9\times10^9\times\frac{1\times1}{(2\times10^3)^2}
=2.25\times10^3\text{N}
Let my mass, m, be 50kg.
Weight of my body, W = mg
\Rightarrow\text{W}=50\times10\text{N}=500\text{N}
Now,
\frac{\text{Weight of my body}}{\text{Force between the charges}}=\frac{500}{2.25\times10^3}
=\frac{1}{4.5}
So, the force between the charges is 4.5 times the weight of my body.
Q525. An electric dipole with dipole moment 4 × 10-9 Cm is aligned at 30° with the direction of a uniform electric field of magnitude 5 × 104 NC-1. Calculate the magnitude 3 Marks
of the torque acting on the dipole.

Ans:Electric dipole moment, p = 4 × 10-9 Cm


Angle made by p with a uniform electric field, θ = 30°
Electric field, E = 5 × 104 NC-1.
Torque acting on the dipole is given by the relation,
Τ = pEsinθ
= 4 × 10-9 × 5 × 104 × Sin 30
= 20 × 10-5 × \frac{1}{2}
= 10-4 Nm
Therefore, the magnitude of the torque acting on the dipole is 10-4 Nm.
Q526. An ac voltage \text{v} =\text{v}_\text{m} \sin \omega\text{t} (vm = 310V and f = 50Hz) is connected to a pure capacitor of capacitance 15\mu\text{F}. Calculate (i) 3 Marks
the reactance of the capacitor, and (ii) the amplitude of the current. Write the expression of current through the capacitor as a function of time.

Ans:1. Calculation of reactance of capacitor

https://bls.smartstudies.co.in/#/exam/pdf-preview/b5b87181-b79b-4b4a-8ebf-8f94fb605fc8/1 72/139
5/17/24, 8:14 PM Exam Automation
2. Calculation of amplitude of current
Writing expression of current
\text{V}=\text{V}_\text{m}\sin\omega\text{t}\ \text{C}=15\times10^{-6}\text{F}
\text{V}=310\sin100\pi\text{t}
1. \text{X}_\text{c}=\frac{1}{\omega\text{C}}
=\frac{1}{100\pi\times15\times10^{-6}}
=\frac{10^4}{15\pi}=212\Omega
2. \text{i}_\text{m}=\frac{\text{V}_\text{m}}{\text{X}_\text{C}}
=\frac{310}{212}=1.46\text{A}
Equation of current
\text{i}=\text{i}_\text{m}\sin\Big(\omega\text{t}+\frac{\pi}{2}\Big)
=1.46\sin\Big(100\pi\text{t}+\frac{\pi}{2}\Big)
Q527. A hollow cylindrical box of length 1 m and area of cross-section 25 cm2 is placed in a three dimensional coordinate system as shown in the figure. The electric field 3 Marks
in the region is given by\overrightarrow{\text{E}} = 50\text{x}\hat{\text{i}}, where E is in NC–1 and x is in metres. Find
1. Net flux through the cylinder.
2. Charge enclosed by the cylinder.

Ans:1.

Given, \vec{E}={50}{x}\vec{i} \text{and}\bigtriangleup{s}={25}\text{C}\text{m}{^2}={25}\times{10}{^-}{^4}\text{m}{^2}


As the electric field is only along the x-axis, so, flux will pass only through the cross-section of cylinder.
Magnitude of Electric Field at cross - section A, \text{E}_\text{A}={50}\times{1}={50}\text{N}\text{C}{^-}{^1}
Magnitude of Electric Field at cross - Section B, \text{E}_\text{B}={50}\times{2}={100}\text{N}\text{C}{^-}{^1}
The Corresponding Electric Fluxes are:
\phi_\text{A}=\vec{\text{E}}.\bigtriangleup\vec{s}={50}\times{25}\times{10}{^-}{^4}\times\cos{180}{^0}=-{0.125}\text{N}\text{m}{^2}\text{C}{^-}{^1}
​\phi_\text{B}=\vec{\text{E}}.\bigtriangleup\vec{s}={100}\times{25}\times{10}{^-}{^4}\times\cos{0}{^0}=-{0.25}\text{N}\text{m}{^2}\text{C}{^-}{^1}
So, the net flux through the cylinder, \phi=\phi_\text{A}+\phi_\text{B}= -{0.125} + {0.25}={0.125}\text{N}\text{m}{^2}\text{C}{^-}{^1}
2. Using Gauss’s law:
\oint\vec{\text{E}}{d}\vec{\text{s}}=\frac{\text{q}}{\epsilon_{0}}\Rightarrow{0.125}=\frac{\text{q}}{{8.85}\times{10}{^-}{^1}{^2}}
\Rightarrow\text{q}={8.85}\times{0.125}\times{10}{^-}{^1}{^2}={1.1}\times{10}{^-}{^1}{^2}\text{C}
Q528. Assume that each atom in a copper wire contributes one free electron. Estimate the number of free electrons in a copper wire having a mass of 6.4g (take the 3 Marks
atomic weight of copper to be 64g mol-1.

Ans:64 grams of copper have 1 mole


6.4 grams of copper have 0.1 mole
1 mole = No atoms
0.1 mole = (no × 0.1) atoms
= 6 × 1023 × 0.1 atoms = 6 × 1022 atoms
1 atom contributes 1 electron
6 × 1022 atoms contributes 6 × 1022 electrons.
Q529. A charge Q is uniformly distributed over a rod of length l. Consider a hypothetical cube of edge l with the centre of the cube at one end of the rod. Find the 3 Marks
minimum possible flux of the electric field through the entire surface of the cube.

Ans:Given:
Total charge on the rod = Q
The length of the rod = edge of the hypothetical cube = l
Portion of the rod lying inside the cube, \text{x}=\frac{\text{l}}2{}
Linear charge density for the rod =\frac{\text{Q}}{\text{l}}
Using Gauss's theorem, flux through the hypothetical cube,
\phi=\Big(\frac{\text{Q}_{\text{in}}}{\epsilon_0}\Big), where Qin = charge enclosed inside the cube
Here, charge per unit length of the rod =\frac{\text{Q}}{\text{l}}
Charge enclosed, \text{Q}_{\text{in}}=\frac{\text{Q}}{\text{l}}\times\frac{\text{l}}{2}=\frac{\text{Q}}{2}
Therefore,
\phi=\frac{\frac{\text{Q}}{2}}{\epsilon_0}=\frac{\text{Q}}{2\epsilon_0}
Q530. 1. Define torque acting on a dipole of dipole moment \overrightarrow{p} placed in a uniform electric field \overrightarrow{\text{E}}. Express it in the vector form 3 Marks
and point out the direction along which it acts.
2. What happens if the field is non-uniform?
3. What would happen if the external field \overrightarrow{\text{E}} is increasing (i) parallel to \overrightarrow{p} and (ii) anti-parallel to \overrightarrow{p}?

Ans:1. \tau=pE\sin\theta\text{ };\text{ }\theta= angle between dipole moment(\overrightarrow{p}) and electric field(\overrightarrow{\text{E}})
\tau=\overrightarrow{p}\times\overrightarrow{\text{E}}
Direction of torque is perpendicular to the plane containing and given by right-hand screw rule.
Alternate Answer

https://bls.smartstudies.co.in/#/exam/pdf-preview/b5b87181-b79b-4b4a-8ebf-8f94fb605fc8/1 73/139
5/17/24, 8:14 PM Exam Automation
Direction of torque is out of the plane of the paper.
2. If the field is non uniform the net force on the dipole will not be zero. There will be translatory motion of the dipole.
3.
1. Net force will be in the direction of increasing electric field.

2. Net force will be in the direction opposite to the increasing field. [or in the direction of decreasing field]

Q531. A nonconducting sheet of large surface area and thickness d contains uniform charge distribution of density \rho. Find the electric field at a point P inside the 3 Marks
plate, at a distance x from the central plane. Draw a qualitative graph of E against x for 0 < x < d.

Ans:Given:
Thickness of the sheet = d
Let the surface area of the sheet be s.
Volume of the sheet = sd
Volume charge density of the sheet, \rho=\frac{\text{Q}}{\text{sd}}
Charge on the sheet = Q

Consider an imaginary plane at a distance x from the central plane of surface area s.
Charge enclosed by this sheet, \text{q}=\rho\text{sx}
For this Guassian surface, using Gauss's Law,we get:
\oint\text{E.ds}=\frac{\text{q}}{\epsilon_0}
\text{E}.\text{s}=\frac{\rho\text{sx}}{\epsilon_0}
\text{E}=\frac{\rho\text{x}}{\epsilon_0}
The electric field outside the sheet will be constant and will be:
\text{E}=\frac{\rho\text{d}}{\epsilon_0}

Q532. It is said that any charge given to a conductor comes to its surface. Should all the protons come to the surface? Should all the electrons come to the surface? 3 Marks
Should all the free electrons come to the surface?

Ans:

https://bls.smartstudies.co.in/#/exam/pdf-preview/b5b87181-b79b-4b4a-8ebf-8f94fb605fc8/1 74/139
5/17/24, 8:14 PM Exam Automation
Protons never take part in any electrical phenomena because they are inside the nuclei and are not able to interact easily. These are the free electrons that are
responsible for all electrical phenomena. So, if a conductor is given a negative charge, the free electrons come to the surface of the conductor. If the conductor is
given a positive charge, electrons move away from the surface and leave a positive charge on the surface of the conductor.
Q533. Two isolated metal spheres A and B have radii R and 2R respectively, and same charge q. Find which of the two spheres have greater energy density just outside 3 Marks
the surface of the spheres.

Ans:Energy density,
\text{U}=\frac{1}{2}\epsilon_0\text{E}^2
But, \text{E}=\frac{\sigma}{\epsilon_0}=\frac{\text{Q}}{\text{A}\epsilon_0}
\therefore\ \text{U}=\frac{1}{2}\frac{\epsilon_0\text{Q}^2}{\text{A}^2\epsilon_0}\ \Rightarrow\ \text{U}=\frac{\text{Q}^2}{2\text{A}^2}
\Rightarrow\ \text{U}\propto\frac{1}{\text{A}^2}\ \Rightarrow\ \text{U}_\text{A}>\text{U}_\text{B}
Q534. A positive point charge (+ q) is kept in the vicinity of an uncharged conducting plate. Sketch electric field lines originating from the point on to the surface of the 3 Marks
plate.
Derive the expression for the electric field at the surface of a charged conductor.

Ans:

Field at the surface of a charged conductor

We have, by Gauss’s law,


\text{E }\delta\text{ S} =\frac{|\sigma| \delta\text{ S}}{\varepsilon_{0}}
\therefore\text{E} = \frac{|\sigma|}{\varepsilon_{0}}.
Q535. A point charge of 2.0 μC is at the centre of a cubic Gaussian surface 9.0 cm on edge. What is the net electric flux through the surface? 3 Marks

Ans:Net electric flux (\phi_{\text{Net}}) through the cubic surface is given by,
\phi_{\text{Net}}=\frac{\text{q}}{\in_0}
Where,
\in_0 = permittivity of free space
= 8.854 × 10-12 N-1C2m-2
q = Net charge contained inside the cube = 2.0 µC = 2 x 10-6 C
\therefore\phi_{\text{Net}}=\frac{2\times10^{-6}}{8.854\times10^{-12}}
= 2.26 × 105 Nm2C-1
The net electric flux through the surface is 2.26 x 105 Nm2C-1.
Q536. Two particles have equal masses of 5.0g each and opposite charges of +4.0 × 10-5 C and -4.0 × 10-5C. They are released from rest with a separation of 1.0m 3 Marks
between them. Find the speeds of the particles when the separation is reduced to 50cm.

Ans:

\text{q}_1=\text{q}_2=4\times10^{-5}
\text{s}=1\text{m},\ \text{m}=5\text{g}
=0.005\text{kg}
\text{F}=\text{K}\frac{\text{q}^2}{\text{r}^2}
=\frac{9\times10^9\times\big(4\times10^{-5}\big)^2}{1^2}
=14.4\text{N}
Acceleration ‘a’ =\frac{\text{F}}{\text{m}}
=\frac{14.4}{0.005}
=2880\text{m/s}^2
Now, \text{u}=0,
\text{s}=50\text{cm}=0.5\text{m}
\text{a}=2880\text{m/s}^2,\ \text{V}=?
\Rightarrow\text{V}=\sqrt{2880}
=53.66\text{m/s}\approx54\text{m/s} for each particle.
Q537. Using Gauss’s theorem, show mathematically that for any point outside the shell, the field due to a uniformly charged thin spherical shell is the same as if the entire 3 Marks
charge of the shell is concentrated at the centre. Why do you expect the electric field inside the shell to be zero according to this theorem?

Ans:\phi =\oint\limits_{s}\overrightarrow{\text{E}}.\overrightarrow{\text{d}}\text{s} = \frac{\text{q}}{\varepsilon_{\circ}}

Derivation: \text{E}\times4\pi\text{r}^{2} =\frac{\sigma}{\varepsilon_{\circ}}4\pi\text{R}^{2}


\therefore \text{E} = \frac{\sigma\text{R}^{2}}{\varepsilon_{\circ}\text{r}^{2}}
where \text{q} = 4\pi\text{R}^{2}\sigma is the total charge on the spherical shell. Electrostatic field is zero, since total charge inside the shell is zero or charge
reside on the surface of the shell.
Q538. Using Gauss’s law in electrostatics, deduce an expression for electric field intensity due to a uniformly charged infinite plane sheet. If another identical sheet is 3 Marks
placed parallel to it, show that there is no electric field in the region between the two sheets.

Ans:

https://bls.smartstudies.co.in/#/exam/pdf-preview/b5b87181-b79b-4b4a-8ebf-8f94fb605fc8/1 75/139
5/17/24, 8:14 PM Exam Automation

By Gauss’s law \oint \overrightarrow{E}.\overrightarrow{ds}=\frac{q}{\epsilon_0}


\therefore{2{\text{EA}}}=\frac{\sigma{\text{A}}}{{\epsilon_0}}
\therefore\text{E}=\frac{\sigma}{2\epsilon_0}\text{ }\text{or}\frac{\sigma}{2\epsilon_0}\text{A}
Electric field between two identical charged sheets

\because Both the sheets have same charge density, their electric fields will be equal and opposite in the region between the two sheets.
Hence the net field is zero.
Alternate Answer
\text{E}_1=\frac{\sigma}{2\epsilon_0}
\text{E}_2=-\frac{\sigma}{2\epsilon_0}
Resultant electric field between the plates = E1 + E2
\frac{\sigma}{2\epsilon_0}-\frac{\sigma}{2\epsilon_0}=0]
Q539. What is the nature of electrostatic force between two point electric charges q1 and q2 if: 3 Marks
1. q1 + q2 > 0?
2. q1 + q2 < 0?

Ans:1. If both q1 and q2 are positive, the electrostatic force between these will be repulsive.
However, if one of these charges is positive and is greater than the other negative charge, the electrostatic force between them will be attractive.
Thus, the nature of force between them can be repulsive or attractive.
2. If both q1 and q2 are -ve, the force between these will be repulsive.
However, if one of them is -ve and it is greater in magnitude than the second +ve charge, the force between them will be attractive.
Thus, the nature of force between them can be repulsive or attractive.
Q540. Two equal charges, 2.0 × 10-7C each, are held fixed at a separation of 20cm. A third charge of equal magmtude is placed midway between the two charges. It is 4 Marks
now moved to a point 20cm from both the charges. How much work is done by the electric field durmg the process?

Ans:

When the charge is placed at A,


\text{E}_1=\frac{\text{Kq}_1\text{q}_2}{\text{r}}+\frac{\text{Kq}_3\text{q}_4}{\text{r}}
=\frac{9\times10^9(2\times10^{-7})^2}{0.1}+\frac{9\times10^9(2\times10^{-7})^2}{0.1}
=\frac{2\times9\times10^9\times4\times10^{-14}}{0.1}
=72\times10^{-4}\text{J}
When charge is placed at B,
\text{E}_2=\frac{\text{Kq}_1\text{q}_2}{\text{r}}+\frac{\text{Kq}_3\text{q}_4}{\text{r}}
=\frac{2\times9\times10^9\times4\times10^{-14}}{0.2}
=36\times10^{-4}\text{J}
Q541. A particle of mass m and charge q is thrown at a speed u against a uniform electric field E. How much distance will it travel before coming to momentary rest? 4 Marks

Ans:

https://bls.smartstudies.co.in/#/exam/pdf-preview/b5b87181-b79b-4b4a-8ebf-8f94fb605fc8/1 76/139
5/17/24, 8:14 PM Exam Automation

Given,
u = Velocity of projection, \vec{\text{E}} = Electric field intensity
q = Charge; m = mass of particle
We know, Force experienced by a particle with charge ‘q’ in an electric field \vec{\text{E}}=\text{qE}
\therefore\ acceleration produced =\frac{\text{qE}}{\text{m}}
As the particle is projected against the electric field, hence deceleration =\frac{\text{qE}}{\text{m}}
So, let the distance covered be ‘s'
Then, v2 = u2 + 2as [where a = acceleration, v = final velocity]
Here 0=\text{u}^2-2\times\frac{\text{qE}}{\text{m}}\times\text{S}
\Rightarrow\text{S}=\frac{\text{u}^2\text{m}}{2\text{qE}}\text{units}
Q542. Positive charge Q is distributed uniformly over a circular ring of radius R. A particle having a mass m and a negative charge q, is placed on its axis at a distance x 4 Marks
from the centre. Find the force on the particle. Assuming x << R, find the time period of oscillation of the particle if it is released from there.

Ans:

We know: Electric field ‘E’ at ‘P’ due to the charged ring


=\frac{\text{KQx}}{\big(\text{R}^2+\text{x}^2\big)^{\frac{3}{2}}}
=\frac{\text{KQx}}{\text{R}^3}
Force experienced ‘F’ =\text{Q}\times\text{E}=\frac{\text{q}\times\text{K}\times\text{Qx}}{\text{R}^3}
Now, amplitude = x
So, \text{T}=2\pi\sqrt{\frac{\text{x}}{\frac{\text{KQqx}}{\text{mR}^3}}}
=2\pi\sqrt{\frac{\text{mR}^3\text{x}}{\text{KQqx}}}
=2\pi\sqrt{\frac{4\pi\in_0\text{mR}^3}{\text{Qq}}}
=\sqrt{\frac{4\pi^2\times4\pi\in_0\text{mR}^3}{\text{qQ}}}
\Rightarrow\text{T}=\bigg[\frac{16\pi^3\in_0\text{mR}^3}{\text{qQ}}\bigg]^{\frac{1}{2}}
Q543. A particle A having a charge of 2.0 × 10-6C and a mass of 100g is placed at the bottom of a smooth inclined plane of inclination 30°. Where should another particle 4 Marks
B, having same charge and mass, be placed on the incline so that it may remain in equilibrium?

Ans:

\text{q}_1=2\times10^{-6}\text{c} Let the distance be r unit


\therefore\text{F}_\text{repulsion}=\frac{\text{kq}_1\text{q}_2}{\text{r}^2}
For equilibrium \frac{\text{kq}_1\text{q}_2}{\text{r}^2}=\text{mg}\sin\theta
\Rightarrow\frac{9\times10^9\times4\times10^{-12}}{\text{r}^2}=\text{m}\times9.8\times\frac{1}{2}
\Rightarrow\text{r}^2=\frac{18\times4\times10^{-3}}{\text{m}\times9.8}
=\frac{72\times10^{-3}}{9.8\times10^{-1}}

https://bls.smartstudies.co.in/#/exam/pdf-preview/b5b87181-b79b-4b4a-8ebf-8f94fb605fc8/1 77/139
5/17/24, 8:14 PM Exam Automation
=7.34\times10^{-2}\ \text{metre}
\Rightarrow\text{r}=2.70924\times10^{-1}\text{metre} from the bottom
Q544. A metallic spherical shell has an inner radius R1 and outer radius R2. A charge Q is placed at the centre of the spherical cavity. What will be surface charge density 4 Marks
on (i) the inner surface, and (ii) the outer surface?

Ans:The situation given in question is shown in figure given below,

When positive charge is placed at the centre of the spherical cavity then an equal amount of negative charge (-Q) appears on inner surface of the sphere due to
induction. This charge is distributed uniformly on the inner surface. Also an equal amount of positive charge (+Q) also appears on outer surface of the sphere.
Now surface charge density on inner surface = (Total charge on inner surface/Area of the inner surface) =\frac{-\text{Q}}{(4\pi\text{R}_1^2)}
Surface charge density on outer surface = (Total charge on outer surface/Area of the outer surface) =\frac{+\text{Q}}{(4\pi\text{R}_1^2)}.
Q545. The electric field in a region is given by \overrightarrow{\text{E}}=\frac{\text{E}_0\text{x}}{\text{l}}\vec{\text{i}}. Find the charge contained inside a cubical volume 4 Marks
bounded by the surfaces x = 0, x = a, y = 0, y = a, z = 0 and z = a. Take E0 = 5 × 103Nc-1, l = 2cm and a = 1cm.

Ans:Given:
Electric field strength, \overrightarrow{\text{E}}=\frac{\text{E}_0\text{x}}{\text{l}}\hat{\text{i}}
Length, l = 2cm
Edge of the cube, a = 1cm
E0 = 5.0 × 103N/C

It is observed that the flux passes mainly through the surfaces ABCD and EFGH. The surfaces AEFB and CHGD are parallel to the electric field. So, electric flux for
these surfaces is zero.
The electric field intensity at the surface EFGH will be zero.
If the charge is inside the cube, then equal flux will pass through the two parallel surfaces ABCD and EFGH. We can calculate flux passing only through one
surface.
Thus,
\overrightarrow{\text{E}}=\frac{\text{E}_0\text{x}}{\text{l}}\hat{\text{i}}
At EFGH, x = 0; thus, the electric field at EFGH is zero.
At ABCD, x = a; thus, the electric field at ABCD is \overrightarrow{\text{E}}=\frac{\text{E}_0\text{a}}{\text{l}}\hat{\text{i}}.
The net flux through the whole cube is only through the side ABCD and is given
by \phi=\overrightarrow{\text{E}}.\overrightarrow{\text{A}}=\Big(\frac{\text{E}_0\text{a}}
{\text{l}}\hat{\text{i}}\Big).\big(\text{a}\hat{\text{i}}\big)=\frac{\text{E}_0\text{a}^2}{\text{l}}
Net flux =\phi=\frac{5\times10^3}{2\times10^{-2}}\times\big(1\times10^{-2}\big)^2\text{Nm}^2/\text{C}
=25\text{Nm}^2/\text{C}
Thus, the net charge,
\text{q}=\epsilon_0\phi
\text{q}=8.85\times10^{-12}\times25
\text{q}=22.125\times10^{-13}
\text{q}=2.2125\times10^{-12}\text{C}
Q546. A long cylindrical volume contains a uniformly distributed charge of density \rho. Find the electric field at a point P inside the cylindrical volume at a distance x 4 Marks
from its axis (figure).

https://bls.smartstudies.co.in/#/exam/pdf-preview/b5b87181-b79b-4b4a-8ebf-8f94fb605fc8/1 78/139
5/17/24, 8:14 PM Exam Automation

Ans:Given:
Volume charge density inside the cylinder =\rho.
Let the radius of the cylinder be r.
Let charge enclosed by the given cylinder be Q
Consider a Gaussian cylindrical surface of radius x and height h.
Let charge enclosed by the cylinder of radius x be q′.

The charge on this imaginary cylinder can be found by taking the product of the volume charge density of the cylinder and the volume of the imaginary cylinder.
Thus,
\text{q}'=\rho\big(\pi\text{x}^2\text{h}\big)
From Gauss's Law,
\oint\text{E.ds}=\frac{\text{q}_{\text{en}}}{\epsilon_0}
\text{E}.2\pi\text{xh}=\frac{\rho(\pi\text{x}^2\text{h})}{\epsilon_0}
\text{E}=\frac{\rho\text{x}}{2\epsilon_0}
Q547. Suppose the second charge in the previous problem is -1.0 × 10-6C. Locate the position where a third charge will not experience a net force. 4 Marks

Ans:

By coulomb's Law, force,


\text{F}=\frac{1}{4\pi\epsilon_0}\frac{\text{Q}_1\text{Q}_2}{\text{r}_2}
So, force on charge q due to q1,
\text{F}=\frac{9\times10^{9}\times2.0\times10^{-6}\times\text{q}}{\text{x}^2}
Force on charge q due to q2
\text{F}'=\frac{9\times10^9\times10^{-6}\times\text{q}}{(10-\text{x})^2}
According to the question,

https://bls.smartstudies.co.in/#/exam/pdf-preview/b5b87181-b79b-4b4a-8ebf-8f94fb605fc8/1 79/139
5/17/24, 8:14 PM Exam Automation
\Rightarrow\text{F}-\text{F}'=0
\Rightarrow\frac{9\times10^{9}\times2.0\times10^{-6}\times\text{q}}{\text{x}^2}=\frac{9\times10^9\times10^{-6}\times\text{q}}{(\text{x}-10)^2}
\Rightarrow\text{x}^2=2(\text{x}-10)^2
\Rightarrow\text{x}^2-40\text{x}+200=0
\Rightarrow\text{x}=20\pm10\sqrt{2}\text{m}
\Rightarrow\text{x}=34.14\text{cm} \Big(\because\ \text{x}\neq20-10\sqrt{2}\Big)
Q548. Two identical pith balls, each carrying a charge q, are suspended from a common point by two strings of equal length l. Find the mass of each ball if the angle 4 Marks
between the strings is 2\theta in equilibrium.

Ans:

Electric force =\frac{\text{kq}^2}{(\ell\sin\text{Q}+\ell\sin\text{Q})^2}


=\frac{\text{kq}^2}{4\ell^2\sin^2}
So, \text{T}\cos\theta=\text{ms} (For equilibrium) \text{T}\sin\theta=\text{Ef},
\text{tan}\theta=\frac{\text{Ef}}{\text{mg}}

\Rightarrow\text{mg}=\text{Ef}\cot\theta
=\frac{\text{kq}^2}{4\ell^2\sin^2\theta}\cot\theta
=\frac{\text{q}^2\cot\theta}{\ell^2\sin^2\theta16\pi\text{E}_0}
\text{m}=\frac{\text{q}^2\cot\theta}{16\pi\text{E}_0\ell^2\sin^2\theta\text{g}}\text{unit}
Q549. The bob of a simple pendulum has a mass of 40g and a positive charge of 4.0 × 10-6C. It makes 20 oscillations in 45s. A vertical electric field pointing upward and 4 Marks
of magnitude 2.5 × 104NC-1 is switched on. How much time will it now take to complete 20 oscillations?

Ans:

\text{m}=40\text{g},\ \text{q}=4\times10^{-6}\text{C}
Time for 20 oscillations =45\sec. Time for 1 oscillation =\frac{45}{20}\sec
When no electric field is applied, \text{T}=2\pi\sqrt{\frac{\ell}{\text{g}}}
\Rightarrow\frac{45}{20}=2\pi\sqrt{\frac{\ell}{10}}
\Rightarrow\frac{\ell}{10}=\Big(\frac{45}{20}\Big)^2\times\frac{1}{4\pi^2}
\Rightarrow\ell=\frac{(45)^2\times10}{(20)^2\times4\pi^2}
=1.2836

https://bls.smartstudies.co.in/#/exam/pdf-preview/b5b87181-b79b-4b4a-8ebf-8f94fb605fc8/1 80/139
5/17/24, 8:14 PM Exam Automation
Time for 1 oscillation = 2.598
Time for 20 oscillation =2.598\times20
=51.96\sec
=52\sec.
Q550. A ball of mass 100g and having a charge of 4.9 × 10-5C is released from rest in a region where a horizontal electric field of 2.0 × 104NC-1 exists. 4 Marks
1. Find the resultant force acting on the ball.
2. What will be the path of the ball?
3. Where will the ball be at the end of 2s?

Ans:\text{m}=100\text{g},\ \text{q}=4.9\times10^{-5},
\text{F}_\text{g}=\text{mg},\ \text{F}_\text{e}=\text{qE}
\vec{\text{E}}=2\times10^4\text{N/C}
So, the particle moves due to the et resultant R

1. \text{R}=\sqrt{\text{F}_\text{g}^2+\text{F}_\text{e}^2}
\sqrt{(0.1\times9.8)^2+(4.9\times10^{-5}\times2\times10^4)^2}
=\sqrt{0.9604+96.04\times10^{-2}}
=\sqrt{1.9208}
=1.3859\text{N}

2. \tan\theta=\frac{\text{F}_\text{g}}{\text{F}_\text{e}}
=\frac{\text{mg}}{\text{qE}}
=1
So, \theta=45^\circ
\therefore\ Hence path is straight along resultant force at an angle 45° with horizontal
Disp. Vertical =\Big(\frac{1}{2}\Big)\times9.8\times2\times2
=19.6\text{m}
3. Disp. Horizontal = \text{S}=\Big(\frac{1}{2}\Big)\text{at}^2
=\frac{1}{2}\times\frac{\text{qE}}{\text{m}}\times\text{t}^2
=\frac{1}{2}\times\frac{0.98}{0.1}\times2\times2
=19.6\text{m}
Net Dispt. =\sqrt{(19.6)^2+(19.6)^2}
=\sqrt{768.32}
=27.7\text{m}
Q551. Ten positively charged particles are kept fixed on the x-axis at points x = 10cm, 20cm, 30cm, ..., 100cm. The first particle has a charge 1.0 × 10-8C, the second 8 4 Marks
× 10-8C, the third 27 × 10-8C and so on. The tenth particle has a charge 1000 × 10-8C. Find the magnitude of the electric force acting on a 1C charge placed at the
origin.

Ans:Electric force feeled by 1C due to 1 × 10-8C.


\text{F}_1=\frac{\text{k}\times1\times10^{-8}\times1}{\big(10\times10^{-2}\big)^2}=\text{k}\times10^{-6}\text{N}. electric force feeled by 1C due to 8 × 10-8c
\text{F}_2=\frac{\text{k}\times8\times10^{-8}\times1}{\big(23\times10^{-2}\big)^2}
=\frac{\text{k}\times8\times10^{-8}\times10^2}{9}
=\frac{28\text{k}\times10^{-6}}{4}
=2\text{k}\times10^{-6}\text{N}.
Similarly \text{F}_3=\frac{\text{k}\times27\times10^{-8}\times1}{\big(30\times10^{-2}\big)^2}=3\text{k}\times10^{-6}\text{N}
So, \text{F}=\text{F}_1+\text{F}_2+\text{F}_3+_{\dots}+\text{F}_{10}\\\ \ =\text{k}\times10^{-6}(1+2+3+_{\dots}+10)\text{N}
=\text{k}\times10^{-6}\times\frac{10\times11}{2}
=55\text{k}\times10^{-6}
=55\times9\times10^9\times10^{-6}\text{N}
=4.95\times10^3\text{N}
Q552. Find the ratio of the electric and gravitational forces between two protons. 4 Marks

Ans:Given:
mass of proton = 1.67\times10^{27}\text{kg}=\text{M}_\text{p}
\text{k}=9\times10^9 Charge of proton = 1.6\times10^{-19}\text{c}=\text{C}_\text{p}

https://bls.smartstudies.co.in/#/exam/pdf-preview/b5b87181-b79b-4b4a-8ebf-8f94fb605fc8/1 81/139
5/17/24, 8:14 PM Exam Automation
\text{G}=6.67\times10^{-11} Let the separation be ‘r’
\text{Fe}=\frac{\text{k}(\text{C}_\text{p})^2}{\text{r}^2},\ \text{fg}=\frac{\text{G}(\text{M}_\text{p})^2}{\text{r}^2}
Now, \text{Fe}:\text{Fg}=\frac{\text{K}(\text{C}_\text{p})^2}{\text{r}^2}\times\frac{\text{r}^2}{\text{G}(\text{M}_\text{p})^2}
=\frac{9\times10^9\times(1.6\times10^{-19})^2}{6.67\times10^{-11}\times(1.67\times10^{-27})^2}
=9\times2.56\times10^{38}\approx1.24\times10^{38}
Q553. A particle A having a charge of 2.0 × 10-6C is held fixed on a horizontal table. A second charged particle of mass 80g stays in equilibrium on the table at a distance 4 Marks
of 10cm from the first charge. The coefficient of friction between the table and this second particle is \mu=0.2. Find the range within which the charge of this
second particle may lie.

Ans:

\text{q}_\text{A}=2\times10^{-6}\text{C},\ \text{M}_\text{b}=80\text{g},\ \mu=0.2


Since B is at equilibrium,
So, \text{Fe}=\mu\text{R}
\Rightarrow\frac{\text{Kq}_\text{A}\text{q}_\text{B}}{\text{r}^2}=\mu\text{R}=\mu\text{m}\times\text{g}
\Rightarrow\frac{9\times10^9\times2\times10^{-6}\times\text{q}_\text{B}}{0.01}=0.2\times0.08\times9.8
\Rightarrow\text{q}_\text{B}=\frac{0.2\times0.08\times9.8\times0.01}{9\times10^9\times2\times10^{-6}}
=8.7\times10^{-8}\text{C}
Range =\pm8.7\times10^{-8}\text{C}
Q554. A small plane area is rotated in an electric field. In which orientation of the area is the flux of electric field through the area maximum? In which orientation is it 4 Marks
zero?

Ans:The flux of an electric field \overrightarrow{\text{E}} through a surface area \Delta\overrightarrow{\text{S}} is given
by \Delta\phi=\overrightarrow{\text{E}}.\Delta\overrightarrow{\text{S}}, where \Delta\phi is the flux.
Therefore, \Delta\phi={\text{E}}\Delta{\text{S}}\cos\theta. Here, \theta is the angle between the electric field \overrightarrow{\text{E}} and the normal to the
surface area.
Thus, for the flux to be maximum, \cos\theta should be maximum. Thus, for \theta=0, the flux is maximum, i.e. the electric field lines are perpendicular to the
surface area.
The flux is minimum if \theta=90. Thus, \cos\theta=0 and, hence, flux is also 0. Thus, if the electric field lines are parallel to the surface area, the flux is minimum.
Q555. Two large conducting plates are placed parallel to each other and they carry equal and opposite charges with surface density \sigma as shown in figure. Find the 4 Marks
electric field:
1. At the left of the plates.
2. In between the plates.
3. At the right of the plates.

Ans:

https://bls.smartstudies.co.in/#/exam/pdf-preview/b5b87181-b79b-4b4a-8ebf-8f94fb605fc8/1 82/139
5/17/24, 8:14 PM Exam Automation

Given :
Surface charge density on the plates =\sigma
The electric field due to plate 1, \text{E}_1=\frac{\sigma}{2\epsilon_0}
The electric field due to plate 2, \text{E}_2=\frac{\sigma}{2\epsilon_0}
1. The strength of the electric field due to both the plates will be same but their directions will be opposite to each other on any point at the left of the two plates.
Thus, the net electric field at a point on the left of plate 1 =\frac{\sigma}{2\epsilon_0}-\frac{\sigma}{2\epsilon_0}=0
2. Here the direction of the fields will be same. So, they will add up to give the resultant field in this region.
Total electric field:
\frac{\sigma}{2\epsilon_0}+\frac{\sigma}{2\epsilon_0}=\frac{\sigma}{\epsilon_0}
3. The strength of the electric field due to both the plates will be same but their directions will be opposite to each other at any point on the right of the two plates.
Thus, the net electric field at a point on the left of plate 2 =\frac{\sigma}{2\epsilon_0}-\frac{\sigma}{2\epsilon_0}=0
Q556. While travelling back to his residence in the car, Dr. Pathak was caught up in a thunderstorm. It became very dark. He stopped driving the car and waited for 4 Marks
thunderstorm to stop. Suddenly he noticed a child walking alone on the road. He asked the boy to come inside the car till the thunderstorm stopped. Dr. Pathak
dropped the boy at his residence. The boy insisted that Dr. Pathak should meet his parents. The parents expressed their gratitude to Dr. Pathak for his concern for
safety of the child.
Answer the following question based on the above information:
1. Why is it safer to sit inside a car during a thunderstorm?
2. Which two values are displayed by Dr. Pathak in his actions?
3. Which values are reflected in parents’ response to Dr. Pathak?
4. Give an example of a similar action on your part in the past from everyday life.

Ans:1. On the basis of electrostatic screening, no electric field exists inside the charged conducting body. During lightening a shower of the charged particles falls on
the earth. So it would be safer to sit inside the car.
2. Dr. Pathak knows the result of lightening during thunderstorm; so he displayed two actions;
1. Shows love, kindness and sympathy to the child.
2. Keeping in view the safety of the child, he allow the boy to sit in the car till the thunderstorm stopped.
3. Parent meets Dr. Pathak; and express their gratitude and heart felt thank for providing the safety to the child from lightening and thunderstorm.
4. Many of us have read in the newspaper that the person either working in the field or in open space have lost their life during thunderstorm, so the persons
belonging to villages must be given advices that they should remain inside the houses (or closed caves etc.) during thunderstorm.
Q557. A charge Q is uniformly distributed on a thin spherical shell. What is the field at the centre of the shell? If a point charge is brought close to the shell, will the field at 4 Marks
the centre change? Does your answer depend on whether the shell is conducting or nonconducting?

Ans:The field at the centre of the shell is zero. As all the charge given to a conductor resides on the surface, the field at any point inside the conducting sphere is zero.
Also, the charge distribution at the surface is uniform so, all the electric field vectors due to these charges at the centre are equal and opposite. So, they cancel
each other, resulting in a zero net value of the field.
When a charge is brought near the shell, due to induction, opposite polarity charges induce on the surface nearer to the charge and the same polarity charges
appear on the face farther from the charge. In this way, a field is generated inside the shell. Hence, the field at the centre is non-zero.
Yes, our answer changes in case of a non-conducting spherical shell. As the charge given to the surface of a non-conducting spherical shell spreads non-
uniformly, there is a net electric field at the centre of the sphere.
Q558. A charge Q is distributed uniformly within the material of a hollow sphere of inner and outer radii r1 and r2 (figure). Find the electric field at a point P a distance x 4 Marks
away from the centre for r1 < x < r2 Draw a rough graph showing the electric field as a function of x for O < x < 2r2 (figure).

Ans:Amount of charge present on the hollow sphere = Q


Inner radii of the hollow sphere = r1
Outer radii of the hollow sphere = r2
Consider an imaginary sphere of radius x.
The charge on the sphere can be found by multiplying the volume charge density of the hollow spherical volume with the volume of the imaginary sphere of radius
(x - r1).
Charge per unit volume of the hollow sphere,
\text{p}=\frac{\text{Q}}{\frac{4}{3}\pi\big(\text{r}_2^3-\text{r}_1^3\big)}

https://bls.smartstudies.co.in/#/exam/pdf-preview/b5b87181-b79b-4b4a-8ebf-8f94fb605fc8/1 83/139
5/17/24, 8:14 PM Exam Automation
Charge enclosed by this imaginary sphere of radius x:
q = ρ × Volume of the part consisting of charge
\text{q}=\frac{\frac{4}{3}\pi\big(\text{x}^3-\text{r}_1^3\big)\text{Q}}{\frac{4}{3}\pi\big(\text{r}_2^3-\text{r}_1^3\big)}
\text{q}=\frac{\big(\text{x}^3-\text{r}_1^3\big)}{\big(\text{r}_2^3-\text{r}_1^3\big)}\text{Q}
According to Gauss's Law,
\oint\text{E.ds}=\frac{\text{q}}{\epsilon_0}
Here, the surface integral is carried out on the sphere of radius x and q is the charge enclosed by this sphere.

\oint\text{E.ds}=\frac{\text{q}}{\epsilon_0}
\text{E}(4\pi\text{x}^2)=\frac{\text{q}}{\epsilon_0}
\text{E}(4\pi\text{x}^2)=\frac{\big(\text{x}^3-\text{r}^3_1\big)\text{Q}}{\big(\text{r}^3_2-\text{r}^3_1\big)\epsilon_0}
\text{E}=\frac{\big(\text{x}^3-\text{r}^3_1\big)\text{Q}}{4\pi\text{x}^2\epsilon_0\big(\text{r}^3_2-\text{r}^3_1\big)}
The electric field is directly proportional to x for r1 < x < r.
The electric field for r2 < x < 2r2,
\text{E}=\frac{\text{Q}}{4\pi\epsilon_0\text{x}^2}
Thus, the graph can be drawn as:

Q559. A particle of mass 1g and charge 2.5 × 10-4C is released from rest in an electric field of 1.2 × 104NC-1. 4 Marks
1. Find the electric force and the force of gravity acting on this particle. Can one of these forces be neglected in comparison with the other for approximate
analysis?
2. How long will it take for the particle to travel a distance of 40cm?
3. What will be the speed of the particle after travelling this distance?
4. How much is the work done by the electric force on the particle during this period?

Ans:\text{m}=1\text{g}=10^{-3},\ \text{u}=0
\text{q}=2.5\times10^{-4}\text{C};\ \text{E}=1.2\times10^4\text{N/c};\ \text{S}=40\text{cm}=4\times10^{-1}\text{m}
1. \text{F}=\text{qE}=2.5\times10^{-4}\times1.2\times10^4=3\text{N}
So, \text{a}=\frac{\text{F}}{\text{m}}
=\frac{3}{10^{-3}}
=3\times10^3
\text{E}_\text{q}=\text{mg}=10^{-3}\times9.8\times10^{-3}\text{N}
2. \text{S}=\frac{1}{2}\text{at}^2
\text{t}=\sqrt{\frac{2\text{a}}{9}}
=\sqrt{\frac{2\times4\times10^{-1}}{3\times10^{3}}}
=1.63\times10^{-2}\text{sec}
3. \text{v}^2=\text{u}^2+2\text{as}
=0+2\times3\times10^3\times4\times10^{-1}
=24\times10^2
\Rightarrow\text{v}=\sqrt{24\times10^2}
4.9\times10=49\text{m/sec}
4. Work done by the electric force \text{w}=\text{F}\rightarrow\text{td}
=3\times4\times10^{-1}
=12\times10^{-1}
=1.2\text{J}
Q560. Find the magnitude of the electric field at a point 4cm away from a line charge of density 2 × 10-6Cm-1. 4 Marks

Ans:Given:
Charge density of the line containing charge, \lambda=2\times10^{-6}\text{Cm}^{-1}
We need to find the electric field at a distance of 4cm away from the line charge.
We take a Gaussian surface around the line charge of cylindrical shape of radius r = 4 × 10-2m and height l.

https://bls.smartstudies.co.in/#/exam/pdf-preview/b5b87181-b79b-4b4a-8ebf-8f94fb605fc8/1 84/139
5/17/24, 8:14 PM Exam Automation

The charge enclosed by the Gaussian surface, \text{q}_{\text{en}}=\lambda\text{l}


Let the magnitude of the electric field at a distance of 4cm away from the line charge be E.
Thus, net flux through the Gaussian surface, \phi=\oint\overrightarrow{\text{E}}.\text{d}\overrightarrow{\text{s}}
There will be no flux through the circular bases of the cylinder; there will be flux only from the curved surface.
The electric field lines are directed radially outward, from the line charge to the cylinder.
Therefore, the lines will be perpendicular to the curved surface.
Thus,
\phi=\oint\overrightarrow{\text{E}}.\text{d}\overrightarrow{\text{s}}=\text{E}\oint\text{ds}
Applying Gauss's theorem,
\phi=\frac{\text{q}_{\text{en}}}{\epsilon_0}
\text{E}\times2\pi\text{rl}=\frac{\lambda\text{l}}{\epsilon_0}
\text{E}=\frac{\lambda}{2\pi\text{r}\epsilon_0}
\text{E}=\frac{2\times10^{-6}}{2\times3.14\times8.85\times10^{-12}\times4\times10^{-2}}
\text{E}=8.99\times10^5\text{N/C}
Q561. Find the magnitude of the electric field at the point P in the configuration shown in figure for d >> a. Take 2qa = p. 4 Marks

Ans:

1. \text{p}=2\text{qa}
2. \text{E}_1\sin\theta=\text{E}_2\sin\theta Electric field intensity

https://bls.smartstudies.co.in/#/exam/pdf-preview/b5b87181-b79b-4b4a-8ebf-8f94fb605fc8/1 85/139
5/17/24, 8:14 PM Exam Automation

=\text{E}_1=\frac{\text{Kqp}}{\text{a}^2+\text{d}^2}
So, \text{E}=\frac{2\text{KPQ}}{\text{a}^2+\text{d}^2}\frac{\text{a}}{\big(\text{a}^2+\text{d}^2\big)^{\frac{1}{2}}}
=\frac{2\text{Kq}\times\text{a}}{\big(\text{a}^2+\text{d}^2\big)^{\frac{3}{2}}}
When a << d
=\frac{2\text{Kqa}}{\big(\text{d}^2\big)^{\frac{3}{2}}}
=\frac{\text{PK}}{\text{d}^3}
=\frac{1}{4\pi\in_0}\frac{\text{p}}{\text{d}^3}
Q562. If the total charge enclosed by a surface is zero, does it imply that the elecric field everywhere on the surface is zero? Conversely, if the electric field everywhere 4 Marks
on a surface is zero, does it imply that net charge inside is zero.

Ans:According to Gauss' law, the flux associated with any closed surface is given by \oint_\text{s}\text{E.dS}=\frac{\text{q}_\text{enclosed}}{\epsilon_0}. The term
qenclosed on the right side of the equation includes the sum of all charges enclosed by the surface called (Gaussian surface).
In left side equation, the electric field is due to all the charges present both inside as well as outside the Gaussian surface.
Thus, despite being total charge enclosed by a surface zero, it doesn't imply that the electric field everywhere on the surface is zero, the field may be normal to the
surface.
Also, conversely if the electric field everywhere on a surface is zero.
Q563. Consider a uniformly charged ring of radius R. Find the point on the axis where the electric field is maximum. 4 Marks

Ans:

Electric field at any point on the axis at a distance x from the center of the ring is
\text{E}=\frac{\text{xQ}}{4\pi\in_0\big(\text{R}^2+\text{x}^2\big)^{\frac{3}{2}}}
=\frac{\text{KxQ}}{\big(\text{R}^2+\text{x}^2\big)^{\frac{3}{2}}}
Differentiating with respect to x
\frac{\text{dE}}{\text{dx}}=\frac{\text{KQ}\big(\text{R}^2+\text{x}^2\big)-\text{KxQ}\big(\frac{3}{2}\big)\big(\text{R}^2+\text{x}^2\big)^{\frac{11}{2}}2\text{x}}
{\big(\text{r}^2+\text{x}^2\big)^3}
Since at a distance x, Electric field is maximum.
\frac{\text{dE}}{\text{dx}}=0
\Rightarrow\text{KQ}\big(\text{R}^2+\text{x}^2\big)^{\frac{3}{2}}-\text{Kx}^2\text{Q}^3\big(\text{R}^2+\text{x}^2\big)^{\frac{1}{2}}=0
\Rightarrow\text{KQ}\big(\text{R}^2+\text{x}^2\big)^{\frac{3}{2}}=\text{Kx}^2\text{Q}3\big(\text{R}^2+\text{x}^2\big)^{\frac{1}{2}}
\Rightarrow\text{R}^2+\text{x}^2=3\text{x}^2
\Rightarrow2\text{x}^2=\text{R}^2
\Rightarrow\text{x}^2=\frac{\text{R}^2}{2}
\Rightarrow\text{x}=\frac{\text{R}}{\sqrt{2}}
Q564. Consider a sphere of radius R with charge density distributed as 4 Marks
\rho\text{(r)}=\text{kr for r}\leq\text{R}
=0\text{ for r}>\text{R}.
Suppose the total charge on the sphere is 2e where e is the electron charge. Where can two protons be embedded such that the force on each of them is zero.
Assume that the introduction of the proton does not alter the negative charge distribution.

https://bls.smartstudies.co.in/#/exam/pdf-preview/b5b87181-b79b-4b4a-8ebf-8f94fb605fc8/1 86/139
5/17/24, 8:14 PM Exam Automation
Ans:The two protons must be placed symmetrically on the opposite sides of the centre along a diameter. This can be shown by the figute given below. Charge on the
sphere,

\text{q}=\int\limits_0^\text{R}\rho\text{dV}=\int\limits_0^\text{R}(\text{Kr})4\pi\text{r}^2\text{dr}
\text{q}=4\pi\text{K}\frac{\text{R}^4}{4}=2\text{e}
\because\ \text{K}=\frac{2\text{e}}{\pi\text{R}^4}
The protons 1 and 2 are embedded at distance r from the centre of the sphere as shown, then attractive force on proton 1 due to charge distribution is
\text{F}_1=\text{eE}=\frac{-\text{eKr}^2}{4\epsilon_0}
and repulsive force on proton 1 due to proton 2 is
\text{F}_2=\frac{\text{e}^2}{4\pi\epsilon_0(2\text{r})^2}
Net force on proton 1, F = F1 + F2
\text{F}=\frac{-\text{eKr}^2}{4\epsilon_0}+\frac{\text{e}^2}{16\pi\epsilon_0\text{r}^2}
So, \text{F}=\bigg[\frac{-\text{er}^2}{4\epsilon_0}\frac{\text{Ze}}{\pi\text{R}^4}+\frac{\text{e}^2}{16\pi\epsilon_0\text{r}^4}\bigg]
Thus, net force on proton 1 will be zero, when
\frac{\text{er}^2 2\text{e}}{4\epsilon_0\pi\text{R}^4}=\frac{\text{e}^2}{16\pi\epsilon_0\text{r}}
\Rightarrow\ \text{r}^4=\frac{\text{R}^4}{8}
Hence the protons must be at a distance \text{r}=\frac{\text{R}}{(8)^\frac{1}{4}} from the centre.
Q565. When a charged particle is placed in an electric field, it experiences an electrical force. If this is the only force on the particle, it must be the net force. The net 5 Marks
force will cause the particle to accelerate according to Newton's second law. So \vec{\text{F}}_\text{e}=\text{q}\vec{\text{E}}=\text{m}\vec{\text{a}}

If \vec{\text{E}} is uniform, then \vec{\text{a}} is constant and \vec{\text{a}}=\text{q}\vec{\text{E}}\text{/ m.} If the particle has a positive charge, its acceleration is
in the direction of the field. If the particle has a negative charge, its acceleration is in the direction opposite to the electric field. Since the acceleration is constant,
the kinematic equations can be used.
1. An electron of mass m, charge e falls through a distance h metre in a uniform electric field E. Then time of fall,
1. \text{t}=\sqrt{\frac{\text{2hm}}{\text{eE}}}
2. \text{t}=\frac{\text{2hm}}{\text{eE}}
3. \text{t}=\sqrt{\frac{\text{2eE}}{\text{hm}}}
4. \text{t}=\frac{\text{2eE}}{\text{hm}}
2. An electron moving with a constant velocity v along X-axis enters a uniform electric field applied along Y-axis. Then the electron moves:
1. With uniform acceleration along Y-axis
2. Without any acceleration along Y-axis
3. In a trajectory represented as y = ax2
4. In a trajectory represented as y = ax
3. Two equal and opposite charges of masses m1 and m2 are accelerated in an uniform electric field through the same distance. What is the ratio of their
accelerations if their ratio of masses is \frac{\text{m}_1}{\text{m}_2}=0.5?
1. \frac{\text{a}_1}{\text{a}_2}=2
2. \frac{\text{a}_1}{\text{a}_2}=0.5
3. \frac{\text{a}_1}{\text{a}_2}=3
4. \frac{\text{a}_1}{\text{a}_2}=1
4. A particle of mass m carrying charge q is kept at rest in a uniform electric field E and then released. The kinetic energy gained by the particle, when it moves
through a distance y is:
1. \frac{1}{2}\text{qEy}^2
2. qEy
3. qEy2
4. qE2y
5. A charged particle is free to move in an electric field. It will travel:
1. Always along a line of force.
2. Along a line of force, if its initial velocity is zero.
3. Along a line of force, if it has some initial velocity in the direction of an acute angle with the line of force.
4. None of these.

Ans:1. (a) \text{t}=\sqrt{\frac{\text{2hm}}{\text{eE}}}


Explanation:
From Newton's law,
\text{F}=\text{m}\vec{\text{a}}
Or \text{qE}=\text{m}\vec{\text{a}}
\Rightarrow\text{a}=\frac{\text{qE}}{\text{m}}=\frac{\text{eE}}{\text{m}}
Using, \text{s}=\text{ut}+\frac{1}{2}\text{at}^2
\therefore\text{ h}=0+\frac{1}{2}\times\frac{\text{eE}}{\text{m}}\text{t}^2
\Rightarrow\text{t}=\sqrt{\frac{\text{2hm}}{\text{eE}}}
2. (c) In a trajectory represented as y = ax2
3. (b) \frac{\text{a}_1}{\text{a}_2}=0.5
Explanation:
Force is same in magnitude for both.

https://bls.smartstudies.co.in/#/exam/pdf-preview/b5b87181-b79b-4b4a-8ebf-8f94fb605fc8/1 87/139
5/17/24, 8:14 PM Exam Automation
\therefore m1a1 = m2a2;
\frac{\text{a}_1}{\text{a}_2}=\frac{\text{m}_2}{\text{m}_1}=\frac{1}{0.5}=2
4. (b) qEy
Explanation:
Here, u = 0;
\text{a}=\frac{\text{qE}}{\text{m}};\text{ s}=\text{y}
Using, v2 - u2 = 2as
\Rightarrow\text{v}^2=2\frac{\text{qE}}{\text{m}}\text{y}
\therefore\text{ K.E.}=\frac{\text{1}}{\text{2}}\text{my}^2=\text{qEy}
5. (b) Along a line of force, if its initial velocity is zero.
Explanation:
If charge particle is put at rest in electric field, then it will move along line of force.
Q566. 1. Derive an expression for the electric field at any point on the equatorial line of an electric dipole. 5 Marks
2. Two identical point charges, q each, are kept 2m apart in air. A third point charge Q of unknown magnitude and sign is placed on the line joining the charges
such that the system remains in equilibrium. Find thea position and nature of Q.

Ans:1. Electric Field for points on the Equatorial Plane

The magnitudes of the electric field due to the two charges +q and -q are given by,
\text{E}_+\text{q}=\frac{\text{q}}{4\pi\epsilon_0}\frac{1}{\text{r}^2+\text{a}^2}\ ...(\text{i})
\text{E}_-\text{q}=\frac{\text{q}}{4\pi\epsilon_0}\frac{1}{\text{r}^2+\text{a}^2}\ ...(\text{ii})
The directions of E+q and E−q are as shown in the figure. The components normal to the dipole axis cancel away. The components along the dipole axis add up.
\therefore Total electric field, \text{E}=-(\text{E}_+\text{q}+\text{E}_{-\text{q}})\cos\theta\ \hat{\text{p}} \big[Negative sign shows that the field is opposite
to \hat{\text{p}}\big]
\text{E}=-\frac{2\text{qa}}{4\pi\epsilon(\text{r}^2+\text{a}^2)^\frac{3}{2}}\hat{\text{p}}\ ...(\text{iii})
At large distances (r>>a), this reduces to,
\text{E}=-\frac{2\text{qa}}{4\pi\epsilon_0\text{r}^3}\hat{\text{p}}\ ...(\text{iv})
\because\overrightarrow{\text{p}}=\text{q}\times\overrightarrow{2\text{a}}\hat{\text{p}}
\therefore\text{E}=\frac{-\overrightarrow{\text{p}}}{4\pi\epsilon_0\text{r}^3}(\text{r}>>\text{a})
2.

\frac{\text{K}(\text{q})(\text{Q})}{\text{x}}=\frac{-\text{K}(\text{q})(\text{q})}{2}
\Rightarrow\text{Q}=\frac{-\text{qx}}{2}
\because\frac{\text{K}\text{q}\text{Q}}{\text{x}}=\frac{\text{KQq}}{\text{y}}
\text{x}=\text{y}
\text{x}+\text{y}=2
\therefore\text{x}=\text{y}=1
\text{Q}=\frac{-\text{q}}{2}
Q567. A pendulum bob of mass 80mg and carrying a charge of 2 × 10-8C is at rest in a uniform, horizontal electric field of 20kVm-1. Find the tension in the thread. 5 Marks

Ans:

https://bls.smartstudies.co.in/#/exam/pdf-preview/b5b87181-b79b-4b4a-8ebf-8f94fb605fc8/1 88/139
5/17/24, 8:14 PM Exam Automation

\text{E}=20\text{Kv/m}=20\times10^3\text{v/m},
\text{m}=80\times10^{-5}\text{kg},\ \text{c}=20\times10^{-5}\text{C}
\tan\theta=\Big(\frac{\text{qE}}{\text{mg}}\Big)^{-1} \big[\text{T}\sin\theta=\text{mg},\ \text{T}\cos\theta=\text{qe}\big]
\tan\theta=\Big(\frac{2\times10^{-8}\times20\times10^3}{80\times10^{-6}\times10}\Big)
=\Big(\frac{1}{2}\Big)^{-1}
1+\tan^2\theta=\frac{1}{4}+1=\frac{5}{4} \Big[\cos\theta=\frac{1}{\sqrt{5}},\sin\theta=\frac{2}{\sqrt{5}}\Big]

\text{T}\sin\theta=\text{mg}
\Rightarrow\text{T}\times\frac{2}{\sqrt{5}}=80\times10^{-6}\times10
\Rightarrow\text{T}=\frac{8\times10^{-4}\times\sqrt{5}}{2}=4\times\sqrt{5}\times10^{-4}
=8.9\times10^{-4}
Q568. 1. Use Gauss’s theorem to find the electric field due to a uniformly charged infinitely large plane thin sheet with surface charge density σ. 5 Marks
2. An infinitely large thin plane sheet has a uniform surface charge density +σ . Obtain the expression for the amount of work done in bringing a point charge q
from infinity to a point, distant r, in front of the charged plane sheet.

Ans:1.

\oint E.ds =\frac{q}{\varepsilon_0}


The electric field E points outwards normal to the sheet. The field lines are parallel to the Gaussian surface except for surfaces 1 and 2. Hence the net flux = \oint
E.ds = EA + EA where A is the area of each of the surface 1 and 2.
\therefore\text{ }\text{ }\text{ }\oint E.ds =\frac{q}{\varepsilon_0}=\frac{\sigma A}{\varepsilon_0}=2EA;
E=\frac{\sigma }{2\varepsilon_0}
2.
W=q\int\limits_{\infty}^{r}\vec{E}.d\vec{r}
=q\int_{\infty}^{r}(-{E}d{r})
=-q\int_{\infty}^{r}\bigg(\frac{\sigma}{2\in_0}\bigg)d{r}
=\frac{q\sigma}{2\in}|\infty-r|
\Longrightarrow\text{ }\text{ }(\infty)
Q569. A rod of length L has a total charge Q distributed uniformly along its length. It is bent in the shape of a semicircle. Find the magnitude of the electric field at the 5 Marks
centre of curvature of the semicircle.

Ans:

https://bls.smartstudies.co.in/#/exam/pdf-preview/b5b87181-b79b-4b4a-8ebf-8f94fb605fc8/1 89/139
5/17/24, 8:14 PM Exam Automation

\lambda= Charge per unit length =\frac{\text{Q}}{\text{L}}


dq1 for a length d =\lambda\times\text{dl}
Electric field at the centre due to charge =\text{k}\times\frac{\text{dq}}{\text{r}^2}
The horizontal Components of the Electric field balances each other. Only the vertical components remain.
\therefore\ Net Electric field along vertical
\text{d}_\text{E}=2\text{E}\cos\theta
=\frac{\text{Kdq}\times\cos\theta}{\text{r}^2}
=\frac{2\text{k}\cos\theta}{\text{r}^2}\times\lambda\times\text{dl} \Big[but \text{d}\theta=\frac{\text{d}\ell}{\text{r}}=\text{d}\ell=\text{rd}\theta\Big]
\Rightarrow\frac{2\text{k}\lambda}{\text{r}^2}\cos\theta\times\text{rd}\theta=\frac{2\text{k}\lambda}{\text{r}}\cos\theta\times\text{d}\theta
\text{E}=\int\limits^{\frac{\pi}{2}}_0\frac{2\text{k}\lambda}{\text{r}}\cos\theta\times\text{d}\theta
=\int\limits_0^{\frac{\pi}{2}}\frac{2\text{k}\lambda}{\text{r}}\sin\theta
=\frac{2\text{k}\lambda\text{l}}{\text{r}}
=\frac{2\text{K}\theta}{\text{Lr}}
but \text{L}=\pi\text{R}\Rightarrow\text{r}=\frac{\text{L}}{\pi}
So, \text{E}=\frac{2\text{k}\theta}{\text{L}\times\big(\frac{\text{L}}{\pi}\big)}
=\frac{2\text{k}\pi\theta}{\text{L}^2}
=\frac{2}{4\pi\in_0}\times\frac{\pi\theta}{\text{L}^2}
=\frac{\theta}{2\in_0\text{L}^2}
Q570. Answer the following Questions. 5 Marks
1. Find expressions for the force and torque on an electric dipole kept in a uniform electric field.
OR
An electric dipole is held in a uniform electric field. (i) Using suitable diagram show that it does not undergo any translatory motion, and (ii) derive an expression for
torque acting on it and specify its direction.
2. Derive an expression for the work done in rotating a dipole from the angle \theta_0 to \theta_1 in a uniform electric field E.
​OR
1. Define torque acting on a dipole of dipole moment \vec{\text{p}} placed in a uniform electric field \vec{\text{E}}. Express it in the vector form and point out the
direction along which it acts.
2. What happens if the field is non-uniform?
3. What would happen if the external field \vec{\text{E}} is increasing (i) parallel to \vec{\text{p}} and (ii) anti-parallel to \vec{\text{p}}?

Ans:Let an electric dipole be rotated in electric field from angle \theta_0 to \theta_1 in the direction of electric field. In this process the angle of orientation is changing
continuously; hence the torque also changes continuously. Let at any time, the angle between dipole moment p and electric field E be then,
Torque on dipole \tau=\text{pE}\sin\theta
The work done in rotating the dipole a further by small angle \text{d}\theta is,
dW = Torque × angular displacement =\text{pE}\sin\theta\text{ d}\theta
Total work done in rotating the dipole from angle \theta_0 to \theta_1 is given by
\text{W}=\int\limits_{\theta_0}^{\theta_1}\text{pE}\sin\theta\text{ d}\theta=\text{pE}[-\cos\theta]^{\theta_1}_{\theta_0}
=-\text{pE}[\cos\theta_1-\theta_0]=\text{pE}(\cos\theta_0-\cos\theta_1)\dots(\text{i})
Special case: If electric dipole is initially in a stable equilibrium position (\theta_0=0^\circ) and rotated through an angle \theta(\theta_1=\theta) then work done
\text{W}=\text{pE}[\cos0^\circ-\cos\theta]=\text{PE}(1-\cos\theta)\dots(\text{ii})
Q571. Two point charges of +5 × 10-19C and +20 × 10-19C are separated by a distance of 2m. Find the point on the line joining them at which electric field intensity is zero. 5 Marks

Ans:Let charges q1 = +5 × 10-19C and q2 = +20 × 10-19C be placed at A and B respectively. Distance AB = 2m.
As charges are similar, the electric field strength will be zero between the charges on the line joining them. Let P be the point (at a distance x from q1) at which
electric field intensity is zero. Then, AP = x metre, BP = (2 - x) metre. The electric field strength at P due to charge q1 is,
\vec{\text{E}}_1=\frac{1}{4\pi\epsilon_0}\frac{\text{q}_1}{\text{x}^2}, along the direction A to P.
The electric field strength at P due to charge q2 is,
\vec{\text{E}}_2=\frac{1}{4\pi\epsilon_0}\frac{\text{q}_2}{(2-\text{x})^2}, along the direction B to P.
Clearly, \vec{\text{E}}_1 and \vec{\text{E}}_2 and are opposite in direction and for net electric field at P to be zero, \vec{\text{E}}_1 and \vec{\text{E}}_2 and must be
equal in magnitude.
So, E1 = E2
\Rightarrow\ \frac{1}{4\pi\epsilon_0}\frac{\text{q}_1}{\text{x}^2}=\frac{1}{4\pi\epsilon_0}\frac{\text{q}_2}{(2-\text{x})^2}

Given, q1 = 5 × 10-19C, q2 = 20 × 10-19C


Therefore, \frac{5\times10^{-19}}{\text{x}^2}=\frac{20\times10^{-19}}{(2-\text{x})^2}
Or \frac{1}{2}=\frac{\text{x}}{2-\text{x}}
Or \text{x}=\frac{2}{3}\text{m}
Q572. A charge Q is placed at the centre of an uncharged, hollow metallic sphere of radius a: 5 Marks

https://bls.smartstudies.co.in/#/exam/pdf-preview/b5b87181-b79b-4b4a-8ebf-8f94fb605fc8/1 90/139
5/17/24, 8:14 PM Exam Automation
1. Find the surface charge density on the inner surface and on the outer surface.
2. If a charge q is put on the sphere, what would be the surface charge densities on the inner and the outer surfaces?
3. Find the electric field inside the sphere at a distance x from the centre in the situations (a) and (b).

Ans:Given:
Amount of charge present at the centre of the hollow sphere = Q
We know that charge given to a hollow sphere will move to its surface.
Due to induction, the charge induced at the inner surface = -Q
Thus, the charge induced on the outer surface = +Q
1. Surface charge density is the charge per unit area, i.e.
\sigma=\frac{\text{Charge}}{\text{Total surface area}}
Surface charge density of the inner surface, \sigma_{\text{in}}=\frac{-\text{Q}}{4\pi\text{a}^2}
Surface charge density of the outer surface, \sigma_{\text{out}}=\frac{\text{Q}}{4\pi\text{a}^2}
2. Now if another charge q is added to the outer surface, all the charge on the metal surface will move to the outer surface. Thus, it will not affect the charge
induced on the inner surface. Hence the inner surface charge density,
\sigma_{\text{in}}=-\frac{\text{q}}{4\pi\text{a}^2}
As the charge has been added to the outer surface, the total charge on the outer surface will become (Q + q).
So the outer surface charge density, \sigma_{\text{out}}=\frac{\text{q}+\text{Q}}{4\pi\text{a}^2}
3. To find the electric field inside the sphere at a distance x from the centre in both the situations,let us assume an imaginary sphere inside the hollow sphere at a
distance x from the centre.
Applying Gauss's Law on the surface of this imaginary sphere,we get:
\oint\text{E.ds}=\frac{\text{Q}}{\epsilon_0}
\text{E}\oint\text{ds}=\frac{\text{Q}}{\epsilon_0}
\text{E}\big(4\pi\text{x}^2\big)=\frac{\text{Q}}{\epsilon_0}
\text{E}=\frac{\text{Q}}{\epsilon_0}\times\frac{1}{4\pi\text{x}^2}=\frac{\text{Q}}{4\pi\epsilon_0\text{x}^2}
Here, Q is the charge enclosed by the sphere.
For situation (b):
As the point is inside the sphere, there is no effect of the charge q given to the shell.
Thus, the electric field at the distance x:
\text{E}=\frac{\text{Q}}{4\pi\epsilon_0\text{x}^2}
Q573. When electric dipole is placed in uniform electric field, its two charges experience equal and opposite forces, which cancel each other and hence net force on 5 Marks
electric dipole in uniform electric field is zero. However, these forces are not collinear, so they give rise to some torque on the dipole. Since net force on electric
dipole in uniform electric field is zero, so no work is done in moving the electric dipole in uniform electric field. However, some work is done in rotating the dipole
against the torque acting on it.

1. The dipole moment of a dipole in a uniform external field \vec{\text{E}} is \vec{\text{P}}. Then the torque 'i acting on the dipole is:
1. \vec{\tau}=\vec{\text{P}}\times\vec{\text{E}}
2. \vec{\tau}=\vec{\text{P}}\cdot\vec{\text{E}}
3. \vec{\tau}=2(\vec{\text{P}}+\vec{\text{E}})
4. \vec{\tau}=(\vec{\text{P}}+\vec{\text{E}})
2. An electric dipole consists of two opposite charges, each of magnitude 1.0\mu\text{C} separated by a distance of 2.0cm. The dipole is placed in an external
field of 105N C-1. The maximum torque on the dipole is:
1. 0.2 × 10-3Nm
2. 1 × 10-3Nm
3. 2 × 10-3Nm
4. 4 × 10-3Nm
3. Torque on a dipole in uniform electric field is minimum when \theta is equal to:
1. 0º
2. 90º
3. 180º
4. Both (a) and (c)
4. When an electric dipole is held at an angle in a uniform electric field, the net force F and torque t on the dipole are:
1. \text{F}=0, \tau=0
2. \text{F}\not=0, \tau\not=0
3. \text{F}=0, \tau\not=0
4. \text{F}\not=0, \tau=0
5. An electric dipole of moment pis placed in an electric field of intensity E. The dipole acquires a position such that the axis of the dipole makes an angle
\theta with the direction of the field. Assuming that the potential energy of the dipole to be zero when \theta=90^\circ the torque and the potential energy of
the dipole will respectively be:
1. \text{pE}\sin\theta,-\text{pE}\cos\theta
2. \text{pE}\sin\theta,-2\text{pE}\cos\theta
3. \text{pE}\sin\theta,2\text{pE}\cos\theta
4. \text{pE}\cos\theta,-2\text{pE}\sin\theta

Ans:1. (a) \vec{\tau}=\vec{\text{P}}\times\vec{\text{E}}


Explanation:
As \tau= either force × perpendicular distance between the two forces.
=\text{qaE}\sin\theta\text{ or }\tau=\text{PE}\sin\theta

https://bls.smartstudies.co.in/#/exam/pdf-preview/b5b87181-b79b-4b4a-8ebf-8f94fb605fc8/1 91/139
5/17/24, 8:14 PM Exam Automation
(\because\text{qa}=\text{P})
Or \vec{\tau}=\vec{\text{P}}\times\vec{\text{E}}
2. (c) 2 × 10-3Nm
Explanation:
The maximum torque on the dipole in an external electric field is given by
\tau=\text{pE}=\text{q}(\text{2a})\times\text{E}
Here, \text{q}=1\mu\text{C}=10^{-6}\text{C,}
2a = 2cm = 2 × 10-2m,
E = 105N C-1,
\tau=?
\therefore\tau=10^{-6}\times2\times10^{-2}\times10^5
=2\times10^{-3}\text{Nm}
3. (d) Both (a) and (c)
Explanation:
When \theta is 0 or 180º, the \tau minimum, which means the dipole moment should be parallel to the direction of the uniform electric field.
4. (c) \text{F}=0, \tau\not=0
Explanation:
Net force is zero and torque acts on the dipole, trying to align p with E.
5. (a) \text{pE}\sin\theta,-\text{pE}\cos\theta
Explanation:
Torque, \tau=\text{pE}\sin\theta and potential energy, \text{U}=-\text{pE}\cos\theta.​​
Q574. Two point charges qA = 3 μC and qB = –3 μC are located 20 cm apart in vaccum. 5 Marks
1. What is the electric field at the midpoint O of the line AB joining the two charges?
2. If a negative test charge of magnitude 1.5 × 10-9 C is placed at this point, what is the force experienced by the test charge?

Ans:1. The situation is represented in the given figure. O is the mid-point of line AB. Distance between the two charges, AB cm

\therefore AO = OB = 10 cm
Net electric field at point O = E
Electric field at point O caused by +3µC charge,
\text{E}1=\frac{3\times10^{-6}}{4\pi\in_0(\text{AO})^2}=\frac{3\times10^{-6}}{4\pi\in_0(10\times10^{-2})^2}\text{N/C}\ \text{along OB}
Where,
\in_0\ = Permittivity of free space
\frac{1}{4\pi\in_0}=9\times10^9\text{ Nm}^2\text{C}^{-2}
Magnitude of electric field at point O caused by - 3µC charge,
\text{E}2=\frac{-3\times10^{-6}}{4\pi\in_0(\text{OB})^2}=\frac{3\times10^{-6}}{4\pi\in_0(10\times10)^{-2}}\text{ N/C along OB}
∴ = E1 + E 2
=2\times\bigg[(9\times10^9)\times\frac{3\times10^{-6}}{(10\times10^{-2})^2}\bigg] [Since the values of E1 and E2 are same, the value is multiplied with 2]
= 5.4 × 106 N/C along OB
Therefore, the electric field at mid-point O is 5.4 × 106 NC-1 along OB
2. A test charge of amount 1.5 × 10-9 C is placed at mid-point O.
q = 1.5 × 10-9 C
Force experienced by the test charge = F
∴ F = qE
= 1.5 × 10-9 × 5.4 × 106
= 8.1 × 10-3 N
The force is directed along line OA. This is because the negative test charge is repelled by the charge placed at point B but attracted towards point A.
Therefore, the force experienced by the test charge is 8.1 × 10-3 N − × along OA.
Q575. 1. Define electric dipole moment. Is it a scalar or a vector? Derive the expression for the electric field of a dipole at a point on the equatorial plane of the dipole. 5 Marks
2. Draw the equipotential surfaces due to an electric dipole. Locate the points where the potential due to the dipole is zero.

Ans:1.

It is defined as the product of either charge and the distance between the two equal and opposite charges
| P| =q.|2a|
It is a vector quantity, so \overrightarrow{\text{p}} = \text{q}.\overrightarrow{\text{2a}}
Derivation At a point of equatorial plane : Consider a point P on broad side on the position of dipole formed of charges + q and - q at separation 2l. The distance of
point P from mid point (O) of electric dipole is r. Let E1 and E2 be the electric field strengths due to charges + q and - q of electric dipole.

https://bls.smartstudies.co.in/#/exam/pdf-preview/b5b87181-b79b-4b4a-8ebf-8f94fb605fc8/1 92/139
5/17/24, 8:14 PM Exam Automation

From fig. AP = BP = \sqrt{\text{r}^{2} + l^{2}}


\therefore \overrightarrow{\text{E}}_{1} = \frac{1}{4\pi\varepsilon_{0}}\frac{\text{q}}{\text{r}^{2} + l^{2}} along B to P
\overrightarrow{\text{E}}_{2} = \frac{1}{4\pi\varepsilon_{0}}\frac{\text{q}}{\text{r}^{2} + l^{2}} along P to A
Clearly \overrightarrow{\text{E}}_{1}\text{ and }\overrightarrow{\text{E}}_{2} are equal in magnitude i.e. |\overrightarrow{\text{E}}_{1}| =
|\overrightarrow{\text{E}}_{2}|or E1 = E2
To find the resultant of \overrightarrow{\text{E}}_{1}\text{ and } \overrightarrow{\text{E}}_{2}, we resolve them along and perpendicular to AB.
Component of \overrightarrow{\text{E}}_{1}along AB = E1 cos \theta, parallel to \text{B}\overrightarrow{\text{A}}
Component of \overrightarrow{\text{E}}_{1}perpendicular to AB = E1 sin \theta along O to P
Component of \overrightarrow{\text{E}}_{2} along AB =E2 cos, \theta parallel to \text{B}\overrightarrow{\text{A}}
Component of \overrightarrow{\text{E}}_{2} perpendicular to AB = E2 sin \thetaalong P to O
Clearly components of \overrightarrow{\text{E}}_{1} and \overrightarrow{\text{E}}_{2}perpendicular to AB : E1 sin \thetaand E2 sin \thetabeing equal and opposite
cancel each other, while the components of \overrightarrow{\text{E}}_{1}and \overrightarrow{\text{E}}_{2} along AB : E1 cos \thetaand E2 cos \theta, being in the
same direction add up and give the resultant electric field whose direction is parallel to \overrightarrow{\text{BA}}
\therefore Resultant electric field at P is E = E1 cos \theta+ E2 cos \theta
But \text{E}_{1} = \text{E}_{2} = \frac{1}{4\pi\varepsilon_{0}}\frac{\text{q}}{(\text{r}^{2} + l^{2})}
and \cos\theta = \frac{\text{OB}}{\text{PB}} = \frac{l}{\sqrt{\text{r}^{2} + l^{2}}} =\frac{l}{(\text{r}^{2} + l^{2})^{1/2}}
\therefore\text{E} = 2 \text{E}_{1}\cos\theta = 2 \times\frac{1}{4\pi\varepsilon_{0}}\frac{\text{q}}{(\text{r}^{2} + l^{2})}.\frac{l}{(\text{r}^{2} + l^{2})^{1/2}}
= \frac{1}{4\pi\varepsilon_{0}}\frac{2\text{ql}}{(\text{r}^{2} + l^{2})^{3/2}}
But q.2l = p =electric dipolemoment
\therefore\text{E} = \frac{1}{4\pi\varepsilon_{0}}\frac{\text{P}}{(\text{r}^{2} + l^{2})^{3/2}} - - - - - - (iii)
\text{E} =\frac{1}{4\pi\varepsilon_{0}}\frac{\text{p}}{(\text{r}^{2})^{3/2}} = \frac{1}{4\pi\varepsilon_{0}}\frac{\text{p}}{\text{r}^{3}}
If dipole is infinitesimal and point P is far away, we have l < < r, so l2 may be neglected as compared to r2 and so equation (3) gives
\therefore\text{E} =\frac{1}{4\pi\varepsilon_{0}}\frac{\text{p}}{(\text{r}^{2})^{3/2}} = \frac{1}{4\pi\varepsilon_{0}}\frac{\text{p}}{\text{r}^{3}}
i.e. electric field strength due to a short dipole at broadside on position
\text{E} = \frac{1}{4\pi\varepsilon_{0}}\frac{\text{P}}{\text{r}^{3}} parallel to \overrightarrow{\text{BA}}
2.

Electric potential is zero at all points in the plane passing through the dipole equator.
Q576. 1. Define electrostatic potential at a point. Write its S.I. unit. Three point charges q1, q2 and q3 are kept respectively at points A, B and C as shown in the figure. 5 Marks
Derive the expression for the electrostatic potential energy of the system.

2. Depict the equipotential surfaces due to.


1. An electric dipole,
2. Two identical positive charges separated by a distance.

Ans:1. Electrostatic potential: Work done by an external force in bringing a unit positive charge from infinity to the given point SI unit- volt or J/C) Net work done in
moving charges q1. q2 & q3 from infinity to A, B and C respectively. \text{𝑊} = 0 + \text{q}_{2}\text{V}_{13} + \text{q}_{3}(\text{V}_{13}\text{V}_{23})
\frac{1}{4\pi\varepsilon_{\circ}}\frac{\text{q}_{1}\text{q}_{2}}{\text{r}_{12}} + \frac{1}{4\pi\varepsilon_{\circ}}\bigg(\frac{\text{q}_{1}\text{q}_{3}}{\text{r}_{13}} +
\frac{\text{q}_{2}\text{q}_{3}}{\text{r}_{23}}\bigg)
But potential energy of the system is equal to the work done.
\therefore \text{u} = \text{w}=\frac{1}{4\pi\varepsilon_{0}}\bigg(\frac{\text{q}_{1}\text{q}_{2}}{\text{r}_{12}} + \frac{\text{q}_{1}\text{q}_{3}}{\text{r}_{23}} +
\frac{\text{q}_{2}\text{q}_{3}}{\text{r}_{23}}\bigg)
2. Equipotential surface due to
1. An electric dipole.

https://bls.smartstudies.co.in/#/exam/pdf-preview/b5b87181-b79b-4b4a-8ebf-8f94fb605fc8/1 93/139
5/17/24, 8:14 PM Exam Automation

2. Two identical positive changes.


Q577. 1. Use Gauss’s law to show that due to a uniformly charged spherical shell of radius R, the electric field at any point situated outside the shell at a distance r from 5 Marks
its centre is equal to the electric field at the same point, when the entire charge on the shell were concentrated at its centre. Also plot the graph showing the
variation of electric field with r, for \text{r}\leq\text{R} and \text{r}\geq\text{R}.
2. Two point charges of +1\mu\text{C} and +\text{4}\mu\text{C} are kept 30cm apart. How far from the +1\mu\text{C} charge on the line joining the two charges,
will the net electric field be zero?

Ans:1.

Consider a spherical Gaussian surface of radius \text{r}\leq\text{R}, concentric with given shell. If \vec{\text{E}} is electric field outside the shell, then by
symmetry, electric field strength has same magnitude \vec{\text{E}_0} on the Gaussian surface and is directed radially outward. Also the directions of normal at
each point is radially outward, so angle between \vec{\text{E}_0} and \vec{\text{ds}} is zero at each point. Hence, electric flux through Gaussian.
=\phi_\text{s} \ \vec{\text{E}_0}\cdot\vec{\text{ds}}
\phi_\text{s}\text{E}_0\text{ds}\cos0^\circ
Surface =\text{E}_04\pi\text{r}^2
Now, Gaussian surface is outside the given charged shell, so charge enclosed by the Gaussian surface is Q.
Hence, by Gauss's theorem,
\phi_\text{s}\ \vec{\text{E}_0}\cdot\vec{\text{ds}}=\frac{1}{\varepsilon_0}\times\text{charge enclosed}
\Rightarrow\text{E}_0\cdot=\frac{1}{4\pi\varepsilon_0}\times\frac{\text{Q}}{\text{r}^2}
Thus, electric field outside a charged thin spherical shell is same as if the whole charge Q is concentrated at the centre. Graphically,

https://bls.smartstudies.co.in/#/exam/pdf-preview/b5b87181-b79b-4b4a-8ebf-8f94fb605fc8/1 94/139
5/17/24, 8:14 PM Exam Automation

For r < R, there is no strength of electric field inside a charged spherical shell.
For r > R, electric field outside a charged thin spherical shell is same as if the whole charge Q is concentrated at the centre.
2. Given that,
First point charge =+1\mu\text{C}
Second point charge =+4\mu\text{C}
Distance = 30cm
Let us consider the net electric field zero at x distance from first charge.
We need to calculate the electric field.
Using formula of electric field,
\text{E}_\text{x}=\text{E}_{30-\text{x}}
\frac{\text{kq}}{\text{r}^2}=\frac{\text{kq}}{\text{r}^2}
Put the value into the formula,
\frac{1\times10^{-6}}{\text{x}^2}=\frac{4\times10^{-6}}{(30-\text{x})^2}
\frac{(30-\text{x})^2}{\text{x}^2}=\frac{4\times10^{-6}}{(30-\text{x})^2}
\frac{(30-\text{x})^2}{\text{x}^2}=\frac{4\times10^{-6}}{1\times10^{-6}}
\frac{30-\text{x}}{\text{x}}=\sqrt{\frac41}
\text{30}-\text{x}=2\text{x}
\text{x}=10{\text{cm}}
Hence, the net electric field will be zero at distance 10cm from +1\mu\text{C}.
Q578. Check that the ratio ke2/G memp is dimensionless. Look up a Table of Physical Constants and determine the value of this ratio. What does the ratio signify? 5 Marks

Ans:The given ratio is \frac{\text{ke}^2}{\text{G}\text{m}_e\text{m}_p}.


Where,
G = Gravitational constant
Its unit is Nm2kg-2
me and mp = Masses of electron and proton.
Their unit is kg.
e = Electric charge.
Its unit is C.
\in_0 = Permittivity of free space
Its unit is Nm2 C-2
Therefore unit of the given ratio \frac{\text{ke}^2}{\text{Gm}_e\text{m}_p}
=\frac{\Big[\text{Nm}^2\text{C}^{+2}\Big]\Big[\text{C}^{+2}\Big]}{\Big[\text{Nm}^2\text{kg}^{-2}\Big]\Big[\text{kg}\Big]\Big[\text{kg}\Big]}
= MºLºTº
Hence, the given ratio is dimensionless.
e = 1.6 × 10-19 C
G = 6.67 × 10-11 Nkg-2
me = 9.1 × 10-31 kg
mp = 1.66 × 10-27 kg
Hence, the numerical value of the given ratio is
\frac{\text{ke}^2}{\text{Gm}_e\text{m}_p}=\frac{9\times10^{9}\times(1.6\times10^{-19})^2}
{6.67\times10^{-11}\times9.1\times10^{-3}\times1.67\times10^{-22}}\approx2.3\times10^{39}
This is the ratio of electric force to the gravitational force between a proton and an electron, keeping distance between them constant.
Q579. Gauss's law and Coulomb's law, although expressed in different forms, are equivalent ways of describing the relation between charge and electric field in static 5 Marks
conditions. Gauss's law is \epsilon_0\phi=\text{q}_{\text{end},} when \text{q}_{\text{encl}} is the net charge inside an imaginary closed surface called Gaussian
surface. \phi=\oint\vec{\text{E}}\cdot\text{d}\vec{\text{A}} gives the electric flux through the Gaussian surface. The two equations hold only when the net charge
is in vacuum or air.

https://bls.smartstudies.co.in/#/exam/pdf-preview/b5b87181-b79b-4b4a-8ebf-8f94fb605fc8/1 95/139
5/17/24, 8:14 PM Exam Automation

1. If there is only one type of charge in the universe, then (\vec{\text{E}}\rightarrow Electric field, \text{d}\vec{\text{s}}\rightarrow Area vector).
1. \oint\vec{\text{E}}\cdot\text{d}\vec{\text{s}}\not=0 on any surface.
2. \oint\vec{\text{E}}\cdot\text{d}\vec{\text{s}} could not be defined.
3. \oint\vec{\text{E}}\cdot\text{d}\vec{\text{s}}=\infty if charge is inside.
4. \oint\vec{\text{E}}\cdot\text{d}\vec{\text{s}}=0 if charge is outside, \oint\vec{\text{E}}\cdot\text{d}\vec{\text{s}}=\frac{\text{q}}{\epsilon_0} if charge is inside.
2. What is the nature of Gaussian surface involved in Gauss law of electrostatic?
1. Magnetic.
2. Scalar.
3. Vector.
4. Electrical.
3. A charge 10\mu\text{C} is placed at the centre of a hemisphere of radius R = 10cm as shown. The electric flux through the hemisphere (in MKS units) is:

1. 20 × 105
2. 10 × 105
3. 6 × 105
4. 2 × 105
4. The electric flux through a closed surface area S enclosing charge Q is \phi. If the surface area is doubled, then the flux is:
1. 2\phi
2. \frac{\phi}{2}
3. \frac{\phi}{4}
4. \phi
5. A Gaussian surface encloses a dipole. The electric flux through this surface is:
1. \frac{\text{q}}{\epsilon_0}
2. \frac{\text{2q}}{\epsilon_0}
3. \frac{\text{q}}{2\epsilon_0}
4. \text{Zero}

Ans:1. (d) \oint\vec{\text{E}}\cdot\text{d}\vec{\text{s}}=0 if charge is outside, \oint\vec{\text{E}}\cdot\text{d}\vec{\text{s}}=\frac{\text{q}}{\epsilon_0} if charge is


inside.
Explanation:
If there is only one type of charge in the universe, then it will produce electric field somehow. Hence, Gauss's law is valid.
2. (c) Vector.
3. (c) 6 × 105
​Explanation:
According to Gauss's theorem,
Electric flux through the sphere =\frac{\text{q}}{\epsilon_0}
\therefore Electric flux through the hemisphere =\frac{1}{2}\frac{\text{q}}{\epsilon_0}
=\frac{10\times10^{-6}}{2\times8.854\times10^{-12}}=0.56\times10^6\text{ Nm}^2\text{ C}^{-1}
= 0.6 × 106 Nm2 C-1 = 6 × 105 Nm2 C-1
4. (d) \phi
​Explanation:
As flux is the total number of tines passing through the surface, for a given charge, it is always the charge enclosed \frac{\text{Q}}{\epsilon_0}. If area is doubled,
the flux remains the same.
5. (d) \text{Zero}

https://bls.smartstudies.co.in/#/exam/pdf-preview/b5b87181-b79b-4b4a-8ebf-8f94fb605fc8/1 96/139
5/17/24, 8:14 PM Exam Automation
Explanation:
As net charge on a dipole is (- q + q) = 0
Thus, when agaussian surface encloses a dipole, as per Gauss's theorem, electric flux through the surface,
\oint\vec{\text{E}}\cdot\text{d}\vec{\text{S}}=\frac{\text{q}}{\epsilon_0}=0​​
Q580. Five charges, q each are placed at the corners of a regular pentagon of side 'a' (Fig.). 5 Marks

1.
1. What will be the electric field at O, the centre of the pentagon?
2. What will be the electric field at O if the charge from one of the corners (say A) is removed?
3. What will be the electric field at O if the charge q at A is replaced by -q?
2. How would your answer to (a) be affected if pentagon is replaced by n-sided regular polygon with charge q at each of its corners?

Ans:1.
1. Here, point O is equidistant from all the charges at the end point of pentagon. Thus ,due to symmetry, the forces due to all the charges are cancelled out; As a
result electric field at O is zero.
2. When charge q is removed, it is equivalent to placing a -q charge at the same point. The electric field at point O would be.
\vec{\text{E}}=\frac{\text{q}}{4\pi\epsilon_0\text{r}^2}\text{along}\vec{\text{ OA}}
3. If charge q at A is replaced by -q, which is equivalent to placing two negative charges i.e., -2q the same point. So value of electric field would be,
\vec{\text{E}'}=\frac{2\text{q}}{4\pi\epsilon_0\text{r}^2}\text{along}\vec{\text{ OA}}
2. If pentagon shown here is replaced by any n sided resular polygon with charge q at each of its comers. the electric firld ar O would continue to be zer.
Q581. 1. An electric dipole of dipole moment \vec{\text{p}} is placed in a uniform electric field \vec{\text{E}} at an angle \theta with it. Derive the expression for torque 5 Marks
(\vec{\tau}) acting on it. Find the orientation of the dipole relative to the electric field for which torque on it is (1) maximum, and (2) half of maximum.
2. Two point charges \text{q}_1=+1\mu\text{C} and \text{q}_2=+4\mu\text{C} are placed 2m apart in air. At what distance from q1 along the line joining the two
charges, will the net electric field be zero?

Ans:1.

From digaram
Magnitude of Torque =(\text{qE})(2\text{a}\sin\theta)
=(2\text{qa})(\text{E}\sin\theta)
=\text{pE}\sin\theta
For direction \vec{\tau}=\vec{\text{p}}\times\vec{\text{E}}
1. For maximum Torque, dipole should be placed perpendicular to the direction of electric field.
\theta=90^\circ=\frac{\pi}{2}
2. For the torque to be half the maximum,
\theta=30^\circ=\frac{\pi}{6}
2.

\text{E}_\text{PA}=\text{E}_\text{PB}\ ;\ \text{E}=\frac{\text{kq}}{\text{r}^2}
\frac{\text{kq}_\text{A}}{\text{x}^2}=\frac{\text{kq}_\text{B}}{(2-\text{x})^2}
\frac{1}{\text{x}^2}=\frac{4}{(2-\text{x})^2}
\frac{1}{\text{x}}=\frac{2}{2-\text{x}}
\text{x}=\frac23\text{m}
Q582. 1. Two point charges q1 and q2 are kept r distance apart in a uniform external electric field \vec{\text{E}}. Find the amount of work done in assembling this 5 Marks
system of charges.
2. A cube of side 20cm is kept in a region as shown in the figure. An electric field \vec{\text{E}}. exists in the region such that the potential at a Point is given by
V = 10x + 5, where V is in volt and x is in m.

https://bls.smartstudies.co.in/#/exam/pdf-preview/b5b87181-b79b-4b4a-8ebf-8f94fb605fc8/1 97/139
5/17/24, 8:14 PM Exam Automation

Find the:
1. Electric field \vec{\text{E}}.
2. Total electric flux through the cube.

Ans:1. Let the charge q1​travels r1​distance and q1​travels.


The work done in bringing the charge q1​in the field to
W 1 = F 1 × r1
= q1 E × r 1
The work done in bringing the second charge,
W 2 = F 2 × r2
= q 2E × r 1
And the work is also done to overcome the force of the charge on one-another.
\text{W}_3=\frac{1}{4\pi\in_0}\cdot\frac{\text{q}_1\text{q}_2}{\text{r}^2}
So, total work =\text{q}_1\text{Er}_1+\text{q}_2\text{Er}_2+\frac{1}{4\pi\in_0}\frac{\text{q}_1\text{q}_2}{\text{r}^2}
2.
1. Given that,
V = 10x + 5
We know
\text{E}=-\frac{\text{dv}}{\text{dx}}
\text{V}=10\text{x}+5
\frac{\text{dV}}{\text{dx}}=\frac{\text{d}}{\text{dx}}(10\text{x}+5)
=10\frac{\text{d}}{\text{dx}}\text{x}+0
=10
\text{E}=-\frac{10\text{N}}{\text{C}}
\vec{\text{E}}=-\frac{10\hat{\text{i}}\text{N}}{\text{C}}
2. Since electric field is constant in negative x-direction.
So, flux enter in the cube will be same as flux come out through the cube,
\phi_\text{in}=\phi_\text{out}
So Net flux from the cube = 0.
Q583. Electric field in the given figure is directed along +X direction and given by Ex = 5Ax + 2B, where E is in NC-1 and x is in metre, A and B are constants with 5 Marks
dimensions. Taking A = 10NC-1 m-1 and B = 5NC-1, calculate:
1. The electric flux through the cube.

2. Net charge enclosed within the cube.

Ans:

https://bls.smartstudies.co.in/#/exam/pdf-preview/b5b87181-b79b-4b4a-8ebf-8f94fb605fc8/1 98/139
5/17/24, 8:14 PM Exam Automation

Given Ex = 5Ax + 2B
The electric field at face M where x = 0 is,
E1 =2B
The electric field at face N where x = 10cm = 0.10m is,
E2 = 5A × 0.10 + 2B = 0.5A + 2B
The electric flux through face M is,
\Phi_1=\vec{\text{E}}_1.\vec{\text{S}}_1=\text{E}_1\text{S}_1\cos\pi=-\text{E}_1\text{S}_1
= -2B × l2 where l = 10cm = 0.01m
The electric flux through face N,
\Phi_2=\vec{\text{E}}_2.\vec{\text{S}}_2=\text{E}_2\text{S}_2\cos0=(0.5\text{A}+2\text{B})\text{l}^2
Net electric flux, \Phi=\Phi_1+\Phi_2
= -2Bl2 + (0.5A + 2B)l2 = 0.5Al2
= 0.5 × 10 × (0.10)2 = 5 × 10-2V m
Q584. 1. Using Gauss’ law, derive an expression for the electric field intensity at any point outside a uniformly charged thin spherical shell of radius R and charge 5 Marks
density σ C/m2. Draw the field lines when the charge density of the sphere is (i) positive, (ii) negative.
2. A uniformly charged conducting sphere of 2.5 m in diameter has a surface charge density of 100 µC/m2. Calculate the
i. Charge on the sphere.
ii. Total electric flux passing through the sphere.

Ans:1.

\oint\overrightarrow{\text{E}}.\overrightarrow{ds} =\frac{\text{q}}{\in}_\circ
\oint \text{Eds cos} \theta = \frac{\sigma4\pi\text{R}^{2}}{\in_0}
\text{E}4\pi\text{r}^{2} = \frac{\sigma4\pi\text{R}^{2}}{\in_\circ}
\therefore \text{E} = \frac{\sigma\text{R}^{2}}{\in_\circ\text{r}^{2}}

2. .
1. \text{q} = \sigma4\pi\text{R}^{2}
= 100\times 10^{-6}\times 4 \times 3.14 \times \bigg(\frac{2.5}{2}\bigg)^{2}
= 19.6 \times 10^{-4}\text{C}
2. flux, \phi = \frac{\text{q}}{\in_{\circ}}
=\frac{19.6\times10^{-4}}{8.85\times10^{-12}}
=2.2\times10^{-4} \text{Nm}^{2}\text{C}^{-1}.
Q585. 1. An electric dipole of dipole moment \overrightarrow{p}consists of point charges +q and -q separated by a distance 2a apart. Deduce the expression for the 5 Marks
electric field \overrightarrow{E} due to the dipole at a distance x from the centre of the dipole on its axial line in terms of the dipole
moment \overrightarrow{P}Hence show that in the limit x >> a , \overrightarrow{E}\rightarrow2\overrightarrow{p}/(4\pi\varepsilon_{0}\text{x}^{3})
2. Given the electric field in the \overrightarrow{E} = 2\text{x}\hat{i}, region , find the net electric flux through the cube and the charge enclosed by it.

Ans:1.

https://bls.smartstudies.co.in/#/exam/pdf-preview/b5b87181-b79b-4b4a-8ebf-8f94fb605fc8/1 99/139
5/17/24, 8:14 PM Exam Automation

Electric field intensity at point P due to charge -q


\overrightarrow{\text{E}_{q}} = \frac{1}{4\pi\varepsilon_{0}}\frac{\text{q}}{(\text{x} +\text{a})^{2}}(\hat{\text{X}})
Due to charge +q
\overrightarrow{\text{E}_{+q}} = \frac{1}{4\pi\varepsilon_{0}}.\frac{\text{q}}{(\text{x} -\text{a})^{2}}(\hat{\text{X}})
Net electric field at point P
\frac{\text{q}}{4\pi\varepsilon_{0}}\times\bigg[\frac{1}{(\text{x} - \text{a})^{2}} -\frac{1}{(\text{x} + \text{a})^{2}}\bigg]^{\overrightarrow{\text{E}} =
\overrightarrow{\text{E}_{-q}}+ \overrightarrow{\text{E}_{+q}}}\hat{(x)}
= \frac{\text{q}}{4\pi\varepsilon_{0}}\bigg[\frac{4\text{aqx}}{(\text{x}^{2} -\text{a}^{2})^{2}}\bigg]\hat{(x)}
\frac{1}{4\pi\varepsilon_{0}}\frac{(\text{q}\times2\text{a})2x}{(\text{x}^{2} - \text{a}^{2})^{2}}\hat{(x)}
\overrightarrow{\text{E}} = \frac{1}{4pi\varepsilon_{0}}.\frac{2\text{px}}{(\text{x}^{2} - \text{a}^{2})^{2}}\hat{x}
For x >> a
(\text{x}^{2} - \text{a}^{2})^{2}\cong\text{x}^{4}
\overrightarrow{\text{E}} = \frac{1}{4\pi\varepsilon_{0}}.\frac{2\text{p}}{x^{3}}\hat{x}
2. Only the faces perpendicular to the direction of x-axis, contribute to the electric flux. The remaining faces of the cube give zero contribution.

Total Flux \phi = \phi_{I} + \phi_{II}


= \oint _{1}\overrightarrow{E}.\overrightarrow{ds} +\oint_{II}\overrightarrow{E}.\overrightarrow{ds}
=0 + 2(\text{a}).\text{a}^{2}
\therefore\phi = 2\text{a}^{3}.
Q586. 1. Define electric flux.Write its S.I. unit. 5 Marks
2. Using Gauss's law, prove that the electric field at a point due to a uniformly charged infinite plane sheet is independent of the distance from it.
3. How is the field directed if (i) the sheet is positively charged, (ii) negatively charged?

Ans:1. Electric flux is defined as the number of electric field lines passing through an area normal to them.
Alternate Answer
Surface integral of the electric field is defined as the electric flux through a closed surface
\varphi - \oint\overrightarrow{\text{E}}.\overline{\text{dS}}
\text{SI unit:}\frac{\text{N}.\text{m}^{2}}{\text{C}}\text{ or volt. metre }
2. Outward flux through the gaussian surface, is
2\text{EA} = \frac{\sigma{\text{A}}}{\varepsilon_{0}}
\therefore\text{E} = \sigma/2\varepsilon_{0}
Vectorically,
\overrightarrow{\text{E}} =\frac{\sigma}{2\varepsilon_{0}}\hat{\text{n}},
where \hat{\text{n}} is a unit vector normal to the plane, away from it. Hence, Electric field is independent of the distance from sheet.
3.
1. For positively charged sheet,
\rightarrow away from the sheet.
2. For negatively charged sheet,
\rightarrow towards the plane sheet.
Q587. Smallest charge that can exist in nature is the charge of an electron. During friction, it is only the transfer of electrons which makes the body charged. Hence, net 5 Marks
charge on any body is an integral multiple of charge of an electron [1.6 x 10-19C] i.e.
q = ± ne
Where n = 1, 2, 3, 4,....

Hence, no body can have a charge represented as 1.1 e, 2. 7e, \frac{3}{5}\text{e,} etc.
Recently, it has been discovered that elementary particles such as protons or neutrons are composed of more elemental units called quarks.
1. Which of the following properties is not satisfied by an electric charge?

https://bls.smartstudies.co.in/#/exam/pdf-preview/b5b87181-b79b-4b4a-8ebf-8f94fb605fc8/1 100/139
5/17/24, 8:14 PM Exam Automation
1. Total charge conservation.
2. Quantization of charge.
3. Two types of charge.
4. Circular line of force.
2. Which one of the following charges is possible?
1. 5.8 × 10-18C
2. 3.2 × 10-18C
3. 4.5 × 10-19C
4. 8.6 × 10-18C
3. If a charge on a body is 1nC, then how many electrons are present on the body?
1. 6.25 × 1027
2. 1.16 × 1019
3. 6.25 × 1028
4. 6.25 × 109
4. If a body gives out 109 electrons every second, how much time is required to get a total charge of 1C from it?
1. 190.19 years
2. 150.12 years
3. 198.19 years
4. 188.21 years
5. A polythene piece rubbed with wool is found to have a negative charge of 3.2 × 10-7 C. Calculate the number of electrons transferred.
1. 2 × 1012
2. 3 × 1012
3. 2 × 1014
4. 3 × 1014

Ans:1. (d) Circular line of force.


2. (b) 3.2 × 10-18C
Explanation:
From, \text{q}=\text{ne,}\text{ n}=\frac{\text{q}}{\text{e}}
=\frac{3.2\times10^{-18}}{1.6\times10^{-19}}=20
As n is an integer, hence this value of charge is possible.
3. (d) 6.25 × 109
Explanation:
Charge on the body is q = ne
\therefore No. of electrons present on the body is
\text{n}=\frac{\text{q}}{\text{e}}=\frac{1\times10^{-9}\text{C}}{1.6\times10^{-19}\text{C}}=6.25\times10^9
4. (c) 198.19 years
Explanation:
Here, n = 109 electrons per second Charge given per second,
q = ne = 109 × 1.6 × 10-19C
q = 1.6 × 10-10C
Total charge, Q = 1 C (given)
\therefore Time required =\frac{\text{Q}}{\text{q}}=\frac{1}{1.6\times10^{-10}}\text{s}=6.25\times10^9\text{s}
\therefore\frac{6.25\times10^9}{3600\times24\times365}\text{year}=198.19\text{year.}
5. (a) 2 × 1012
​Explanation:
As \text{q}=\text{ne,}\text{ n}=\frac{3.2\times10^{-7}}{1.6\times10^{-19}}
\Rightarrow\text{n}=2\times10^{12}\text{ electrons.}
Q588. Two identical pith balls are charged by rubbing against each other. They are suspended from a horizontal rod through two strings of length 20cm each, the 5 Marks
separation between the suspension points being 5cm. In equilibrium, the separation between the balls is 3cm. Find the mass of each ball and the tension in the
strings. The charge on each ball has a magnitude 2.0 × 10-8C.

Ans:

\text{q}=2.0\times10^{-8}\text{c},\ \text{n}=?,\ \sin\theta=\frac{1}{20}


Force between the charges
\text{F}=\frac{\text{Kq}_1\text{q}_2}{\text{r}^2}
=\frac{9\times10^9\times2\times10^{-8}\times2\times10^{-8}}{\big(3\times10^{-2}\big)^2}
=4\times10^{-3}\text{N}

https://bls.smartstudies.co.in/#/exam/pdf-preview/b5b87181-b79b-4b4a-8ebf-8f94fb605fc8/1 101/139
5/17/24, 8:14 PM Exam Automation

\text{mg}\sin\theta=\text{F}
\Rightarrow\text{m}=\frac{\text{F}}{\text{g}\sin\theta}
=\frac{4\times10^{-3}}{10\times\Big(\frac{1}{20}\Big)}
=8\times10^{-3}
=8\text{gm}
\cos\theta=\sqrt{1-\sin^2\theta}
=\sqrt{1-\frac{1}{400}}
=\sqrt{\frac{400-1}{400}}
=0.99=1
So, \text{T}=\text{mg}\cos\theta
\text{T}=8\times10^{-3}10\times0.99
=8\times10^{-2}\text{M}
Q589. State and Prove Gauss theorem in electrostatics. 5 Marks

Ans:Statement: The net-outward normal electric flux through any closed surface of any shape is equal to \frac{1}{\epsilon_0} times the total charge contained within
that surface, \frac{1}{\epsilon_0} i.e.,
\oint\text{S }\vec{\text{E}}.\vec{\text{ds}}=\frac{1}{\epsilon_0}\Sigma\text{q}
Where \oint\limits_{\text{S}} indicates the surface integral over the whole of the closed surface, \Sigma\text{q}
Is the algebraic sum of all the charges (i.e., net charge in coulombs) enclosed by surface S and remain unchanged with the size and shape of the surface.
Proof: Let a point charge +q be placed at centre O of a sphere S. Then S is a Gaussian surface. Electric field at any point on S is given by,
\text{E}=\frac{1}{4\pi\epsilon_0}.\frac{\text{q}}{\text{r}^2}
Image
The electric field and area element points radially outwards, so \theta=0^\circ,
Flux through area \vec{\text{dS}} is,
\text{d}\Phi=\vec{\text{E}}.\text{dS}=\text{E dS}\cos0^\circ=\text{E dS}
Total flux through surface S is,
\Phi=\oint\limits_{\text{S}}\text{d}\Phi=\oint\limits_{\text{S}}\text{E dS}=\text{E}\oint\limits_{\text{S}}\text{dS}
= E × Area of Sphere
=\frac{1}{4\pi\epsilon_0}.\frac{\text{q}}{\text{x}^2}.4\pi\text{r}^2
or, \Phi=\frac{\text{q}}{\epsilon_0} which proves Gauss's theorem.
Q590. 1. Define electric flux. Write its SI unit. 5 Marks
2. The electric field components due to a charge inside the cube of side 0.1 m are as shown:

\text{E}_{x} = \alpha\text{x},\text{ where }\alpha = 500\text{N/C-m}


\text{E}_{y} = 0,\text{E}_{z} =0.
Calculate (i) the flux through the cube and (ii) the charge inside the cube.

Ans:Definition: Number of Eletric field lines passing through a surface normally


\text{S.I.}\text{Unit of flux} = \text{Nm}^{2}\text{C}^{-1}

https://bls.smartstudies.co.in/#/exam/pdf-preview/b5b87181-b79b-4b4a-8ebf-8f94fb605fc8/1 102/139
5/17/24, 8:14 PM Exam Automation
\text{Net flux }\Phi = (\text{E}_{L} - \text{E}_{R})\text{A} = 500(-0.1 + 0.2).01
\text{Calculation and Result} = 0.5\text{Nm}^{2}\text{C}^{-1}
\text{Charge enclosed Q} = \in_{0}\Phi
\text{Calculation and Result } =4.42\times10^{-12}\text{C} = 8.85\times10^{-12}\times0.5\text{C}
Alternate Answer
0.5\in_{0}\text{Nm}^{2}\text{C}^{-1}.
Q591. A system has two charges qA = 2.5 × 10-7 C and qB = –2.5 × 10-7 C located at points A: (0, 0, –15 cm) and B: (0, 0, +15 cm), respectively. What are the total charge 5 Marks
and electric dipole moment of the system?

Ans:Both the charges can be located in a coordinate frame of reference as shown in the given figure.

At A, amount of charge, qA = 2.5 × 10-7 C


At B, amount of charge, qB = -2.5 × 10-7 C
Total charge of the system,
q = qA + q B
q = 2.5 × 10-7 C - 2.5 × 10-7 C
=0
Distance between two charges at points A and B,
d = 15 + 15 = 30 cm = 0.3 m
Electric dipole moment of the system is given by,
p = qA × d = q B × d
= 2.5 × 10-7 × 0.3
= 7.5 × 10-8 Cm along positive z-axis
Therefore, the electric dipole moment of the system is 7.5 10-8 Cm along positive z-axis.
Q592. Two particles A and B, having opposite charges 2.0 × 10-6C and 2.0 × 10-6C, are placed at a separation of 1.0cm. 5 Marks
1. Write down the electric dipole moment of this pair.
2. Calculate the electric field at a point on the axis of the dipole 1.0m awa.y from the centre.
3. Calculate the electric field at a point on the perpendicular bisector of the dipole and 1.0m away from the centre.

Ans:1. Dipolemoment =\text{q}\times\ell


(Where q = magnitude of charge \ell= Separation between the charges)

=2\times10^{-6}\times10^{-2}\text{cm}
=2\times10^{-8}\text{cm}
2. We know, Electric field at an axial point of the dipole

https://bls.smartstudies.co.in/#/exam/pdf-preview/b5b87181-b79b-4b4a-8ebf-8f94fb605fc8/1 103/139
5/17/24, 8:14 PM Exam Automation

=\frac{2\text{KP}}{\text{r}^3}
=\frac{2\times9\times10^9\times2\times10^{-8}}{\big(1\times10^{-2}\big)^3}
=36\times10^7\text{N/C}
3. We know, Electric field at a point on the perpendicular bisector about 1m away from centre of dipole.

=\frac{\text{KP}}{\text{r}^3}
=\frac{9\times10^9\times2\times10^{-8}}{1^3}
=180\text{N/C}
Q593. 1. Deduce the expression for the torque acting on a dipole of dipole moment \overrightarrow{\text{P}} in the presence of a uniform electric 5 Marks
field \overrightarrow{\text{E}}.
2. Consider two hollow concentric spheres, S1 and S2, enclosing charges 2Q and 4Q respectively as shown in the figure. (i) Find out the ratio of the electric flux
through them. (ii) How will the electric flux through the sphere S1 change if a medium of dielectric constant ‘\varepsilon_{r}’ is introduced in the space inside S1
in place of air? Deduce the necessary expression.

Ans:1. The forces, acting on the two charges of the dipole, are + \text{q}\overrightarrow{\text{E}} \text{ and } - \text{q}\overrightarrow{\text{E}}

The net force on the dipole is zero. The two forces are, however, equivalent to a torque having a magnitude.
\tau=\text{ (qE)AC}
= qE. 2a sin \theta
= pE sin \theta

https://bls.smartstudies.co.in/#/exam/pdf-preview/b5b87181-b79b-4b4a-8ebf-8f94fb605fc8/1 104/139
5/17/24, 8:14 PM Exam Automation
The direction of this torque is that of the cross product –(\overrightarrow{\text{p}}\times\overrightarrow{\text{E}}) . Hence, the torque acting on the dipole, is given
by
\overrightarrow{\tau} =\overrightarrow{\text{p}}\times\overrightarrow{\text{E}}).
2. As per Guass’s Theorem
Electric Flux = \oint_{s}\overrightarrow{\text{E}}.\overrightarrow{\text{dS}} = \frac{\text{q enclosed}}{\in_{o}}
\thereforeFor sphere S1, flux enclosed = \phi_{1} = \frac{2\text{Q}}{\in_{o}}
For sphere S2, flux enclosed = \phi_{2} = \frac{2\text{Q} + 4 \text{Q}}{\in_{o}} =\frac{6\text{Q}}{\in_{o}}
\therefore\frac{\phi_{1}}{\phi_{2}} = \frac{1}{3}
When a medium of dielectric consistent ∈r is introduced in sphere S1 the flux through \text{S}_{1}\text{would be } \phi'_{1} = \frac{2\text{Q}}{\in_{r}}.
Q594. One end of a 10cm long silk thread is fixed to a large vertical surface of a charged nonconducting plate and the other end is fastened to a small ball having a mass 5 Marks
of 10g and a charge of 4.0 × 10-5C. In equilibrium, the thread makes an angle of 60° with the vertical. Find the surface charge density on the plate.

Ans:

There are two forces acting on the ball. These are:


1. Weight of the ball, W = mg
2. Coulomb force acting on the charged ball due to the electric field of the plate, F = qE
Due to these forces, a tension develops in the thread.
Let the surface charge density on the plate be \sigma.
Electric field of a plate,
\text{E}=\frac{\sigma}{2\epsilon_0}
It is given that in equilibrium, the thread makes an angle of 60° with the vertical.
Resolving the tension in the string along horizontal and vertical directions, we get:
\text{T}\cos60^\circ=\text{mg}
\text{T}\sin60^\circ=\text{qE}
\Rightarrow\tan60^\circ=\frac{\text{qE}}{\text{mg}}
\Rightarrow\text{E}=\frac{\text{mg}\tan60^\circ}{\text{q}}
Also, electric field due to a plate,
\text{E}=\frac{\sigma}{2\epsilon_0}=\frac{\text{mg}\tan60^\circ}{\text{q}}
\sigma=\frac{2\epsilon_0\text{mg}\tan60^\circ}{\text{q}}
\sigma=\frac{2\times\big(8.85\times10^{-12}\big)\times\big(10\times10^{-3}\times9.8\big)\times1.732}{4.0\times10^{-6}}
\sigma=7.5\times10^{-7}\text{C/m}^2
Q595. Explain the principle of a device that can build up high voltages of the order of a few million volts. 5 Marks
Draw a schematic diagram and explain the working of this device.
Is there any restriction on the upper limit of the high voltages set up in this machine? Explain.

Ans:

Principle: The inner smaller sphere is at a higher potential than the larger outer sphere. Hence even a small positive charge, on the small sphere, flows immediately
to the larger sphere when both are connected bya conducting wire, even when the charge on the larger sphere is quite large.
Alternate Answer
1. Surface charge densityof sharp points of a conductor is extremely high and hence charge is continuously being sprayed out from sharp points.
2. Charge given to inner surface of a hollow conductor is transferred to its outer surface and is uniformly distributed over it.

https://bls.smartstudies.co.in/#/exam/pdf-preview/b5b87181-b79b-4b4a-8ebf-8f94fb605fc8/1 105/139
5/17/24, 8:14 PM Exam Automation

Working: The belt continuously carries positive charge, sprayed on to it by a brush at ground level, to the top. There it transfers its positive charge to another
conducting brush connected to the large shell. Thus positive charge is transferred to the shell, where it spreads out uniformly on the outer surface. In thisway
voltage difference is built up. Yes, high voltages can be built up only upto the breakdown field of the surrounding medium.
Q596. ln practice, we deal with charges much greater in magnitude than the charge on an electron, so we can ignore the quantum nature of charges and imagine that the 5 Marks
charge is spread in a region in a continuous manner. Such a charge distribution is known as continuous charge distribution. There are three types of continuous
charge distribution : (i) Line charge distribution (ii) Surface charge distribution (iii) Volume charge distribution as shown in figure.

1. Statement 1: Gauss's law can't be used to calculate an electric field near an electric dipole.
Statement 2: Electric dipole don't have symmetrical charge distribution.
1. Statement 1 and statement 2 are true.
2. Statement 1 is false but statement 2 is true.
3. Statement 1 is true but statement 2 is false.
4. Both statements are false.
2. An electric charge of 8.85 × 10-13C is placed at the centre of a sphere of radius 1m. The electric flux through the sphere is:
1. 0.2NC-1 m2
2. 0.1NC-1 m2
3. 0.3NC-1 m2
4. 0.01NC-1 m2
3. The electric field within the nucleus is generally observed to be linearly dependent on r. So,

1. a = 0
2. \text{a}=\frac{\text{R}}{2}
3. a = R
4. \text{a}=\frac{\text{2R}}{3}
4. What charge would be required to electrify a sphere of radius 25cm so as to get a surface charge density of \frac{3}{\pi}\text{Cm}^{-2}?
1. 0.75C
2. 7.5C
3. 75C
4. Zero
5. The SI unit of linear charge density is:
1. Cm
2. Cm-1
3. Cm-2
4. Cm-3

Ans:1. (a) Statement 1 and statement 2 are true.


Explanation:
Gauss's law is applicable for any closed surface. Gauss's law is most useful in situation where the charge distribution has spherical or cylindrical symmetry or is
distributed uniformly over the plane. Whereas electric dipole is a system of two equal and opposite point charges separated by a very small and finite distance.

https://bls.smartstudies.co.in/#/exam/pdf-preview/b5b87181-b79b-4b4a-8ebf-8f94fb605fc8/1 106/139
5/17/24, 8:14 PM Exam Automation
-1 2
2. (b) 0.1NC m
Explanation:
According to Gauss's law, the electric flux through the sphere is
\phi=\frac{\text{q}_{in}}{\epsilon_0}=\frac{8.85\times10^{-13}\text{C}}{8.85\times10^{-12}\text{C}^2\text{N}^{-1}\text{m}^{-2}}
= 0.1NC-1 m2
3. (c) a = R
Explanation:
For uniformly volume charge density,
\text{E}=\frac{\text{Pr}}{3\epsilon_0}
\text{E}\propto\text{r}
4. (a) 0.75C
Explanation:
r = 25cm = 0.25m,
\sigma=\frac{3}{\pi}\text{C/m}^2
As, \sigma=\frac{\text{q}}{4\pi\text{r}^2}
\Rightarrow\text{q}=4\pi\times(0.25)^2\times\frac{3}{\pi}=0.75\text{C.}
5. (b) Cm-1
Explanation:​​
The line charge density at a point on a tine is the charge per unit length of the line at that point
\lambda=\frac{\text{bq}}{\text{bL}}
Thus, the SI unit for \lambda is Cm-1​.
Q597. 1. Define electric flux. Is it a scalar or a vector quantity? 5 Marks
A point charge q is at a distance of \frac{\text{d}}{2} directly above the centre of a square of side d, as shown in the figure. Use Gauss’ law to obtain the
expression for the electric flux through the square.

2. If the point charge is now moved to a distance ‘d’ from the centre of the square and the side of the square is doubled, explain how the electric flux will be
affected.

Ans:1. Electric Flux: It is the total number of electric field lines passing normally through a surface held in electric field.
Alternate Answer
The electric flux linked with a surface is the surface integral of the electric field over the surface. It is a scalar quantity.

We will draw a closed gaussian surface in the form of cube of side 'd'.
By Gauss’s Law,
\phi=\frac{\text{q}}{\epsilon_0}
For any surface, flux will be
\phi=\frac{\text{q}}{6\epsilon_0}
2. Electric flux will not be affected because electric flux does not depend on dimensions or area of surface.

https://bls.smartstudies.co.in/#/exam/pdf-preview/b5b87181-b79b-4b4a-8ebf-8f94fb605fc8/1 107/139
5/17/24, 8:14 PM Exam Automation

\phi=\frac{\text{q}}{6\epsilon_0}
Q598. The radius of a gold nucleus (Z = 79) is about 7.0 × 10-15m. Assume that the positive charge is distributed uniformly throughout the nuclear volume. Find the 5 Marks
strength of the electric field at:
1. The surface of the nucleus.
2. At the middle point of a radius. Remembering that gold is a conductor, is it justified to assume that the positive charge is uniformly distributed over the entire
volume of the nucleus and does not come to the outer surface?

Ans:Given:
Atomic number of gold = 79
Charge on the gold nucleus, \text{Q}=79\times\big(1.6\times10^{-19}\big)\text{C}
The charge is distributed across the entire volume. So, using Gauss's Law, we get:
1. \phi=\oint\overrightarrow{\text{E}}.\text{d}\overrightarrow{\text{s}}=\frac{\text{Q}}{\epsilon_0}
\Rightarrow\oint\text{Eds}=\frac{\text{Q}}{\epsilon_0}
The value of E is fixed for a particular radius.
\Rightarrow\text{E}\oint\text{ds}=\frac{\text{Q}}{\epsilon_0}
\Rightarrow\text{E}\times4\pi\text{r}^2=\frac{\text{Q}}{\epsilon_0}
\Rightarrow\text{E}=\frac{\text{Q}}{\epsilon_0\times4\pi\text{r}^2}
\Rightarrow\text{E}=\frac{79\times(1.6\times10^{-10})}{(8.85\times10^{-13})\times4\times3.14\times(7\times10^{-10})^2}
\Rightarrow\text{E}=2.315131\times10^{21}\text{N/C}

2. To find the electric field at the middle point of the radius:


Radius, \text{r}=\frac{7}{2}\times10^{-10}\text{m}
Volume,
\text{V}=\frac{4}{3}\pi\text{r}^3
=\frac{4}3{}\times\frac{22}{7}\times\frac{343}{8}\times10^{-30}
Net charge = 79 × 1.6 × 10-19C
Volume charge density
=\frac{79\times1.6\times10^{-19}}{\frac{4}{3}\pi\times343\times10^{-30}}

So, the charge enclosed by this imaginary sphere of radius r = 3.5 × 10-10
=\frac{79\times1.6\times10^{-19}}{\frac{4}{3}\pi\times343\times10^{-30}}\times\frac{4}3{}\pi\times\frac{343}{8}\times10^{-30}
=\frac{79\times1.6\times10^{-19}}{8}
\Rightarrow\text{E}=\frac{79\times1.6\times10^{-19}}{8\times4\pi\epsilon_0\text{x}^2} at \text{r}=3.5\times10^{-10}
=1.16\times10^{21}\text{N/C}
As electric charge is given to a conductor, it gets distributed on its surface. But nucleons are bound by the strong force inside the nucleus. Thus, the nuclear
charge does not come out and reside on the surface of the conductor. Thus, the charge can be assumed to be uniformly distributed in the entire volume of the
nucleus.
Q599. 1. Derive an expression for the electric field E due to a dipole of length ‘2a’ at a point distant r from the centre of the dipole on the axial line. 5 Marks
2. Draw a graph of E versus r for r >> a.
3. If this dipole were kept in a uniform external electric field E0, diagrammatically represent the position of the dipole in stable and unstable equilibrium and write
the expressions for the torque acting on the dipole in both the cases.

Ans:1.

Electric field at P due to charge (+q)=E_1=\frac{1}{4\pi\varepsilon_0}\frac{q}{(r-a)^2}


Electric field at P due to charge (-q)=E_2=\frac{1}{4\pi\varepsilon_0}\frac{q}{(r+a)^2}
Net electric Field at \text{P}=E_1-E_2=\frac{1}{4\pi\varepsilon_0}\frac{q}{(r-a)^2}-\frac{1}{4\pi\varepsilon_0}\frac{q}{(r+a)^2}
=\frac{1}{4\pi\varepsilon_0}\frac{2pr}{(r^2-a^2)^2}\text{ }\text{ }\text{ }\text{ }(p=q.2a)
Its direction is parallel to \overrightarrow{p}
2.

https://bls.smartstudies.co.in/#/exam/pdf-preview/b5b87181-b79b-4b4a-8ebf-8f94fb605fc8/1 108/139
5/17/24, 8:14 PM Exam Automation

3.

Torque = 0 for (i) as well as case (ii).


Q600. A point charge causes an electric flux of –1.0 × 103 Nm2 /C to pass through a spherical Gaussian surface of 10.0 cm radius centred on the charge. 5 Marks
1. If the radius of the Gaussian surface were doubled, how much flux would pass through the surface?
2. What is the value of the point charge?

Ans:1. Electric flux, \phi = -1.0 x 103 Nm2/C


Radius of the Gaussian surface,
r = 10.0 cm
Electric flux piercing out through a surface depends on the net charge enclosed inside a body. It does not depend on the size of the body. If the radius of the
Gaussian surface is doubled, then the flux passing through the surface remains the same i.e., -103 N m2/C.
2. Electric flux is given by the relation,
\phi=\frac{\text{q}}{\in_0}
Where,
q = Net charge enclosed by the spherical surface
\in_0 = Permittivity of free space = 8.854 x 10-12 N-1C2m-2
\therefore\text{q}=\phi\in_0
= -1.0 × 103 × 8.854 × 10-12
= -8.854 × 10-9 C
= -8.854 nC
Therefore, the value of the point charge is -8.854 nC.
Q601. Using Gauss’ law deduce the expression for the electric field due to a uniformly charged spherical conducting shell of radius R at a point (i) outside and (ii) inside 5 Marks
the shell.
Plot a graph showing variation of electric field as a function of r > R and r < R. (r being the distance from the centre of the shell)

Ans:1. Electric field intensity at a point outside a uniformly charged thin spherical shell: Consider a uniformly charged thin spherical shell of radius R carrying charge Q.
To find the electric field outside the shell, we consider a spherical Gaussian surface of radius r (>R), concentric with given shell. If \overrightarrow{\text{E}} is
electric field outside the shell, then by symmetry electric field strength has same magnitude E0 on the Gaussian surface and is directed radially outward. Also the
directions of normal at each point is radially outward, so angle between \overrightarrow{\text{E}}_{i} and d \overrightarrow{\text{S}} is zero at each point. Hence,
electric flux through Gaussian

surface \oint_{s} = \overrightarrow{\text{E}}_{0}.\text{d}\overrightarrow{\text{S}}.


\oint =\text{E}_{0}\text{ds}\cos 0 =\text{E}_{0}.4\pi\text{r}^{2}
Now, Gaussian surface is outside the given charged shell, so charge enclosed by Gaussian surface is Q.
Hence, by Gauss's theorem
\oint_{s} =\overrightarrow{\text{E}}_{0}.\text{d}\overrightarrow{\text{S}} = \frac{1}{\varepsilon_{0}}\times\text{ charged enclosed}
\Rightarrow\text{E}_{0}4\pi\text{r}^{2} =\frac{1}{\varepsilon_{0}}\times\text{Q}\Rightarrow\text{E}_{0} =\frac{1}{4\pi\varepsilon_{0}}\frac{\text{Q}}{\text{r}^{2}}
Thus, electric field outside a charged thin spherical shell is the same as if the whole charge Q is concentrated at the centre.
If \sigma is the surface charge density of the spherical shell, then
\text{Q} = 4\pi\text{R}^{2}\sigma\text{C}
\therefore\text{E}_{0} =\frac{1}{4\pi\varepsilon_{0}}\frac{4\pi\text{R}^{2}\sigma}{\text{r}^{2}} = \frac{\text{R}^{2}\sigma}{\varepsilon_{0}\text{r}^{2}}
2. Electric field inside the shell (hollow charged conducting sphere): The charge resides on the surface of a conductor. Thus a hollow charged conductor is
equivalent to a charged spherical shell. To find the electric field inside the shell, we consider a spherical Gaussian surface of radius r (<R), concentric with
the given shell. If \overrightarrow{\text{E}} is the electric field inside the shell, then by symmetry electric field strength has the same magnitude Ei on the
Gaussian surface and is directed radially outward.
Also the directions of normal at each point is radially outward, so angle between \overrightarrow{\text{E}}_{i} and d\overrightarrow{\text{S}}is zero at each point.
Hence, electric flux through Gaussian surface
= \int_{s}\overrightarrow{\text{E}}_{i}.\text{d}\overrightarrow{\text{S}}
= \int\text{E}_{i}\text{ds}\cos 0 =\text{E}_{i}.4\pi\text{r}^{2}
Now, Gaussian surface is inside the given charged shell, so charge enclosed by Gaussian surface is zero.

https://bls.smartstudies.co.in/#/exam/pdf-preview/b5b87181-b79b-4b4a-8ebf-8f94fb605fc8/1 109/139
5/17/24, 8:14 PM Exam Automation
Hence, by Gauss’s theorem
\int_{s}\overrightarrow{\text{E}}_{i}.\overrightarrow{\text{dS}} =\frac{1}{\varepsilon_{0}}\times\text{ chargeenclosed}
\Rightarrow\text{E}_{i}.4\pi\text{r}^{2} =\frac{1}{\varepsilon_{0}}\times0\Rightarrow\text{E}_{i} = 0

Thus, electric field at each point inside a charged thin spherical shell is zero. The graph is shown in fig.
Q602. Derive an expression for the electric field at a point on the equatorial plane of an electric dipole consisting of charges q and - q separated by a distance 2a. The 5 Marks
distance of a far off point on the equatorial plane of an electric dipole is halved. How will the electric field be affected for the dipole? Two identical electric dipoles
are placed along the diagonals of a square ABCD of side \sqrt{2} m as shown in the figure. Obtain the magnitude and direction of the net electric field at the centre
(O) of the square.

Ans:1. Derivation of electric field


2. Effect on electric field
3. Finding magnitude and direction of electric field

\text{E}_\text{+q}=\frac{\text{q}}{4\pi\in_\text{o}}\times\frac{1}{\text{r}^2+\text{a}^2}
\text{E}_\text{-q}=\frac{\text{q}}{4\pi\in_\text{o}}\times\frac{1}{\text{r}^2+\text{a}^2}
The components normal to dipole axis cancel away. The components along the dipole axis add up. Total electric field is opposite to dipole moment.
\overrightarrow{\text{E}}=-(\text{E}_\text{+q}+\text{E}_\text{-q})\cos\theta\hat{\text{p}}
=\frac{-2\text{qa}}{4\pi\in_\text{o}(\text{r}^2+\text{a}^2)^\frac{3}{2}}\hat{\text{p}}
=\frac{-\overrightarrow{\text{p}}}{4\pi\in_\text{0}(\text{r}^2+\text{a}^2)^\frac{3}{2}}
Deduct ½ mark if the expression of electric field is not in vector form.
At far off point r >> a
\overrightarrow{\text{E}}=\frac{-\overrightarrow{\text{p}}}{4\pi\in_\text{0}\text{ r}^3}
When distance is halved.
\overrightarrow{\text{E}}=\frac{-\overrightarrow{\text{p}}}{4\pi\in_\text{0}(\frac{\text{r}}{2})^3}
=\frac{-8\overrightarrow{\text{p}}}{4\pi\in_\text{0}\text{ r}^3}
\overrightarrow{\text{E}} becomes 8 times

p1 q × 2Cm (along OA)

https://bls.smartstudies.co.in/#/exam/pdf-preview/b5b87181-b79b-4b4a-8ebf-8f94fb605fc8/1 110/139
5/17/24, 8:14 PM Exam Automation
p2 q × 2Cm (along OD)
\text{p}_\text{net}=\sqrt{\text{p}_1^2+\text{p}_2^2}
=2\sqrt{2}\text{q cm}
Electric field at centre O
\text{E}=\frac{\text{kp}_\text{net}}{(\text{r}^2+\text{a}^2)^\frac{3}{2}}
at point O, r = 0, a = 1m
\text{E}=\frac{\text{k}\times2\sqrt{2}\text{q}}{1^3}=2\sqrt{2}\text{kq}=\frac{2\sqrt{2}\text{q}}{4\pi\in_\text{o}}
Along DC
Alternatively

\text{E}=\frac{\text{kq}}{\text{r}^2}
AC = BD = 2m
r = OA = OB = OC = OD = 1m
Electric field at O due to charges at B and D
E 1 = EB + ED
\text{E}_1=\frac{\text{kq}}{\text{1}^2}+\frac{\text{kq}}{\text{1}^2} along OB
= 2kq
Electric field at O due to charges at A and C
E 2 = EA + EC
\text{E}_2=\frac{\text{kq}}{\text{1}^2}+\frac{\text{kq}}{\text{1}^2} along OB
= 2kq
\text{E}_\text{net}=\sqrt{\text{E}_1^2+\text{E}_2^2}
=2\sqrt{2}\text{kq}=\frac{2\sqrt{2}\text{q}}{4\pi\in_\text{o}}
Along DC
Alternatively

Considering AB as dipole, electric field at O


\text{E}_1=\frac{2\text{kq}\times\text{a}}{((\frac{1}{\sqrt{2}})^2+(\frac{1}{\sqrt{2}})^2)^\frac{3}{2}}=\frac{2\text{kqa}}{(\frac{1}{2}+\frac{1}{2})^\frac{3}
{2}}=2\text{kqa}
Similarly considering DC as another dipole, electric field at O
\text{E}_2=\frac{2\text{kq}\times\text{a}}{((\frac{1}{\sqrt{2}})^2+(\frac{1}{\sqrt{2}})^2)^\frac{3}{2}}=\frac{2\text{kqa}}{(\frac{1}{2}+\frac{1}{2})^\frac{3}
{2}}=2\text{kqa}
Enet = E1 + E2 = 4kqa = \frac{1}{4\pi\in_\text{o}}\times4\times\frac{1}{\sqrt{2}}\times\text{q}
=2\sqrt{2}\text{kq}=\frac{2\sqrt{2}\text{q}}{4\pi\in_\text{o}}
Along DC
Q603. 1. Use Gauss’ law to derive the expression for the electric field (\vec{\text{E}}) due to a straight uniformly charged infinite line of charge 5 Marks
density \lambda\frac{\text{C}}{\text{m}}.
2. Draw a graph to show the variation of E with perpendicular distance r from the line of charge.
3. Find the work done in bringing a charge q from perpendicular distance r1 to r2(r2 > r1).

Ans:1.

https://bls.smartstudies.co.in/#/exam/pdf-preview/b5b87181-b79b-4b4a-8ebf-8f94fb605fc8/1 111/139
5/17/24, 8:14 PM Exam Automation
Consider a line charge of density \lambda coulomb per metre. Choose a Gaussian as a right circular cylinder of unit length and of radius r with its length as axis. On
the top and bottom, \vec{\text{E}} and \Delta\vec{\text{s}} are perpendicular to each other, so there is no electric flux through the faces. On the sides they are
parallel if \lambda is positive, anti parallel if \lambda is negative. Therefore, the total outward flux \phi through S is,
\phi=\oint\vec{\text{E}}.\Delta\vec{\text{S}}\\ \ \ \ \text{curved}\\ \ \ \ \text{surface}
=\text{E}\oint\Delta\text{s}\cos0^\circ=\text{E}(2\pi\text{rl})
Also, By Gauss’s Law \phi=\frac{\text{q}}{\epsilon_0}=\frac{\lambda\text{l}}{\epsilon_0}\ (\because\ \text{q}=\lambda\text{l})
From (i) and (ii)
\text{E}(2\pi\text{rl})=\frac{\lambda\text{l}}{\epsilon_0}
\text{E}=\frac{\lambda}{2\pi\text{r}\epsilon_0}=\frac{2\text{k}\lambda}{\text{r}}
2.

3.

Work done to move charge q by small distance dx.dw = F.dx.cos180° = –qE.dx


=\frac{-\text{q}\lambda}{2\pi\epsilon_0\text{x}}\text{dx}\ \Big(\because\ \text{E}=\frac{\lambda}{2\pi\epsilon_0\text{x}}\Big)
So, work done to move from r1 to r2
\text{w}=\int_{\text{r}_1}^{\text{r}_2} \text{dw}
=\int_{\text{r}_1}^{\text{r}_2}\frac{-\text{q}\lambda}{2\pi\epsilon_0\text{x}}\text{dx}
\text{w}=\frac{-\text{q}\lambda}{2\pi\epsilon_0}\int_{\text{r}_1}^{\text{r}_2}\frac{\text{dx}}{\text{x}}
\text{w}=\frac{-\text{q}\lambda}{2\pi\epsilon_0}(\log_\text{e}\text{r}_2-\log_\text{e}\text{r}_1)
\text{w}=\frac{\text{q}\lambda}{2\pi\epsilon_0}\Big[\log_\text{e}\Big(\frac{\text{r}_1}{\text{r}_2}\Big)\Big]
Q604. A long cylindrical wire carries a positive charge of linear density 2.0 × 10-8Cm-1. An electron revolves around it in a circular path under the influence of the 5 Marks
attractive electrostatic force. Find the kinetic energy of the electron. Note that it is independent of the radius.

Ans:Let the linear charge density of the wire be \lambda.


The electric field due to a charge distributed on a wire at a perpendicular distance r from the wire,
\text{E}=\frac{\lambda}{2\pi\epsilon_0\text{r}}
The electrostatic force on the electron will provide the electron the necessary centripetal force required by it to move in a circular orbit. Thus,
\text{qE}=\frac{\text{mv}^2}{\text{r}}
\Rightarrow\text{mv}^2=\text{qEr}\ \dots(1)
Kinetic energy of the electron, \text{K}=\frac{1}{2}\text{mv}^2
From (1),
\text{K}=\frac{\text{qEr}}{2}
\text{K}=\frac{\text{qr}}{2}\frac{\lambda}{2\pi\epsilon_0\text{r}} \Big[\because\text{E}=\frac{\lambda}{2\pi\epsilon_0\text{r}}\Big]
\text{K}=\big(1.6\times10^{-19}\big)\times\big(2\times10^{-8}\big)\times\big(9\times10^{9}\big)\text{J}
\text{K}=2.88\times10^{-17}\text{J}
Q605. A charge Q located at a point \vec{\text{r}} is in equilibrium under the combined electric field of three charges q1, q2, q3. If the charges q1, q2 are located at 5 Marks
points \vec{\text{r}}_1 and \vec{\text{r}}_2 respectively, find the direction of the force on Q, due to q3 in terms of q1, q2, \vec{\text{r}}_1,\
\vec{\text{r}}_2 and \vec{\text{r}}.

Ans:\vec{\text{F}}_1+\vec{\text{F}}_2+\vec{\text{F}}_3=0
\Rightarrow\ \frac{1}{4\pi\epsilon_0}\frac{\text{Qq}_1}{|\vec{\text{r}}-\vec{\text{r}}_1|^3}(\vec{\text{r}}-\vec{\text{r}}_1)+\frac{1}{4\pi\epsilon_0}\frac{\text{Qq}_2}
{|\vec{\text{r}}-\vec{\text{r}}_2|^3}(\vec{\text{r}}-\vec{\text{r}}_2)
+\frac{1}{4\pi\epsilon_0}\frac{\text{Qq}_3}{|\vec{\text{r}}-\vec{\text{r}}_3|^3}(\vec{\text{r}}-\vec{\text{r}}_3)=0
\Rightarrow\ \frac{\text{q}_1}{|\vec{\text{r}}-\vec{\text{r}}_1|^3}(\vec{\text{r}}-\vec{\text{r}}_1)+\frac{\text{q}_2}{|\vec{\text{r}}-\vec{\text{r}}_2|^3}(\vec{\text{r}}-
\vec{\text{r}}_2)=-\frac{\text{q}_3}{|\vec{\text{r}}-\vec{\text{r}}_3|^3}(\vec{\text{r}}-\vec{\text{r}}_3)
\Rightarrow\ \vec{\text{r}}- \vec{\text{r}}_3=-\frac{| \vec{\text{r}}- \vec{\text{r}}_3|^3}{\text{q}_3}\Big[\frac{\text{q}_1( \vec{\text{r}}- \vec{\text{r}}_1)}{|
\vec{\text{r}}- \vec{\text{r}}_1|^3}+\frac{\text{q}_2( \vec{\text{r}}- \vec{\text{r}}_2)}{| \vec{\text{r}}- \vec{\text{r}}_2|^3}\Big]
Direction of force on Q due to q3 along ( \vec{\text{r}}- \vec{\text{r}}_3) is given by,

https://bls.smartstudies.co.in/#/exam/pdf-preview/b5b87181-b79b-4b4a-8ebf-8f94fb605fc8/1 112/139
5/17/24, 8:14 PM Exam Automation
\frac{\text{q}_1}{| \vec{\text{r}}- \vec{\text{r}}_1|^3}( \vec{\text{r}}_1- \vec{\text{r}})+\frac{\text{q}_1}{| \vec{\text{r}}- \vec{\text{r}}_2|^3}( \vec{\text{r}}_2-
\vec{\text{r}})
Q606. Two charges q and -3q are placed fixed on x-axis separated by distance 'd'. Where should a third charge 2q be placed such that it will not experience any force? 5 Marks

Ans:The force on any charge will be zero only if net electric field at the position of charge is zero. Let electric field is zero at a distance x from charge q.

At point \text{P},\vec{\text{E}_\text{A}}+\vec{\text{E}_\text{B}}=0\Rightarrow|\vec{\text{E}_\text{A}}|=|\vec{\text{E}_\text{B}}|
\Rightarrow\ \frac{\text{q}}{4\pi\epsilon_0\text{x}^2}=\frac{3\text{q}}{4\pi\epsilon_0(\text{x}+\text{d})^2}
\Rightarrow\ (\text{x}+\text{d})^2=3\text{x}^2
\Rightarrow\ \text{x}^2+\text{d}^2=2\text{dx}=3\text{x}^2
\therefore\ 2\text{x}^2-2\text{dx}-\text{d}^2=0
or \text{x}=\frac{\text{d}}{2}\pm\frac{\sqrt{3\text{d}}}{2}
(Negative sign lies between q and -3q and hence is unaceptable.)
Hence \text{x}=-\frac{\text{d}}{2}+\frac{\sqrt{3\text{d}}}{2}=\frac{\text{d}}{2}(1+\sqrt{3})
Hence if charge 2q is placed at a distance \frac{\text{d}}{2}(1+\sqrt{3}) to the left of q.
Q607. 1. State Gauss’s law in electrostatics. Show, with the help of a suitable example along with the figure, that the outward flux due to a point charge ‘q’, 5 Marks
in vacuum within a closed surface, is independent of its size or shape and is given by \text{q}/ \varepsilon_{\circ}.
2. Two parallel uniformly charged infinite plane sheets, ‘1’ and ‘2’, have charge +\sigma densities -2 \sigma and respectively. Give the magnitude and direction of
the net electric field at a point.
1. In between the two sheets and.
2. outside near the sheet ‘1’.

Ans:1. Gauss Law: Electric flux through a closed surface is \frac{1}{\in}_{\circ} times the total charge enclosed by the surface.
Alternate Answer
\phi = \frac{1}{\in_{0}}.\text{q}.
The term q equals the sum of all charges enclosed by the surface and remain unchanged with the size and shape of the surface.
Alternate Answer
The total number of electric field lines emanating from the enclosed charge "q‟ are same for all surfaces 1, 2 & 3.

2. We have \text{|E}_{1}| = \frac{\sigma}{\in_{o}} ; \text{|E}_{2}| \frac{2\sigma}{\in_{o}}


1. Between the plates.
= \frac{\sigma}{2\in_{o}} + \frac{2\sigma}{2\in_{o}} =\frac{\sigma+2\sigma}{2\in_0}= \frac{3\sigma}{2\in_{o}}
\text{E}_\text{in} = \text{E}_1 + \text{E}_2
2. Outside near the sheet "1‟
\text{E}_\text{out} = \text{E}_2 − \text{E}_1
\frac{2\sigma}{2\in}_{\circ} - \frac{\sigma}{2\in}_{\circ} = \frac{\sigma}{2\in}_{\circ}
Q608. 1. Two insulated charged copper spheres A and B have their centres separated by a distance of 50 cm. What is the mutual force of electrostatic repulsion if the 5 Marks
charge on each is 6.5 × 10-7 C? The radii of A and B are negligible compared to the distance of separation.
2. What is the force of repulsion if each sphere is charged double the above amount, and the distance between them is halved?

Ans:1. Charge on sphere A, qA = Charge on sphere B, qB = 6.5 x 10-7 C


Distance between the spheres, r = 50 cm = 0.5 m
Force of repulsion between the two spheres,
\text{F}=\frac{\text{q}_\text{A}\text{q}_\text{B}}{4\pi\in_0\text{r}^2}
Where,
\in_0 = Free space perrnittivity
\frac{1}{4\pi\in_0}=9\times10^9\text{Nm}^2\text{C}^{-2}
\therefore\text{F}=\frac{9\times10^9\times(6.5\times10^{-7})^2}{(0.5)^2}
= 1.52 × 10-2 N
Therefore, the force between the two spheres is 1.52 x 10-2 N.
2. After doubling the charge, charge on sphere A, qA = Charge on sphere B, qB = 2 × 6.5 × 10-7 C = 1.3 × 10-6 C
The distance between the spheres is halved.
\therefore\text{r}=\frac{0.5}{2}=0.25 \text{m}
Force of repulsion between the two spheres,
\text{F}=\frac{\text{q}_\text{A}\text{q}_\text{B}}{4\pi\in_0\text{r}^2}
=\frac{9\times10^9\times1.3\times10^{-6}\times1.3\times10^{-6}}{(0.25)^2}
= 16 × 1.52 × 10-2
= 0.243 N
Therefore, the force between the two spheres is 0.243 N.
Q609. Repeat the previous problem if the particle C is displaced through a distance x along the line AB. 5 Marks

Ans:

https://bls.smartstudies.co.in/#/exam/pdf-preview/b5b87181-b79b-4b4a-8ebf-8f94fb605fc8/1 113/139
5/17/24, 8:14 PM Exam Automation

\text{F}_\text{AC}=\frac{\text{KQq}}{(\ell+\text{x})^2}
\text{F}_\text{CA}=\frac{\text{KQq}}{(\ell-\text{x})^2}
Net force =\text{KQq}\Big[\frac{1}{(\ell-\text{x})^2}-\frac{1}{(\ell-\text{x})^2}\Big]
=\text{KQq}\bigg[\frac{(\ell+\text{x})^2-(\ell-\text{x})^2}{(\ell+\text{x})^2(\ell-\text{x})^2}\bigg]
=\text{KQq}\bigg[\frac{4\ell\text{x}}{(\ell^2-\text{x}^2)^2}\bigg]
\text{x}<<<\text{l}=\frac{\text{d}}{2} neglecting x w.r.t. \ell We get
net \text{F}=\frac{\text{KQq}4\ell\text{x}}{\ell^4}=\frac{\text{KQq}4\text{x}}{\ell^3} acceleration =\frac{4\text{KQqx}}{\text{m}\ell^3}
Time period =2\pi\sqrt{\frac{\text{displacement}}{\text{acceleration}}}
=2\pi\sqrt{\frac{\text{xm}\ell^3}{4\text{KQqx}}}
=2\pi\sqrt{\frac{\text{m}\ell^3}{4\text{KQq}}}
=\sqrt{\frac{4\pi^2\text{m}\ell^34\pi\in_0}{4\text{Qq}}}
=\sqrt{\frac{4\pi^3\text{m}\ell^3\in_0}{\text{Qq}}}
=\sqrt{4\pi^3\text{md}^3\in_08\text{Qq}}
=\bigg[\frac{\pi^3\text{md}^3\in_0}{2\text{Qq}}\bigg]^{\frac{1}{2}}
Q610. Draw a labelled diagram of Van de Graaff generator. State its working principle to show how by introducing a small charged sphere into a larger sphere, a large 5 Marks
amount of charge can be transferred to the outer sphere. State the use of this machine and also point out its limitations.

Ans:

1. Potential inside and on the surface, of the conducting sphere pf radius ‘R’:
\text{V}'_{R} = \frac{1}{4\pi\in_{o}}.\frac{\text{Q}}{\text{R}}
2. Potential due to small sphere of radius ‘r’ carrying a charge ‘q’:
At the surface of the smaller sphere: \text{V}'_{r} = \frac{1}{4\pi\in_{o}}.\frac{\text{q}}{\text{r}}
At the surface of the larger sphere: \text{V}''_{R} = \frac{1}{4\pi\in_{o}}.\frac{\text{Q}}{\text{R}}
\therefore The difference of potential between the smaller and the larger sphere:
= Δ\text{V} =\frac{1}{4\pi\in_{o}}.\bigg[\bigg(\frac{\text{Q}}{\text{R}} + \frac{\text{q}}{\text{r}}\bigg) - \bigg(\frac{\text{Q}}{\text{R}} + \frac{\text{q}}
{\text{r}}\bigg)\bigg]
=\frac{\text{q}}{4\pi\in_{o}}\bigg(\frac{1}{\text{r}} - \frac{1}{\text{R}}\bigg)
When ‘q’ is positive, the inner sphere would always be at a higer potential with respect to outer sphere, irrespective of the amount of charges on the two.
\thereforeWhen both the spheres are connected, charge will flow from the smaller sphere to the larger sphere. Thus for a set up of the type shown, charge would
keep on pilling up on the larger sphere.
Use: This machine is used to accelerate charged particles (electron, protons, ions) to high energies.
Limitation: It can build up potentials upto a few million volts only.
Q611. Four equal charges 2.0 × 10-6C each are fixed at the four corners of a square of side 5cm. Find the Coulomb force experienced by one of the charges due to the 5 Marks
rest three.

Ans:

https://bls.smartstudies.co.in/#/exam/pdf-preview/b5b87181-b79b-4b4a-8ebf-8f94fb605fc8/1 114/139
5/17/24, 8:14 PM Exam Automation

q1 = q2 = q3 = q4 = 2 × 10-6C
\text{v}=5\text{cm}=5\times10^{-2}\text{m}
So force on \bar{\text{C}}=\bar{\text{F}}_\text{CA}+\bar{\text{F}}_\text{CB}+\bar{\text{F}}_\text{CD}
So Force along × Component =\bar{\text{F}}_\text{CD}+\bar{\text{F}}_\text{CA}\cos45^\circ+0
=\frac{\text{k}\big(2\times10^{-6}\big)^2}{\big(5\times10^{-2}\big)^2}+\frac{\text{k}\big(2\times10^{-6}\big)^2}{\big(5\times10^{-2}\big)^2}\frac{1}{2\sqrt{2}}
=\text{kq}^2\Big(\frac{1}{25\times10^{-4}}+\frac{1}{50\sqrt{2}\times10^{-4}}\Big)
\frac{9\times10^9\times4\times10^{12}}{24\times10^{-4}}\Big(1+\frac{1}{2\sqrt{2}}\Big)
=1.44(1.35)=19.49 Force along % component = 19.49
So, Resultant \text{R}=\sqrt{\text{Fx}^2+\text{Fy}^2}
=19.49\sqrt{2}
=27.56
Q612. Three point charges of +2\mu\text{C}, -3\mu\text{C} and -3\mu\text{C} are kept at the vertices A, B and C respectively of an equilateral triangle of side 20cm as 5 Marks
shown in figure. What should be the sign and magnitude of charge to be placed at the midpoint M of side BC so that charge at A remains in equilibrium?

Ans:

https://bls.smartstudies.co.in/#/exam/pdf-preview/b5b87181-b79b-4b4a-8ebf-8f94fb605fc8/1 115/139
5/17/24, 8:14 PM Exam Automation

Let charge placed at M be qM. The forces acting on charge (\text{qA}=+2\mu\text{C}) are FAB, FAC and FAM as shown in figure.
\vec{\text{F}}_\text{AB}=\frac{1}{4\pi\epsilon_0}\frac{\text{q}_\text{A}\text{q}_\text{B}}{\text{r}^2_\text{AB}}\text{along }\vec{\text{AB}}
=9\times10^9\times\frac{(2\times10^{-6})(3\times10^{-6})}{(0.20)^2}\ \text{along}\ \vec{\text{AB}}=1.35\text{ N}\text{ along }\vec{\text{AB}}
\vec{\text{F}}_\text{AC}=\frac{1}{4\pi\epsilon_0}\frac{\text{q}_\text{A}\text{q}_\text{C}}{\text{r}^2_\text{AC}}
=9\times10^9\times\frac{(2\times10^{-6})(3\times10^{-6})}{(0.20)^2}=1.35\text{ N}\text{ along }\vec{\text{AC}}
\vec{\text{F}}_\text{AM}=\frac{1}{4\pi\epsilon_0}\frac{\text{q}_\text{A}\text{q}_\text{M}}{(\text{r}_\text{AM})^2}=9\times10^9\frac{(2\times10^{-6})
(\text{q}_\text{M})}{(\sqrt{3}\times10^{-1})^2}
=6\times10^5_\text{qM}\text{ N along }\vec{\text{MA}}
For equilibrium of charge qA; the resultant of \vec{\text{F}}_\text{AB} and \vec{\text{F}}_\text{AC} must be equal and opposite to \vec{\text{F}}_\text{AM}.
i.e., \text{F}_\text{AB}\cos30^\circ+\text{F}_\text{AC}\cos30^\circ=\text{F}_\text{AM}
\Rightarrow\ 1.35\times\frac{\sqrt{3}}{2}+1.35\frac{\sqrt{3}}{2}=6\times10^5\text{qM}
\Rightarrow\ \text{qM}=\frac{1.35\sqrt{3}}{6\times10^5}=0.225\sqrt{3}\times10^{-5}\text{C}=2.25\sqrt{3}\mu\text{C}
Q613. Two large conducting plates are placed parallel to each other with a separation of 2.00cm between them. An electron starting from rest near one of the plates 5 Marks
reaches the other plate in 2.00 microseconds. Find the surface charge density on the inner surfaces.

Ans:Distance travelled by the electron, d = 2cm


Time taken to cross the region, t = 2 × 10-6s
Let the surface charge density at the conducting plates be \sigma.
Let the acceleration of the electron be a.
Applying the 2nd equation of motion, we get:
\text{d}=\frac{1}{2}\text{at}^2
\Rightarrow\text{a}=\frac{2\text{d}}{\text{t}^2}
This acceleration is provided by the Coulombic force. So,
\text{a}=\frac{\text{qE}}{\text{m}}=\frac{2\text{d}}{\text{t}^2}
\Rightarrow\text{E}=\frac{2\text{md}}{\text{qt}^2}
\Rightarrow\text{E}=\frac{2\times\big(9.1\times10^{-31}\big)\times\big(2\times10^{-2}\big)}{\big(1.6\times10^{-19}\big)\times\big(4\times10^{-12}\big)}
\Rightarrow\text{E}=5.6875\times10^{-2}\text{N/C}
Also, we know that electric field due to a plate,
\text{E}=\frac{\sigma}{\epsilon_0}
\Rightarrow\sigma=\epsilon_ 0\text{E}
\Rightarrow\sigma=\big(8.85\times10^{-12}\big)\times\big(5.68\times10^{-2}\big)\text{C/m}^2
\Rightarrow\sigma=50.33\times10^{-14}\text{C/m}^2
\Rightarrow\sigma=0.503\times10^{-12}\text{C/m}^2
Q614. Answer the following questions: 5 Marks
1. Define electric flux. Write its SI unit.
2. Using Gauss’s law, prove that the electric field at a point due to a uniformly charged infinite plane sheet is independent of the distance from it.
3. How is the field directed if:
The sheet is positively charged.
Negatively charged?

Ans:1. Electric flux: It is defined as the total number of electric field lines passing through an area normal to its surface.
\Phi=\oint\vec{\text{E}}.\vec{\text{dS}}
The SI unit is Nm2/ C or volt-metre.
2. Image
Let electric charge be uniformly distributed over the surface of a thin, non-conducting infinite sheet. Let the surface charge density (i.e., charge per unit surface
area) be σ. We need to calculate the electric field strength at any point distant r from the sheet of charge.
To calculate the electric field strength near the sheet, we now consider a cylindrical Gaussian surface bounded by two plane faces A and B lying on the opposite
sides and parallel to the charged sheet and the cylindrical surface perpendicular to the sheet (fig). By symmetry the electric field strength at every point on the flat
surface is the same and its direction is normal outwards at the points on the two plane surfaces and parallel to the curved surface.
Total electric flux,
or \oint\limits_\text{S}\vec{\text{E}}.\vec{\text{dS}}=\oint\limits_{\text{S}_1}\vec{\text{E}}.\vec{\text{dS}_1}+\oint\limits_{\text{S}_2}\vec{\text{E}}.\vec{\text{dS}_2}+\oint\limits_{\text{S
\oint\limits_\text{S}\vec{\text{E}}.\vec{\text{dS}}=\oint\limits_{\text{S}_1}\text{E dS}_1\cos0^\circ+\oint\limits_{\text{S}_2}\text{E
dS}_2\cos0^\circ+\oint\limits_{\text{S}_3}\text{E dS}_3\cos90^\circ
=\text{E}\oint\text{dS}_1+\text{E}\oint\text{dS}_2=\text{Ea}+\text{Ea}=2\text{Ea}
\therefore Total electric flux = 2Ea
As \sigma is charge per unit area of sheet and a is the intersecting area, the charge enclosed by Gaussian surface =\sigma\text{a}
Total electric flux =\frac{1}{\epsilon_0}\times (total charge enclosed by the surface)
i.e., 2\text{Ea}=\frac{1}{\epsilon_0}(\sigma\text{a})
\therefore\ \text{E}=\frac{\text{p}}{2\epsilon_0}.
Thus electric field strength due to an infinite flat sheet of charge is independent of the distance of the point.
3. ​
If \sigma is positive, \vec{\text{E}} points normally outwards/ away from the sheet.
If \sigma is negative, \vec{\text{E}} points normally inwards/ towards the sheet.
Q615. Consider the situation of the previous problem: 5 Marks

https://bls.smartstudies.co.in/#/exam/pdf-preview/b5b87181-b79b-4b4a-8ebf-8f94fb605fc8/1 116/139
5/17/24, 8:14 PM Exam Automation
1. Find the tension in the string in equilibrium.
2. Suppose the ball is slightly pushed aside and released. Find the time period of the small oscillations.

Ans:

In equilibrium state, the thread makes an angle of 60° with the vertical.
The tension in the thread is resolved into horizontal and vertical components.
Then, tension in the string in equilibrium,
\text{T}\cos60^\circ=\text{mg}
\text{T}\times\frac{1}{2}=\big(10\times10^{-3}\big)\times10
\text{T}=\big(10\times10^{-3}\big)\times10\times2=0.20\text{N}
(b) As it is displaced from equilibrium, net force on the ball,
\text{F}=\sqrt{(\text{mg})^2+\Big(\frac{\text{q}\sigma}{2\epsilon_0}\Big)^2}
As F = ma
\Rightarrow\text{a}=\sqrt{(\text{g})^2+\Big(\frac{\text{q}\sigma}{m2\epsilon_0}\Big)^2}
The surface charge density of the plate (as calculated in the previous question), \sigma=7.5\times10^{-7}\text{C/m}^2
Charge on the ball, q = 4 × 10-6C
Mass of the ball, m = The time period of oscillation of the given simple pendulum,
\text{T}=2\pi\sqrt{\frac{\text{l}}{\text{g}}}
=2\pi\sqrt{\frac{10\times10^{-2}}{9.8}}
=0.45\text{sec}
Q616. An electric dipole of dipole moment \vec{\text{P}} is placed in a uniform electric field \vec{\text{E}}. Write the expression for the torque \vec\tau experienced by 5 Marks
the dipole. Identify two pairs of perpendicular vectors in the expression. Show diagrammatically the orientation of the dipole in the field for which the torque is (i)
Maximum (ii) Half the maximum value (iii) Zero.

Ans:Torque experienced by an electric dipole,


\vec{\tau}=\vec{\text{p}}\times\vec{\text{E}}
Pairs of perpendicular vectors,
1. (\vec{\tau},\vec{\text{p}})
2. (\vec{\tau},\vec{\text{E}})
1. Magnitude of torque \tau=\text{pE}\sin\theta
For maximum torque (\sin\theta)\max=1\Rightarrow\theta=90^\circ
Orientation is shown in figure (i).

2. For \tau=\frac{1}{2}\tau_\max
\text{pE}\sin\theta=\frac{1}{2}\text{pE}
\Rightarrow\ \sin\theta=\frac{1}{2}\ \text{or }\theta=30^\circ
Orientation is shown in figure (ii).

https://bls.smartstudies.co.in/#/exam/pdf-preview/b5b87181-b79b-4b4a-8ebf-8f94fb605fc8/1 117/139
5/17/24, 8:14 PM Exam Automation

3. For zero torque, \sin\theta=0\Rightarrow\theta=0


The orientations is shown in the figure (iii).

Q617. 1. Explain, using suitable diagrams, the difference in the behaviour of a (i) conductor and (ii) dielectric in the presence of an external electric field. Define the 5 Marks
terms polarization of a dielectric and write its relation with susceptibility
2. A thin metallic spherical shell of radius R carries a charge Q on its surface. A point charge \frac{\text{Q}}{2} placed at its centre C and an othercharge +2Q is
placed outside the shell at a distance x from the centre as shown in the figure. Find (i) the force on the charge at the centre of shell and at the point A and (ii)
the electric flux through the shell.

Ans:

In the presence of electric field, the free charge carriers, in a conductor, move the charge distribution in the conductor readjusts itself so that the net lectric field
within the conductor becomes zero. In a dielectric, the external electric field induces a net dipole moment, by stretching/reorienting the molecules. The electric
field, due to this induced dipole moment, opposes ,but does not exactly cancel, the external electric field. Polarisation: Induced dipole moment, per unit volume, is
called the polarization. For linear isotropic dielectrics having a susceptibility Xc , we have P = XeE.
Net Force on the charge \frac{\text{Q}}{2} ,placed at the centre of the shell,is zero.
Force on charge '2Q' kept at point A
\text{F} = \text{E}\times2\text{Q} =\frac{1\bigg(\frac{3\text{q}}{2}\bigg)2\text{Q}}{4\pi\varepsilon_{o}\text{r}^{2}} = \frac{\text{(k)}3\text{Q}^{2}}{\text{r}^{2}}
Electric flux through the shell
\phi =\frac{\text{Q}}{2\varepsilon_{0}}
Q618. Two charges 2.0 × 10-6C and 1.0 × 10-6C are placed at a separation of 10cm. Where should a third charge be placed such that it experiences no net force due to 5 Marks
these charges?

Ans:Given:
\text{q}_1=2.0\times10^{-6}\text{C}
\text{q}_2=1.0\times10^{-6}\text{C}
Let the third charge, q, be placed at a distance of x cm from charge q1, as shown in the figure.

https://bls.smartstudies.co.in/#/exam/pdf-preview/b5b87181-b79b-4b4a-8ebf-8f94fb605fc8/1 118/139
5/17/24, 8:14 PM Exam Automation

By Coulomb's Law, force,


\text{F}=\frac{1}{4\pi\in_0}\frac{\text{Q}_1\text{Q}_2}{\text{r}^2}
Force on charge q due to q1,
\text{F}=\frac{9\times10^9\times2.0\times10^{-6}\times\text{q}}{\text{x}^2}
Force on charge q due to q2,
\text{F}'=\frac{9\times10^9\times10^{-6}\times\text{q}}{(10-\text{x})^2}
According to the question,
\text{F}-\text{F}'=0
\Rightarrow\text{F}=\text{F}'
\Rightarrow\frac{9\times10^9\times2\times10^{-6}\times\text{q}}{\text{x}^2}=\frac{9\times10^9\times10^{-6}\times\text{q}}{(10-\text{x})^2}
\Rightarrow\text{x}^2=2(10-\text{x})^2
\Rightarrow\text{x}^2-40\text{x}+200=0
\Rightarrow\text{x}=20\pm10\sqrt{2}
\Rightarrow\text{x}=5.9\text{cm} \Big(\because\text{x}\neq20+10\sqrt{2}\Big)
So, the third charge should be placed at a distance of 5.9cm from q1.
Q619. Electric field strength is proportional to the density of lines of force i.e., electric field strength at a point is proportional to the number of lines of force cutting a unit 5 Marks
area element placed normal to the field at that point. As illustrated in the given figure, the electric field at P is stronger that at Q.

1. Electric lines of force about a positive point charge are:


1. Radially outwards.
2. Circular clockwise.
3. Radially inwards.
4. Parallel straight lines.
2. Which of the following is false for electric lines of force?
1. They always start from positive charges and terminate on negative charges.
2. They are always perpendicular to the surface of a charged conductor.
3. They always form closed loops.
4. They are parallel and equally spaced in a region of uniform electric field.
3. Which one of the following pattern of electric line of force in not possible in filed due to stationary charges?
1.

2.

https://bls.smartstudies.co.in/#/exam/pdf-preview/b5b87181-b79b-4b4a-8ebf-8f94fb605fc8/1 119/139
5/17/24, 8:14 PM Exam Automation

3.

4.

4. Electric lines of force are curved:


1. In the field of a single positive or negative charge.
2. In the field of two equal and opposite charges.
3. In the field of two like charges.
4. Both (b) and (c).
5. The figure below shows the electric field lines due to two positive charges. The magnitudes EA, EB and EC of the electric fields at points A, Band C respectively
are related as:

https://bls.smartstudies.co.in/#/exam/pdf-preview/b5b87181-b79b-4b4a-8ebf-8f94fb605fc8/1 120/139
5/17/24, 8:14 PM Exam Automation

1. EA > EB > EC
2. EB > EA > EC
3. EA = EB > EC
4. EA > EB = EC

Ans:1. (a) Radially outwards.


2. (c) They always form closed loops.
Explanation:
Electric lines of force do not form any closed loops.
3. (c)


​Explanation:
Electric field tines can't be closed.
4. (d) Both (b) and (c).
5. (a) EA > EB > EC
Q620. In 1909, Robert Millikan was the first to find the charge of an electron in his now-famous oil-drop experiment. In that experiment, tiny oil drops were sprayed into a 5 Marks
uniform electric field between a horizontal pair of oppositely charged plates. The drops were observed with a magnifying eyepiece, and the electric field was
adjusted so that the upward force on some negatively charged oil drops was just sufficient to balance the downward force of gravity. That is, when suspended,
upward force qE just equaled Mg. Millikan accurately measured the charges on many oil drops and found the values to be whole number multiples of 1.6 × 10-19C
the charge of the electron. For this, he won the Nobel Prize.

https://bls.smartstudies.co.in/#/exam/pdf-preview/b5b87181-b79b-4b4a-8ebf-8f94fb605fc8/1 121/139
5/17/24, 8:14 PM Exam Automation

1. If a drop of mass 1.08 × 10-14kg remains stationary in an electric field of 1.68 × 105NC-1, then the charge of this drop is:
1. 6.40 × 10-19C
2. 3.2 × 10-19C
3. 1.6 × 10-19C
4. 4.8 × 10-19C
2. Extra electrons on this particular oil drop (given the presently known charge of the electron) are:
1. 4
2. 3
3. 5
4. 8
3. A negatively charged oil drop is prevented from falling under gravity by applying a vertical electric field 100V m-1. If the mass of the drop is 1.6 × 10-3g, the
number of electrons carried by the drop is (g = 10m s-2)
1. 1018
2. 1015
3. 1012
4. 109
4. The important conclusion given by Millikan's experiment about the charge is:
1. Charge is never quantized.
2. Charge has no definite value.
3. Charge is quantized.
4. Charge on oil drop always increases.
5. If in Millikan's oil drop experiment, charges on drops are found to be 8\mu\text{C, }12\mu\text{C, }20\mu\text{C, } then quanta of charge is:
1. 8\mu\text{C}
2. 20\mu\text{C}
3. 12\mu\text{C}
4. 4\mu\text{C}

Ans:1. (a) 6.40 × 10-19C


Explanation:
As, qE = mg
\Rightarrow\text{q}=\frac{1.08\times10^{-14}\times9.8}{1.68\times10^5}
= 6.40 × 10-19C
2. (a) 4
Explanation:
q = ne or
\Rightarrow\text{n}=\frac{6.4\times10^{-19}}{1.6\times10^{-19}}=4
3. (c) 1012
​Explanation:
For the drop to be stationary,
Force on the drop due to electric field = Weight of the drop qE = mg.
\text{q}=\frac{\text{mg}}{\text{E}}=\frac{1.6\times10^{-6}\times10}{100}=1.6\times10^{-7}\text{C}
Number of electrons carried by the drop is
\text{n}=\frac{\text{q}}{\text{e}}=\frac{1.6\times10^{-7}\text{C}}{1.6\times10^{-19}\text{C}}=10^{12}
4. (c) Charge is quantized.
5. (d) 4\mu\text{C}
​Explanation:
Millikan's experiment confirmed that the charges are quantized, i.e., charges are small integer multiples of the base value which is charge on electron. The charges
on the drops are found to be multiple of 4. Hence, the quanta of charge is 4\mu\text{C}.
Q621. State Gauss theorem in electrostatics. Apply this theorem to obtain the expression for the electric field at a point due to an infinitely long, thin, uniformly charged 5 Marks
straight wire of linear charge density \lambda\text{Cm}^{-1}.

Ans:Gauss Theorem: It states that total electric flux over the closed surface S is \frac{1}{\epsilon_0} times the total charge (q) contained in side S.
\therefore\ \Phi_\text{E}=\oint\limits_{\text{S}}\vec{\text{E}}.\vec{\text{dS}}=\frac{\text{q}}{\epsilon_0}
Electric field due to infinitely long, thin and uniformly charged straight wire: Consider an infinitely long line charge having linear charge density \lambda coulomb
metre-1 (linear charge density means charge per unit length). To find the electric field strength at a distance r, we consider a cylindrical Gaussian surface of radius r
and length l coaxial with line charge. The cylindrical Gaussian surface may be divided into three parts:
1. Curved surface S1.
2. Flat surface S2.
3. Flat surface S3.

https://bls.smartstudies.co.in/#/exam/pdf-preview/b5b87181-b79b-4b4a-8ebf-8f94fb605fc8/1 122/139
5/17/24, 8:14 PM Exam Automation
By symmetry, the electric field has the same magnitude E at each point of curved surface S1 and is directed radially outward.
We consider small elements of surfaces S1, S2 and S3 The surface element vector \vec{\text{dS}}_1 is directed along the direction of electric field (i.e., angle
between \vec{\text{E}} and \vec{\text{dS}}_1 is zero); the elements \vec{\text{dS}}_2 and \vec{\text{dS}}_3 are directed perpendicular to field
vector \vec{\text{E}} (i.e., angle between \vec{\text{dS}}_2 and \vec{\text{E}} is 90° and so also angle between \vec{\text{dS}}_3 and \vec{\text{E}}).
Electric Flux through the cylindrical surface,
Image
\oint\limits_{\text{S}}\vec{\text{E}}.\vec{\text{dS}}=\oint\limits_{\text{S}_1}\vec{\text{E}}.\vec{\text{dS}_1}+\oint\limits_{\text{S}_2}\vec{\text{E}}.\vec{\text{dS}_2}+\oint\limits_{\text{S}_3}\
=\oint\limits_{\text{S}_1}{\text{E}}.{\text{dS}_1}\cos0^\circ+\oint\limits_{\text{S}_2}{\text{E}}.{\text{dS}_2}\cos90^\circ+\oint\limits_{\text{S}_3}{\text{E}}.
{\text{dS}_3}\cos90^\circ
=\oint\text{E dS}_1+0+0
=\text{E}\oint\text{dS}_1 (since electric field E is the same at each point of curved surface)
=\text{E}2\pi\text{rl} (since area of curved surface =2\pi\text{rl})
As \lambda is charge per unit length and length of cylinder is l therefore, charge enclosed by assumed surface =(\lambda\text{l})
\therefore By Gauss's theorem
\int\vec{\text{E}}.\vec{\text{dS}}=\frac{1}{\epsilon_0}\times\text{charge enclosed}
\Rightarrow\ \text{E}.2\pi\text{rl}=\frac{1}{\epsilon_0}(\lambda\text{l})
\Rightarrow\ \text{E}=\frac{\lambda}{2\pi\epsilon_0\text{r}}
Thus, the electric field strength due to a line charge is inversely proportional to r.
Q622. Net electric flux through a cube is the sum of fluxes through its six faces. Consider a cube as shown in figure, having sides of length L = 10.0cm. The electric field 5 Marks
3 -1
is uniform, has a magnitude E = 4.00 × 10 N C and is parallel to the xy plane at an angle of 37º measured from the + x - axis towards the + y - axis.

1. Electric flux passing through surface S6 is:


1. -24N m2 C-1
2. 24N m2 C-1
3. 32N m2 C-1
4. -32N m2 C-1
2. Electric flux passing through surface S1 is:
1. -24N m2 C-1
2. 24N m2 C-1
3. 32N m2 C-1
4. -32N m2 C-1
3. The surfaces that have zero flux are:
1. S1 and S3
2. S5 and S6
3. S2 and S4
4. S1 and S2
4. The total net electric flux through all faces of the cube is:
1. 8N m2 C-1
2. -8N m2 C-1
3. 24N m2 C-1
4. Zero.
5. The dimensional formula of surface integral \oint\vec{\text{E}}\cdot\text{d}\vec{\text{S}} of an electric field is:
1. [M L2 T-2 A-1]
2. [M L3 T-3 A-1]
3. [M L-1 T3 A-3]
4. [M L-3 T-3 A-1]

Ans:1. (d) -32N m2 C-1


Explanation:
Electric flux, \phi=\vec{\text{E}}\cdot\vec{\text{A}}=\text{EA} \cos\theta.
Where \vec{\text{A}}=\widehat{\text{An}}
For electric flux passing through \text{S}_6,\widehat{\text{n}}_{\text{s}_6}=-\widehat{\text{i}}\text{ (Back})
\therefore \phi_{\text{s}{_6}}=-(4\times10^3\text{ N}\text{ C}^{-1})(0.1\text{m})^2 \cos37^\circ
-32N m2 C-1
2. (a) -24N m2 C-1
Explanation:
For electric flux passing through \text{S}_1,\widehat{\text{n}}_{\text{s}_1}=-\widehat{\text{j}}\text{ (Left})
\therefore \phi_{\text{s}_{1}}=-(4\times10^3\text{ N}\text{ C}^{-1})(0.1\text{m})^2 \cos(90^\circ-37^\circ)
-24 N m2 C-1

https://bls.smartstudies.co.in/#/exam/pdf-preview/b5b87181-b79b-4b4a-8ebf-8f94fb605fc8/1 123/139
5/17/24, 8:14 PM Exam Automation
3. (c) S2 and S4
Explanation:
Here, \widehat{\text{n}}_{\text{s}_{2}} = + \widehat{\text{k}} \text{ (Top)}
\therefore \phi_{\text{s}_{2}}=-(4\times10^3\text{ N}\text{ C}^{-1})(0.1\text{m})^2 \cos90^\circ=0
\widehat{\text{n}}_{\text{s}_{3}} = + \widehat{\text{j}} \text{ (Right)}
\widehat{\text{n}}_{\text{s}_{4}} = - \widehat{\text{k}} \text{ (Bottom)}
\therefore \phi_{\text{s}_{4}}=-(4\times10^3\text{ N}\text{ C}^{-1})(0.1\text{m})^2 \cos90^\circ=0
And, \widehat{\text{n}}_{\text{s}_{5}} = - \widehat{\text{i}} \text{ (Fornt)}
\therefore \phi_{\text{s}_{5}}=-(4\times10^3\text{ N}\text{ C}^{-1})(0.1\text{m})^2 \cos37^\circ
= 32N m2 C-1
S2 and S4 surface have zero flux.
4. (d) Zero.
Explanation:
As the field is uniform, the total flux through the cube must be zero, i.e., any flux entering the cube must leave it.
5. (b) [M L3 T-3 A-1]
Explanation:
Surface integral \oint\vec{\text{E}}\cdot\text{d}\vec{\text{S}} is the net electric flux over a closed surface S.
\therefore[\phi_\text{E}]=[\text{M}\text{ L}^3\text{ T}^{-3}\text{ A}^{-1}]
Q623. Fig. represents a crystal unit of cesium chloride, CsCl. The cesium atoms, represented by open circles are situated at the corners of a cube of side 0.40nm, 5 Marks
whereas a Cl atom is situated at the centre of the cube. The Cs atoms are deficient in one electron while the Cl atom carries an excess electron.

1. What is the net electric field on the Cl atom due to eight Cs atoms?
2. Suppose that the Cs atom at the corner A is missing. What is the net force now on the Cl atom due to seven remaining Cs atoms?

Ans:1. From the given figure, we can observe that Cl‒ atom is at the centre whereas Cs+ atoms are at eight corners at equal distance. So, by symmetry net force on Cl‒
atom due to other Cs+ atoms is zero. Due to which, net electric field due to other Cs+ atoms is also zero \Big(\text{as},|\text{E}|=\frac{\text{F}}{\text{q}}\Big).
2. In the given question, removing Cs atom at the corner A is equivalent to adding a singly charged negative Cs ion at point A.
So,
\text{Net force}=\frac{\text{q}^2}{4\pi\epsilon_0\text{r}^2}
Here, q = charge on electron, r = distance between Cl and Cs atoms
Applying Pythagoras theorem, we have
\text{r}=\sqrt{(0.02)^2+(0.20)^2+(0.20)^2}\times10^{-9}\text{m}=0.346\times10^{-9}\text{m}
Now,
\text{Not force}=\frac{\text{q}^2}{4\pi\epsilon_0\text{r}^2}=\frac{9\times10^9(1.6\times10^{-19})^2}{(0.346\times10^{-9})^2}=1.92\times10^{-9}\text{N}
The direction of force is from A to Cl‒.
Q624. 1. Using Gauss Theorem show mathematically that for any point outside the shell, the field due to a uniformly charged spherical shell is same as the entire 5 Marks
charge on the shell, is concentrated at the centre.
2. Why do you expect the electric field inside the shell to be zero according to this theorem?
OR
A thin conducting spherical shell of radius R has charge Q spread uniformly over its surface. Using Gauss’s theorem, derive an expression for the electric field at a
point outside the shell.
Draw a graph of electric field E(r) with distance r from the centre of the shell for 0\leq\text{r}\le\infty.
OR
Find the electric field intensity due to a uniformly charged spherical shell at a point (i) outside the shell and (ii) inside the shell. Plot the graph of electric field with
distance from the centre of the shell.
OR
Using Gauss’s law obtain the expression for the electric field due to a uniformly charged thin spherical shell of radius R at a point outside the shell. Draw a graph
showing the variation of electric field with r, for r > R and r < R.

Ans:Image
1. Electric field intensity at a point outside a uniformly charged thin spherical shell: Consider a uniformly charged thin spherical shell of radius R carrying charge
Q. To find the electric field outside the shell, we consider a spherical Gaussian surface of radius r (>R), concentric with given shell. If \vec{\text{E}} is electric
field outside the shell, then by symmetry electric field strength has same magnitude E0 on the Gaussian surface and is directed radially outward. Also the
directions of normal at each point is radially outward, so angle between \vec{\text{E}}_\text{i} and \vec{\text{dS}} is zero at each point. Hence, electric flux
through Gaussian surface.
\oint\text{S}=\vec{\text{E}}_0.\vec{\text{dS}}
\oint=\text{E}_0\text{dS}\cos0=\text{E}_0.4\pi\text{r}^2
Now, Gaussian surface is outside the given charged shell, so charge enclosed by Gaussian surface is Q.
Hence, by Gauss’s theorem,
\oint\text{S}=\vec{\text{E}}_0.\vec{\text{dS}}=\frac{1}{\epsilon_0}\times\text{charged enclosed}
\Rightarrow\ \text{E}_0.4\pi\text{r}^2=\frac{1}{\epsilon_0}\times\text{Q}
\Rightarrow\ \text{E}_0=\frac{1}{4\pi\epsilon_0}\frac{\text{Q}}{\text{r}^2}
Thus, electric field outside a charged thin spherical shell is the same as if the whole charge Q is concentrated at the centre.
If \sigma is the surface charge density of the spherical shell, then
\text{Q}=4\pi\text{R}^2\sigma coulomb
\therefore\ \text{E}_0=\frac{1}{4\pi\epsilon_0}\frac{4\pi\text{R}^2\sigma}{\text{r}^2}=\frac{\text{R}^2\sigma}{\epsilon_0\text{r}^2}
Image
1. Electric field inside the shell (hollow charged conducting sphere): The charge resides on the surface of a conductor. Thus a hollow charged conductor is
equivalent to a charged spherical shell. To find the electric field inside the shell, we consider a spherical Gaussian surface of radius r (< R) concentric with the
given shell. If \vec{\text{E}} is the electric field inside the shell, then by symmetry electric field strength has the same magnitude Ei on the Gaussian surface and
is directed radially outward. Also the directions of normal at each point is radially outward, so angle between \vec{\text{E}}_\text{i} and \vec{\text{dS}} is zero at
each point.
Hence, electric flux through Gaussian surface,
=\int\limits_\text{S}\vec{\text{E}}_{\text{i}}.\vec{\text{dS}}=\int\text{E}_\text{i}.\text{dS}\cos0=\text{E}_\text{i}.4\pi\text{r}^2

https://bls.smartstudies.co.in/#/exam/pdf-preview/b5b87181-b79b-4b4a-8ebf-8f94fb605fc8/1 124/139
5/17/24, 8:14 PM Exam Automation
Now, Gaussian surface is inside the given charged shell, so charge enclosed by Gaussian surface is zero.
Hence, by Gauss’s theorem,
\int\limits_\text{S}\vec{\text{E}}_\text{i}.\vec{\text{dS}}=\frac{1}{\epsilon_0}\times\text{charged enclosed}
\Rightarrow\ \text{E}_\text{i}4\pi\text{r}^2=\frac{1}{\epsilon_0}\times0\Rightarrow\ \text{E}_\text{i}=0
Image
Thus, electric field at each point inside a charged thin spherical shell is zero. The graph is shown in fig.
Q625. 1. Derive an expression for the energy stored in a parallel plate capacitor of capacitance C when charged up to voltage V. How is this energy stored in the 5 Marks
capacitor?
2. A capacitor of capacitance 1\mu\text{F} is charged by connecting a battery of negligible internal resistance and emf 10V across it. Calculate the amount of
charge supplied by the battery in charging the capacitor fully.

Ans:1. Work done in adding a charge \text{dq}=\text{dW}


=\text{Vdq}
=\frac{\text{q}}{\text{c}}\text{dq}
\therefore Total amount of work (W) in charging a capacitor.
\text{W}=\int\text{dw}=\frac1{\text{C}}\int\limits_\text{0}^\text{Q}\text{qdp}
\text{W}=\frac{\text{Q}^2}{\text{2C}}
=\frac{(\text{CV})^2}{\text{2C}}=\frac12\text{CV}^2
The electrostatic Energy/ potential energy is stored in the electric field between the plates.
2. \text{C}=1\mu\text{F}=1\times10^{-6}\text{F;}\ \text{V}=10\text{ volt}
\text{Q}=\text{CV}
=1\times10^{-6}\times10
=10^{-5}\text{coulomb}.
Q626. Sketch the electric field lines for a uniformly charged hollow cylinder shown in Fig. 5 Marks

Ans:The electric field lines starts from positive charges and move towards infinity and meet plane surface normally as shown in the figure below:

Important point: No electric field lines will be present inside the cylinder because of electrostatic shielding. Electrostatic shielding/screening is the phenomenon of
protecting a certain region of space from external electric field. Sensitive instruments and appliances are affected seriously with strong external electrostatic fields.
Their working suffers and they may start misbehaving under the effect of unwanted fields.

The electrostatic shielding can be achieved by protecting and enclosing the sensitive instruments inside a hollow conductor because inside hollow conductors,
electric fields is zero.
Q627. Coulomb's law states that the electrostatic force of attraction or repulsion acting between two stationary point charges is given by: 5 Marks
\text{F}=\frac{1}{4\pi\epsilon_0}\frac{\text{q}_1\text{q}_2}{\text{r}^2}

Where F denotes the force between two charges q1 and q2 separated by a distance r in free space, \epsilon_0 is a constant known as permittivity of free space.
Free space is vacuum and may be taken to be air practically. If free space is replaced by a medium, then \epsilon_0 is replaced
by (\epsilon_0\text{k}) or (\epsilon_0\epsilon_\text{r}) where k is known as dielectric constant or relative permittivity.
1. In coulomb's law, \text{F}=\text{k}\frac{\text{q}_1\text{q}_2}{\text{r}^2} then on which of the following factors does the proportionality constant k depends?
1. Electrostatic force acting between the two charges.
2. Nature of the medium between the two charges.
3. Magnitude of the two charges.
4. Distance between the two charges.

https://bls.smartstudies.co.in/#/exam/pdf-preview/b5b87181-b79b-4b4a-8ebf-8f94fb605fc8/1 125/139
5/17/24, 8:14 PM Exam Automation
2. Dimensional formula for the permittivity constant \epsilon_0 of free space is:
1. [ML-3 T4 A2]
2. [M-1 L3 T2 A2]
3. [M-1 L-3 T4 A2]
4. ML-3 T4 A-2]
3. The force of repulsion between two charges of 1C each, kept 1m apart in vaccum is:
1. \frac{1}{9\times10^9}\text{N}
2. 9 × 109N
3. 9 × 107N
4. \frac{1}{9\times10^{12}}\text{N}
4. Two identical charges repel each other with a force equal to 10 mgwt when they are 0.6m apart in air.(g = 10m s-2). The value of each charge is:
1. 2mC
2. 2 × 10-7mC
3. 2 nC
4. 2\mu\text{C}
5. Coulomb's law for the force between electric charges most closely resembles with:
1. Law of conservation of energy.
2. Newton's law of gravitation.
3. Newton's 2nd law of motion.
4. Law of conservation of charge.

Ans:1. (b) Nature of the medium between the two charges.


Explanation:
The proportionality constant k depends on the nature of the medium between the two charges.
2. (c) [M-1 L-3 T4 A2]
Explanation:
As, [\epsilon_0]=\frac{1}{4\pi\text{F}}.\frac{\text{q}_1\text{q}_2}{\text{r}^2}
=\frac{[\text{AT]}^2}{[\text{M L T}^{-2}][\text{L}^2]}
\text{[M}^{-1} \text{ L}^{-3} \text{ T}^4 \text{A}^2]
3. (b) 9 × 109N
4. 2\mu\text{C}
Explanation:
\text{F}=\frac{1}{4\pi\epsilon_0}\frac{\text{q}_1\text{q}_2}{\text{d}^2}
\therefore(10\times10^{-3})\times10=\frac{(9\times10^9)\times\text{q}^2}{(0.6)^2}
Or \text{q}^2=\frac{10^{-1}\times0.36}{9\times10^9}=4\times10^{-12}
Or \text{q}=2\times10^{-6}\text{ C}=2\mu\text{C}
5. (b) Newton's law of gravitation.
Q628. 1. Derive an expression for the torque experienced by an electric dipole kept in a uniform electric field. 5 Marks
2. Calculate the work done to dissociate the system of three charges placed on the vertices of a triangle as shown.
Here q = 1.6 × 10–10C.

Ans:1.

Electric force on + q = qE
Electric force on – q = –qE
\therefore\text{Magnitude of torque } (\pi) = \text{qE. } 2\text{a}\sin\theta
\tau = \big(\text{q}.2a\big)E \sin \theta
\tau = \text{pE}\sin\theta
The correct direction of \overrightarrow{\tau} is given by \overrightarrow{\text{P}} \times\overrightarrow{\text{E}}
Hence \overrightarrow{\tau} = \overrightarrow{\text{p}} \times\overrightarrow{\text{E}}
2. Work done (W) = -(electric P.E. of the system)
\text{w} = \frac{1}{4\pi\in_{\circ}l}\big(\text{q}_{1}\text{q}_{2} + \text{q}_{2}\text{q}_{3} + \text{q}_{3}\text{q}_{1}\big)
= - \frac{9\times10^{9}}{10^{-1}} \big( - 4\text{q}\times\text{q} + \text{q} \times2\text{q} + 2\text{q}-4\text{q}\big) = 9\times10^{10}\times10\text{q}^{2}
= 9\times10^{11}\times\big(1.6\times10^{-10}\big)^{2}\text{J}
= 2.3\times 10^{-8}\text{J}.
Q629. Fig. shows the electric field lines around three point charges A, B and C. 5 Marks

1. Which charges are positive?


2. Which charge has the largest magnitude? Why?
3. In which region or regions of the picture could the electric field be zero? Justify your answer.
1. Near A,
2. Near B,
3. Near C,
4. Nowhere.

Ans:1. Charges A and C are positive since lines of force emanate from them.
2. Charge C has the largest magnitude since maximum numbers of field lines are associated with it.
3.

https://bls.smartstudies.co.in/#/exam/pdf-preview/b5b87181-b79b-4b4a-8ebf-8f94fb605fc8/1 126/139
5/17/24, 8:14 PM Exam Automation
1. Near A. There is no neutral point between a positive and a negative charge. A neutral point may exist between two like charges. From the figure we see that a
neutral point exists between charges A and C. Also between two like charges the neutral point is closer to the charge with smaller magnitude. Thus, electric field
is zero near charge A.
Q630. Surface charge density is defined as charge per unit surface area of surface charge distribution. i.e., \sigma=\frac{\text{dq}}{\text{dS}}. Two large, thin metal 5 Marks
plates are parallel and close to each other. On their inner faces, the plates have surface charge densities of opposite signs having magnitude of 17.0 × 10-22Cm-2 as
shown. The intensity of electric field at a point is \text{E}=\frac{\sigma}{\epsilon_0}, where\epsilon_0= permittivity of free space.

1. E in the outer region of the first plate is:


1. 17 × 10-22 N/C
2. 1.5 × 10-25 N/C
3. 1.9 × 10-10 N/C
4. Zero.
2. E in the outer region of the second plate is:
1. 17 × 10-22 N/C
2. 1.5 × 10-15 N/C
3. 1.9 × 10-10 N/C
4. Zero.
3. E between the plates is:
1. 17 × 10-22 N/C
2. 1.5 × 10-15 N/C
3. 1.9 × 10-10 N/C
4. Zero.
4. The ratio of E from right side of B at distances 2cm and 4cm, respectively is:
1. 1 : 2
2. 2 : 1
3. 1 : 1
4. 1:\sqrt{2}
5. ln order to estimate the electric field due to a thin finite plane metal plate, the Gaussian surface considered is:
1. Spherical.
2. Spherical.
3. Straight line.
4. None of these.

Ans:1. (d) Zero.


Explanation:
There are two plates A and B having surface charge densities,

\sigma_\text{A}=17.0\times10^{-22}\text{C/m}^2
on A and \sigma_\text{B}=-17.0\times10^{-22}\text{C/m}^2on B, respectively. According to Gauss' theorem, if the plates have same surface charge density but
having opposite signs, then the electric field in region I is zero.

https://bls.smartstudies.co.in/#/exam/pdf-preview/b5b87181-b79b-4b4a-8ebf-8f94fb605fc8/1 127/139
5/17/24, 8:14 PM Exam Automation
\text{E}_\text{I}=\text{E}_\text{A}+\text{E}_\text{B}
=\frac{\sigma}{2\epsilon_0}+\Big(-\frac{\sigma}{2\epsilon_0}\Big)=0
2. (d) Zero.
Explanation:
The electric field in region III is also zero.
\text{E}_\text{III}=\text{E}_\text{A}+\text{E}_\text{B}
=\frac{\sigma}{2\epsilon_0}+\Big(-\frac{\sigma}{2\epsilon_0}\Big)=0
3. (c) 1.9 × 10-10 N/C
Explanation:
In region II or between the plates, the electric field.
\text{E}_\text{II}=\text{E}_\text{A}-\text{E}_\text{B}
=\frac{\sigma}{2\epsilon_0}+=\frac{\sigma}{2\epsilon_0}
=\frac{\sigma(\sigma_\text{A}\text{ or }\sigma_\text{B})}{\epsilon_0}=\frac{17.0\times10^{-22}}{8.85\times10^{-12}}
E = 1.9 × 10-10 N/C
4. (c) 1 : 1
Explanation:
Since electric field due to an infinite-plane sheet of charge does not depend on the distance of observation point from the plane sheet of charge. So, for the given
distances, the ratio of E will be 1 : 1.
5. (b) Spherical.
Explanation:
ln order to estimate the electric field due to a thin finite plane metal plate, we take a cylindrical cross-sectional area A and length 2r as the gaussian surface.
Q631. Two fixed, identical conducting plates (\alpha\ \&\ \beta), each of surface area S are charged to -Q and q, respectively, where Q > q > 0. A third identical plate 6 Marks
(\gamma), free to move is located on the other side of the plate with charge q at a distance d (Fig.). The third plate is released and collides with the plate \beta.
Assume the collision is elastic and the time of collision is sufficient to redistribute charge amongst \beta\ \&\ \gamma.

Find the charges on \beta and \gamma after the collision.

Ans:During collision, plates \beta and \gamma are in contact with each other, hence their potentials become same.
Suppose charge on plate \beta is q1 and charge on plate \gamma is q2. At any point O, in between the two plates, the electric field must be zero.

Electric field at o due to plate \alpha,\vec{\text{E}_\alpha}=\frac{\text{Q}}{\text{S}(2\epsilon_0)}(-\hat{\text{i}})


Electric field at O due to plate \beta,\vec{\text{E}_2}=\frac{\text{q}_1}{\text{S}(2\epsilon_0)}(\hat{\text{i}})
Electric field at O due to plate \gamma,\vec{\text{E}_\gamma}=\frac{\text{q}_2}{\text{S}(2\epsilon_0)}(-\hat{\text{i}})
As the electric field at O is zero, therefore
\frac{\text{Q}+\text{q}_2}{\text{S}(2\epsilon_0)}=\frac{\text{q}_1}{\text{S}(2\epsilon_0)}
\because\ \text{Q}+\text{q}_2=\text{q}_1
As there is no loss of charge on collision,
Q + q = q1 + q 2
On solving Eqs. (i) and (ii), we get
\text{q}_1=\Big(\text{Q}+\frac{\text{q}}{2}\Big)=\text{Charge on plate }\beta
\text{q}_1=\Big(\frac{\text{q}}{2}\Big)=\text{Charge on plate }\gamma
Q632. Two particles A and B, each carrying a charge Q, are held fixed with a separation d between them. A particle C having mass m and charge q is kept at the middle 6 Marks
point of the line AB.
1. If it is displaced through a distance x perpendicular to AB, what would be the electric force experienced by it.
2. Assuming x << d, show that this force is proportional to x.
3. Under what conditions will the particle C execute simple harmonic motion if it is released after such a small displacement?
Find the time period of the oscillations if these conditions are satisfied.

Ans:

https://bls.smartstudies.co.in/#/exam/pdf-preview/b5b87181-b79b-4b4a-8ebf-8f94fb605fc8/1 128/139
5/17/24, 8:14 PM Exam Automation

1. Let Q = charge on A & B Separated by distance d


q = charge on c displaced \bot to -AB
So, force on 0=\overline{\text{F}}_\text{AB}+\overline{\text{F}}_\text{BO}
But \text{F}_\text{AO}\cos\theta=\text{F}_\text{BO}\cos\theta
So, force on ‘0’ in due to vertical component.
\overline{\text{F}}=\text{F}_\text{AO}\sin\theta+\text{F}_\text{BO}\sin\theta |\text{F}_\text{AO}|=|\text{F}_\text{BO}|
=2\frac{\text{KQq}}{\Big(\frac{\text{d}}{2^2+\text{x}^2}\big)}\sin\theta
\text{F}=\frac{2\text{KQq}}{\Big(\frac{\text{d}}{2}\Big)^2+\text{x}^2}\sin\theta
=\frac{4\times2\times2\text{KQq}}{(\text{d}^2+4\text{x}^2)}\times\frac{\text{x}}{\Big[\big(\frac{\text{d}}{2}\big)^2+\text{x}^2\Big]^{\frac{1}{2}}}
=\frac{2\text{kQq}}{\Big[\big(\frac{\text{d}}{2}\big)^2+\text{x}^2\Big]^{\frac{3}{2}}}\text{x} = Electric force \Rightarrow\text{F}\propto\text{x}
2. When x << d
\text{F}=\frac{2\text{kQq}}{\Big[\big(\frac{\text{d}}{2}\big)^2+\text{x}^2\Big]^{\frac{3}{2}}}\text{x} x << d
\Rightarrow\text{F}=\frac{2\text{kQq}}{\Big(\frac{\text{d}^2}{4}\Big)^{\frac{3}{2}}}\text{x}\Rightarrow\text{F}\propto\text{x}
\text{a}=\frac{\text{F}}{\text{m}}=\frac{1}{\text{m}}\Bigg[\frac{2\text{kQqx}}{\Big[\big(\frac{\text{d}^2}{4}\big)+\ell^2}\Bigg]
So time period \text{T}=2\pi\sqrt{\frac{\ell}{\text{g}}}
=2\pi\sqrt{\frac{\ell}{\text{a}}}
Q633. Two fixed, identical conducting plates (\alpha\ \&\ \beta), each of surface area S are charged to -Q and q, respectively, where Q > q > 0. A third identical plate 6 Marks
(\gamma), free to move is located on the other side of the plate with charge q at a distance d (Fig.). The third plate is released and collides with the plate \beta.
Assume the collision is elastic and the time of collision is sufficient to redistribute charge amongst \beta\ \&\ \gamma.

Find the electric field acting on the plate \gamma before collision.

Ans:Net electric field at plate \gamma before collision is vector sum of electric field at plate \gamma due to plate \alpha and \beta.
The electric field at plate \gamma due to plate \alpha is \vec{\text{E}_1}=\frac{\text{Q}}{\text{S}(2\epsilon_0)}(-\hat{\text{i}}),
The electric field at palte \gamma due to plate \beta is \vec{\text{E}_2}=\frac{\text{q}}{\text{S}(2\epsilon_0)}(\hat{\text{i}}),
Hence, the net electric field at plate \gamma before collision is
\vec{\text{E}}=\vec{\text{E}_1}+\vec{\text{E}_2}=\frac{\text{q}-\text{Q}}{\text{S}(2\epsilon_0)}\hat{\text{(i)}}
\vec{\text{E}}=\vec{\text{E}_1}+\vec{\text{E}_2}=\frac{\text{Q}-\text{q}}{\text{S}(2\epsilon_0)}(-\hat{\text{i}})
\frac{\text{Q}-\text{q}}{\text{S}(2\epsilon_0)} to the left, if Q > q.
Q634. Two identical balls, each having a charge of 2.00 × 10-7C and a mass of 100g, are suspended from a common point by two insulating strings each 50cm long. The 6 Marks
balls are held at a separation 5.0cm apart and then released. Find,
1. The electric force on one of the charged balls.
https://bls.smartstudies.co.in/#/exam/pdf-preview/b5b87181-b79b-4b4a-8ebf-8f94fb605fc8/1 129/139
5/17/24, 8:14 PM Exam Automation
2. The components of the resultant force on it along and perpendicular to the string.
3. The tension in the string.
4. The acceleration of one of the balls. Answers are to be obtained only for the instant just after the release.

Ans:

\text{q}_1=\text{q}_2=2\times10^{-7}\text{c},\ \text{m}=100\text{g}
\text{l}=50\text{cm}=5\times10^{-2}\text{m},\ \text{d}=5\times10^{-2}\text{m}
1. Now Electric force
\text{F}=\text{K}\frac{\text{q}^2}{\text{r}^2}
=\frac{9\times10^9\times4\times10^{-14}}{25\times10^{-4}}\text{N}
=14.4\times10^{-2}\text{N}
=0.144\text{N}
2. The components of Resultant force along it is zero, because mg balances \text{T}\cos\theta and so also.
\text{F}=\text{mg}=\text{T}\sin\theta
3. Tension on the string
\text{T}\sin\theta=\text{F},\ \text{T}\cos\theta=\text{mg}
\text{Tan}\theta=\frac{\text{F}}{\text{mg}}
=\frac{0.144}{100\times10^{-3}\times9.8}
=0.14693
4. But \text{T}\cos\theta=10^2\times10^{-3}\times10=1\text{N}
\Rightarrow\text{T}=\frac{1}{\cos\theta}=\sec\theta
\Rightarrow\text{T}=\frac{\text{F}}{\sin\theta},
\sin\theta=0.145369
\cos\theta=0.989378
Q635. In 1959 Lyttleton and Bondi suggested that the expansion of the Universe could be explained if matter carried a net charge. Suppose that the Universe is made up 6 Marks
of hydrogen atoms with a number density N, which is maintained a constant. Let the charge on the proton be: ep = -(1 + y)e where e is the electronic charge.
Find the critical value of y such that expansion may start.

Ans:Let the Universe have a radius R. Assume that the hydrogen atoms are uniformly distributed. The Expansion starts if the Coulumb repulsion on a hydrogen atom, at
R, is larger than the gravitational attraction.
The hydrogen atom contains one proton and one electron, charge on each hydrogen atom.
eH = eP + e = -(1 + y)e + e = -ye = |ye|
Let E be elecrtric field intensity at distance R, on the surface of the sphere, the according to Gauss' theorem,
\oint\overrightarrow{\text{E}}.\text{d}\overrightarrow{\text{S}}=\frac{\text{q}_\text{enclosed}}{\epsilon_0}
\Rightarrow\ \text{E}(4\pi\text{R}^2)=\frac{4}{3}\frac{\pi\text{R}^3\text{N}|\text{ye}|}{\epsilon_0}
\Rightarrow\ \text{E}=\frac{1}{3}\frac{\text{N}|\text{ye}|\text{R}}{\epsilon_0}\ .....(\text{i})
Let us suppose the mass of each nydrogen atom = mp = Mass of a proton and GR = gravitational rield at distance R on the sphere.
Then -4\pi\text{R}^2\text{G}_\text{R}=4\pi\text{Gm}_\text{p}\Big(\frac{4}{3}\pi\text{R}^3\Big)\text{N}
\Rightarrow\ \text{G}_\text{R}=\frac{-4}{3}\pi\text{Gm}_\text{p}\text{NR}\ .....(\text{ii})
\therefore Gravitational force on this atom is
\text{F}_\text{G}=\text{m}_\text{p}\times\text{G}_\text{p}=\frac{-4\pi}{3}\text{Gm}_\text{p}^2\text{NR}\ .....(\text{iii})
Coulomb force on hydrogen atom at R is
\text{F}_\text{C}=(\text{ye})\text{E}=\frac{1}{3}\frac{\text{Ny}^2\text{e}^2\text{R}}{\epsilon_0}\ \ [\text{from Eq. (i)}]
Now, to start expansion FC > FG and aritical value of y to start expansion wolud be when,
FC = FG
\Rightarrow\ \frac{1}{3}\frac{\text{Ny}^2\text{e}^2\text{R}}{\epsilon_0}=\frac{4\pi}{3}\text{Gm}_\text{p}^2\text{NR}
\Rightarrow\ \text{y}^2=(4\pi\epsilon_0)\text{G}\Big(\frac{\text{m}_\text{p}}{\text{e}}\Big)^2
=\frac{1}{9\times10^9}\times(6.67\times10^{-11})
=\frac{1}{9\times10^9}\times(6.67\times10^{-11})\Big(\frac{(1.66\times10^{-27})^2}{(1.6\times10^{-19})^2}\Big)
\Rightarrow\ \text{y}=\sqrt{79.8\times10^{-38}}=8.9\times10^{-19}\simeq10^{-18}
Hence 10-18 is the required critical value of y corresponding to which expansion of universe would start.
Q636. A paisa coin is made up of Al-Mg alloy and weighs 0.75g. It has a square shape and its diagonal measures 17mm. It is electrically neutral and contains equal 6 Marks
amounts of positive and negative charges.
Treating the paisa coins made up of only Al, find the magnitude of equal number of positive and negative charges. What conclusion do you draw from this
magnitude?

Ans:1 Molar mass M of Al has NA = 6.023 × 1023 atoms.


\therefore m = mass of Al patsa coin has \text{N}=\frac{\text{N}_\text{A}}{\text{M}}\times\text{m }\text{atoms}
\therefore Number of aliminium atoms in one paisa coin,
\text{N}=\frac{6.023\times10^{23}}{26.9815}\times0.75=1.6742\times10^{22}
As charge number of AI is 13, each atom of AI contians 13 protons and 13 electrons.
\therefore Magnitude of positive and negative charges in one paisa coin = N.Ze
= 1.6782 × 1022 × 13 × 1.60 × 10-19C
= 3.48 × 104C = 34.8kC
This is an enormous amount of charge. Thus, we can conclude that ordinary neutral matter contains large amount of positive and negative charges.
Q637. Two conducting plates X and Y, each having large surface area A (on one side), are placed parallel to each other as shown in figure. The plate X is given a charge 6 Marks
Q whereas the other is neutral. Find:

https://bls.smartstudies.co.in/#/exam/pdf-preview/b5b87181-b79b-4b4a-8ebf-8f94fb605fc8/1 130/139
5/17/24, 8:14 PM Exam Automation

1. The surface charge density at the inner surface of the plate X.


2. The electric field at a point to the left of the plates.
3. The electric field at a point in between the plates.
4. The electric field at a point to the right of the plates.

Ans:1. Given that the charge present on the plate is Q. The other plate will get the same charge Q due to convection.
Let the surface charge densities on both sides of the plate be \sigma_1 and \sigma_2.
Now, electric field due to a plate,
\text{E}=\frac{\sigma}{2\epsilon_0}
So, the magnitudes of the electric fields due to this plate on each side =\frac{\sigma_1}{2\epsilon_0} and \frac{\sigma_2}{2\epsilon_0}
The plate has two sides, each of area A. So, the net charge given to the plate will be equally distributed on both the sides.This implies that the charge developed on
each side will be:
\text{q}_1=\text{q}_2=\frac{\text{Q}}{2}
This implies that the net surface charge density on each side =\frac{\text{Q}}{2\text{A}}
2. Electric field to the left of the plates
On the left side of the plate surface, charge density,
\sigma=\frac{\text{Q}}{2\text{A}}
Hence, electric field =\frac{\text{Q}}{2\text{A}\epsilon_0}
This must be directed towards the left, as 'X' is the positively-charged plate.
3. Here, the charged plate 'X' acts as the only source of electric field, with positive in the inner side. Plate Y is neutral. So, a negative charge will be induced on its
inner side. 'Y' attracts the charged particle towards itself. So, the middle portion E is towards the right and is equal to \frac{\text{Q}}{2\text{A}\epsilon_0}.
4. Similarly for the extreme right, the outer side of plate 'Y' acts as positive and hence it repels to the right with \text{E}=\frac{\text{Q}}{2\text{A}\epsilon_0}.
Q638. What will be the total flux through the faces of the cube (Fig.) with side of length a if a charge q is placed at: 6 Marks
1. A: a corner of the cube.
2. B: mid-point of an edge of the cube.
3. C: centre of a face of the cube.
4. D: mid-point of B and C.

Ans:1. Use of symmetry consideration may be useful in problems of flux calculation. We can imagine the charged particle is placed at the centre of a cube of side 2a.
We can observe that the charge is being shared equally by 8 cubes. Therefore, total flux through the faces of the given cube =\frac{\text{q}}{8\epsilon_0}.

2. If the charge q is placed at B, middle point of an edge of the cube, it is being shared equally by 4 cubes. Therefore, total flux through the faces of the given cube
=\frac{\text{q}}{4\epsilon_0}.

https://bls.smartstudies.co.in/#/exam/pdf-preview/b5b87181-b79b-4b4a-8ebf-8f94fb605fc8/1 131/139
5/17/24, 8:15 PM Exam Automation

3. If the charge q is placed at C, the centre of a face of the cube, it is being shared equally by 2 cubes. Therefore, total flux through the faces of the given
cube =\frac{\text{q}}{2\epsilon_0}.

4. Finally, if charge q is placed at D, the mid-point of B and C, it is being shared equally by 2 cubes. Therefore, total flux through the faces of the given
cube =\frac{\text{q}}{2\epsilon_0}.

Q639. Two charges -q each are fixed separated by distance 2d. A third charge q of mass m placed at the mid-point is displaced slightly by x (x < < d) perpendicular to 6 Marks
the line joining the two fixed charged as shown in Fig. Show that q will perform simple harmonic oscillation of time period.

\text{T}=\Big[\frac{8\pi^3\epsilon_0\text{md}^3}{\text{q}^2}\Big]^\frac{1}{2}.

Ans:Net force F on q towards the centre O


\text{F}=2\frac{\text{q}^2}{4\pi\epsilon_0\text{r}^2}\cos\theta=-\frac{2\text{q}^2}{4\pi\epsilon_0\text{r}^2}.\frac{\text{x}}{\text{r}}
\text{F}=-\frac{2\text{q}^2}{4\pi\epsilon_0}.\frac{\text{x}}{(\text{d}^2+\text{x}^2)\frac{3}{2}}
\approx-\frac{2\text{q}^2}{4\pi\epsilon_0\text{d}^3}\text{x}=-\text{k};\text{ for x}<<\text{d}
Thus, the force on the third charge q is proportional to the displacement and is towards the centre of the two other charges.
Therefore, the motion of the third charges is harmonic with frequency
\omega=\sqrt{\frac{\text{K}}{\text{m}}}
\Rightarrow\ \text{T}=\frac{2\pi}{\omega}=2\pi\sqrt{\frac{\text{K}}{\text{m}}}
\Rightarrow\ =2\pi\sqrt{\frac{\text{m}.4\pi\epsilon_0\text{d}^3}{2\text{q}^2}}=\Big[\frac{8\pi^3\epsilon_0\text{md}^3}{\text{q}^2}\Big]^\frac{1}{2}.
Q640. Two small spheres, each having a mass of 20g, are suspended from a common point by two insulating strings of length 40cm each. The spheres are identically 6 Marks
charged and the separation between the balls at equilibrium is found to be 4cm. Find the charge on each sphere.

Ans:

https://bls.smartstudies.co.in/#/exam/pdf-preview/b5b87181-b79b-4b4a-8ebf-8f94fb605fc8/1 132/139
5/17/24, 8:15 PM Exam Automation

\text{T}\cos\theta=\text{mg}\ \dots(1)
\text{T}\sin\theta=\text{Fe}\ \dots(2)
Solving, \frac{(2)}{(1)} we get, \tan\theta=\frac{\text{Fe}}{\text{mg}}
=\frac{\text{kq}^2}{\text{r}}\times\frac{1}{\text{mg}}
\Rightarrow\frac{2}{\sqrt{1596}}=\frac{9\times10^9\times\text{q}^2}{(0.04)^2\times0.02\times9.8}
\Rightarrow\text{q}^2=\frac{(0.04)^2\times0.02\times9.8\times2}{9\times10^9\times\sqrt{1596}}
=\frac{6.27\times10^{-4}}{9\times10^9\times39.95}
=17\times10^{-16}\text{c}^2
\Rightarrow\text{q}=\sqrt{17\times10^{-16}}
=4.123\times10^{-8}\text{c}
Q641. Total charge -Q is uniformly spread along length of a ring of radius R. A. small test charge +q of mass m is kept at the centre of the ring and is given a gentle push 6 Marks
along the axis of the ring.
1. Show that the particle executes a simple harmonic oscillation.
2. Obtain its time period.

Ans:Let the charge q is dispaced slightly by z (z < < R) along the axis of ring. Let force on the charge q be towards O. The motion of charge q to be simple harmonic, if
the force on charge q must be proportional to Z and is directed towads O.
Electric field at axis of the rign at a distance Z from the centre of ring

\text{E}=\frac{1}{4\pi\epsilon_0}\frac{\text{Qz}}{(\text{R}^2+\text{z}^2)^\frac{3}{2}};\text{towards O}
Net force on the charge Fnet = qE
\text{F}_\text{net}=\frac{1}{4\pi\epsilon_0}\frac{\text{qQz}}{(\text{R}^2+\text{z}^2)^\frac{3}{2}}
\Rightarrow\ \text{F}_\text{net}=\frac{1}{4\pi\epsilon_0}\frac{\text{qQz}}{\text{R}^3\Big(\frac{1+\text{z}^2}{\text{R}^2}\Big)^\frac{3}{2}}
As z < < R then, \text{F}_\text{net}=\frac{1}{4\pi\epsilon_0}\frac{\text{qQz}}{\text{R}^3}
or \vec{\text{F}}_\text{Net}=-\text{K}\vec{\text{z}}
Where \text{K}=\frac{\text{Qq}}{4\pi\epsilon_0\text{R}^3}=\text{constant}
Clearly, force on q is proportional to negative of its displacement. therefore, motion of q is simlpe harmonic.
\text{W}=\sqrt{\frac{\text{K}}{\text{m}}}\text{ and T}=\frac{2\pi}{\omega}=2\pi\sqrt{\frac{\text{m}}{\text{K}}}
\text{T}=2\pi\sqrt{\frac{\text{m}4\pi\epsilon_0\text{R}^3}{\text{Qq}}}
\Rightarrow\ \text{T}=2\pi\sqrt{\frac{\text{m}4\pi\epsilon_0\text{R}^3}{\text{Qq}}}
Q642. Two particles A and B, each having a charge Q, are placed a distance d apart. Where should a particle of charge q be placed on the perpendicular bisector of AB 6 Marks
so that it experiences maximum force What is the magnitude of this maximum force?

Ans:

https://bls.smartstudies.co.in/#/exam/pdf-preview/b5b87181-b79b-4b4a-8ebf-8f94fb605fc8/1 133/139
5/17/24, 8:15 PM Exam Automation

Force on the charge particle ‘q’ at ‘c’ is only the x component of 2 forces
So, \text{F}_\text{on c}=\text{F}_\text{CB}\sin\theta+\text{F}_\text{AC}\sin\theta
But |\bar{\text{F}}_\text{CB}|=|\bar{\text{F}}_\text{AC}|
2\text{F}_\text{CB}\sin\theta
=2\frac{\text{KQq}}{\text{x}^2+\Big(\frac{\text{d}}{2}\Big)^2}\times\frac{\text{x}}{\Big[\frac{\text{x}^2+\text{d}^2}{4}\Big]^{\frac{1}{2}}}
=\frac{2\text{k}\theta\text{qx}}{\Big(\frac{\text{x}^2+\text{d}^2}{4}\Big)^{\frac{3}{2}}}
=\frac{16\text{kQq}}{\big(4\text{x}^2+\text{d}^2\big)^{\frac{3}{2}}}\text{x}
For maximum force \frac{\text{dF}}{\text{dx}}=0
\frac{\text{d}}{\text{dx}}\Bigg(\frac{16\text{kQqx}}{\big(4\text{x}^2+\text{d}^2\big)^{\frac{3}{2}}}\Bigg)=0
\Rightarrow\text{K}\begin{bmatrix}\frac{(4\text{x}^2+\text{d}^2)-\text{x}\bigg[\frac{3}{2\big[4\text{x}^2+\text{d}^2\big]^{\frac{1}{2}}}8\text{x}\bigg]}
{\big[4\text{x}^2+\text{d}^2\big]^3}\end{bmatrix}=0
\Rightarrow\frac{\text{K}\big(4\text{x}^2+\text{d}^2\big)^{\frac{1}{2}}\Big[(4\text{x}^2+\text{d}^2)^3-12\text{x}^2\Big]}{\big(4\text{x}^2+\text{d}^2\big)^3}=0
\Rightarrow\big(4\text{x}^2+\text{d}^2\big)^3=12\text{x}^2
\Rightarrow16\text{x}^4+\text{d}^4+8\text{x}^2\text{d}^2=12\text{x}^2 [\text{d}^4+8\text{x}^2\text{d}^2=0]
\Rightarrow\text{d}^2=0 [\text{d}^2+8\text{x}^2=0]
\Rightarrow\text{d}^2=8\text{x}^2
\text{d}=\frac{\text{d}}{2\sqrt{2}}
Q643. There is another useful system of units, besides the SI/mks A system, called the cgs (centimeter-gram-second) system. In this system Coloumb’s law is given by 6 Marks
\text{F}=\frac{\text{Qq}}{\text{r}^2}\hat{\text{r}}
where the distance r is measured in cm (= 10-2m), F in dynes (=10-5N) and the charges in electrostatic units (es units), where
les unit of charge =\frac{1}{[3]}\times10^{-9}\text{C}
The number [3] actually arises from the speed of light in vaccum which is now taken to be exactly given by c = 2.99792458 × 108 m/s. An approximate value of c
then is c = [3] × 108 m/s.
1. Show that the coloumb law in cgs units yields
1 esu of charge = 1 (dyne)1/2 cm.
Obtain the dimensions of units of charge in terms of mass M, length L and time T. Show that it is given in terms of fractional powers of M and L.
2. Write 1 esu of charge = x C, where x is a dimensionless number. Show that this gives
\frac{1}{4\pi\epsilon_0}=\frac{10^{-9}}{\text{x}^2}\frac{\text{N.m}^2}{\text{C}^2}
With \text{x}=\frac{1}{[3]}\times10^{-9}, we have
\frac{1}{4\pi\epsilon_0}=[3]^2\times10^9\frac{\text{Nm}^2}{\text{C}^2}
Or, \frac{1}{4\pi\epsilon_0}=(2.99792458)^2\times10^{9}\frac{\text{Nm}^2}{\text{C}^2}\text{(exactly).}

Ans:1. \text{F}=\frac{\text{Qq}}{\text{r}^2}=1\text{dyne}=\frac{[1\text{esu of charge}]^2}{[1\text{cm}]^2}


1 esu of charge =1\text{(dyne)}^\frac{1}{2}\text{(cm)}
Hence, \text{[1 esu of charge]}=\text{[F]}^\frac{1}{2}\text{L}=\text{[MLT}^2]^\frac{1}{2}\text{L}=\text{M}^\frac{1}{2}\text{L}^\frac{3}{2}\text{T}^{-1}
\text{[1 esu of charge]}=\text{M}^\frac{1}{2}\text{L}^\frac{3}{2}\text{T}^{-1}
Thus charge in egs units is expressed as fractional powers \Big(\frac{1}{2}\Big) of M and \Big(\frac{3}{2}\Big) of L.
2. Consider the coloumb force on two charges, each of magnitude 1 esu of charge separated by a distance of 1 cm:
The force is then 1 dyne = 10-5N.
This situation is equivalent to two charges of magnitude x C separated by 10-2m.
This given:
\text{F}=\frac{1}{4\pi\epsilon_0}\frac{\text{x}^2}{10^{-4}}
which should be 1 dyne = 10-5N. thus
\frac{1}{4\pi\epsilon_0}\frac{\text{x}^2}{10^{-4}}=10^{-5}\Rightarrow\ \frac{1}{4\pi\epsilon_0}\frac{10^{-9}}{\text{x}^2}\frac{\text{Nm}^2}{\text{C}^2}
With \text{x}=\frac{1}{[3]\times10^9}, this yields
\frac{1}{4\pi\epsilon_0}=10^{-9}\times[3]^2\times10^{18}=[3]^2\times10^9\frac{\text{Nm}^2}{\text{C}^2}
With [3]\rightarrow2.99792458, we have
\frac{1}{4\pi\epsilon_0}=8.98755\ ...\times10^9\frac{\text{Nm}^2}{\text{C}^2} exactly.
Q644. Two particles A and B having charges q and 2q respectively are placed on a smooth table with a separation d. A third particle C is to be clamped on the table in 6 Marks
such a way that the particles A and B remain at rest on the table under electrical forces. What should be the charge on C and where should it be clamped?

Ans:

https://bls.smartstudies.co.in/#/exam/pdf-preview/b5b87181-b79b-4b4a-8ebf-8f94fb605fc8/1 134/139
5/17/24, 8:15 PM Exam Automation

Let the charge on C = q


So, net force on c is equal to zero
So, \text{F}_{\overline{\text{AC}}}+\text{F}_{\overline{\text{BA}}}=0,
But \text{F}_{\overline{\text{AC}}}=\text{F}_{\overline{\text{BC}}}
\Rightarrow\frac{\text{kqQ}}{\text{x}^2}=\frac{\text{k}2\text{qQ}}{(\text{d}-\text{x})^2}
\Rightarrow2\text{x}^2=(\text{d}-\text{x})^2
\Rightarrow\sqrt{2}\text{x}=\text{d}-\text{x}
\Rightarrow\text{x}=\frac{\text{d}}{\sqrt{2}+1}
=\frac{\text{d}}{\big(\sqrt{2}+1\big)}\times\frac{\big(\sqrt{2}-1\big)}{\big(\sqrt{2}-1\big)}
=\text{d}\big(\sqrt{2}-1\big)
For the charge on rest, \text{F}_\text{AC}+\text{F}_\text{AB}=0
(2.414)^2\frac{\text{kqQ}}{\text{d}^2}+\frac{\text{kq}(2\text{q})}{\text{d}^2}=0
\Rightarrow\frac{\text{kq}}{\text{d}^2}\big[(2.414)^2\text{Q}+2\text{q}\big]=0
\Rightarrow2\text{q}=-(2.414)^2\text{Q}
\Rightarrow\text{Q}=\frac{2}{-\big(\sqrt{2}+1\big)^2}\text{q}
=-\Big(\frac{2}{3+2\sqrt{2}}\Big)\text{q}
=-(0.343)\text{q}
=-\big(6-4\sqrt{2}\big)
Q645. Suppose all the electrons of 100g water are lumped together to form a negatively charged particle and all the nuclei are lumped together to form a positively 6 Marks
charged particle. If these two particles are placed 10.0cm away from each other, find the force of attraction between them. Compare it with your weight.

Ans:Molecular weight of \text{H}_2\text{O}=2\times1\times16=16


No. of electrons present in one molecule of \text{H}_2\text{O}=10
18gm of H2O has 6.023\times1023\times10\text{ electrons}
100gm of H2O has \frac{6.023\times10^{-24}}{18}\times100\text{ electrons}
So number of protons =\frac{6.023\times10^{26}}{18}\text{protons} (since atom is electrically neutral)
Charge of protons =\frac{1.6\times10^{-19}\times6.023\times10^{26}}{18}\text{coulomb}=\frac{1.6\times6.023\times10^7}{18}\text{coulomb}
Charge of electrons =\frac{1.6\times6.023\times10^7}{18}\text{coulomb}
Hence Electrical force =\frac{9\times10^9\Big(\frac{1.6\times6.023\times10^7}{18}\Big)\times\Big(\frac{1.6\times6.023\times10^7}{18}\Big)}{(10\times10^{-2})^2}
=\frac{8\times6.023}{18}\times16.\times6.023^{25}
=2.56\times10^{25}\text{ Newton}
Q646. A uniform electric field \vec{\text{E}}=\text{E}_\text{x}\hat{\text{i}} N/ C for x > 0 and \vec{\text{E}}=-\text{E}_\text{x}\hat{\text{i}} N/ C for x < 0 are given. A right 6 Marks
circular cylinder of length l cm and radius r cm has its centre at the origin and its axis along the X-axis. Find out the net outward flux. Using Gauss’s law, write the
expression for the net charge within the cylinder.

Ans:Electric flux through flat surface S1,


\Phi_1=\oint\limits_{\text{S}_1}\vec{\text{E}}_1.\vec{\text{dS}}_1=\oint\limits_{\text{S}_1}(\text{E}_\text{x}\hat{\text{i}}).
(\text{dS}_1\hat{\text{i}})=\text{E}_\text{x}\text{S}_1
Electric flux through flat surface S2,
\Phi_2=\oint\limits_{\text{S}_2}\vec{\text{E}}_2.\vec{\text{dS}}_2=\oint\limits_{\text{S}_2}(-\text{E}_\text{x}\hat{\text{i}}).(-
\text{dS}_1\hat{\text{i}})=\oint\limits_{\text{S}_2}\text{E}_\text{x}\text{dS}_2
=\text{E}_\text{x}\text{S}_2
Electric flux through curved surface S3,
\Phi_3=\oint\limits_{\text{S}_3}\Big(\vec{\text{E}}_3.\vec{\text{dS}}_3\Big)=\oint\limits_{\text{S}_3}\text{E}_3\text{dS}_3\cos90^\circ=0
\therefore Net electric flux, \Phi=\Phi_1+\Phi_2=\text{E}_\text{x}(\text{S}_1+\text{S}_2)
But \text{S}_1=\text{S}_2=\pi(\text{r}\times10^{-2})^2\text{m}^2=\pi\text{r}^2\times10^{-4}\text{m}^2
\therefore\ \Phi=\text{E}_\text{x}2(\pi\text{r}^2\times10^{-4})\text{ units}
By Gauss's law, \Phi=\frac{1}{\epsilon_0}\text{q}
\text{q}=\epsilon_0\Phi=\epsilon_0\text{E}_\text{x}(2\pi\text{r}^2\times10^{-4})
=2\pi\epsilon_0\text{E}_\text{x}\text{r}^2\times10^{-4}=4\pi\epsilon_0\Big(\frac{\text{E}_\text{x}\text{r}^2\times10^{-4}}{2}\Big)
=\frac{1}{9\times10^9}\Big[\frac{\text{E}_\text{x}\text{r}^2\times10^{-4}}{2}\Big]
=5.56\text{E}_\text{x}\text{r}^2\times10^{-11} coulomb.

Q647. Consider the following very rough model of a beryllium atom. The nucleus has four protons and four neutrons confined to a small volume of radius 10-15m. The two 6 Marks
1s electrons make a spherical charge cloud at an average distance of 1.3 × 10-11m from the nucleus, whereas the two 2s electrons make another spherical cloud at
an average distance of 5.2 × 10-11m from the nucleus. Find the electric field at:
1. A point just inside the 1s cloud.
2. A point just inside the 2s cloud.

https://bls.smartstudies.co.in/#/exam/pdf-preview/b5b87181-b79b-4b4a-8ebf-8f94fb605fc8/1 135/139
5/17/24, 8:15 PM Exam Automation
Ans:1. Let us consider the three surfaces as three concentric spheres A, B and C.
Let us take q = 1.6 × 10-19C.
Sphere A is the nucleus; so, the charge on sphere A, q1 = 4q
Sphere B is the sphere enclosing the nucleus and the 2 1s electrons; so charge on this sphere, q2 = 4q - 2q = 2q.
Sphere C is the sphere enclosing the nucleus and the 4 electrons of Be; so, the charge enclosed by this sphere, q3 = 4q - 4q = 0.
Radius of sphere A, r1 = 10-15m
Radius of sphere B, r2 = 1.3 × 10-11m
Radius of sphere C, r3 = 5.2 × 10-11m
As the point 'P' is just inside the spherical cloud 1s, its distance from the centre
x = 1.3 × 10-11m
Electric field,
\text{E}=\frac{\text{q}}{4\pi\epsilon_0\text{x}^2}
Here, the charge enclosed is due to the charge of the 4 protons inside the nucleus. So,
\text{E}=\frac{4\times\big(1.6\times10^{-19}\big)}{4\times3.14\times\big(8.85\times10^{-12}\big)\times\big(1.3\times10^{-11}\big)^2}
\text{E}=3.4\times10^{13}\text{N/C}
2. For a point just inside the 2s cloud, the total charge enclosed will be due to the 4 protons and 2 electrons. Charge enclosed,
qen = 2q = 2 × (1.6 × 10-19)C
Hence, electric field,
\text{E}=\frac{\text{q}_{\text{en}}}{4\pi\epsilon_0\text{x}^2}
\text{x}=5.2\times10^{-11}\text{m}
\text{E}=\frac{2\times\big(1.6\times10^{-19}\big)}{4\times3.14\times\big(8.5\times10^{-12}\big)\times\big(5.2\times10^{-11}\big)^2}
\text{E}=1.065\times\text{10}^{12}\text{N/C}
Thus, \text{E}=1.1\times\text{10}^{12}\text{N/C}
Q648. Consider a coin of Example 1.20. It is electrically neutral and contains equal amounts of positive and negative charge of magnitude 34.8kC. Suppose that these 6 Marks
equal charges were concentrated in two point charges seperated by,
1. 1cm \Big(\sim\frac{1}{2}\times\text{diagonalof theone paisa coin}\Big),
2. 100m (~ length of a long building), and
3. 106m (radius of the earth). Find the force on each such point charge in each of the three cases. What do you conclude from these results?

Ans:We know force between two point charges separated at a distance r,


\text{F}=\frac{|\text{q}|^2}{4\pi\epsilon_0\text{r}^2}
Here, \text{q}=\pm34.8\text{kC}=\pm3.48\times10^4\text{C}
\text{r}_1=1\text{cm}=10^{-2}\text{m},\text{r}_2=100\text{m},\text{r}_3=10^6\text{m}
1. \text{F}_1=\frac{|\text{q}|^2}{4\pi\epsilon_0\text{r}_1^2}=\frac{9\times10^9(3.48\times10^4)^2}{(10^{-2})^2}=1.09\times10^{23}\text{N}
2. \text{F}_2=\frac{|\text{q}|^2}{4\pi\epsilon_0\text{r}_2^2}=\frac{9\times10^9(3.48\times10^4)^2}{(100)^2}=1.09\times10^{15}\text{N}
3. \text{F}_3=\frac{|\text{q}|^2}{4\pi\epsilon_0\text{r}_3^2}=\frac{9\times10^9(3.48\times10^4)^2}{(10^6)^2}=1.09\times10^{7}\text{N}
Conclusion: Here we can observe that when positive and negative charges in ordinary neutral matter are separated as point charges, they exert very large force. It
means, it is very difficult to disturb electrical neutrality of matter.
Q649. Two fixed, identical conducting plates (\alpha\ \&\ \beta), each of surface area S are charged to -Q and q, respectively, where Q > q > 0. A third identical plate 6 Marks
(\gamma), free to move is located on the other side of the plate with charge q at a distance d (Fig.). The third plate is released and collides with the plate \beta.
Assume the collision is elastic and the time of collision is sufficient to redistribute charge amongst \beta\ \&\ \gamma.

Find the velocity of the plate \gamma after the collision and at a distance d from the plate \beta.

Ans:Let the velocity be v at the distance d from plate \beta after the collision. If m is the mass of the palte \gamma, then the gain in K.E. over the round trip must be
equal to the work done by the electric field.
After the collision, electric field at plate \gamma is
\vec{\text{E}_2}=\frac{\text{Q}}{2\epsilon_0\text{S}}(-\hat{\text{i}})+\frac{\Big(\text{Q}+\frac{\text{q}}{2}\Big)}
{2\epsilon_0\text{S}}\hat{\text{i}}=\frac{\frac{\text{q}}{2}}{2\epsilon_0\text{S}}\hat{\text{i}}
Just befcore collision, electric field at palte \gamma is \vec{\text{E}_1}=\frac{\text{Q}-\text{q}}{2\epsilon_0\text{S}}\hat{\text{i}}
If F1 is force on plate \gamma before collision, then \vec{\text{E}_1}=\vec{\text{E}_1}\text{Q}=\frac{(\text{Q}-\text{q})\text{Q}}{2\epsilon_0\text{S}}\hat{\text{i}}
And \vec{\text{F}_2}=\vec{\text{E}_2}\frac{\text{q}}{2}=\frac{\Big(\frac{\text{q}}{2}\Big)^2}{2\epsilon_0\text{S}}\hat{\text{i}}
Total work done by the elecric field is round trip movement of plate \gamma.
\text{W}=(\text{F}_1+\text{F}_2)\text{d}
=\frac{\Big[(\text{Q}-\text{q})\text{Q}+\Big(\frac{\text{q}}{2}\Big)^2\Big]\text{d}}{2\epsilon_0\text{S}}=\frac{\Big(\text{Q}-\frac{\text{q}}{2}^2\text{d}\Big)}
{2\epsilon_0\text{S}}
If m is the mass of plate \gamma the KE gained by the plate =\frac{1}{2}\text{mv}^2
According to work-energy priniciple, =\frac{1}{2}\text{mv}^2=\text{W}\Rightarrow\ \frac{1}{2}\text{mv}^2=\frac{\Big(\text{Q}-\frac{\text{q}}{2}\Big)^2\text{d}}
{2\epsilon_0\text{S}}
\Rightarrow\ \text{v}=\Big(\text{Q}-\frac{\text{q}}{2}\Big)\Big(\frac{\text{d}}{\text{m}\epsilon_0\text{S}}\Big)^\frac{1}{2}
Q650. A charge is distributed uniformly over a ring of radius ‘a’. Obtain an expression for the electric intensity E at a point on the axis of the ring. Hence show that for 6 Marks
point’s at large distances from the ring, it behaves like a point charge.

Ans:Image
Consider a point P on the axis of uniformly charged ring at a distance x from its centre O. Point P is at distance \text{r}=\sqrt{\text{a}^2+\text{x}^2} from each
element dl of ring. If q is total charge on ring, then, charge per metre length, \lambda=\frac{\text{q}}{2\pi\text{a}}.
The ring may be supposed to be formed of a large number of ring elements.
Consider an element of length dl situated at A.
The charge on element, \text{dp}=\lambda\text{dl}
\therefore The electric field at P due to this element,

https://bls.smartstudies.co.in/#/exam/pdf-preview/b5b87181-b79b-4b4a-8ebf-8f94fb605fc8/1 136/139
5/17/24, 8:15 PM Exam Automation
\vec{\text{dE}}_1=\frac{1}{4\pi\epsilon_0}\frac{\text{dq}}{\text{r}^2}=\frac{1}{4\pi\epsilon_0}\frac{\lambda\text{dl}}{\text{r}^2},\ \text{along}\ \vec{\text{PC}}
The electric field strength due to opposite symmetrical element of length dl at B is,
\vec{\text{dE}}_2=\frac{1}{4\pi\epsilon_0}\frac{\text{dq}}{\text{r}^2}=\frac{1}{4\pi\epsilon_0}\frac{\lambda\text{dl}}{\text{r}^2},\ \text{along}\ \vec{\text{PD}}
If we resolve \vec{\text{dE}}_1 and \vec{\text{dE}}_2 along the axis and perpendicular to axis, we note that the components perpendicular to axis are oppositely
directed and so get cancelled, while those along the axis are added up. Hence, due to symmetry of the ring, the electric field strength is directed along the axis.
The electric field strength due to charge element of length dl, situated at A, along the axis will be,
\text{dE}=\text{dE}_1\cos\theta=\frac{1}{4\pi\epsilon_0}\frac{\lambda\text{dl}}{\text{r}^2}\cos\theta
But, \cos\theta=\frac{\text{x}}{\text{r}}
\therefore\ \text{dE}=\frac{1}{4\pi\epsilon_0}\frac{\lambda\text{dl x}}{\text{r}^3}=\frac{1}{4\pi\epsilon_0}\frac{\lambda\text{x}}{\text{r}^3}\text{dl}
The resultant electric field along the axis will be obtained by adding fields due to all elements of the ring, i.e.,
\therefore\ \text{E}=\int\frac{1}{4\pi\epsilon_0}\frac{\lambda\text{x}}{\text{r}^3}\text{dl}
=\frac{1}{4\pi\epsilon_0}\frac{\lambda\text{x}}{\text{r}^3}\int\text{dl}
But, \int\text{dl}= whole length of ring =2\pi\text{a} and \text{r}=(\text{a}^2+\text{x}^2)^{1/2}
\therefore\ \text{E}=\frac{1}{4\pi\epsilon_0}\frac{\lambda\text{x}}{(\text{a}^2+\text{x}^2)^{3/2}}2\pi\text{a}
As, \lambda=\frac{\text{q}}{2\pi\text{a}}, we have \text{E}=\frac{1}{4\pi\epsilon_0}\frac{\big(\frac{\text{q}}{2\pi\text{a}}\big)\text{x}}
{(\text{a}^2+\text{x}^2)^{3/2}}2\pi\text{}\text{a}
\text{E}=\frac{1}{4\pi\epsilon_0}\frac{\text{qx}}{(\text{a}^2+\text{x}^2)^{3/2}}
or, \text{E}=\frac{1}{4\pi\epsilon_0}\frac{\text{qx}}{(\text{a}^2+\text{x}^2)^{3/2}}, along the axis
At large distances i.e., x >> a, \text{E}=\frac{1}{4\pi\epsilon_0}\frac{\text{q}}{\text{x}^2}
i.e., the electric field due to a point charge at a distance x.
For points on the axis at distances much larger than the radius of ring, the ring behaves like a point charge.
Q651. Consider a system of n charges q1, q2, ... qn with position vectors \vec{\text{r}}_1,\vec{\text{r}}_2,\vec{\text{r}}_3,...\vec{\text{r}}_\text{n} relative to some origin ‘O’. 7 Marks
Deduce the expression for the net electric field \vec{\text{E}} at a point P with position vector \vec{\text{r}}_\text{p}, due to this system of charges.

Ans:

Electric field due to a system of point charges.


Consider a system of N point charges q1,q2,.....qn, having position vectors \vec{\text{r}}_1,\vec{\text{r}}_2,\vec{\text{r}}_3,...\vec{\text{r}}_\text{n}, with respect to
origin O. We wish to determine the electric field at point P whose position vector is \vec{\text{r}}.
According to Coulomb’s law, the force on charge q0 due to charge q1 is,
\vec{\text{F}}_1=\frac{1}{4\pi\epsilon_0}.\frac{\text{q}_1\text{q}_0}{\text{r}^2_{2}\text{p}}\hat{\text{r}}_{1}\text{p}
Where \hat{\text{r}}_1\text{p} is a unit vector in the direction from q1 to P and r1p is the distance between q1 and P.
Hence the electric field at point P due to charge q1 is,
\vec{\text{E}}_1=\frac{\vec{\text{F}}_1}{\text{q}_0}=\frac{1}{4\pi\epsilon_0}.\frac{\text{q}_1}{\text{r}^2_{1}\text{p}}\hat{\text{r}}_{1}\text{p}
Similarly, electric field at P due to charge q2 is,
\vec{\text{E}}_2=\frac{1}{4\pi\epsilon_0}.\frac{\text{q}_2}{\text{r}^2_{2}\text{p}}\hat{\text{r}}_{2}\text{p}
According to the principle of superposition of electric fields, the electric field at any point due to a group of point charges is equal to the vector sum of the electric
fields produced by each charge individually at that point, when all other charges are assumed to be absent.
Hence, the electric field at point P due to the system of N charges is
\vec{\text{E}}=\vec{\text{E}}_1+\vec{\text{E}}_2+\dots+\vec{\text{E}}_\text{n}
=\frac{1}{4\pi\epsilon_0}\Big[\frac{\text{q}_1}{\text{r}^2_{1}\text{p}}\hat{\text{r}}_{1}\text{p}+\frac{\text{q}_2}
{\text{r}^2_{2}\text{p}}\hat{\text{r}}_{2}\text{p}+\dots+\frac{\text{q}_\text{n}}{\text{r}^2_{\text{n}}\text{p}}\hat{\text{r}}_{\text{n}}\text{p}\Big]
=\frac{1}{4\pi\epsilon_0}\Sigma^{\text{n}}_\text{i=1}\frac{\text{q}_\text{i}}{\text{r}^2_{\text{i}}\text{p}}\hat{\text{r}}_\text{i}\text{p}
Q652. Find an expression for the electric field strength at a distant point situated (i) on the axis and (ii) along the equatorial line of an electric dipole. 8 Marks
OR
Derive an expression for the electric field intensity at a point on the equatorial line of an electric dipole of dipole moment \vec{\text{p}} and length 2a. What is the
direction of this field?

Ans:Consider an electric dipole AB. The charges -q and +q of dipole are situated at A and B respectively as shown in the figure. The separation between the charges is
2a.
Electric dipole moment, p = q.2a
The direction of dipole moment is from -q to +q.
Image
1. At axial or end-on position: Consider a point P on the axis of dipole at a distance r from mid-point O of electric dipole.
The distance of point P from charge +q at B is BP = r - a.
And distance of point P from charge -q at A is, AP = r + a.
Let E1 and E2 be the electric field strengths at point P due to charges +q and -q respectively.
We know that the direction of electric field due to a point charge is away from positive charge and towards the negative charge. Therefore,
\text{E}_1=\frac{1}{4\pi\epsilon_0}\frac{\text{q}}{(\text{r}-\text{a})^2} (from B to P)
and \text{E}_2=\frac{1}{4\pi\epsilon_0}\frac{\text{q}}{(\text{r}+\text{a})^2} (from P to A)
Clearly the directions of electric field strengths \vec{\text{E}}_1 and \vec{\text{E}}_2 along the same line but opposite to each other and E1 > E2 because positive
charge is nearer.
\therefore The resultant electric field due to electric dipole has magnitude equal to the difference of E1 and E2 direction from B to P i.e.,
E = E 1 - E2
=\frac{1}{4\pi\epsilon_0}\frac{\text{q}}{(\text{r}-\text{a})^2}-​​\frac{1}{4\pi\epsilon_0}\frac{\text{q}}{(\text{r}+\text{a})^2}

https://bls.smartstudies.co.in/#/exam/pdf-preview/b5b87181-b79b-4b4a-8ebf-8f94fb605fc8/1 137/139
5/17/24, 8:15 PM Exam Automation
=\frac{\text{q}}{4\pi\epsilon_0}=\Big[\frac{1}{(\text{r}-\text{a})^2}-\frac{1}{(\text{r}+\text{a})^2}\Big]=\frac{\text{q}}
{4\pi\epsilon_0}\bigg[\frac{(\text{r}+\text{a})^2-(\text{r}-\text{a})^2}{(\text{r}^2-\text{a}^2)^2}\bigg]
=\frac{\text{q}}{4\pi\epsilon_0}\frac{4\text{ra}}{(\text{r}^2-\text{a}^2)^2}=\frac{1}{4\pi\epsilon_0}\frac{2(\text{q}2\text{a})\text{r}}{(\text{r}^2-\text{a}^2)^2}
But q.21 = p (electric dipole moment)
\therefore\ \text{E}=\frac{1}{4\pi\epsilon_0}\frac{2\text{pr}}{(\text{r}^2-\text{a}^2)^2}\dots(\text{i})
If the dipole is infinitely small and point P is far away from the dipole, then r >> l, therefore equation (i) may be expressed as,
\text{E}=\frac{1}{4\pi\epsilon_0}\frac{2\text{pr}}{\text{r}^4} or \text{E}=\frac{1}{4\pi\epsilon_0}\frac{2\text{p}}{\text{r}^3}\dots(\text{ii})
This is the expression for the electric field strength at axial position due to a short electric dipole.
2. At a point of equatorial line: Consider a point P on broad side on the position of dipole formed of charges +q and -q at separation 2a. The distance of point P
from mid-point (O) of electric dipole is r. Let \vec{\text{E}}_1 and \vec{\text{E}}_2 be the electric field strengths due to charges +q and -q of electric dipole.
From fig. \text{AP}=\text{BP}=\sqrt{\text{r}^2+\text{a}^2}
\therefore\ \vec{\text{E}}=\frac{1}{4\pi\epsilon_0}\frac{\text{q}}{\text{r}^2+\text{a}^2} along B to P
\vec{\text{E}}_2=\frac{1}{4\pi\epsilon_0}\frac{\text{q}}{\text{r}^2+\text{a}^2} along P to A
Clearly \vec{\text{E}}_1 and \vec{\text{E}}_2 are equal in magnitude i.e., |\vec{\text{E}}_1|=|\vec{\text{E}}_2| or E1 = E2
To find the resultant of \vec{\text{E}}_1 and \vec{\text{E}}_2, we reslove them into rectangular components.
Component of \vec{\text{E}}_1 Parallel to ​\text{AB}=\text{E}_1\cos\theta, in the direction to \vec{\text{BA}}
Component of \vec{\text{E}}_1 perpendicular to ​\text{AB}=\text{E}_1\sin\theta, along OP
Component of \vec{\text{E}}_2 Parallel to ​\text{AB}=\text{E}_2\cos\theta, in the direction to \vec{\text{BA}}
Component of \vec{\text{E}}_2 perpendicular to ​\text{AB}=\text{E}_2\sin\theta, along PO
Clearly, comoponents of \vec{\text{E}}_1 and \vec{\text{E}}_2 perpendicular to ​\text{AB}=\text{E}_1\sin\theta and \text{E}_2\sin\theta being equal and opposite
cancel each other, while the components of \vec{\text{E}}_1 and \vec{\text{E}}_2 parallel to ​\text{AB}=\text{E}_1\cos\theta and \text{E}_2\cos\theta, being in the
same direction add up and give the resultant electric field whose direction is parallel to \vec{\text{BA}}.
\therefore Resultant electric field at P is ​\text{E}=\text{E}_1\cos\theta+\text{E}_2\cos\theta
But \text{E}_1=\text{E}_2=\frac{1}{4\pi\epsilon_0}\frac{\text{q}}{(\text{r}^2+\text{a}^2)}
From the figure, \cos\theta=\frac{\text{OB}}{\text{PB}}=\frac{\text{l}}{\sqrt{\text{r}^2+\text{a}^2}}=\frac{\text{l}}{(\text{r}^2+\text{a}^2)^{1/2}}
\text{E}=2\text{E}_1\cos\theta=2\times\frac{1}{4\pi\epsilon_0}\frac{\text{q}}{(\text{r}^2+\text{a}^2)}.\frac{\text{l}}{(\text{r}^2+\text{a}^2)^{1/2}}
=\frac{1}{4\pi\epsilon_0}\frac{2\text{ql}}{(\text{r}^2+\text{a}^2)^{3/2}}
But q.2l = p = electric dipole moment ...(iii)
\therefore\ \text{E}=\frac{1}{4\pi\epsilon_0}\frac{\text{p}}{(\text{r}^2+\text{a}^2)^{3/2}}
If dipole is infinitesimal and point P is far away, we have a << r, so l2 may be neglected as compared to r2 and so equation (iii) gives,
\text{E}=\frac{1}{4\pi\epsilon_0}\frac{\text{p}}{(\text{r}^2)^{3/2}}=\frac{1}{4\pi\epsilon_0}\frac{\text{p}}{\text{r}^3}
i.e., electric field strength due to a short dipole at broadside on position.
\text{E}=\frac{1}{4\pi\epsilon_0}\frac{\text{p}}{\text{r}^3} in the direction parallel to \vec{\text{BA}}\dots(\text{iv})
Its direction is parallel to the axis of dipole from positive to negative charge. It may be noted clearly from equations (ii) and (iv) that electric field strength due to a
short dipole at any point is inversely proportional to the cube of its distance from the dipole and the electric field strength at axial position is twice that at broad-
side on position for the same distance.
Important: Note the important point that the electric field due to a dipole at large distances falls off as \frac{1}{\text{r}^3} and not as \frac{1}{\text{r}^2} as in the
case of a point charge.

Q653. Apply Gauss’s Theorem to find the electric field near a charged conductor. 8 Marks
OR
Show that the electric field at the surface of a charged conductor is \vec{\text{E}}=\frac{\text{p}}{\epsilon_0}\hat{\text{n}} where \sigma is surface charge density
and \hat{\text{n}} is a unit vector normal to the surface in the outward direction.

Ans:

Let a charge Q be given to a conductor, this charge under electrostatic equilibrium will redistribute and the electric field inside the conductor is zero (i.e., Ein = 0).

https://bls.smartstudies.co.in/#/exam/pdf-preview/b5b87181-b79b-4b4a-8ebf-8f94fb605fc8/1 138/139
5/17/24, 8:15 PM Exam Automation
Let us consider a point P at which electric field strength is to be calculated, just outside the surface of the conductor. Let the surface charge density on the surface
of the conductor in the neighbourhood of P be \sigma coulomb/ metre2 . Now consider a small cylindrical box CD having one base C passing through P; the other
base D lying inside the conductor and the curved surface being perpendicular to the surface of the conductor.
Let the area of each flat base be a. As the surface of the conductor is equipotential surface, the electric field strength E at P, just outside the surface of the
conductor is perpendicular to the surface of the conductor in the neighbourhood of P.
The flux of electric field through the curved surface of the box is zero, since there is no component of electric field E normal to curved surface. Also the flux of
electric field through the base D is zero, as electric field strength inside the conductor is zero. Therefore the resultant flux of electric field through the entire
surface of the box is same as the flux through the face C. This may be analytically seen as:
If S1 and S2 are flat surfaces at C and D and S3 is curved surface, then
Total electric flux,
\oint\limits_{\text{S}}\vec{\text{E}}.\vec{\text{dS}}=\oint\limits_{\text{S}_1}\vec{\text{E}}.\vec{\text{dS}_1}+\oint\limits_{\text{S}_2}\vec{\text{E}}.\vec{\text{dS}_2}+\oint\limits_{\text{S}_3}\
=\oint\limits_{\text{S}_1}{\text{E}}.{\text{dS}_1}\cos0+\oint\limits_{\text{S}_2}{\vec0}.{\text{dS}_2}+\oint\limits_{\text{S}_3}{\text{E}}.{\text{dS}_3}\cos90^\circ
\oint\limits_{\text{S}}\text{E dS}_1=\text{Ea}
As the charge enclosed by the cylinder is (\sigma\text{a}) coulomb, we have, using Gauss’s theorem, =\frac{1}{\epsilon_0}\times\text{charge enclosed}
\text{Ea}=\frac{1}{\epsilon_0}(\sigma\text{a})
or \text{E}=\frac{\text{p}}{\epsilon_0}\dots(\text{i})
Thus the electric field strength at any point close to the surface of a charged conductor of any shape is equal to \frac{1}{\epsilon_0} times the surface charge
density \sigma. This is known as Coulomb’s law. The electric field strength is directed radially away from the conductor if \sigma is positive and towards the
conductor if \sigma is negative.
If \hat{\text{n}} is unit vector normal to surface in outward direction, then
\vec{\text{E}}=\frac{\text{p}}{\epsilon_0}\hat{\text{n}}
Obviously electric field strength near a plane conductor is twice of the electric field strength near a non-conducting thin sheet of charge.

https://bls.smartstudies.co.in/#/exam/pdf-preview/b5b87181-b79b-4b4a-8ebf-8f94fb605fc8/1 139/139

You might also like